Download as pdf or txt
Download as pdf or txt
You are on page 1of 531

WOMEN’S HEALTH

1. Injectable contraceptives
Depo Provera is the main injectable contraceptive used in the UK*. It contains
medroxyprogesterone acetate 150mg. It is given via in intramuscular injection every
12 weeks. It can however be given up to 14 weeks after the last dose without the
need for extra precautions**

The main method of action is by inhibiting ovulation. Secondary effects include


cervical mucus thickening and endometrial thinning.

Disadvantages include the fact that the injection cannot be reversed once given.
There is also a potential delayed return to fertility (maybe up to 12 months)

Adverse effects

• irregular bleeding
• weight gain
• may potentially increase risk of osteoporosis: should only be used in
adolescents if no other method of contraception is suitable
• not quickly reversible and fertility may return after a varying time

Contraindications

• breast cancer: current breast cancer is UKMEC 4, past breast cancer is UKMEC
3

*Noristerat, the other injectable contraceptive licensed in the UK, is rarely used in
clinical practice. It is given every 8 weeks

**the BNF gives different advice, stating a pregnancy test should be done if the
interval is greater than 12 weeks and 5 days - this is however not commonly adhered
to in the family planning community

Explanation: Condoms work as a barrier contraceptive and do not affect ovulation


and hence do not delay fertility.

The intrauterine system (IUS) works by thickening cervical mucous and in some
women may prevent ovulation, however the majority of women still ovulate. After
removal of the IUS the majority of women regain fertility immediately.

The combined oral contraceptive pill can delay return to normal menstrual cycle in
some women but the majority will be able to conceive within a month of stopping.
The progesterone only pill is less likely to delay return to normal cycle as it does not
contain oestrogen.

Because Depo-Provera lasts up to 12 weeks, it can take several months for the body
to return to the normal menstrual cycle and hence delay fertility. For this reason, it is
the least appropriate method for this woman who wants to return to ovulatory cycles
immediately
2. Vaginal candidiasis

Vaginal candidiasis ('thrush') is an extremely common condition which many women


diagnose and treat themselves. Around 80% of cases of Candida albicans, with the
remaining 20% being caused by other candida species.

The majority of women will have no predisposing factors. However, certain factors
may make vaginal candidiasis more likely to develop:

• diabetes mellitus
• drugs: antibiotics, steroids
• pregnancy
• immunosuppression: HIV

Features

• 'cottage cheese', non-offensive discharge


• vulvitis: superficial dyspareunia, dysuria
• itch
• vulval erythema, fissuring, satellite lesions may be seen
Investigations

• a high vaginal swab is not routinely indicated if the clinical features are
consistent with candidiasis

Management

• options include local or oral treatment


• local treatments include clotrimazole pessary (e.g. clotrimazole 500mg PV stat)
• oral treatments include itraconazole 200mg PO bd for 1 day or fluconazole
150mg PO stat
• if pregnant then only local treatments (e.g. cream or pessaries) may be used -
oral treatments are contraindicated

Recurrent vaginal candidiasis


• BASHH define recurrent vaginal candidiasis as 4 or more episodes per year
• compliance with previous treatment should be checked
• confirm the diagnosis of candidiasis
o high vaginal swab for microscopy and culture
o consider a blood glucose test to exclude diabetes
• exclude differential diagnoses such as lichen sclerosus
• consider the use of an induction-maintenance regime
o induction: oral fluconazole every 3 days for 3 doses
o maintenance: oral fluconazole weekly for 6 months
Metronidazole is used in the treatment of bacterial vaginosis and Trichomonas
vaginalis.

3. Antenatal care: timetable

NICE issued guidelines on routine care for the healthy pregnant woman in March
2008. They recommend:

• 10 antenatal visits in the first pregnancy if uncomplicated


• 7 antenatal visits in subsequent pregnancies if uncomplicated
• women do not need to be seen by a consultant if the pregnancy is
uncomplicated

Gestation Purpose of visit


8 - 12 weeks (ideally < Booking visit
10 weeks)
•general information e.g. diet, alcohol, smoking, folic
acid, vitamin D, antenatal classes
• BP, urine dipstick, check BMI
Booking bloods/urine

• FBC, blood group, rhesus status, red cell alloantibodies,


haemoglobinopathies
• hepatitis B, syphilis
• HIV test is offered to all women
• urine culture to detect asymptomatic bacteriuria
10 - 13+6 weeks Early scan to confirm dates, exclude multiple pregnancy
11 - 13+6 weeks Down's syndrome screening including nuchal scan
16 weeks Information on the anomaly and the blood results. If Hb < 11
g/dl consider iron
Routine care: BP and urine dipstick
18 - 20+6 weeks Anomaly scan
25 weeks (only if primip) Routine care: BP, urine dipstick, symphysis-fundal height (SFH)
Gestation Purpose of visit
28 weeks Routine care: BP, urine dipstick, SFH
Second screen for anaemia and atypical red cell
alloantibodies. If Hb < 10.5 g/dl consider iron
First dose of anti-D prophylaxis to rhesus negative women
31 weeks (only if primip) Routine care as above
34 weeks Routine care as above
Second dose of anti-D prophylaxis to rhesus negative women*
Information on labour and birth plan
36 weeks Routine care as above
Check presentation - offer external cephalic version if
indicated
Information on breast feeding, vitamin K, 'baby-blues'
38 weeks Routine care as above
40 weeks (only if primip) Routine care as above
Discussion about options for prolonged pregnancy
41 weeks Routine care as above
Discuss labour plans and possibility of induction

*the evidence base suggests that there is little difference in the efficacy of single-
dose (at 28 weeks) and double-dose regimes (at 28 & 34 weeks). For this reason the
RCOG in 2011 advised that either regime could be used 'depending on local factors'

4. Polycystic ovarian syndrome: management

Polycystic ovarian syndrome (PCOS) is a complex condition of ovarian dysfunction


thought to affect between 5-20% of women of reproductive age. Management is
complicated and problem based partly because the aetiology of PCOS is not fully
understood. Both hyperinsulinaemia and high levels of luteinizing hormone are seen
in PCOS and there appears to be some overlap with the metabolic syndrome.

General

• weight reduction if appropriate


• if a women requires contraception then a combined oral contraceptive (COC)
pill may help regulate her cycle and induce a monthly bleed (see below)

Hirsutism and acne

• a COC pill may be used help manage hirsutism. Possible options include a
third generation COC which has fewer androgenic effects or co-cyprindiol
which has an anti-androgen action. Both of these types of COC may carry an
increased risk of venous thromboembolism
• if doesn't respond to COC then topical eflornithine may be tried
• spironolactone, flutamide and finasteride may be used under specialist
supervision

Infertility

• weight reduction if appropriate


• the management of infertility in patients with PCOS should be supervised by a
specialist. There is an ongoing debate as to whether metformin, clomifene or a
combination should be used to stimulate ovulation
• a 2007 trial published in the New England Journal of Medicine suggested
clomifene was the most effective treatment. There is a potential risk of
multiple pregnancies with anti-oestrogen* therapies such as clomifene. The
RCOG published an opinion paper in 2008 and concluded that on current
evidence metformin is not a first line treatment of choice in the management
of PCOS
• metformin is also used, either combined with clomifene or alone, particularly
in patients who are obese
• gonadotrophins

*work by occupying hypothalamic oestrogen receptors without activating them. This


interferes with the binding of oestradiol and thus prevents negative feedback
inhibition of FSH secretion

Infertility in PCOS - clomifene is superior to metformin

Whilst metformin has a role in the management of infertility it should be used


second-line to anti-oestrogens such as clomifene. Similar questions to this often
appear in which clomifene is not an option, in this case metformin is clearly the right
answer.
5. Cervical cancer screening

The UK has a well established cervical cancer screening program which is estimated
to prevent 1,000-4,000 deaths per year. The main aim of cervical screening is to
detect pre-malignant changes rather than to detect cancer. It should be noted
that cervical adenocarcinomas, which account for around 15% of cases, are
frequently undetected by screening

Who is screened and how often?


A smear test is offered to all women between the ages of 25-64 years

• 25-49 years: 3-yearly screening


• 50-64 years: 5-yearly screening
• cervical screening cannot be offered to women over 64 (unlike breast
screening, where patients can self refer once past screening age)

Special situations

• cervical screening in pregnancy is usually delayed until 3 months post-


partum unless missed screening or previous abnormal smears.
• women who have never been sexually active have very low risk of developing
cervical cancer therefore they may wish to opt-out of screening

How is performed?

There is currently a move away from traditional Papanicolaou (Pap) smears to liquid-
based cytology (LBC). Rather than smearing the sample onto a slide the sample is
either rinsed into the preservative fluid or the brush head is simply removed into the
sample bottle containing the preservative fluid.

Advantages of LBC includes

• reduced rate of inadequate smears


• increased sensitivity and specificity

It is said that the best time to take a cervical smear is around mid-cycle. Whilst there
is limited evidence to support this it is still the current advice given out by the NHS.

6. Combined oral contraceptive pill: advantages/disadvantages

Advantages of combined oral contraceptive pill

• highly effective (failure rate < 1 per 100 woman years)


• doesn't interfere with sex
• contraceptive effects reversible upon stopping
• usually makes periods regular, lighter and less painful
• reduced risk of ovarian, endometrial - this effect may last for several
decades after cessation
• reduced risk of colorectal cancer
• may protect against pelvic inflammatory disease
• may reduce ovarian cysts, benign breast disease, acne vulgaris

Disadvantages of combined oral contraceptive pill

• people may forget to take it


• offers no protection against sexually transmitted infections
• increased risk of venous thromboembolic disease
• increased risk of breast and cervical cancer
• increased risk of stroke and ischaemic heart disease (especially in smokers)
• temporary side-effects such as headache, nausea, breast tenderness may be
seen

Whilst some users report weight gain whilst taking the combined oral contraceptive
pill a Cochrane review did not support a causal relationship.

7. Infertility: initial investigations

Infertility affects around 1 in 7 couples. Around 84% of couples who have regular sex
will conceive within 1 year, and 92% within 2 years

Causes

• male factor 30%


• unexplained 20%
• ovulation failure 20%
• tubal damage 15%
• other causes 15%

Basic investigations

• semen analysis
• serum progesterone 7 days prior to expected next period. For a typical 28 day
cycle, this is done on day 21.

Interpretation of serum progestogen

Level Interpretation
< 16 nmol/l Repeat, if consistently low refer to specialist
16 - 30 nmol/l Repeat
> 30 nmol/l Indicates ovulation
Key counselling points

• folic acid
• aim for BMI 20-25
• advise regular sexual intercourse every 2 to 3 days
• smoking/drinking advice

8. Hypertension in pregnancy

NICE published guidance in 2010 on the management of hypertension in pregnancy.


They also made recommendations on reducing the risk of hypertensive disorders
developing in the first place. Women who are at high risk of developing pre-
eclampsia should take aspirin 75mg od from 12 weeks until the birth of the baby.
High risk groups include:

• hypertensive disease during previous pregnancies


• chronic kidney disease
• autoimmune disorders such as SLE or antiphospholipid syndrome
• type 1 or 2 diabetes mellitus

The classification of hypertension in pregnancy is complicated and varies. Remember,


in normal pregnancy:

• blood pressure usually falls in the first trimester (particularly the diastolic), and
continues to fall until 20-24 weeks
• after this time the blood pressure usually increases to pre-pregnancy levels by
term

Hypertension in pregnancy in usually defined as:

• systolic > 140 mmHg or diastolic > 90 mmHg


• or an increase above booking readings of > 30 mmHg systolic or > 15 mmHg
diastolic

After establishing that the patient is hypertensive they should be categorised into
one of the following groups

Pregnancy-induced hypertension
Pre-existing (PIH, also known as gestational
hypertension hypertension) Pre-eclampsia
A history of hypertension Hypertension (as defined above) Pregnancy-induced
before pregnancy or an occurring in the second half of hypertension in association
Pregnancy-induced hypertension
Pre-existing (PIH, also known as gestational
hypertension hypertension) Pre-eclampsia
elevated blood pressure pregnancy (i.e. after 20 weeks) with proteinuria (> 0.3g / 24
> 140/90 mmHg before hours)
20 weeks gestation No proteinuria, no oedema
Oedema may occur but is
No proteinuria, no Occurs in around 5-7% of now less commonly used as
oedema pregnancies a criteria

Occurs in 3-5% of Resolves following birth (typically Occurs in around 5% of


pregnancies and is more after one month). Women with PIH pregnancies
common in older women are at increased risk of future pre-
eclampsia or hypertension later in
life

9. Contraception: mode of action

The table below is based on documents produced by the Faculty for Sexual and
Reproductive Health (FSRH).

Standard contraceptives:

Contraceptive Mode of action


Combined oral contraceptive pill Inhibits ovulation
Progestogen-only pill (excluding desogestrel) Thickens cervical mucus
Desogestrel-only pill Primary: Inhibits ovulation
Also: thickens cervical mucus
Injectable contraceptive Primary: Inhibits ovulation
(medroxyprogesterone acetate) Also: thickens cervical mucus
Implantable contraceptive (etonogestrel) Primary: Inhibits ovulation
Also: thickens cervical mucus
Intrauterine contraceptive device Decreases sperm motility and survival
Intrauterine system (levonorgestrel) Primary: Prevents endometrial proliferation
Also: Thickens cervical mucus

Methods of emergency contraception:

Contraceptive Mode of action


Levonorgestrel Inhibits ovulation
Contraceptive Mode of action
Ulipristal Inhibits ovulation
Intrauterine contraceptive device Primary: Toxic to sperm and ovum
Also: Inhibits implantation

10. Urinary incontinence

Urinary incontinence (UI) is a common problem, affecting around 4-5% of the


population. It is more common in elderly females.

Risk factors

• advancing age
• previous pregnancy and childbirth
• high body mass index
• hysterectomy
• family history

Classification

• overactive bladder (OAB)/urge incontinence: due to detrusor overactivity


• stress incontinence: leaking small amounts when coughing or laughing
• mixed incontinence: both urge and stress
• overflow incontinence: due to bladder outlet obstruction, e.g. due to prostate
enlargement

Initial investigation

• bladder diaries should be completed for a minimum of 3 days


• vaginal examination to exclude pelvic organ prolapse and ability to initiate
voluntary contraction of pelvic floor muscles ('Kegel' exercises)
• urine dipstick and culture
• urodynamic studies

Management depends on whether urge or stress UI is the predominant picture. If


urge incontinence is predominant:

• bladder retraining (lasts for a minimum of 6 weeks, the idea is to gradually


increase the intervals between voiding)
• bladder stabilising drugs: antimuscarinics are first-line. NICE recommend
oxybutynin (immediate release), tolterodine (immediate release) or darifenacin
(once daily preparation). Immediate release oxybutynin should, however,
be avoided in 'frail older women'
• mirabegron (a beta-3 agonist) may be useful if there is concern about
anticholinergic side-effects in frail elderly patients

If stress incontinence is predominant:

• pelvic floor muscle training: NICE recommend at least 8 contractions


performed 3 times per day for a minimum of 3 months
• surgical procedures: e.g. retropubic mid-urethral tape procedures
• duloxetine may be offered to women if they decline surgical procedures
o a combined noradrenaline and serotonin reuptake inhibitor
o mechanism of action: increased synaptic concentration of
noradrenaline and serotonin within the pudendal nerve → increased
stimulation of urethral striated muscles within the sphincter →
enhanced contraction
Stress incontinence is an involuntary leakage of urine on effort or exertion, or on
sneezing or coughing. Diagnosis is symptomatic and a bladder diary may be helpful
in assessing the severity of symptoms. Treatment is with lifestyle measures such as
reducing caffeine, losing weight and avoiding excess fluid. Pelvic floor exercises
should also be advised. If symptoms continue despite these measures, surgery can
be offered. If the woman is unsuitable for surgery or does not wish to have surgery,
duloxetine which is a serotonin-norepinephrine reuptake inhibitors (SNRI) can be
offered.

A ring pessary is used in the treatment of vaginal prolapse and would not be
effective in the treatment of stress incontinence. Sertraline is a SSRI (Selective
serotonin reuptake inhibitor) which is used in the treatment of depression, obsessive-
compulsive disorder and anxiety. O

xybutynin and tolterodine are anticholinergic medications used in the treatment of


urge incontinence.
11. Epilepsy: pregnancy and breast feeding

The risks of uncontrolled epilepsy during pregnancy generally outweigh the risks of
medication to the fetus. All women thinking about becoming pregnant should be
advised to take folic acid 5mg per day well before pregnancy to minimise the risk of
neural tube defects. Around 1-2% of newborns born to non-epileptic mothers have
congenital defects. This rises to 3-4% if the mother takes antiepileptic medication.

Other points

• aim for monotherapy


• there is no indication to monitor antiepileptic drug levels
• sodium valproate: associated with neural tube defects
• carbamazepine: often considered the least teratogenic of the older
antiepileptics
• phenytoin: associated with cleft palate
• lamotrigine: studies to date suggest the rate of congenital malformations may
be low. The dose of lamotrigine may need to be increased in pregnancy

Breast feeding is generally considered safe for mothers taking antiepileptics with the
possible exception of the barbiturates

It is advised that pregnant women taking phenytoin are given vitamin K in the last
month of pregnancy to prevent clotting disorders in the newborn

Sodium valproate

The November 2013 issue of the Drug Safety Update also carried a warning about
new evidence showing a significant risk of neurodevelopmental delay in children
following maternal use of sodium valproate.

The update concludes that sodium valproate should not be used during pregnancy
and in women of childbearing age unless clearly necessary. Women of childbearing
age should not start treatment without specialist neurological or psychiatric advice.

12. Miscarriage

Threatened miscarriage

• painless vaginal bleeding occurring before 24 weeks, but typically occurs at 6 -


9 weeks
• the bleeding is often less than menstruation
• cervical os is closed
• complicates up to 25% of all pregnancies

Missed (delayed) miscarriage

• a gestational sac which contains a dead fetus before 20 weeks without the
symptoms of expulsion
• mother may have light vaginal bleeding / discharge and the symptoms of
pregnancy which disappear. Pain is not usually a feature
• cervical os is closed
• when the gestational sac is > 25 mm and no embryonic/fetal part can be seen
it is sometimes described as a 'blighted ovum' or 'anembryonic pregnancy'
Inevitable miscarriage

• heavy bleeding with clots and pain


• cervical os is open

Incomplete miscarriage

• not all products of conception have been expelled


• pain and vaginal bleeding
• cervical os is open

13. Oligohydramnios

In oligohydramnios there is reduced amniotic fluid. Definitions vary but include less
than 500ml at 32-36 weeks and an amniotic fluid index (AFI) < 5th percentile.

Causes

• premature rupture of membranes


• fetal renal problems e.g. renal agenesis
• intrauterine growth restriction
• post-term gestation
• pre-eclampsia
Oligohydraminos is a conditions where there is a deficiency of amniotic fluid during
pregnancy. This can often present as smaller symphysiofundal height.

Renal agenesis is a cause of oligohydraminos (abnormally low volume of amniotic


fluid) as the amniotic fluid is mainly derived from foetal urine.
14. Pre-eclampsia

Pre-eclampsia is a condition seen after 20 weeks gestation characterised by


pregnancy-induced hypertension in association with proteinuria (> 0.3g / 24 hours).
Oedema used to be third element of the classic triad but is now often not included in
the definition as it is not specific

Pre-eclampsia is important as it predisposes to the following problems

• fetal: prematurity, intrauterine growth retardation


• eclampsia
• haemorrhage: placental abruption, intra-abdominal, intra-cerebral
• cardiac failure
• multi-organ failure
NICE divide risk factors into high and moderate risk:

High risk factors Moderate risk factors


• hypertensive disease in a • first pregnancy
previous pregnancy • age 40 years or older
• chronic kidney disease • pregnancy interval of more than 10
• autoimmune disease, such as years
systemic lupus erythematosus or • body mass index (BMI) of 35 kg/m² or
antiphospholipid syndrome more at first visit
• type 1 or type 2 diabetes • family history of pre-eclampsia
• chronic hypertension • multiple pregnancy

Features of severe pre-eclampsia

• hypertension: typically > 170/110 mmHg and proteinuria as above


• proteinuria: dipstick ++/+++
• headache
• visual disturbance
• papilloedema
• RUQ/epigastric pain
• hyperreflexia
• platelet count < 100 * 106/l, abnormal liver enzymes or HELLP syndrome

Management

• a woman at moderate or high risk of pre-eclampsia should take aspirin 75mg


daily from 12 weeks gestation until the birth
• consensus guidelines recommend treating blood pressure > 160/110 mmHg
although many clinicians have a lower threshold
• oral labetalol is now first-line following the 2010 NICE guidelines. Nifedipine
(e.g. if asthmatic) and hydralazine may also be used
• delivery of the baby is the most important and definitive management step.
The timing depends on the individual clinical scenario

15. Shoulder dystocia

Shoulder dystocia is a complication of vaginal cephalic delivery. It entails inability to


deliver the body of the fetus using gentle traction, the head having already been
delivered. Shoulder dystocia is a cause of both maternal and fetal morbidity. It is
associated with postpartum haemorrhage and perineal tears with respect to the
former, and brachial plexus injury with respect to the latter, amongst other
complications. Neonatal death occasionally occurs.
Key risk factors for shoulder dystocia include fetal macrosomia, high maternal body
mass index, diabetes mellitus and prolonged labour. It usually occurs due to
impaction of the anterior fetal shoulder on the maternal pubic symphysis.
Additionally help should be called as soon as shoulder dystocia is identified and
McRoberts' manoeuvre should be performed. This manoeuvre entails flexion and
abduction of the maternal hips, bringing the mother's thighs towards her abdomen.
This rotation increases the relative anterior-posterior angle of the pelvis and often
facilitates a successful delivery.

An episiotomy will not relieve the bony obstruction but is sometimes used to allow
better access for internal manoeuvres. Symphysiotomy and the Zavanelli manoeuvre
can cause significant maternal morbidity and are not first-line options. Oxytocin
administration is not indicated in shoulder dystocia.
(RCOG Green-top Guideline No. 42)

16. Mittelschmerz

Mittelschmerz literally translates to 'middle pain' and refers to abdominal pain


associated with ovulation. This mid-cyclical pain is experienced by 20% of women
and there are several theories as to why it occurs. One explanation is that is occurs
due to a leakage of follicular fluid containing prostaglandins at the time of ovulation,
which causes the pain. Another explanation is that the growth of the follicle stretches
the surface of the ovary, causing pain.

Presentation

• Sudden onset of pain in either iliac fossa which then manifests as a


generalised pelvic pain.
• Typically, the pain is not severe and varies in duration, lasting from minutes to
hours.
• It is self-limiting and resolves within 24 hours of onset.
• Pain may switch side from month to month, depending on the site of
ovulation

Investigations

• There is no specific test to confirm Mittelschmerz and it diagnosed clinically,


after taking a full history and examination to exclude other conditions
• No abnormal signs on abdominal or pelvic examination.

Management
• Mittelschmerz is not harmful and can be controlled with simple analgesia.

17. Chickenpox exposure in pregnancy

Chickenpox is caused by primary infection with varicella-zoster virus. Shingles is


caused by the reactivation of dormant virus in dorsal root ganglion. In pregnancy,
there is a risk to both the mother and also the fetus, a syndrome now termed fetal
varicella syndrome

Risks to the mother

• 5 times greater risk of pneumonitis

Fetal varicella syndrome (FVS)

• risk of FVS following maternal varicella exposure is around 1% if occurs before


20 weeks gestation
• studies have shown a very small number of cases occurring between 20-28
weeks gestation and none following 28 weeks
• features of FVS include skin scarring, eye defects (microphthalmia), limb
hypoplasia, microcephaly and learning disabilities

Other risks to the fetus

• shingles in infancy: 1-2% risk if maternal exposure in the second or third


trimester
• severe neonatal varicella: if the mother develops rash between 5 days before
and 2 days after birth there is a risk of neonatal varicella, which may be fatal to
the newborn child in around 20% of cases

Management of chickenpox exposure in pregnancy, i.e. post-exposure prophylaxis


(PEP)

• if there is any doubt about the mother previously having chickenpox maternal
blood should be urgently checked for varicella antibodies
• if the pregnant woman <= 20 weeks gestation is not immune to varicella she
should be given varicella-zoster immunoglobulin (VZIG) as soon as possible
o RCOG and Greenbook guidelines suggest VZIG is effective up to 10
days post exposure
• if the pregnant woman > 20 weeks gestation is not immune to varicella then
either VZIG or antivirals (aciclovir or valaciclovir) should be given days 7 to 14
after exposure
o the Public Health England (PHE) guidelines state that 'The decision on
choice of PEP for women exposed from 20 weeks of pregnancy should
take into account patient and health professional preference as well as
the ability to offer and provide PEP in a timely manner'
o why wait until days 7-14? From the PHE guidelines: 'In a study
evaluating the comparative effectiveness of 7 days course of aciclovir
given either immediately after exposure or starting at day 7 after
exposure to healthy children, the incidence and severity of varicella
infection was significantly higher in those given aciclovir immediately
(10/13 (77%) who received aciclovir immediately developed clinical
varicella compared with 3/14 (21%) who started aciclovir at day 7)'
o it seems part of this guidance is related to a limited supply of VZIG
within the NHS

Management of chickenpox in pregnancy

• if a pregnant woman develops chickenpox in pregnancy then specialist advice


should be sought
• there is an increased risk of serious chickenpox infection (i.e. maternal risk)
and fetal varicella risk (i.e. fetal risk) balanced against theoretical concerns
about the safety of aciclovir in pregnancy
• consensus guidelines (Health Protection Authority and RCOG) suggest oral
aciclovir should be given if the pregnant women is ≥ 20 weeks and she
presents within 24 hours of onset of the rash
• if the woman is < 20 weeks the aciclovir should be 'considered with caution'

18. Contraception: a very basic introduction

The development of effective methods of contraception over the past 50 years has
been one of the most significant developments in medicine.

Methods of contraception

Barrier methods

• condoms

Daily methods

• combined oral contraceptive pill


• progesterone only pill
Long-acting methods of reversible contraception (LARCs)

• implantable contraceptives
• injectable contraceptives
• intrauterine system (IUS): progesterone releasing coil
• intrauterine device (IUD): copper coil

Contraceptive Method of action Notes


Condoms Physical barrier Relatively low success rate,
particularly when used by young
people
Help protects against STIs
Combined oral Inhibits ovulation Increases risk of venous
contraceptive pill thromboembolism
Increases risk of breast and cervical
cancer
Progestogen-only pill Thickens cervical mucus Irregular bleeding a common side-
(excluding effect
desogestrel*)
Injectable Primary: Inhibits ovulation Lasts 12 weeks
contraceptive Also: thickens cervical
(medroxyprogesterone mucus
acetate)
Implantable Primary: Inhibits ovulation Irregular bleeding a common side-
contraceptive Also: thickens cervical effect
(etonogestrel) mucus Last 3 years
Intrauterine Decreases sperm motility
contraceptive device and survival
Intrauterine system Primary: Prevents Irregular bleeding a common side-
(levonorgestrel) endometrial proliferation effect
Also: Thickens cervical
mucus

*desogestrel is a type of progestogen-only pill which also inhibits ovulation


The combined pill is completely contraindicated in this scenario (category 4)
according to the UKMEC guidelines for contraception use as it drastically reduces
breast milk volume. These guidelines are a valuable source for contraception advice
and can be found here http://www.fsrh.org/documents/ukmec-2009/

The progesterone only pill is safe to use (category 1) as is Depo-Provera injections


(category 2). Natural birth control is an acceptable method in all scenarios. The
Mirena coil is relatively contraindicated (category 3) due to the increased risk of
perforation but is not absolutely contraindicated.

19. Vaginal candidiasis

Vaginal candidiasis ('thrush') is an extremely common condition which many women


diagnose and treat themselves. Around 80% of cases of Candida albicans, with the
remaining 20% being caused by other candida species.

The majority of women will have no predisposing factors. However, certain factors
may make vaginal candidiasis more likely to develop:

• diabetes mellitus
• drugs: antibiotics, steroids
• pregnancy
• immunosuppression: HIV

Features

• 'cottage cheese', non-offensive discharge


• vulvitis: superficial dyspareunia, dysuria
• itch
• vulval erythema, fissuring, satellite lesions may be seen

Investigations

• a high vaginal swab is not routinely indicated if the clinical features are
consistent with candidiasis

Management

• options include local or oral treatment


• local treatments include clotrimazole pessary (e.g. clotrimazole 500mg PV stat)
• oral treatments include itraconazole 200mg PO bd for 1 day or fluconazole
150mg PO stat
• if pregnant then only local treatments (e.g. cream or pessaries) may be used -
oral treatments are contraindicated

Recurrent vaginal candidiasis

• BASHH define recurrent vaginal candidiasis as 4 or more episodes per year


• compliance with previous treatment should be checked
• confirm the diagnosis of candidiasis
o high vaginal swab for microscopy and culture
o consider a blood glucose test to exclude diabetes
• exclude differential diagnoses such as lichen sclerosus
• consider the use of an induction-maintenance regime
o induction: oral fluconazole every 3 days for 3 doses
o maintenance: oral fluconazole weekly for 6 months
This patient is presenting with symptoms consistent with thrush (vaginal candidiasis).
Over the counter treatments include antifungals (such as clotrimazole cream) and are
usually successful for management of one-off episodes of vaginal candidiasis.

As this patient is experiencing recurrent symptoms, an oral preparation of


fluconazole is recommended in the British National Formulary. It is important to
ensure that the patient is not taking SSRI medications or has hypersensitivity to
'azole' antifungal medications.

IM ceftriaxone and oral ciprofloxacin can be used to manage Neisseria


gonorrhoea not thrush.

Oral metronidazole can be used to manage bacterial vaginosis, not thrush.

Topical fluconazole is an uncommon preparation as it is strongly hydrophilic and is


not as effective in comparison with other antifungals (such as topical clotrimazole).

20. Combined oral contraceptive pill: contraindications

The decision of whether to start a women on the combined oral contraceptive pill is
now guided by the UK Medical Eligibility Criteria (UKMEC). This scale categorises the
potential cautions and contraindications according to a four point scale, as detailed
below:

• UKMEC 1: a condition for which there is no restriction for the use of the
contraceptive method
• UKMEC 2: advantages generally outweigh the disadvantages
• UKMEC 3: disadvantages generally outweigh the advantages
• UKMEC 4: represents an unacceptable health risk

Examples of UKMEC 3 conditions include

• more than 35 years old and smoking less than 15 cigarettes/day


• BMI > 35 kg/m^2*
• family history of thromboembolic disease in first degree relatives < 45 years
• controlled hypertension
• immobility e.g. wheel chair use
• carrier of known gene mutations associated with breast cancer (e.g.
BRCA1/BRCA2)
• current gallbladder disease

Examples of UKMEC 4 conditions include

• more than 35 years old and smoking more than 15 cigarettes/day


• migraine with aura
• history of thromboembolic disease or thrombogenic mutation
• history of stroke or ischaemic heart disease
• breast feeding < 6 weeks post-partum
• uncontrolled hypertension
• current breast cancer
• major surgery with prolonged immobilisation

Diabetes mellitus diagnosed > 20 years ago is classified as UKMEC 3 or 4 depending


on severity

Changes in 2016

• breast feeding 6 weeks - 6 months postpartum was changed from UKMEC 3 →


2
Oestrogen-containing contraceptives should preferably be discontinued 4 weeks
before major elective surgery and all surgery to the legs or surgery which involves
prolonged immobilisation of a lower limb. A progestogen-only contraceptive may be
offered as an alternative and the oestrogen-containing contraceptive restarted after
mobilisation.

These recommendations do not apply to minor surgery with short duration of


anaesthesia, e.g. laparoscopic sterilisation or tooth extraction, or to women using
oestrogen-free hormonal contraceptives.
21. Antenatal care: specific points

NICE issued guidelines on routine care for the healthy pregnant woman in March
2008

Nausea and vomiting

• natural remedies - ginger and acupuncture on the 'p6' point (by the wrist) are
recommended by NICE
• antihistamines should be used first-line (BNF suggests promethazine as first-
line)
Vitamin D

• NICE recommend 'All women should be informed at the booking appointment


about the importance for their own and their baby's health of maintaining
adequate vitamin D stores during pregnancy and whilst breastfeeding'
• 'women may choose to take 10 micrograms of vitamin D per day, as found in
the Healthy Start multivitamin supplement'. This was confirmed in 2012 when
the Chief Medical Officer advised: 'All pregnant and breastfeeding women
should take a daily supplement containing 10micrograms of vitamin D, to
ensure the mothers requirements for vitamin D are met and to build adequate
fetal stores for early infancy'
• particular care should be taken with women at risk (e.g. Asian, obese, poor
diet)

Alcohol

• in 2016 the Chief Medical Officer proposed new guidelines in relation to the
safe consumption of alcohol following an expert group report.
• the government now recommend pregnant women should not drink. The
wording of the official advice is 'If you are pregnant or planning a pregnancy,
the safest approach is not to drink alcohol at all, to keep risks to your baby to
a minimum. Drinking in pregnancy can lead to long-term harm to the baby,
with the more you drink the greater the risk.'
This patient has a low risk of conceiving a child with neural tube defects and she is
not at high risk of getting vitamin D deficiency. She is therefore recommended the
standard care. All pregnant women should take a daily supplement containing
10micrograms of vitamin D and 400 microgram of Folic acid.

22. Endometriosis

Endometriosis is a common condition characterised by the growth of ectopic


endometrial tissue outside of the uterine cavity. Around 10% of women of a
reproductive age have a degree of endometriosis.

Clinical features

• chronic pelvic pain


• dysmenorrhoea - pain often starts days before bleeding
• deep dyspareunia
• subfertility
• non-gynaecological: urinary symptoms e.g. dysuria, urgency, haematuria.
Dyschezia (painful bowel movements)
• on pelvic examination reduced organ mobility, tender nodularity in the
posterior vaginal fornix and visible vaginal endometriotic lesions may be seen

Investigation

• laparoscopy is the gold-standard investigation


• there is little role for investigation in primary care (e.g. ultrasound)- if the
symptoms are significant the patient should be referred for a definitive
diagnosis

Management depends on clinical features - there is poor correlation between


laparoscopic findings and severity of symptoms. NICE published guidelines in 2017:

• NSAIDs and/or paracetamol are the recommended first-line treatments for


symptomatic relief
• if analgesia does help then hormonal treatments such as the combined oral
contraceptive pill or progestogens e.g. medroxyprogesterone acetate
should be tried

If analgesia/hormonal treatment does not improve symptoms or if fertility is a


priority the patient should be referred to secondary care. Secondary treatments
include:

• GnRH analogues - said to induce a 'pseudomenopause' due to the low


oestrogen levels
• drug therapy unfortunately does not seem to have a significant impact on
fertility rates
• surgery: some treatments such as laparoscopic excision and laser treatment of
endometriotic ovarian cysts may improve fertility

23. Postpartum haemorrhage

Postpartum haemorrhage (PPH) is defined as blood loss of > 500mls and may be
primary or secondary

Primary PPH

• occurs within 24 hours


• affects around 5-7% of deliveries
• most common cause of PPH is uterine atony (90% of cases). Other causes
include genital trauma and clotting factors
Risk factors for primary PPH include*:

• previous PPH
• prolonged labour
• pre-eclampsia
• increased maternal age
• polyhydramnios
• emergency Caesarean section
• placenta praevia, placenta accreta
• macrosomia
• ritodrine (a beta-2 adrenergic receptor agonist used for tocolysis)

Management

• ABC including two peripheral cannulae, 14 gauge


• IV syntocinon (oxytocin) 10 units or IV ergometrine 500 micrograms
• IM carboprost
• if medical options failure to control the bleeding then surgical options will
need to be urgently considered
• the RCOG state that the intrauterine balloon tamponade is an appropriate
first-line ‘surgical’ intervention for most women where uterine atony is the
only or main cause of haemorrhage
• other options include: B-Lynch suture, ligation of the uterine arteries or
internal iliac arteries
• if severe, uncontrolled haemorrhage then a hysterectomy is sometimes
performed as a life-saving procedure

Secondary PPH

• occurs between 24 hours - 12 weeks**


• due to retained placental tissue or endometritis

*the effect of parity on the risk of PPH is complicated. It was previously though
multiparity was a risk factor but more modern studies suggest nulliparity is actually a
risk factor

**previously the definition of secondary PPH was 24 hours - 6 weeks. Please see the
RCOG guidelines for more details

Uterine atony is the most common cause of primary postpartum haemorrhage. It


entails failure of the uterus to contract fully following the delivery of the placenta,
which hinders the achievement of haemostasis. Uterine atony is associated with
overdistension, which may be due to multiple gestation, macrosomia,
polyhydramnios or other causes.

In addition to the usual steps taken in an episode of PPH (including an ABC approach
if the patient is unstable), the following management should be initiated in sequence:

• bimanual uterine compression to manually stimulate contraction


• intravenous oxytocin and/or ergometrine
• intramuscular carboprost
• intramyometrial carboprost
• rectal misoprostol
• surgical intervention such as balloon tamponade

24. Ectopic pregnancy

Implantation of a fertilized ovum outside the uterus results in an ectopic pregnancy

A typical history is a female with a history of 6-8 weeks amenorrhoea who presents
with lower abdominal pain and later develops vaginal bleeding

• lower abdominal pain


o due to tubal spasm
o typically the first symptom
o pain is usually constant and may be unilateral.
• vaginal bleeding
o usually less than a normal period
o may be dark brown in colour
• history of recent amenorrhoea
o typically 6-8 weeks from the start of last period
o if longer (e.g. 10 wks) this suggest another causes e.g. inevitable
abortion
• peritoneal bleeding can cause shoulder tip pain and pain on defecation /
urination
• dizziness, fainting or syncope may be seen
• symptoms of pregnancy such as breast tenderness may also be reported

Examination findings

• abdominal tenderness
• cervical excitation (also known as cervical motion tenderness)
• adnexal mass: NICE advise NOT to examine for an adnexal mass due to an
increased risk of rupturing the pregnancy. A pelvic examination to check for
cervical excitation is however recommended

In the case of pregnancy of unknown location, serum bHCG levels >1,500 points
toward a diagnosis of an ectopic pregnancy
The correct answer here is an ectopic pregnancy, as this is a common presentation
for a ruptured ectopic. 7 weeks gestation is a common time for ectopics to become
symptomatic, since they have by this point grown large enough to stretch the tubes
and therefore cause pain.

A threatened miscarriage means that at the moment, the fetus is viable but the lady
may be having some bleeding. A threatened miscarriage would, by definition, be
seen on ultrasound scan, thus cannot be the explanation in this case. You cannot
diagnose a threatened miscarriage without seeing a viable fetus on ultrasound scan.

When the beta HCG is greater than 1000 it implies that the fetus is large enough to
be seen on an ultrasound scan, and therefore the second option here is unlikely.

A complete miscarriage means that the products of conception have all passed, and
thus the womb is now empty. Since this lady has only suffered from mild bleeding, it
does not sound like she has miscarried yet.

Fibroids can sometimes make it difficult to visualise the inside of the womb,
especially if there are many. However, with the modern transvaginal scanners it is
becoming more possible to view the gestational sac even if there are fibroids
present, and therefore this should never be the presumed cause of an inconclusive
scan. Other pathology, especially ectopic pregnancies, should be excluded first.
Ectopic pregnancy: investigation and management

Women who are stable are typically investigated and managed in an early pregnancy
assessment unit. If a woman is unstable then she should be referred to the
emergency department.

Investigation

A pregnancy test will be positive.

The investigation of choice for ectopic pregnancy is a transvaginal ultrasound.


Management

There are 3 ways to manage ectopic pregnancies. And the following criteria can help
to guide you which method your patient will be able to have.

Expectant management Medical management Surgical management


Size <35mm Size <35mm Size >35mm
Unruptured Unruptured Can be ruptured
Asymptomatic No significant pain Pain
No fetal heartbeat No fetal heartbeat Visible fetal heartbeat
serum B-hCG <1,000IU/L serum B-hCG <1,500IU/L serum B-hCG >1,500IU/L
Compatible if another Not suitable if Compatible with another
intrauterine pregnancy intrauterine pregnancy intrauterine pregnancy
Expectant management Medical management Surgical management can
involves closely monitoring involves giving the involve salpingectomy or
the patient over 48 hours and patient methotrexate and salpingotomy
if B-hCG levels rise again or can only be done if the
symptoms manifest patient is willing to
intervention is performed. attend follow up.

25. Rubella and pregnancy

Rubella, also known as German measles, is a viral infection caused by the togavirus.
Following the introduction of the MMR vaccine it is now rare. If contracted during
pregnancy there is a risk of congenital rubella syndrome. Remember that the
incubation period is 14-21 days and individuals are infectious from 7 days before
symptoms appear to 4 days after the onset of the rash.

Risk

• in first 8-10 weeks risk of damage to fetus is as high as 90%


• damage is rare after 16 weeks

Features of congenital rubella syndrome

• sensorineural deafness
• congenital cataracts
• congenital heart disease (e.g. patent ductus arteriosus)
• growth retardation
• hepatosplenomegaly
• purpuric skin lesions
• 'salt and pepper' chorioretinitis
• microphthalmia
• cerebral palsy

Diagnosis

• suspected cases should be discussed immediately with the local Health


Protection Unit (HPU) as type/timing of investigations may vary
• IgM antibodies are raised in women recently exposed to the virus
• it should be noted that it is very difficult to distinguish rubella from parvovirus
B19 clinically. It is therefore important to also check parvovirus B19 serology
as there is a 30% risk of transplacental infection, with a 5-10% risk of fetal loss

Management

• suspected cases of rubella in pregnancy should be discussed with the local


Health Protection Unit
• since 2016, rubella immunity is no longer routinely checked at the booking
visit
• if a woman is however tested at any point and no immunity is demonstrated
they should be advised to keep away from people who might have rubella
• non-immune mothers should be offered the MMR vaccination in the post-
natal period
o MMR vaccines should not be administered to women known to be
pregnant or attempting to become pregnant
MMR cannot be given during pregnancy.

26. Folic acid

Folic acid is converted to tetrahydrofolate (THF). Green, leafy vegetables are a good
source of folic acid.

Functions

• THF plays a key role in the transfer of 1-carbon units (e.g. methyl, methylene,
and formyl groups) to the essential substrates involved in the synthesis of
DNA & RNA

Causes of folic acid deficiency:

• phenytoin
• methotrexate
• pregnancy
• alcohol excess

Consequences of folic acid deficiency:

• macrocytic, megaloblastic anaemia


• neural tube defects

Prevention of neural tube defects (NTD) during pregnancy:

• all women should take 400mcg of folic acid until the 12th week of pregnancy
• women at higher risk of conceiving a child with a NTD should take 5mg of
folic acid from before conception until the 12th week of pregnancy
• women are considered higher risk if any of the following apply:
o either partner has a NTD, they have had a previous pregnancy affected
by a NTD, or they have a family history of a NTD
o the woman is taking antiepileptic drugs or has coeliac disease, diabetes,
or thalassaemia trait.
o the woman is obese (defined as a body mass index [BMI] of 30
kg/m2 or more).

27. Emergency contraception

There are two methods currently available in the UK:

Emergency hormonal contraception

There are now two methods of emergency hormonal contraception ('emergency pill',
'morning-after pill'); levonorgestrel and ulipristal, a progesterone receptor
modulator.

Levonorgestrel

• mode of action not fully understood - acts both to stop ovulation and inhibit
implantation
• should be taken as soon as possible - efficacy decreases with time
• must be taken within 72 hours of unprotected sexual intercourse (UPSI)*
• single dose of levonorgestrel 1.5mg (a progesterone)
o the dose should be doubled for those with a BMI >26 or weight over
70kg
• 84% effective is used within 72 hours of UPSI
• levonorgestrel is safe and well-tolerated. Disturbance of the current menstrual
cycle is seen in a significant minority of women. Vomiting occurs in around 1%
• if vomiting occurs within 3 hours then the dose should be repeated
• can be used more than once in a menstrual cycle if clinically indicated
• hormonal contraception can be started immediately after using levornogestrel
(Levonelle) for emergency contraception

Ulipristal (ellaOne)

• a selective progesterone receptor modulator currently marketed as EllaOne.


The primary mode of action is thought to be inhibition of ovulation
• 30mg oral dose taken as soon as possible, no later than 120 hours after
intercourse
• concomitant use with levonorgestrel is not recommended
• Ulipristal may reduce the effectiveness of hormonal
contraception. Contraception with the pill, patch or ring should be started, or
restarted, 5 days after having ulipristal. Barrier methods should be used during
this period
• caution should be exercised in patients with severe asthma
• repeated dosing within the same menstrual cycle was previously not
recommended - however, this has now changed and ulipristal can be used
more than once in the same cycle
• breastfeeding should be delayed for one week after taking ulipristal. There are
no such restrictions on the use of levonorgestrel

Intrauterine device (IUD)

• must be inserted within 5 days of UPSI, or


• if a women presents after more than 5 days then an IUD may be fitted up to 5
days after the likely ovulation date
• may inhibit fertilisation or implantation
• prophylactic antibiotics may be given if the patient is considered to be at
high-risk of sexually transmitted infection
• is 99% effective regardless of where it is used in the cycle
• may be left in-situ to provide long-term contraception. If the client wishes for
the IUD to be removed it should be at least kept in until the next period

*may be offered after this period as long as the client is aware of reduced
effectiveness and unlicensed indication
28. Polycystic ovarian syndrome: management

Polycystic ovarian syndrome (PCOS) is a complex condition of ovarian dysfunction


thought to affect between 5-20% of women of reproductive age. Management is
complicated and problem based partly because the aetiology of PCOS is not fully
understood. Both hyperinsulinaemia and high levels of luteinizing hormone are seen
in PCOS and there appears to be some overlap with the metabolic syndrome.

General

• weight reduction if appropriate


• if a women requires contraception then a combined oral contraceptive (COC)
pill may help regulate her cycle and induce a monthly bleed (see below)

Hirsutism and acne

• a COC pill may be used help manage hirsutism. Possible options include a
third generation COC which has fewer androgenic effects or co-cyprindiol
which has an anti-androgen action. Both of these types of COC may carry an
increased risk of venous thromboembolism
• if doesn't respond to COC then topical eflornithine may be tried
• spironolactone, flutamide and finasteride may be used under specialist
supervision

Infertility

• weight reduction if appropriate


• the management of infertility in patients with PCOS should be supervised by a
specialist. There is an ongoing debate as to whether metformin, clomifene or a
combination should be used to stimulate ovulation
• a 2007 trial published in the New England Journal of Medicine suggested
clomifene was the most effective treatment. There is a potential risk of
multiple pregnancies with anti-oestrogen* therapies such as clomifene. The
RCOG published an opinion paper in 2008 and concluded that on current
evidence metformin is not a first line treatment of choice in the management
of PCOS
• metformin is also used, either combined with clomifene or alone, particularly
in patients who are obese
• gonadotrophins

*work by occupying hypothalamic oestrogen receptors without activating them. This


interferes with the binding of oestradiol and thus prevents negative feedback
inhibition of FSH secretion
29. Ovarian cancer

Ovarian cancer is the fifth most common malignancy in females. The peak age of
incidence is 60 years and it generally carries a poor prognosis due to late diagnosis.

Pathophysiology

• around 90% of ovarian cancers are epithelial in origin, with 70-80% of cases
being due to serous carcinomas
• interestingly, it is now increasingly recognised that the distal end of the
fallopian tube is often the site of origin of many 'ovarian' cancers

Risk factors

• family history: mutations of the BRCA1 or the BRCA2 gene


• many ovulations*: early menarche, late menopause, nulliparity

Clinical features are notoriously vague

• abdominal distension and bloating


• abdominal and pelvic pain
• urinary symptoms e.g. Urgency
• early satiety
• diarrhoea

Investigations

• CA125
o NICE recommends a CA125 test is done initially. Endometriosis,
menstruation, benign ovarian cysts and other conditions may also raise
the CA125 level
o if the CA125 is raised (35 IU/mL or greater) then an urgent ultrasound
scan of the abdomen and pelvis should be ordered
o a CA125 should not be used for screening for ovarian cancer in
asymptomatic women
• ultrasound

Diagnosis is difficult and usually involves diagnostic laparotomy

Management
• usually a combination of surgery and platinum-based chemotherapy

Prognosis

• 80% of women have advanced disease at presentation


• the all stage 5-year survival is 46%

*It is traditionally taught that infertility treatment increases the risk of ovarian cancer,
as it increases the number of ovulations. Recent evidence however suggests that
there is not a significant link. The combined oral contraceptive pill reduces the risk
(fewer ovulations) as does having many pregnancies.
The risk factors for ovarian cancer are hormonal in nature. A woman's risk is greater if
ovulation is not suppressed. In this way, early menarche and late menopause, both of
which would increase ovulation, are risk factors for ovarian cancer. Hormone
replacement therapy (HRT) and obesity, rather than low body weight, are also risk
factors.

Pregnancy, which suppresses ovulation, is protective against ovarian cancer as is


combined oral contraceptive use.

Ovarian cancer has received a lot of attention in the media, and often patients read
about vague symptoms such as bloating being precursors to ovarian cancer. While
this is true to an extent, it is important to reassure patients and carry out a thorough
history and examination and identify and risk factors.

30. Postcoital bleeding

Postcoital bleeding describes vaginal bleeding after sexual intercourse.

Causes

• no identifiable pathology is found in around 50% of cases


• cervical ectropion is the most common identifiable causes, causing around
33% of cases. This is more common in women on the combined oral
contraceptive pill
• cervicitis e.g. secondary to Chlamydia
• cervical cancer
• polyps
• trauma

31. Intrauterine contraceptive devices


Intrauterine contraceptive devices comprise both conventional copper intrauterine
devices (IUDs) and levonorgestrel-releasing intrauterine systems (IUS, Mirena®). The
IUS is also used in the management of menorrhagia

Effectiveness

• both the IUD and IUS are more than 99% effective

Mode of action

• IUD: primary mode of action is prevention of fertilisation by causing decreased


sperm motility and survival (possibly an effect of copper ions)
• IUS: levonorgestrel prevents endometrial proliferation and causes cervical
mucous thickening

Counselling

• IUD
o can be relied upon immediately following insertion
o the majority of IUDs with copper on the stem only are effective for 5
years, whereas some of the IUDs that have copper on the stem and the
arms of the T may be effective for up to 10 years
• IUS
o can be relied upon after 7 days
o the most common IUS (i.e. Mirena® - levonorgestrel 20 mcg/24 hrs) is
effective for 5 years
o if used as endometrial protection for women taking oestrogen-only
hormone replacement therapy they are only licensed for 4 years

Potential problems

• IUDs make periods heavier, longer and more painful


• the IUS is associated with initial frequent uterine bleeding and spotting. Later
women typically have intermittent light menses with less dysmenorrhoea and
some women become amenorrhoeic
• uterine perforation: up to 2 per 1000 insertions and higher in breastfeeding
women
• the proportion of pregnancies that are ectopic is increased but the absolute
number of ectopic pregnancies is reduced, compared to a woman not using
contraception
• infection: there is a small increased risk of pelvic inflammatory disease in the
first 20 days after insertion but after this period the risk returns to that of a
standard population
• expulsion: risk is around 1 in 20, and is most likely to occur in the first 3
months

New IUS systems

The Jaydess® IUS is licensed for 3 years. It has a smaller frame, narrower inserter
tube and less levonorgestrel (LNG) than the Mirena® coil (13.5 mg compared to 52
mg). This results in lower serum levels of LNG.

The Kyleena® IUS has 19.5mg LNG and is also smaller than the Mirena® but is
licensed for 5 years. It also results in lower serum levels of LNG. The rate of
amenorrhoea is less with Kyleena® compared to Mirena®.

Migraine with aura is a complete contraindication to using the combined oral


contraceptive pill due to an increased risk of ischaemic stroke (UKMEC class 4). The
progestogen-based methods of contraception and the levonorgestrel-releasing
intrauterine device are classed as UKMEC 2 in patients who suffer from migraine with
aura, indicating that 'the advantages of using the method generally outweigh the
theoretical or proven risks'. The only form of contraception that is recommended by
the UKMEC as having no contraindication in this condition - i.e. UKMEC 1 - is the
copper intrauterine device

32. Chickenpox exposure in pregnancy

Chickenpox is caused by primary infection with varicella-zoster virus. Shingles is


caused by the reactivation of dormant virus in dorsal root ganglion. In pregnancy,
there is a risk to both the mother and also the fetus, a syndrome now termed fetal
varicella syndrome

Risks to the mother

• 5 times greater risk of pneumonitis

Fetal varicella syndrome (FVS)

• risk of FVS following maternal varicella exposure is around 1% if occurs before


20 weeks gestation
• studies have shown a very small number of cases occurring between 20-28
weeks gestation and none following 28 weeks
• features of FVS include skin scarring, eye defects (microphthalmia), limb
hypoplasia, microcephaly and learning disabilities
Other risks to the fetus

• shingles in infancy: 1-2% risk if maternal exposure in the second or third


trimester
• severe neonatal varicella: if the mother develops rash between 5 days before
and 2 days after birth there is a risk of neonatal varicella, which may be fatal to
the newborn child in around 20% of cases

Management of chickenpox exposure in pregnancy, i.e. post-exposure prophylaxis


(PEP)

• if there is any doubt about the mother previously having chickenpox maternal
blood should be urgently checked for varicella antibodies
• if the pregnant woman <= 20 weeks gestation is not immune to varicella she
should be given varicella-zoster immunoglobulin (VZIG) as soon as possible
o RCOG and Greenbook guidelines suggest VZIG is effective up to 10
days post exposure
• if the pregnant woman > 20 weeks gestation is not immune to varicella then
either VZIG or antivirals (aciclovir or valaciclovir) should be given days 7 to 14
after exposure
o the Public Health England (PHE) guidelines state that 'The decision on
choice of PEP for women exposed from 20 weeks of pregnancy should
take into account patient and health professional preference as well as
the ability to offer and provide PEP in a timely manner'
o why wait until days 7-14? From the PHE guidelines: 'In a study
evaluating the comparative effectiveness of 7 days course of aciclovir
given either immediately after exposure or starting at day 7 after
exposure to healthy children, the incidence and severity of varicella
infection was significantly higher in those given aciclovir immediately
(10/13 (77%) who received aciclovir immediately developed clinical
varicella compared with 3/14 (21%) who started aciclovir at day 7)'
o it seems part of this guidance is related to a limited supply of VZIG
within the NHS

Management of chickenpox in pregnancy

• if a pregnant woman develops chickenpox in pregnancy then specialist advice


should be sought
• there is an increased risk of serious chickenpox infection (i.e. maternal risk)
and fetal varicella risk (i.e. fetal risk) balanced against theoretical concerns
about the safety of aciclovir in pregnancy
• consensus guidelines (Health Protection Authority and RCOG) suggest oral
aciclovir should be given if the pregnant women is ≥ 20 weeks and she
presents within 24 hours of onset of the rash
• if the woman is < 20 weeks the aciclovir should be 'considered with caution'
Rudimentary digits, limb hypoplasia and microcephaly- These are the classical signs
of primary infection with varicella zoster in pregnancy. The greatest risk to the foetus
occurs before 20 weeks gestation and can result in scarring of the skin, limb
hypoplasia, microcephaly and eye defects. If there is any doubt regarding the
chickenpox history in a pregnant lady, they should be checked for varicella
antibodies and given varicella zoster immunoglobulins if they are not immune.

Antenatal cytomegalovirus infection can cause cerebral calcification, microcephaly


and sensorineural deafness. HIV causes no physical risk to the developing foetus but
there is a risk of perinatal transmission. Antenatal rubella infections are associated
with deafness, congenital cataracts and cardiac complications. Parvovirus B19 causes
hydrops fetalis and death.

33. Amenorrhoea

Amenorrhoea may be divided into primary (failure to start menses by the age of 16
years) or secondary (cessation of established, regular menstruation for 6 months or
longer).

Causes of primary amenorrhoea

• Turner's syndrome
• testicular feminisation
• congenital adrenal hyperplasia
• congenital malformations of the genital tract

Secondary amenorrhoea is defined as when menstruation has previously occurred


but has now stopped for at least 6 months.

Causes of secondary amenorrhoea (after excluding pregnancy)

• hypothalamic amenorrhoea (e.g. Stress, excessive exercise)


• polycystic ovarian syndrome (PCOS)
• hyperprolactinaemia
• premature ovarian failure
• thyrotoxicosis*
• Sheehan's syndrome
• Asherman's syndrome (intrauterine adhesions)

Initial investigations

• exclude pregnancy with urinary or serum bHCG


• gonadotrophins: low levels indicate a hypothalamic cause where as raised
levels suggest an ovarian problem (e.g. Premature ovarian failure)
• prolactin
• androgen levels: raised levels may be seen in PCOS
• oestradiol
• thyroid function tests
*hypothyroidism may also cause amenorrhoea

Low-level gonadotrophins indicate a hypothalamic cause.

Premature ovarian failure would have high gonadotrophins.

Polycystic ovarian syndrome would have high LH and androgens.

Her prolactin level is within the normal range.

Her thyroxine level is within the normal range


34. HIV and pregnancy

With the increased incidence of HIV infection amongst the heterosexual population
there are an increasing number of HIV positive women giving birth in the UK. In
London the incidence may be as high as 0.4% of pregnant women. The aim of
treating HIV positive women during pregnancy is to minimise harm to both the
mother and fetus, and to reduce the chance of vertical transmission.

Guidelines regularly change on this subject and most recent guidelines can be found
using the links provided.

Factors which reduce vertical transmission (from 25-30% to 2%)

• maternal antiretroviral therapy


• mode of delivery (caesarean section)
• neonatal antiretroviral therapy
• infant feeding (bottle feeding)

Screening
• NICE guidelines recommend offering HIV screening to all pregnant women

Antiretroviral therapy

• all pregnant women should be offered antiretroviral therapy regardless of


whether they were taking it previously

Mode of delivery

• vaginal delivery is recommended if viral load is less than 50 copies/ml at 36


weeks, otherwise caesarian section is recommended
• a zidovudine infusion should be started four hours before beginning the
caesarean section

Neonatal antiretroviral therapy

• zidovudine is usually administered orally to the neonate if maternal viral load


is <50 copies/ml. Otherwise triple ART should be used. Therapy should be
continued for 4-6 weeks.

Infant feeding

• in the UK all women should be advised not to breast feed


35. Premature ovarian failure

Premature ovarian failure is defined as the onset of menopausal symptoms and


elevated gonadotrophin levels before the age of 40 years. It occurs in around 1 in
100 women.

Causes of premature menopause include:

• idiopathic
o the most common cause
o there may be a family history
• bilateral oophorectomy
o having a hysterectomy with preservation of the ovaries has also been
shown to advance the age of menopause
• radiotherapy
• chemotherapy
• infection: e.g. mumps
• autoimmune disorders
• resistant ovary syndrome: due to FSH receptor abnormalities
Features are similar to those of the normal climacteric but the actual presenting
problem may differ

• climacteric symptoms: hot flushes, night sweats


• infertility
• secondary amenorrhoea
• raised FSH, LH levels
o e.g. FSH > 40 iu/l
• low oestradiol
o e.g. < 100 pmol/l
Oestrogen has been shown to be protective to both bone health and cardiovascular
disease. In patients with premature menopause, all-cause mortality is increased with
an associated increase in the risk of developing ischaemic heart disease and
osteoporosis. Hormone replacement therapy (HRT) is usually recommended until the
normal age of menopause. Beyond the normal age of menopause risks and benefits
of continuing HRT should be discussed.

A NICE review in 2015 on the use of HRT and cardiovascular outcomes for
menopausal women concluded the following -

• The baseline risk of coronary heart disease and stroke for women around
menopausal age varies from one woman to another according to the presence
of cardiovascular risk factors
• HRT with oestrogen alone is associated with no, or reduced, risk of coronary
heart disease
• HRT with oestrogen and progestogen is associated with little or no increase in
the risk of coronary heart disease.
• Taking oral (but not transdermal) oestrogen is associated with a small increase
in the risk of stroke.

36. Post-partum contraception

After giving birth women require contraception after day 21.

Progestogen only pill (POP)

• the FSRH advise 'postpartum women (breastfeeding and non-breastfeeding)


can start the POP at any time postpartum.'
• after day 21 additional contraception should be used for the first 2 days
• a small amount of progestogen enters breast milk but this is not harmful to
the infant
Combined oral contraceptive pill (COC)

• absolutely contraindicated - UKMEC 4 - if breast feeding < 6 weeks post-


partum
• UKMEC 2 - if breast feeding 6 weeks - 6 months postpartum*
• the COC may reduce breast milk production in lactating mothers
• may be started from day 21 - this will provide immediate contraception
• after day 21 additional contraception should be used for the first 7 days

The intrauterine device or intrauterine system can be inserted within 48 hours of


childbirth or after 4 weeks.

Lactational amenorrhoea method (LAM)

• is 98% effective providing the woman is fully breast-feeding (no


supplementary feeds), amenorrhoeic and < 6 months post-partum

An inter-pregnancy interval of less than 12 months between childbirth and


conceiving again is associated with an increased risk of preterm birth, low
birthweight and small for gestational age babies.

*this changed from UKMEC 3 in 2016


Emergency contraception (EC) is not required before day 21 postpartum. The earliest
date of ovulation in a non-breastfeeding woman is thought to be day 28 postpartum.
Therefore, contraception is required from day 21 onwards, as sperm can survive for
up to 7 days. A woman who is exclusively breastfeeding will take longer to ovulate,
however, contraception should still be advised if pregnancy is not desired.

After day 21 postpartum, progesterone only EC (Levonelle and ellaOne) can be used
in both breastfeeding and non-breastfeeding woman.

The Cu-IUD should not be inserted before day 28 postpartum, due to the increased
risk of uterine perforation if inserted before this time.

It would be important to discuss future contraception and the risk of sexually


transmitted infections with the patient in the above scenario.

37. Pelvic pain

In women the most common cause of pelvic pain is primary dysmenorrhoea. Some
women also experience transient pain in the middle of their cycle secondary to
ovulation (mittelschmerz). The table below gives characteristic features for other
conditions causing pelvic pain:

Usually acute

Condition Notes
Ectopic A typical history is a female with a history of 6-8 weeks amenorrhoea
pregnancy who presents with lower abdominal pain and later develops vaginal
bleeding
Shoulder tip pain and cervical excitation may be seen
Urinary tract Dysuria and frequency are common but women may experience
infection suprapubic burning secondary to cystitis
Appendicitis Pain initial in the central abdomen before localising to the right iliac
fossa
Anorexia is common
Tachycardia, low-grade pyrexia, tenderness in RIF
Rovsing's sign: more pain in RIF than LIF when palpating LIF
Pelvic Pelvic pain, fever, deep dyspareunia, vaginal discharge, dysuria and
inflammatory menstrual irregularities may occur
disease Cervical excitation may be found on examination
Ovarian Usually sudden onset unilateral lower abdominal pain. Onset may
torsion coincide with exercise.
Nausea and vomiting are common
Unilateral, tender adnexal mass on examination
Miscarriage Vaginal bleeding and crampy lower abdominal pain following a period
of amenorrhoea

Usually chronic

Condition Notes
Endometriosis Chronic pelvic pain
Dysmenorrhoea - pain often starts days before bleeding
Deep dyspareunia
Subfertility
Irritable bowel Extremely common. The most consistent features are abdominal pain,
syndrome bloating and change in bowel habit
Features such as lethargy, nausea, backache and bladder symptoms
may also be present
Ovarian cyst Unilateral dull ache which may be intermittent or only occur during
intercourse. Torsion or rupture may lead to severe abdominal pain
Condition Notes
Large cysts may cause abdominal swelling or pressure effects on the
bladder
Urogenital Seen in older women
prolapse Sensation of pressure, heaviness, 'bearing-down'
Urinary symptoms: incontinence, frequency, urgency

38. Ovarian cysts: types

Benign ovarian cysts are extremely common. They may be divided into physiological
cysts, benign germ cell tumours, benign epithelial tumours and benign sex cord
stromal tumours.

Complex (i.e. multi-loculated) ovarian cysts should be biopsied to exclude


malignancy.

Physiological cysts (functional cysts)

Follicular cysts

• commonest type of ovarian cyst


• due to non-rupture of the dominant follicle or failure of atresia in a non-
dominant follicle
• commonly regress after several menstrual cycles

Corpus luteum cyst

• during the menstrual cycle if pregnancy doesn't occur the corpus luteum
usually breaks down and disappears. If this doesn't occur the corpus luteum
may fill with blood or fluid and form a corpus luteal cyst
• more likely to present with intraperitoneal bleeding than follicular cysts

Benign germ cell tumours

Dermoid cyst

• also called mature cystic teratomas. Usually lined with epithelial tissue and
hence may contain skin appendages, hair and teeth
• most common benign ovarian tumour in woman under the age of 30 years
• median age of diagnosis is 30 years old
• bilateral in 10-20%
• usually asymptomatic. Torsion is more likely than with other ovarian tumours

Benign epithelial tumours

Arise from the ovarian surface epithelium

Serous cystadenoma

• the most common benign epithelial tumour which bears a resemblance to the
most common type of ovarian cancer (serous carcinoma)
• bilateral in around 20%

Mucinous cystadenoma

• second most common benign epithelial tumour


• they are typically large and may become massive
• if ruptures may cause pseudomyxoma peritonei

39. Menstrual cycle

The menstrual cycle may be divided into the following phases:

Days
Menstruation 1-4
Follicular phase (proliferative phase) 5-13
Ovulation 14
Luteal phase (secretory phase) 15-28

Further details are given in the table below

Follicular phase (proliferative


phase) Luteal phase (secretory phase)
Ovarian A number of follicles develop. Corpus luteum
histology
One follicle will become dominant
around the mid-follicular phase
Follicular phase (proliferative
phase) Luteal phase (secretory phase)
Endometrial Proliferation of endometrium Endometrium changes to secretory
histology lining under influence of
progesterone
Hormones A rise in FSH results in the Progesterone secreted by corpus
development of follicles which in turn luteum rises through the luteal
secrete oestradiol phase.

When the egg has matured, it If fertilisation does not occur the
secretes enough oestradiol to trigger corpus luteum will degenerate and
the acute release of LH. This in turn progesterone levels fall
leads to ovulation
Oestradiol levels also rise again
during the luteal phase
Cervical Following menstruation the mucus is Under the influence of
mucus thick and forms a plug across the progesterone it becomes thick,
external os scant, and tacky

Just prior to ovulation the mucus


becomes clear, acellular, low viscosity.
It also becomes 'stretchy' - a quality
termed spinnbarkeit
Basal body Falls prior to ovulation due to the Rises following ovulation in
temperature influence of oestradiol response to higher progesterone
levels

40. Miscarriage: management

In the 2019 NICE guidelines, 3 types of management for miscarriage were discussed:

Expectant management

• 'Waiting for a spontaneous miscarriage'


• First-line and involves waiting for 7-14 days for the miscarriage to complete
spontaneously
• If expectant management is unsuccessful then medical or surgical
management may be offered

Some situations are better managed with medically or surgically. NICE list the
following:

• increased risk of haemorrhage


o she is in the late first trimester
o if she has coagulopathies or is unable to have a blood transfusion
• previous adverse and/or traumatic experience associated with pregnancy (for
example, stillbirth, miscarriage or antepartum haemorrhage)
• evidence of infection

Medical management:

• 'Using tablets to expedite the miscarriage'


• Vaginal misoprostol
o Prostaglandin analogue, binds to myometrial cells to cause strong
myometrial contractions leading to the expulsion of tissue
• The addition of oral mifepristone is not currently recommended by NICE in
contrast to US guidelines
• Advise them to contact the doctor if the bleeding hasn't started in 24 hours.
• Should be given with antiemetics and pain relief

Surgical management

• 'Undergoing a surgical procedure under local or general anaesthetic'


• The two main options are vacuum aspiration (suction curettage) or surgical
management in theatre
• Vacuum aspiration is done under local anaesthetic as an outpatient
- Surgical management is done in theatre under general anaesthetic. This was
previously referred to as 'Evacuation of retained products of conception'
NICE guidelines recommend expectant management as first line in the treatment of
miscarriage, unless one of the following factors is present: there is an increased risk
of bleeding, there are previous adverse experiences associated with pregnancy, there
is increased risk from the effects of haemorrhage or there is evidence of infection.
The most appropriate option in the above case, which represents an infected
miscarriage with the patient progressing to septic shock, is to evacuate the
pregnancy as soon as possible through surgical management.

Methotrexate is used in ectopic pregnancies and mifepristone is not to be used in


the medical management of miscarriage.

41. Combined oral contraceptive pill: counselling

Women who are considering taking the combined oral contraceptive pill (COC)
should be counselled in a number of areas:
Potential harms and benefits, including

• the COC is > 99% effective if taken correctly


• small risk of blood clots
• very small risk of heart attacks and strokes
• increased risk of breast cancer and cervical cancer

Advice on taking the pill, including

• if the COC is started within the first 5 days of the cycle then there is no need
for additional contraception. If it is started at any other point in the cycle then
alternative contraception should be used (e.g. condoms) for the first 7 days
• should be taken at the same time every day
• the COCP is conventionally taken for 21 days then stopped for 7 days - similar
uterine bleeding to menstruation. However, there was a major change
following the 2019 guidelines. 'Tailored' regimes should now be discussed
with women. This is because there is no medical benefit from having a
withdrawal bleed. Options include never having a pill-free interval or
'tricycling' - taking three 21 day packs back-to-back before having a 4 or 7
day break
• advice that intercourse during the pill-free period is only safe if the next pack
is started on time

Discussion on situations where efficacy may be reduced*

• if vomiting within 2 hours of taking COC pill


• medication that induce diarrhoea or vomiting may reduce effectiveness of oral
contraception (for example orlistat)
• if taking liver enzyme-inducing drugs

Other information

• discussion on STIs

*Concurrent antibiotic use

• for many years doctors in the UK have advised that the concurrent use of
antibiotics may interfere with the enterohepatic circulation of oestrogen and
thus make the combined oral contraceptive pill ineffective - 'extra-
precautions' were advised for the duration of antibiotic treatment and for 7
days afterwards
• no such precautions are taken in the US or the majority of mainland Europe
• in 2011 the Faculty of Sexual & Reproductive Healthcare produced new
guidelines abandoning this approach. The latest edition of the BNF has been
updated in line with this guidance
• precautions should still be taken with enzyme inducing antibiotics such as
rifampicin

42. Menopause: management

Menopause is defined as the permanent cessation of menstruation. It is caused by


the loss of follicular activity. Menopause is a clinical diagnosis usually made in
primary care when a woman has not had a period for 12 months.

Menopausal symptoms are very common and affect roughly 75% of postmenopausal
women. Symptoms typically last for 7 years but may resolve quicker and in some
cases take much longer. The duration and severity are also variable and may develop
before the start of the menopause and in some cases may start years after the onset
of menopause.

The CKS has very thorough and clear guidance on the management of menopause
and is summarised below.

The management of menopause can be split into three categories:

• Lifestyle modifications
• Hormone replacement therapy (HRT)
• Non-hormone replacement therapy

Management with lifestyle modifications

Hot flushes: regular exercise, weight loss and reduce stress

Sleep disturbance: avoiding late evening exercise and maintaining good sleep
hygiene

Mood: sleep, regular exercise and relaxation

Cognitive symptoms: regular exercise and good sleep hygiene

Management with HRT

Contraindications:
• Current or past breast cancer
• Any oestrogen-sensitive cancer
• Undiagnosed vaginal bleeding
• Untreated endometrial hyperplasia

Roughly 10% of women will have some form of HRT to treat their menopausal
symptoms. There is a current drive by NICE to increase this number as they have
found that women were previously being undertreated due to worries about
increased cancer risk. If the woman has a uterus then it is important not to give
unopposed oestrogens as this will increase her risk of endometrial cancer. Therefore
oral or transdermal combined HRT is given.

If the woman does not have a uterus then oestrogen alone can be given either orally
or in a transdermal patch.

Women should be advised that the symptoms of menopause typically last for 2-5
years and that treatment with HRT brings certain risks:

• Venous thromboembolism: a slight increase in risk with all forms of oral HRT.
No increased risk with transdermal HRT.
• Stroke: slightly increased risk with oral oestrogen HRT.
• Coronary heart disease: combined HRT may be associated with a slight
increase in risk.
• Breast cancer: there is an increased risk with all combined HRT although the
risk of dying from breast cancer is not raised.
• Ovarian cancer: increased risk with all HRT.

Management with non-HRT

Vasomotor symptoms:
fluoxetine, citalopram or venlafaxine

Vaginal dryness:
vaginal lubricant or moisturiser

Psychological symptoms:
self-help groups, cognitive behaviour therapy or antidepressants

Urogenital symptoms

• if suffering from urogenital atrophy vaginal oestrogen can be prescribed. This


is appropriate if they are taking HRT or not
• vaginal dryness can be treated with moisturisers and lubricants. These can be
offered alongside vaginal oestrogens if required.

Stopping treatment

For vasomotor symptoms, 2-5 years of HRT may be required with regular attempts
made to discontinue treatment. Vaginal oestrogen may be required long term. When
stopping HRT it is important to tell women that gradually reducing HRT is effective at
limiting recurrence only in the short term. In the long term, there is no difference in
symptom control.

Although menopausal symptoms can be managed mainly in primary care, there are
some instances when a woman should be referred to secondary care. She should be
referred to secondary care if treatment has been ineffective, if there are ongoing side
effects or if there is unexplained bleeding.

43. Menorrhagia: causes

Menorrhagia was previously defined as total blood loss > 80 ml per menses, but it is
obviously difficult to quantify. The assessment and management of heavy periods
has therefore shifted towards what the woman considers to be excessive and aims to
improve quality of life measures.

Causes

• dysfunctional uterine bleeding: this describes menorrhagia in the absence of


underlying pathology. This accounts for approximately half of patients
• anovulatory cycles: these are more common at the extremes of a women's
reproductive life
• uterine fibroids
• hypothyroidism
• intrauterine devices*
• pelvic inflammatory disease
• bleeding disorders, e.g. von Willebrand disease

*this refers to normal copper coils. Note that the intrauterine system (Mirena) is used
to treat menorrhagia
*Testing for coagulation disorders (for example, von Willebrand's disease) should be
considered in women who have had heavy menstrual bleeding since menarche and
have personal or family history suggesting a coagulation disorder. NICE CG44
44. Group B Streptococcus

Group B Streptococcus (GBS) is the most common cause of early-onset severe


infection in the neonatal period. It is thought around 20-40% of mothers have GBS
present in their bowel flora and may therefore be thought of as 'carriers' of GBS.
Infants may be exposed to maternal GBS during labour and subsequently develop
potentially serious infections.

Risk factors for Group B Streptococcus (GBS) infection:

• prematurity
• prolonged rupture of the membranes
• previous sibling GBS infection
• maternal pyrexia e.g. secondary to chorioamnionitis

Management

The Royal College of Obstetricians and Gynaecologists (RCOG) published guidelines


on GBS in 2017.

The main points are as follows:

• universal screening for GBS should not be offered to all women


• the guidelines also state a maternal request is not an indication for screening
• women who've had GBS detected in a previous pregnancy should be informed
that their risk of maternal GBS carriage in this pregnancy is 50%. They should
be offered intrapartum antibiotic prophylaxis (IAP) OR testing in late
pregnancy and then antibiotics if still positive
• if women are to have swabs for GBS this should be offered at 35-37 weeks or
3-5 weeks prior to the anticipated delivery date
• IAP should be offered to women with a previous baby with early- or late-onset
GBS disease
• IAP should be offered to women in preterm labour regardless of their GBS
status
• women with a pyrexia during labour (>38ºC) should also be given IAP
• benzylpenicillin is the antibiotic of choice for GBS prophylaxis
45. Urinary incontinence

Urinary incontinence (UI) is a common problem, affecting around 4-5% of the


population. It is more common in elderly females.
Risk factors

• advancing age
• previous pregnancy and childbirth
• high body mass index
• hysterectomy
• family history

Classification

• overactive bladder (OAB)/urge incontinence: due to detrusor overactivity


• stress incontinence: leaking small amounts when coughing or laughing
• mixed incontinence: both urge and stress
• overflow incontinence: due to bladder outlet obstruction, e.g. due to prostate
enlargement

Initial investigation

• bladder diaries should be completed for a minimum of 3 days


• vaginal examination to exclude pelvic organ prolapse and ability to initiate
voluntary contraction of pelvic floor muscles ('Kegel' exercises)
• urine dipstick and culture
• urodynamic studies

Management depends on whether urge or stress UI is the predominant picture. If


urge incontinence is predominant:

• bladder retraining (lasts for a minimum of 6 weeks, the idea is to gradually


increase the intervals between voiding)
• bladder stabilising drugs: antimuscarinics are first-line. NICE recommend
oxybutynin (immediate release), tolterodine (immediate release) or darifenacin
(once daily preparation). Immediate release oxybutynin should, however,
be avoided in 'frail older women'
• mirabegron (a beta-3 agonist) may be useful if there is concern about
anticholinergic side-effects in frail elderly patients

If stress incontinence is predominant:

• pelvic floor muscle training: NICE recommend at least 8 contractions


performed 3 times per day for a minimum of 3 months
• surgical procedures: e.g. retropubic mid-urethral tape procedures
• duloxetine may be offered to women if they decline surgical procedures
o a combined noradrenaline and serotonin reuptake inhibitor
o mechanism of action: increased synaptic concentration of
noradrenaline and serotonin within the pudendal nerve → increased
stimulation of urethral striated muscles within the sphincter →
enhanced contraction
explanation: Other examples of muscarinic antagonists used in urinary incontinence
include oxybutynin and solifenacin. Examples of muscarinic antagonists used in
different conditions include ipratropium (chronic obstructive pulmonary disease) and
procyclidine (Parkinson's disease).
Tamsulosin is an alpha blocker.

46. Menorrhagia: causes

Menorrhagia was previously defined as total blood loss > 80 ml per menses, but it is
obviously difficult to quantify. The assessment and management of heavy periods
has therefore shifted towards what the woman considers to be excessive and aims to
improve quality of life measures.

Causes

• dysfunctional uterine bleeding: this describes menorrhagia in the absence of


underlying pathology. This accounts for approximately half of patients
• anovulatory cycles: these are more common at the extremes of a women's
reproductive life
• uterine fibroids
• hypothyroidism
• intrauterine devices*
• pelvic inflammatory disease
• bleeding disorders, e.g. von Willebrand disease

*this refers to normal copper coils. Note that the intrauterine system (Mirena) is used
to treat menorrhagia

47. Pregnancy: jaundice

Intrahepatic cholestasis of pregnancy

Intrahepatic cholestasis of pregnancy (also known as obstetric cholestasis) occurs in


around 1% of pregnancies and is generally seen in the third trimester. It is the most
common liver disease of pregnancy.

Features
• pruritus, often in the palms and soles
• no rash (although skin changes may be seen due to scratching)
• raised bilirubin

Management

• ursodeoxycholic acid is used for symptomatic relief


• weekly liver function tests
• women are typically induced at 37 weeks

Complications include an increased rate of stillbirth. It is not generally associated


with increased maternal morbidity

Acute fatty liver of pregnancy

Acute fatty liver of pregnancy is rare complication which may occur in the third
trimester or the period immediately following delivery.

Features

• abdominal pain
• nausea & vomiting
• headache
• jaundice
• hypoglycaemia
• severe disease may result in pre-eclampsia

Investigations

• ALT is typically elevated e.g. 500 u/l

Management

• support care
• once stabilised delivery is the definitive management

Gilbert's, Dubin-Johnson syndrome, may be exacerbated during pregnancy

HELLP

• Haemolysis, Elevated Liver enzymes, Low Platelets


48. Hypertension in pregnancy

NICE published guidance in 2010 on the management of hypertension in pregnancy.


They also made recommendations on reducing the risk of hypertensive disorders
developing in the first place. Women who are at high risk of developing pre-
eclampsia should take aspirin 75mg od from 12 weeks until the birth of the baby.
High risk groups include:

• hypertensive disease during previous pregnancies


• chronic kidney disease
• autoimmune disorders such as SLE or antiphospholipid syndrome
• type 1 or 2 diabetes mellitus

The classification of hypertension in pregnancy is complicated and varies. Remember,


in normal pregnancy:

• blood pressure usually falls in the first trimester (particularly the diastolic), and
continues to fall until 20-24 weeks
• after this time the blood pressure usually increases to pre-pregnancy levels by
term

Hypertension in pregnancy in usually defined as:

• systolic > 140 mmHg or diastolic > 90 mmHg


• or an increase above booking readings of > 30 mmHg systolic or > 15 mmHg
diastolic

After establishing that the patient is hypertensive they should be categorised into
one of the following groups

Pregnancy-induced
hypertension
Pre-existing (PIH, also known as gestational
hypertension hypertension) Pre-eclampsia
A history of hypertension Hypertension (as defined above) Pregnancy-induced
before pregnancy or an occurring in the second half of hypertension in
elevated blood pressure > pregnancy (i.e. after 20 weeks) association with
140/90 mmHg before 20 proteinuria (> 0.3g / 24
weeks gestation hours)
Pregnancy-induced
hypertension
Pre-existing (PIH, also known as gestational
hypertension hypertension) Pre-eclampsia
No proteinuria, no No proteinuria, no oedema
oedema
Oedema may occur but is
now less commonly used
Occurs in around 5-7% of as a criteria
Occurs in 3-5% of pregnancies
pregnancies and is more
common in older women Resolves following birth (typically Occurs in around 5% of
after one month). Women with pregnancies
PIH are at increased risk of future
pre-eclampsia or hypertension
later in life

49. Termination of pregnancy

The current law surround abortion is based on the 1967 Abortion Act. In 1990 the act
was amended, reducing the upper limit from 28 weeks gestation to 24 weeks*

Key points

• two registered medical practitioners must sign a legal document (in an


emergency only one is needed)
• only a registered medical practitioner can perform an abortion, which must be
in a NHS hospital or licensed premise

The method used to terminate pregnancy depend upon gestation

• less than 9 weeks: mifepristone (an anti-progestogen, often referred to as


RU486) followed 48 hours later by prostaglandins to stimulate uterine
contractions
• less than 13 weeks: surgical dilation and suction of uterine contents
• more than 15 weeks: surgical dilation and evacuation of uterine contents or
late medical abortion (induces 'mini-labour')

1967 Abortion Act


Subject to the provisions of this section, a person shall not be guilty of an offence under
the law relating to abortion when a pregnancy is terminated by a registered medical
practitioner if two registered medical practitioners are of the opinion, formed in good
faith

• that the pregnancy has not exceeded its 24th week and that the continuance of
the pregnancy would involve risk, greater than if the pregnancy were
terminated, of injury to the physical or mental health of the pregnant woman or
any existing children of her family; or
• that the termination is necessary to prevent grave permanent injury to the
physical or mental health of the pregnant woman; or
• that the continuance of the pregnancy would involve risk to the life of the
pregnant woman, greater than if the pregnancy were terminated; or
• that there is a substantial risk that if the child were born it would suffer from
such physical or mental abnormalities as to be seriously handicapped.

*these limits do not apply in cases where it is necessary to save the life of the
woman, there is evidence of extreme fetal abnormality, or there is risk of serious
physical or mental injury to the woman.

50. Dysmenorrhoea

Dysmenorrhoea is characterised by excessive pain during the menstrual period. It is


traditionally divided into primary and secondary dysmenorrhoea.

Primary dysmenorrhoea

In primary dysmenorrhoea there is no underlying pelvic pathology. It affects up to


50% of menstruating women and usually appears within 1-2 years of the menarche.
Excessive endometrial prostaglandin production is thought to be partially
responsible.

Features

• pain typically starts just before or within a few hours of the period starting
• suprapubic cramping pains which may radiate to the back or down the thigh
Management

• NSAIDs such as mefenamic acid and ibuprofen are effective in up to 80% of


women. They work by inhibiting prostaglandin production
• combined oral contraceptive pills are used second line

Secondary dysmenorrhoea

Secondary dysmenorrhoea typically develops many years after the menarche and is
the result of an underlying pathology. In contrast to primary dysmenorrhoea the pain
usually starts 3-4 days before the onset of the period. Causes include:

• endometriosis
• adenomyosis
• pelvic inflammatory disease
• intrauterine devices*
• fibroids

Clinical Knowledge Summaries recommend referring all patients with secondary


dysmenorrhoea to gynaecology for investigation.

*this refers to normal copper coils. Note that the intrauterine system (Mirena) may
help dysmenorrhoea

51. Heavy menstrual bleeding: management

Heavy menstrual bleeding (also known as menorrhagia) was previously defined as


total blood loss > 80 ml per menses, but it is obviously difficult to quantify. The
management has therefore shifted towards what the woman considers to be
excessive. Prior to the 1990's many women underwent a hysterectomy to treat heavy
periods but since that time the approach has altered radically. The management of
menorrhagia now depends on whether a woman needs contraception.

Investigations

• a full blood count should be performed in all women


• NICE recommend arranging a routine transvaginal ultrasound scan if
symptoms (for example, intermenstrual or postcoital bleeding, pelvic pain
and/or pressure symptoms) suggest a structural or histological abnormality.
Other indications include abnormal pelvic exam findings.
Does not require contraception

• either mefenamic acid 500 mg tds (particularly if there is dysmenorrhoea as


well) or tranexamic acid 1 g tds. Both are started on the first day of the period
• if no improvement then try other drug whilst awaiting referral

Requires contraception, options include

• intrauterine system (Mirena) should be considered first-line


• combined oral contraceptive pill
• long-acting progestogens

Norethisterone 5 mg tds can be used as a short-term option to rapidly stop heavy


menstrual bleeding.

52. Prematurity: risks

Risk of prematurity

• increased mortality depends on gestation


• respiratory distress syndrome
• intraventricular haemorrhage
• necrotizing enterocolitis
• chronic lung disease, hypothermia, feeding problems, infection, jaundice
• retinopathy of newborn, hearing problems
This woman is now in premature labour, although at 3cm dilated it is still in an early
stage. Therefore, it may be stopped by administering tocolytic medication. In case
the labour continues and delivery is required, steroids are given as a pre-emptively to
help the foetal lungs mature. Antibiotics are not required as there is no indication of
an infection. Syntocinon injection contains oxytocin which strengthens the
contractions of the uterus!

53. Pregnancy: diabetes mellitus

Diabetes mellitus may be a pre-existing problem or develop during pregnancy,


gestational diabetes. It complicates up to 1 in 20 pregnancies. NICE estimate the
following breakdown:

• 87.5% have gestational diabetes


• 7.5% have type 1 diabetes
• 5% have type 2 diabetes

Risk factors for gestational diabetes

• BMI of > 30 kg/m²


• previous macrosomic baby weighing 4.5 kg or above
• previous gestational diabetes
• first-degree relative with diabetes
• family origin with a high prevalence of diabetes (South Asian, black Caribbean
and Middle Eastern)

Screening for gestational diabetes

• women who've previously had gestational diabetes: oral glucose tolerance test
(OGTT) should be performed as soon as possible after booking and at 24-28
weeks if the first test is normal. NICE also recommend that early self-
monitoring of blood glucose is an alternative to the OGTTs
• women with any of the other risk factors should be offered an OGTT at 24-28
weeks

Diagnostic thresholds for gestational diabetes

• these have recently been updated by NICE, gestational diabetes is diagnosed


if either:
• fasting glucose is >= 5.6 mmol/l
• 2-hour glucose is >= 7.8 mmol/l
Management of gestational diabetes

• newly diagnosed women should be seen in a joint diabetes and antenatal


clinic within a week
• women should be taught about selfmonitoring of blood glucose
• advice about diet (including eating foods with a low glycaemic index) and
exercise should be given
• if the fasting plasma glucose level is < 7 mmol//l a trial of diet and exercise
should be offered
• if glucose targets are not met within 1-2 weeks of altering diet/exercise
metformin should be started
• if glucose targets are still not met insulin should be added to
diet/exercise/metformin
• if at the time of diagnosis the fasting glucose level is >= 7 mmol/l insulin
should be started
• if the plasma glucose level is between 6-6.9 mmol/l, and there is evidence of
complications such as macrosomia or hydramnios, insulin should be offered
• glibenclamide should only be offered for women who cannot tolerate
metformin or those who fail to meet the glucose targets with metformin but
decline insulin treatment

Management of pre-existing diabetes

• weight loss for women with BMI of > 27 kg/m^2


• stop oral hypoglycaemic agents, apart from metformin, and commence insulin
• folic acid 5 mg/day from pre-conception to 12 weeks gestation
• aspirin 75mg/day from 12 weeks until the birth of the baby, to reduce the risk
of pre-eclampsia
• detailed anomaly scan at 20 weeks including four-chamber view of the heart
and outflow tracts
• tight glycaemic control reduces complication rates
• treat retinopathy as can worsen during pregnancy

Targets for self-monitoring of pregnant women (pre-existing and gestational


diabetes)

Time Target
Fasting 5.3 mmol/l
1 hour after meals 7.8 mmol/l, or:
2 hour after meals 6.4 mmol/l
Patients with diabetes (type 1 and 2) should take aspirin 75mg daily from 12 weeks
gestation to reduce the risk of pre-eclampsia. They are also at higher risk of neural
tube defects, therefore should take the higher dose of folic acid, 5mg daily, whilst
trying to conceive until 12 weeks gestation. Pregnant women who have risk factors
such as this should be referred at booking to Consultant lead antenatal care.

All pregnant and breastfeeding women are advised to take vitamin D 10mcg daily.
A vitamin B12 supplement may be advised for pregnant women who eat a vegan
diet.
See the link to the NICE guidance for hypertension in pregnancy for details of other
risk factors for pre-eclampsia which may indicate aspirin therapy in pregnancy.

54. Endometrial cancer

Endometrial cancer is classically seen in post-menopausal women but around 25% of


cases occur before the menopause. It usually carries a good prognosis due to early
detection

The risk factors for endometrial cancer are as follows*:

• obesity
• nulliparity
• early menarche
• late menopause
• unopposed oestrogen. The addition of a progestogen to oestrogen reduces
this risk (e.g. In HRT). The BNF states that the additional risk is eliminated if a
progestogen is given continuously
• diabetes mellitus
• tamoxifen
• polycystic ovarian syndrome
• hereditary non-polyposis colorectal carcinoma

Features
• postmenopausal bleeding is the classic symptom
• premenopausal women may have a change intermenstrual bleeding
• pain and discharge are unusual features

Investigation
• women >= 55 years who present with postmenopausal bleeding should be
referred using the suspected cancer pathway
• first-line investigation is trans-vaginal ultrasound - a normal endometrial
thickness (< 4 mm) has a high negative predictive value
• hysteroscopy with endometrial biopsy

Management

• localised disease is treated with total abdominal hysterectomy with bilateral


salpingo-oophorectomy. Patients with high-risk disease may have post-
operative radiotherapy
• progestogen therapy is sometimes used in frail elderly women not consider
suitable for surgery

*the combined oral contraceptive pill and smoking are protective

55. Bleeding in pregnancy

The table below outlines the major causes of bleeding during pregnancy.
Antepartum haemorrhage is defined as bleeding after 24 weeks

1st trimester 2nd trimester 3rd trimester


Spontaneous abortion Spontaneous abortion Bloody show
Ectopic pregnancy Hydatidiform mole Placental abruption
Hydatidiform mole Placental abruption Placenta praevia
Vasa praevia

Alongside the pregnancy related causes, conditions such as sexually transmitted


infections and cervical polyps should be excluded.

The table below outlines the key features of each condition:

Threatened miscarriage - painless vaginal bleeding typically around


6-9 weeks
Missed (delayed) miscarriage - light vaginal bleeding and symptoms
of pregnancy disappear
Inevitable miscarriage - complete or incomplete depending or
whether all fetal and placental tissue has been expelled. Incomplete
Spontaneous miscarriage - heavy bleeding and crampy, lower abdo
abortion pain. Complete miscarriage - little bleeding
Ectopic Typically history of 6-8 weeks amenorrhoea with lower abdominal pain
pregnancy (usually unilateral) initially and vaginal bleeding later. Shoulder tip pain
and cervical excitation may be present
Hydatidiform Typically bleeding in first or early second trimester associated with
mole exaggerated symptoms of pregnancy e.g. hyperemesis. The uterus may
be large for dates and serum hCG is very high
Threatened miscarriage - painless vaginal bleeding typically around
6-9 weeks
Missed (delayed) miscarriage - light vaginal bleeding and symptoms
of pregnancy disappear
Inevitable miscarriage - complete or incomplete depending or
whether all fetal and placental tissue has been expelled. Incomplete
Spontaneous miscarriage - heavy bleeding and crampy, lower abdo
abortion pain. Complete miscarriage - little bleeding
Placental Constant lower abdominal pain and, woman may be more shocked than
abruption is expected by visible blood loss. Tender, tense uterus* with normal lie
and presentation. Fetal heart may be distressed
Placental Vaginal bleeding, no pain. Non-tender uterus* but lie and presentation
praevia may be abnormal
Vasa praevia Rupture of membranes followed immediately by vaginal bleeding. Fetal
bradycardia is classically seen

*vaginal examination should not be performed in primary care for suspected


antepartum haemorrhage - women with placenta praevia may haemorrhage

56. Progestogen only pill: missed pill

The missed pill rules for the progestogen only pill (POP) are simpler than those used
for the combined oral contraceptive pill, but it is important not to confuse the two.

'Traditional' POPs (Micronor,


Noriday, Nogeston, Femulen) Cerazette (desogestrel)
If less than 3 hours late If less than 12 hours late
no action required, continue as normal no action required, continue as normal

If more than 3 hours late (i.e. more If more than 12 hours late (i.e. more than
than 27 hours since the last pill was 36 hours since the last pill was taken)
taken) action needed - see below
action needed - see below

Action required, if needed:

• take the missed pill as soon as possible. If more than one pill has been missed
just take one pill. Take the next pill at the usual time, which may mean taking
two pills in one day
• continue with rest of pack
• extra precautions (e.g. condoms) should be used until pill taking has been re-
established for 48 hours
57. Breastfeeding: contraindications

The major breastfeeding contraindications tested in exams relate to drugs (see


below). Other contraindications of note include:

• galactosaemia
• viral infections - this is controversial with respect to HIV in the developing world.
This is because there is such an increased infant mortality and morbidity
associated with bottle feeding that some doctors think the benefits outweigh
the risk of HIV transmission

Drug contraindications

The following drugs can be given to mothers who are breastfeeding:

• antibiotics: penicillins, cephalosporins, trimethoprim


• endocrine: glucocorticoids (avoid high doses), levothyroxine*
• epilepsy: sodium valproate, carbamazepine
• asthma: salbutamol, theophyllines
• psychiatric drugs: tricyclic antidepressants, antipsychotics**
• hypertension: beta-blockers, hydralazine
• anticoagulants: warfarin, heparin
• digoxin

The following drugs should be avoided:

• antibiotics: ciprofloxacin, tetracycline, chloramphenicol, sulphonamides


• psychiatric drugs: lithium, benzodiazepines
• aspirin: Aspirin is best avoided in children due to the risk of Reye's syndrome
• carbimazole
• methotrexate
• sulfonylureas
• cytotoxic drugs
• amiodarone

*the BNF advises that the amount is too small to affect neonatal hypothyroidism
screening
**clozapine should be avoided

**Aspirin must be avoided in breastfeeding due to a possible risk of Reye's


syndrome; regular use of high doses could impair platelet function and produce
hypoprothrombinaemia in infants if neonatal vitamin K stores are low.

58. Epilepsy: contraception

There are a number of factors to consider for women with epilepsy:

• the effect of the contraceptive on the effectiveness of the anti-epileptic


medication
• the effect of the anti-epileptic on the effectiveness of the contraceptive
• the potential teratogenic effects of the anti-epileptic if the woman becomes
pregnant

Given the points above, the Faculty of Sexual & Reproductive Healthcare (FSRH)
recommend the consistent use of condoms, in addition to other forms of
contraception.
For women taking phenytoin,carbamazepine, barbiturates, primidone, topiramate,
oxcarbazepine:

• UKMEC 3: the COCP and POP


• UKMEC 2: implant
• UKMEC 1: Depo-Provera, IUD, IUS

For lamotrigine:
• UKMEC 3: the COCP
• UKMEC 1: POP, implant, Depo-Provera, IUD, IUS

If a COCP is chosen then it should contain a minimum of 30 µg of ethinylestradiol.

59.Uterine fibroids

Fibroids are benign smooth muscle tumours of the uterus. They are thought to occur
in around 20% of white and around 50% of black women in the later reproductive
years.

• more common in Afro-Caribbean women


• rare before puberty, develop in response to oestrogen

• may be asymptomatic
• menorrhagia: may result in iron-deficiency anaemia
• lower abdominal pain: cramping pains, often during menstruation
• bloating
• urinary symptoms, e.g. frequency, may occur with larger fibroids
• subfertility
• rare features: polycythaemia secondary to autonomous production of
erythropoietin

Diagnosis: transvaginal ultrasound

Management

Asymptomatic fibroids

• no treatment is needed other than periodic review to monitor size and growth

Management of menorrhagia secondary to fibroids

• levonorgestrel intrauterine system (LNG-IUS)


o useful if the woman also requires contraception
o cannot be used if there is distortion of the uterine cavity
• NSAIDs e.g. mefenamic acid
• tranexamic acid
• combined oral contraceptive pill
• oral progestogen
• injectable progestogen

Treatment to shrink/remove fibroids


• medical
o GnRH agonists may reduce the size of the fibroid but are typically useful
for short-term treatment
o ulipristal acetate has been in the past but not currently due to concerns
about rare but serious liver toxicity
• surgical
o myomectomy: this may be performed abdominally, laparoscopically or
hysteroscopically
o hysteroscopic endometrial ablation
o hysterectomy
• uterine artery embolization

Prognosis and complications: Fibroids generally regress after the menopause.

Some of the complications such as subfertility and iron-deficiency anaemia have


been mentioned previously.

Other complications: red degeneration - haemorrhage into tumour - commonly


occurs during pregnanc

60. Cervical cancer screening: interpretation of results

Around 50% of cases of cervical cancer occur in women under the age of 45 years,
with incidence rates for cervical cancer in the UK are highest in people aged 25-29
years, according to Cancer Research UK. It may be divided into:

• squamous cell cancer (80%)


• adenocarcinoma (20%)

Features

• may be detected during routine cervical cancer screening


• abnormal vaginal bleeding: postcoital, intermenstrual or postmenopausal
bleeding
• vaginal discharge

Human papillomavirus (HPV), particularly serotypes 16,18 & 33 is by far the most
important factor in the development of cervical cancer. Other risk factors include:

• smoking
• human immunodeficiency virus
• early first intercourse, many sexual partners
• high parity
• lower socioeconomic status
• combined oral contraceptive pill*

Mechanism of HPV causing cervical cancer

• HPV 16 & 18 produces the oncogenes E6 and E7 genes respectively


• E6 inhibits the p53 tumour suppressor gene
• E7 inhibits RB suppressor gene

*the strength of this association is sometimes debated but a large study published in
the Lancet (2007 Nov 10;370(9599):1609-21) confirmed the link

The cervical cancer screening programme has undergone a significant evolution in


recent years. For many years the smears were examined for signs of dyskaryosis
which may indicate cervical intraepithelial neoplasia - management was based solely
on the degree of dyskaryosis. The introduction of HPV testing allowed patients with
mild dyskaryosis to be further risk-stratified, i.e. as HPV is such a strong risk factor
patients who were HPV negative could be treated as having normal results.

The NHS has now moved to an HPV first system, i.e. a sample is tested for high-risk
strains of human papillomavirus (hrHPV) first and cytological examination is only
performed if this is positive.

Management of results

Negative hrHPV

• return to normal recall, unless


o the test of cure (TOC) pathway: individuals who have been treated for
CIN1, CIN2, or CIN3 should be invited 6 months after treatment for a test
of cure repeat cervical sample in the community
o the untreated CIN1 pathway
o follow-up for incompletely excised cervical glandular intraepithelial
neoplasia (CGIN) / stratified mucin producing intraepithelial lesion
(SMILE) or cervical cancer
o follow-up for borderline changes in endocervical cells

Positive hrHPV

• samples are examined cytologically


• if the cytology is abnormal → colposcopy
o this includes the following results:
o borderline changes in squamous or endocervical cells.
o low-grade dyskaryosis.
o high-grade dyskaryosis (moderate).
o high-grade dyskaryosis (severe).
o invasive squamous cell carcinoma.
o glandular neoplasia
• if the cytology is normal (i.e. hrHPV +ve but cytologically normal) the test is
repeated at 12 months
o if the repeat test is now hrHPV -ve → return to normal recall
o if the repeat test is still hrHPV +ve and cytology still normal → further
repeat test 12 months later:
o If hrHPV -ve at 24 months → return to normal recall
o if hrHPV +ve at 24 months → colposcopy

If the sample is 'inadequate'

• repeat the sample within 3 months


• if two consecutive inadequate samples then → colposcopy

The follow-up of patients who've previously had CIN is complicated but as a first
step, individuals who've been treated for CIN1, CIN2, or CIN3 should be invited 6
months after treatment for a test of cure repeat cervical sample in the community.

** Repeating the cervical smear in 5 years - this is routine recall for someone between 50-65
years of age. Even though this patient had a previous smear that was positive for high-risk
strains of human papillomavirus (hrHPV), the most recent smear was negative for hrHPV.
Therefore she should return to routine recall.

Cytological examination of the smear is incorrect as it is not indicated. The NHS has now
moved to an HPV first system so cytological examination is only performed if the smear is
positive for hrHPV.

Referral for colposcopy is incorrect as it is also not indicated. Colposcopy would be indicated
if either she had a hrHPV positive smear with cytological evidence of dyskaryosis, she has 3
successive annual smears that are hrHPV positive but with no cytological evidence of
dyskaryosis, or she has 2 smears that are inadequate 3 months apart.

Repeating the cervical smear in 3 years is incorrect because that is routine recall under the
cervical screening programme for people between 25-49 years of age. As this patient is 50
years old, this is not the appropriate timeframe for recall.

Repeating the cervical smear after 12 months is incorrect because it would be indicated if the
most recent smear was also hrHPV positive but with no cytological abnormalities.

61. Vulval carcinoma


Around 80% of vulval cancers are squamous cell carcinomas. Most cases occur in
women over the age of 65 years. Vulval cancer is relatively rare with only around
1,200 cases diagnosed in the UK each year.

Risk factors include:

• Age
• Human papilloma virus (HPV) infection
• Vulval intraepithelial neoplasia (VIN)
• Immunosuppression
• Lichen sclerosus

Features: lump or ulcer on the labia majora, inguinal lymphadenopathy, may be


associated with itching, irritation

** Vulval carcinomas are commonly ulcerated and can present on the labium majora.
Melanomas are usually pigmented. Vulval intraepithelial neoplasia tend to be white or
plaque like and don't tend to ulcerate. Herpes simplex tend to be smaller vesicles and
chancre tends to be painless and is seen in the first phase of syphilis.

62. Progestogen only pill: advantages/disadvantages

Advantages

• highly effective (failure rate = 1 per 100 woman years)


• doesn't interfere with sex
• contraceptive effects reversible upon stopping
• can be used whilst breast-feeding
• can be used in situations where the combined oral contraceptive pill is
contraindicated e.g. in smokers > 35 years of age and women with a history of
venous thromboembolic disease

Disadvantages

• irregular periods: some users may not have periods whilst others may have
irregular or light periods. This is the most common adverse effect
• doesn't protect against sexually transmitted infections
• increased incidence of functional ovarian cysts
• common side-effects include breast tenderness, weight gain, acne and
headaches. These symptoms generally subside after the first few months

63. Abdominal pain in pregnancy


Early pregnancy

Cause Notes
This is the single most important cause of abdominal pain to exclude
in early pregnancy
0.5% of all pregnancies are ectopic
Risk factors (anything slowing the ovum's passage to the uterus)

• damage to tubes (salpingitis, surgery)


• previous ectopic
• IVF (3% of pregnancies are ectopic)

A typical history is a female with a history of 6-8 weeks amenorrhoea


who presents with lower abdominal pain and later develops vaginal
Ectopic
bleeding
pregnancy
• lower abdominal pain: typically the first symptom. Pain is
usually constant and may be unilateral. Due to tubal spasm
• vaginal bleeding: usually less than a normal period, may be
dark brown in colour
• history of recent amenorrhoea: typically 6-8 weeks from start of
last period; if longer (e.g. 10 wks) this suggest another causes
e.g. inevitable abortion
• peritoneal bleeding can cause shoulder tip pain and pain on
defecation / urination

Threatened miscarriage

• painless vaginal bleeding occurring before 24 weeks, but


typically occurs at 6 - 9 weeks
• cervical os is closed
• complicates up to 25% of all pregnancies

Missed (delayed) miscarriage


Miscarriage
• a gestational sac which contains a dead fetus before 20 weeks
without the symptoms of expulsion
• mother may have light vaginal bleeding / discharge and the
symptoms of pregnancy which disappear
• when the gestational sac is > 25 mm and no embryonic/fetal
part can be seen it is sometimes described as a 'blighted ovum'
or 'anembryonic pregnancy'
Cause Notes

Inevitable miscarriage

• cervical os is open
• heavy bleeding with clots and pain

Incomplete miscarriage

• not all products of conception have been expelled

Late pregnancy

Cause Notes
Regular tightening of the abdomen which may be painful in
Labour
the later stages
Placental abruption describes separation of a normally sited
placenta from the uterine wall, resulting in maternal
haemorrhage into the intervening space

Occurs in approximately 1/200 pregnancies

Clinical features
Placental abruption
• shock out of keeping with visible loss
• pain constant
• tender, tense uterus
• normal lie and presentation
• fetal heart: absent/distressed
• coagulation problems
• beware pre-eclampsia, DIC, anuria

Ligament laxity increases in response to hormonal changes of


Symphysis pubis pregnancy
dysfunction Pain over the pubic symphysis with radiation to the groins and
the medial aspects of the thighs. A waddling gait may be seen
Associated with hypertension, proteinuria. Patients with HELLP
also have haemolysis, elevated liver enzymes and a low platelet
Pre-eclampsia/HELLP
count.
syndrome
The pain is typically epigastric or in the RUQ
Cause Notes
Ruptures usually occur during labour but occur in third
trimester
Uterine rupture Risk factors: previous caesarean section
Presents with maternal shock, abdominal pain and vaginal
bleeding to varying degree

Any point in pregnancy

Cause Notes
Occurs in 1:1,000-2:1,000 pregnancies, making it the most
common non-obstetric surgical emergency
Higher morbidity and mortality in pregnancy
Appendicitis
Location of pain changes depending on gestation, moving up
from the RLQ in the first trimester to the umbilicus in the second
and the RUQ in the third
Urinary tract 1 in 25 women develop in UTI in pregnancy
infection (UTI) Associated with an increased risk of pre-term delivery and IUGR

64. Implantable contraceptives

Implanon was the original non-biodegradable subdermal contraceptive implant


which has been replaced by Nexplanon. From a pharmacological perspective
Nexplanon is the same as Implanon. The two main differences are:

• the applicator has been redesigned to try and prevent 'deep' insertions (i.e.
subcutaneous/intramuscular)
• it is radiopaque and therefore easier to locate if impalpable

Both versions slowly releases the progestogen hormone etonogestrel. They are
typically inserted in the proximal non-dominant arm, just overlying the tricep. The
main mechanism of action is preventing ovulation. They also work by thickening the
cervical mucus.

Key points

• highly effective: failure rate 0.07/100 women-years - it is the most effective form
of contraception
• long-acting: lasts 3 years
• doesn't contain oestrogen so can be used if past history of thromboembolism,
migraine etc
• can be inserted immediately following a termination of pregnancy
Disadvantages include

• the need for a trained professional to insert and remove device


• additional contraceptive methods are needed for the first 7 days if not inserted
on day 1 to 5 of a woman's menstrual cycle

Adverse effects

• irregular/heavy bleeding is the main problem: this is sometimes managed


using a co-prescription of the combined oral contraceptive pill. It should be
remembered to do a speculum exam/STI check if the bleeding continues
• 'progestogen effects': headache, nausea, breast pain

Interactions

• enzyme-inducing drugs such as certain antiepileptic and rifampicin may reduce


the efficacy of Nexplanon
• the FSRH advises that women should be advised to switch to a method
unaffected by enzyme-inducing drugs or to use additional contraception until
28 days after stopping the treatment

Contraindications

• UKMEC 3*: ischaemic heart disease/stroke (for continuation, if initiation then


UKMEC 2), unexplained, suspicious vaginal bleeding, past breast cancer, severe
liver cirrhosis, liver cancer
• UKMEC 4**: current breast cancer

*proven risks generally outweigh the advantages


**a condition which represents an unacceptable risk if the contraceptive method is
used

65. Rhesus negative pregnancy

A basic understanding of the pathophysiology is essential to understand the


management of Rhesus negative pregnancies

• along with the ABO system the Rhesus system is the most important antigen
found on red blood cells. The D antigen is the most important antigen of the
rhesus system
• around 15% of mothers are rhesus negative (Rh -ve)
• if a Rh -ve mother delivers a Rh +ve child a leak of fetal red blood cells may
occur
• this causes anti-D IgG antibodies to form in mother
• in later pregnancies these can cross placenta and cause haemolysis in fetus
• this can also occur in the first pregnancy due to leaks

Prevention

• test for D antibodies in all Rh -ve mothers at booking


• NICE (2008) advise giving anti-D to non-sensitised Rh -ve mothers at 28 and 34
weeks
• the evidence base suggests that there is little difference in the efficacy of single-
dose (at 28 weeks) and double-dose regimes (at 28 & 34 weeks). For this reason
the RCOG in 2011 advised that either regime could be used 'depending on local
factors'
• anti-D is prophylaxis - once sensitization has occurred it is irreversible
• if event is in 2nd/3rd trimester give large dose of anti-D and perform Kleihauer
test - determines proportion of fetal RBCs present

Anti-D immunoglobulin should be given as soon as possible (but always within 72


hours) in the following situations:

• delivery of a Rh +ve infant, whether live or stillborn


• any termination of pregnancy
• miscarriage if gestation is > 12 weeks
• ectopic pregnancy (if managed surgically, if managed medically with
methotrexate anti-D is not required)
• external cephalic version
• antepartum haemorrhage
• amniocentesis, chorionic villus sampling, fetal blood sampling
• abdominal trauma

Tests

• all babies born to Rh -ve mother should have cord blood taken at delivery for
FBC, blood group & direct Coombs test
• Coombs test: direct antiglobulin, will demonstrate antibodies on RBCs of baby
• Kleihauer test: add acid to maternal blood, fetal cells are resistant

Affected fetus

• oedematous (hydrops fetalis, as liver devoted to RBC production albumin falls)


• jaundice, anaemia, hepatosplenomegaly
• heart failure
• kernicterus
• treatment: transfusions, UV phototherapy
66. Barrier methods of contraception

Barrier methods are still widely used as methods of contraception. Unlike other types
of contraception condoms also provide some protection against sexually transmitted
infections (STIs). The two most common types of barrier methods used in the UK are:

• condoms
• diaphragms and cervical caps

Efficacy (defined as percentage of women experiencing an unintended pregnancy


within the first
year of use)

• male condoms: perfect use - 98%, typical use - 80%


• female condoms: perfect use - 95%, typical use - 80%
• diaphragms and cervical caps: if used with spermicide then 92-96%

Usage

• oil based lubricants should not be used with latex condoms


• polyurethane condoms should be used in patients allergic to latex

** Polyurethane is a suitable alternative for those with latex allergies. Although lambskin (a
misnomer as they are actually made from sheep intestines) would protect against unwanted
pregnancy they will not protect against sexually transmitted infections and are therefore not
recommended. Latex is a rubber derivative so a rubber condom would not help. Regular
condoms contain latex.

67. Gestational trophoblastic disorders

Describes a spectrum of disorders originating from the placental trophoblast:

• complete hydatidiform mole


• partial hydatidiform mole
• choriocarcinoma

Complete hydatidiform mole

Benign tumour of trophoblastic material. Occurs when an empty egg is fertilized by a


single sperm that then duplicates its own DNA, hence the all 46 chromosomes are of
paternal origin

• bleeding in first or early second trimester


• exaggerated symptoms of pregnancy e.g. hyperemesis
• uterus large for dates
• very high serum levels of human chorionic gonadotropin (hCG)
• hypertension and hyperthyroidism* may be seen

Management

• urgent referral to specialist centre - evacuation of the uterus is performed


• effective contraception is recommended to avoid pregnancy in the next 12
months

Around 2-3% go on to develop choriocarcinoma


In a partial mole a normal haploid egg may be fertilized by two sperms, or by one
sperm with duplication of the paternal chromosomes. Therefore the DNA is both
maternal and paternal in origin. Usually triploid - e.g. 69 XXX or 69 XXY. Fetal parts
may be seen
*hCG can mimic thyroid-stimulating hormone (TSH)

**Although placental abruption, placenta praevia and ectopic pregnancy can cause
vaginal bleeding they do not present with a non-tender, large-for-dates uterus.
Gestational diabetes is not associated with vaginal bleeding or hyperemesis.

Molar pregnancy is a form of gestational trophoblastic disease. In a normal


pregnancy, half of the chromosomes come from the father and half come from the
mother. Molar pregnancies may be partial or complete. A complete hydatidiform
mole has a 46 XX or 46 XY karyotype with all of the genetic material deriving from
the father. This is most commonly due to abnormal fertilisation of an empty ovum. A
partial hydatidiform mole has a karyotype of 69 XXX or 69 XXY and contains both
maternal and paternal chromosomes. Neither pregnancy can produce a viable fetus.

The most common presenting symptom of a molar pregnancy is vaginal bleeding,


which may be light or heavy. Uterine size may be large-for-dates in around one
quarter of complete molar pregnancies. Complete hydatidiform moles contain large
amounts of abnormal chorionic villi, which produce high volumes of beta hCG. This
can lead to sequelae such as hyperemesis, hyperthyroidism and other symptoms.
Women aged under 20 years and over 35 years are at higher risk of having a molar
pregnancy.

68. Absolute contra-indications for the combined oral contraceptive pill (category 4)
include:

• Migraine with aura


• Breastfeeding <6 weeks post-partum
• Age 35 or over smoking 15 or more cigarettes/day
• Systolic 160mmHg or diastolic 95mmHg
• Vascular disease
• History of VTE
• Current VTE (on anticoagulants)
• Major surgery with prolonged immobilisation
• Known thrombogenic mutations
• Current and history of ischaemic heart disease
• Stroke (including TIA)
• Complicated valvular and congenital heart disease
• Current breast cancer
• Nephropathy/retinopathy/neuropathy
• Other vascular disease
• Severe (decompensated) cirrhosis
• Hepatocellular adenoma
• Hepatoma
• Raynaud's disease with lupus anticoagulant
• Positive antiphospholipid antibodies

69. Chickenpox exposure in pregnancy

Chickenpox is caused by primary infection with varicella-zoster virus. Shingles is


caused by the reactivation of dormant virus in dorsal root ganglion. In pregnancy,
there is a risk to both the mother and also the fetus, a syndrome now termed fetal
varicella syndrome

Risks to the mother

• 5 times greater risk of pneumonitis

Fetal varicella syndrome (FVS)

• risk of FVS following maternal varicella exposure is around 1% if occurs before


20 weeks gestation
• studies have shown a very small number of cases occurring between 20-28
weeks gestation and none following 28 weeks
• features of FVS include skin scarring, eye defects (microphthalmia), limb
hypoplasia, microcephaly and learning disabilities

Other risks to the fetus

• shingles in infancy: 1-2% risk if maternal exposure in the second or third


trimester
• severe neonatal varicella: if the mother develops rash between 5 days before
and 2 days after birth there is a risk of neonatal varicella, which may be fatal to
the newborn child in around 20% of cases

Management of chickenpox exposure in pregnancy, i.e. post-exposure prophylaxis


(PEP)

• if there is any doubt about the mother previously having chickenpox maternal
blood should be urgently checked for varicella antibodies
• if the pregnant woman <= 20 weeks gestation is not immune to varicella she
should be given varicella-zoster immunoglobulin (VZIG) as soon as possible
o RCOG and Greenbook guidelines suggest VZIG is effective up to 10 days
post exposure
• if the pregnant woman > 20 weeks gestation is not immune to varicella then
either VZIG or antivirals (aciclovir or valaciclovir) should be given days 7 to 14
after exposure
o the Public Health England (PHE) guidelines state that 'The decision on
choice of PEP for women exposed from 20 weeks of pregnancy should take
into account patient and health professional preference as well as the
ability to offer and provide PEP in a timely manner'
o why wait until days 7-14? From the PHE guidelines: 'In a study evaluating
the comparative effectiveness of 7 days course of aciclovir given either
immediately after exposure or starting at day 7 after exposure to healthy
children, the incidence and severity of varicella infection was significantly
higher in those given aciclovir immediately (10/13 (77%) who received
aciclovir immediately developed clinical varicella compared with 3/14
(21%) who started aciclovir at day 7)'
o it seems part of this guidance is related to a limited supply of VZIG within
the NHS

Management of chickenpox in pregnancy

• if a pregnant woman develops chickenpox in pregnancy then specialist advice


should be sought
• there is an increased risk of serious chickenpox infection (i.e. maternal risk) and
fetal varicella risk (i.e. fetal risk) balanced against theoretical concerns about the
safety of aciclovir in pregnancy
• consensus guidelines (Health Protection Authority and RCOG) suggest oral
aciclovir should be given if the pregnant women is ≥ 20 weeks and she presents
within 24 hours of onset of the rash
• if the woman is < 20 weeks the aciclovir should be 'considered with caution'
The development of chickenpox in a pregnant woman is associated with increased
morbidity, including pneumonia, hepatitis and encephalitis. Therefore, pregnant
women who develop a chickenpox rash should see their GP immediately.

Oral aciclovir should be prescribed in pregnant women with chickenpox, if they


present within 24 hours of the onset of the rash and are 20 weeks gestation or
beyond. (Use before 20 weeks can be considered).
Aciclovir reduces the duration of fever and symptomatology of varicella, if
commenced within 24 hours of developing the rash. Symptomatic treatment can be
used alongside aciclovir.
Pregnant women with severe chickenpox should be referred to hospital for
intravenous aciclovir. In addition, a referral to fetal medicine would be necessary (in
the above scenario), due to the small risk of fetal varicella syndrome in the first 28
weeks of pregnancy.
It is important to advise anyone with chickenpox to avoid contact with other
potentially susceptible people, such a pregnant women and neonates until the last
lesions have crusted over (usually 5 days after the onset of the rash).
VZIG has no therapeutic benefit once the rash has started. RCOG Greentop
guidelines, Chickepox in pregnancy.

70.Pelvic inflammatory disease

Pelvic inflammatory disease (PID) is a term used to describe infection and


inflammation of the female pelvic organs including the uterus, fallopian tubes,
ovaries and the surrounding peritoneum. It is usually the result of ascending
infection from the endocervix.

Causative organisms: Chlamydia trachomatis+ the most common cause

• Neisseria gonorrhoeae
• Mycoplasma genitalium
• Mycoplasma hominis

Features

• lower abdominal pain


• fever
• deep dyspareunia
• dysuria and menstrual irregularities may occur
• vaginal or cervical discharge
• cervical excitation

Investigation
• a pregnancy test should be done to exclude an ectopic pregnancy
• high vaginal swab
o these are often negative
• screen for Chlamydia and Gonorrhoea

Management

• due to the difficulty in making an accurate diagnosis, and the potential


complications of untreated PID, consensus guidelines recommend having a low
threshold for treatment
• oral ofloxacin + oral metronidazole or intramuscular ceftriaxone + oral
doxycycline + oral metronidazole
• RCOG guidelines suggest that in mild cases of PID intrauterine contraceptive
devices may be left in. The more recent BASHH guidelines suggest that the
evidence is limited but that ' Removal of the IUD should be considered and may
be associated with better short term clinical outcomes'

Complications

• perihepatitis (Fitz-Hugh Curtis Syndrome)


o occurs in around 10% of cases
o it is characterised by right upper quadrant pain and may be confused
with cholecystitis
• infertility - the risk may be as high as 10-20% after a single episode
• chronic pelvic pain
• ectopic pregnancy

NICE guidelines state that the diagnosis of pelvic inflammatory disease should be
made on clinical grounds and that clinicians should have a low threshold for
initiating treatment in the form of antibiotics. Although investigations should be
performed - including taking endocervical and high vaginal swabs for microscopy
and culture - these should not delay treatment. Negative swab results do not rule out
the diagnosis. Blood cultures are unlikely to be indicated unless the patient appears
systemically unwell. Transvaginal ultrasound is not first-line but may be indicated if
an abscess is suspected.

71. Amenorrhoea

Amenorrhoea may be divided into:

• primary: defined as the failure to establish menstruation by 15 years of age in


girls with normal secondary sexual characteristics (such as breast development),
or by 13 years of age in girls with no secondary sexual characteristics
• secondary: cessation of menstruation for 3-6 months in women with previously
normal and regular menses, or 6-12 months in women with previous
oligomenorrhoea

Causes

Secondary amenorrhoea (after


Primary amenorrhoea
excluding pregnancy)
• gonadal dysgenesis (e.g. Turner's
syndrome) - the most common
• hypothalamic amenorrhoea (e.g.
causes
secondary stress, excessive exercise)
• testicular feminisation
• polycystic ovarian syndrome (PCOS)
• congenital malformations of the
• hyperprolactinaemia
genital tract
• premature ovarian failure
• functional hypothalamic
• thyrotoxicosis*
amenorrhoea (e.g. secondary to
• Sheehan's syndrome
anorexia)
• Asherman's syndrome (intrauterine
• congenital adrenal hyperplasia
adhesions)
• imperforate hymen

Investigation and management

Initial investigations

• exclude pregnancy with urinary or serum bHCG


• full blood count, urea & electrolytes, coeliac screen, thyroid function tests
• gonadotrophins
o low levels indicate a hypothalamic cause where as raised levels suggest
an ovarian problem (e.g. Premature ovarian failure)
o raised if gonadal dysgenesis (e.g. Turner's syndrome)
• prolactin
• androgen levels
o raised levels may be seen in PCOS
• oestradiol

Management

• primary amenorrhoea:
o investigate and treat any underlying cause
o with primary ovarian insufficiency due to gonadal dysgenesis (e.g.
Turner's syndrome) are likely to benefit from hormone replacement
therapy (e.g. to prevent osteoporosis etC)
• secondary amenorrhoea
o exclude pregnancy, lactation, and menopause (in women 40 years of age
or older)
o treat the underlying cause

*hypothyroidism may also cause amenorrhoea

72. Postpartum haemorrhage

Postpartum haemorrhage (PPH) is defined as blood loss of > 500mls and may be
primary or secondary

Primary PPH

• occurs within 24 hours


• affects around 5-7% of deliveries
• most common cause of PPH is uterine atony (90% of cases). Other causes
include genital trauma and clotting factors

Risk factors for primary PPH include*:

• previous PPH
• prolonged labour
• pre-eclampsia
• increased maternal age
• polyhydramnios
• emergency Caesarean section
• placenta praevia, placenta accreta
• macrosomia
• ritodrine (a beta-2 adrenergic receptor agonist used for tocolysis)

Management

• ABC including two peripheral cannulae, 14 gauge


• IV syntocinon (oxytocin) 10 units or IV ergometrine 500 micrograms
• IM carboprost
• if medical options failure to control the bleeding then surgical options will need
to be urgently considered
• the RCOG state that the intrauterine balloon tamponade is an appropriate first-
line ‘surgical’ intervention for most women where uterine atony is the only or
main cause of haemorrhage
• other options include: B-Lynch suture, ligation of the uterine arteries or internal
iliac arteries
• if severe, uncontrolled haemorrhage then a hysterectomy is sometimes
performed as a life-saving procedure

Secondary PPH

• occurs between 24 hours - 12 weeks**


• due to retained placental tissue or endometritis

*the effect of parity on the risk of PPH is complicated. It was previously though
multiparity was a risk factor but more modern studies suggest nulliparity is actually a
risk factor

**previously the definition of secondary PPH was 24 hours - 6 weeks. Please see the
RCOG guidelines for more details

**Uterine atony is the most common cause of primary postpartum haemorrhage. It


entails failure of the uterus to contract fully following the delivery of the placenta,
which hinders the achievement of haemostasis. Uterine atony is associated with
overdistension, which may be due to multiple gestation, macrosomia,
polyhydramnios or other causes.

In addition to the usual steps taken in an episode of PPH (including an ABC approach
if the patient is unstable), the following management should be initiated in sequence:

•bimanual uterine compression to manually stimulate contraction

•intravenous oxytocin and/or ergometrine

•intramuscular carboprost

•intramyometrial carboprost

•rectal misoprostol

•surgical intervention such as balloon tamponade

73. Pregnancy: risks of smoking, alcohol and illegal drugs

The table below summarises some of the risks associated with drug use during
pregnancy:
Drug Risks
Increased risk of miscarriage (increased risk of around 47%)
Increased risk of pre-term labour
Smoking Increased risk of stillbirth
IUGR
Increased risk of sudden unexpected death in infancy
Fetal alcohol syndrome (FAS)

• learning difficulties
• characteristic facies: smooth philtrum, thin vermilion, small
Alcohol
palpebral fissures, epicanthic folds, microcephaly
• IUGR & postnatal restricted growth

Binge drinking is a major risk factor for FAS


Cannabis Similar to smoking risks due to tobacco content
Maternal risks

• hypertension in pregnancy including pre-eclampsia


• placental abruption

Cocaine
Fetal risk

• prematurity
• neonatal abstinence syndrome

Heroin Risk of neonatal abstinence syndrome

74. Pregnancy: diabetes - complications

Maternal complications

• polyhydramnios - 25%, possibly due to fetal polyuria


• preterm labour - 15%, associated with polyhydramnios

Neonatal complications

• macrosomia (although diabetes may also cause small for gestational age babies)
• hypoglycaemia (secondary to beta cell hyperplasia)
• respiratory distress syndrome: surfactant production is delayed
• polycythaemia: therefore more neonatal jaundice
• malformation rates increase 3-4 fold e.g. sacral agenesis, CNS and CVS
malformations (hypertrophic cardiomyopathy)
• stillbirth
• hypomagnesaemia
• hypocalcaemia
• shoulder dystocia (may cause Erb's palsy)

75. Vaginal discharge

Vaginal discharge is a common presenting symptom and is not always pathological

Common causes

• physiological
• Candida
• Trichomonas vaginalis
• bacterial vaginosis

Less common causes

• Gonorrhoea
• Chlamydia can cause a vaginal discharge although this is rarely the presenting
symptoms
• ectropion
• foreign body
• cervical cancer

Key features of the common causes are listed below

Condition Key features


'Cottage cheese' discharge
Candida Vulvitis
Itch
Offensive, yellow/green, frothy discharge
Trichomonas vaginalis Vulvovaginitis
Strawberry cervix
Bacterial vaginosis Offensive, thin, white/grey, 'fishy' discharge

76. Post-term pregnancy


The World Health Organization defines a post-term pregnancy as one that has
extended to or beyond 42 weeks.

Potential complications/consequences:

Neonatal Maternal
Reduced placental Increased rates of intervention including forceps and
perfusion caesarean section
Oligohydramnios Increased rates of labour induction

The pregnancy is now post term. A woman who has reached 42 weeks gestation can
be offered induction of labour, or alternatively she can choose expectant
management. At this gestation the risks to the foetus are increased, and women with
either pregnancy-induced hypertension or pre-eclampsia are usually delivered.
Medical induction of labour would be the preferred choice. Caesarean section would
usually only be indicated if there was foetal compromise. This level of blood pressure
does not require treatment.

77. Post-partum mental health problems

Post-partum mental health problems range from the 'baby-blues' to puerperal


psychosis.

The Edinburgh Postnatal Depression Scale may be used to screen for depression:

• 10-item questionnaire, with a maximum score of 30


• indicates how the mother has felt over the previous week
• score > 13 indicates a 'depressive illness of varying severity'
• sensitivity and specificity > 90%
• includes a question about self-harm

'Baby-blues' Postnatal depression Puerperal psychosis


Seen in around 60- Affects approximately 0.2% of
70% of women Affects around 10% of women women

Typically seen 3-7 Most cases start within a month and Onset usually within the first
days following birth typically peaks at 3 months 2-3 weeks following birth
and is more
common in primips Features are similar to depression Features include severe
seen in other circumstances swings in mood (similar to
Mothers are bipolar disorder) and
'Baby-blues' Postnatal depression Puerperal psychosis
characteristically disordered perception (e.g.
anxious, tearful and auditory hallucinations)
irritable
As with the baby blues reassurance
and support are important
Admission to hospital is
Cognitive behavioural therapy may be usually required
Reassurance and
beneficial. Certain SSRIs such as
support, the health
sertraline and paroxetine* may be There is around a 25-50% risk
visitor has a key role
used if symptoms are severe** - whilst of recurrence following future
they are secreted in breast milk it is pregnancies
not thought to be harmful to the
infant

*paroxetine is recommended by SIGN because of the low milk/plasma ratio


**fluoxetine is best avoided due to a long half-life

**The mother may be suffering from puerperal psychosis and needs urgent
admission to allow psychiatric evaluation.

Whilst there is not a full complement of psychotic features there are a number of
pointers towards significant mental health problems:

•poor interaction with the baby: this is very unusual, including in women with
postnatal depression

•'talking in an incoherent fashion about the future'

•stating that the baby 'has been brought into a very bad world' is odd and somewhat
worrying

78. Female sterilisation

Female sterilisation

• failure rate: 1 per 200*


• usually done by laparoscopy under general anaesthetic
• generally done as a day case
• many different techniques involving clips (e.g. Filshie clips) , blockage, rings
(Falope rings) and salpingectomy
• complications: increased risk of ectopic if sterilisation fails, general complications
of GA/laparoscopy
• the current success rate of female sterilisation reversal is between 50-60%

79. Contraception: mode of action

The table below is based on documents produced by the Faculty for Sexual and
Reproductive Health (FSRH).
Standard contraceptives:

Contraceptive Mode of action


Combined oral contraceptive pill Inhibits ovulation
Progestogen-only pill (excluding desogestrel) Thickens cervical mucus
Primary: Inhibits ovulation
Desogestrel-only pill
Also: thickens cervical mucus
Injectable contraceptive (medroxyprogesterone Primary: Inhibits ovulation
acetate) Also: thickens cervical mucus
Primary: Inhibits ovulation
Implantable contraceptive (etonogestrel)
Also: thickens cervical mucus
Decreases sperm motility and
Intrauterine contraceptive device
survival
Primary: Prevents endometrial
Intrauterine system (levonorgestrel) proliferation
Also: Thickens cervical mucus

Methods of emergency contraception:

Contraceptive Mode of action


Levonorgestrel Inhibits ovulation
Ulipristal Inhibits ovulation
Primary: Toxic to sperm and ovum
Intrauterine contraceptive device
Also: Inhibits implantation

80.Cardiotocography

Cardiotocography (CTG) records pressure changes in the uterus using internal or


external pressure transducers
The normal fetal heart rate varies between 100-160 / min

Feature Description Causes


Baseline Increased fetal vagal tone,
Heart rate < 100 /min
bradycardia maternal beta-blocker use
Maternal pyrexia,
Baseline
Heart rate > 160 /min chorioamnionitis, hypoxia,
tachycardia
prematurity
Loss of baseline
< 5 beats / min Prematurity, hypoxia
variability
Deceleration of the heart rate which
Usually an innocuous
Early commences with the onset of a
feature and indicates
deceleration contraction and returns to normal on
head compression
completion of the contraction
Deceleration of the heart rate which lags
Indicates fetal distress e.g.
Late the onset of a contraction and does not
asphyxia or placental
deceleration returns to normal until after 30 seconds
insufficiency
following the end of the contraction
Variable May indicate cord
Independent of contractions
decelerations compression

81. Reduced fetal movements

Introduction

Reduced fetal movements can represent fetal distress, as a method of fetal


compensation to reduce oxygen consumption as a response to chronic hypoxia in
utero. This is concerning, as it reflects risk of stillbirth and fetal growth restriction. It is
believed that there may also be a link between reduced fetal movements and
placental insufficiency.

Physiology

The first onset of recognised fetal movements is known as quickening. This usually
occurs between 18-20 weeks gestation, and increase until 32 weeks gestation at
which point the frequency of movement tends to plateau. Multiparous women will
usually experience fetal movements sooner, from 16-18 weeks gestation. Towards
the end of pregnancy, fetal movements should not reduce.

Expectant mothers will usually quickly recognise a pattern to these movements. The
nature of the movements themselves can be very variable. There is no established
definition for what constitutes reduced fetal movements (RFM), but the RCOG
considers less than 10 movements within 2 hours (in pregnancies past 28 weeks
gestation) an indication for further assessment.

Epidemiology

Fetal movements should be established by 24 weeks gestation.

Reduced fetal movements is a fairly common presentation, affecting up to 15% of


pregnancies. 3-5% of pregnant women will have recurrent presentations with RFM.

Risk factors for reduced fetal movements

• Posture
o There can be positional changes in fetal movement awareness, generally
being more prominent during lying down and less when sitting and
standing
• Distraction
o Awareness of fetal movements can be distractable, and if a woman is
busy or concentrating on something else, these can be less prominent
• Placental position
o Patient with anterior placentas prior to 28 weeks gestation may have
lesser awareness of fetal movements
• Medication
o Both alcohol and sedative medications like opiates or benzodiazepines
can temporarily cause reduced fetal movements
• Fetal position
o Anterior fetal position means movements are less noticeable
• Body habitus
o Obese patients are less likely to feel prominent fetal movements
• Amniotic fluid volume
o Both oligohydramnios and polyhydramnios can cause reduction in fetal
movements
• Fetal size
o Up to 29% of women presenting with RFM have a SGA fetus

Investigations

Fetal movements are usually based solely on maternal perception, though it can also
be objectively assessed using handheld Doppler or ultrasonography.

As per RCOG Green-top guidelines, investigations are dependent of gestation at


onset of RFM.
• If past 28 weeks gestation:
o Initially, handheld Doppler should be used to confirm fetal heartbeat.
o If no fetal heartbeat detectable, immediate ultrasound should be offered.
o If fetal heartbeat present, CTG should be used for at least 20 minutes to
monitor fetal heart rate which can assist in excluding fetal compromise.
o If concern remains, despite normal CTG, urgent (within 24 hours)
ultrasound can be used. Ultrasound assessment should include
abdominal circumference or estimated fetal weight (to exclude SGA), and
amniotic fluid volume measurement
• If between 24 and 28 weeks gestation, a handheld Doppler should be used to
confirm presence of fetal heartbeat.
• If below 24 weeks gestation, and fetal movements have previously been felt, a
handheld Doppler should be used.
• If fetal movements have not yet been felt by 24 weeks, onward referral should
be made to a maternal fetal medicine unit.

If RFM are recurrent, further investigations are also required to consider structural or
genetic fetal abnormalities.

Prognosis

Concern regarding absent or reduced fetal movements stems for the potential for
this presentation to represent fetal distress or impending demise. Between 40-55% of
women who suffer from stillbirth experience reduced fetal movements prior to
diagnosis.

However, in 70% of pregnancies with a single episode of reduced fetal movement,


there is no onward complication.

82. Contraceptive Choices for Young People

The Faculty of Sexual and Reproductive Health (FRSH) produced guidelines in 2010
concerning the provision of contraception to young people. Much of the following is
based on those guidelines. Please see the link for more details.

Legal and ethical issues

• the age of consent for sexual activity in the UK is 16 years. Practitioners may
however provide advice and contraception if they feel that the young person is
'competent'. This is usually assessed using the Fraser guidelines (see below)
• children under the age of 13 years are considered unable to consent for sexual
intercourse and hence consultations regarding this age group should
automatically trigger child protection measures
The Fraser Guidelines state that all the following requirements should be fulfilled:

• the young person understands the professional's advice


• the young person cannot be persuaded to inform their parents
• the young person is likely to begin, or to continue having, sexual intercourse
with or without contraceptive treatment
• unless the young person receives contraceptive treatment, their physical or
mental health, or both, are likely to suffer
• the young person's best interests require them to receive contraceptive advice
or treatment with or without parental consent

Sexual Transmitted Infections (STIs)

• young people should be advised to have STI tests 2 and 12 weeks after an
incident of unprotected sexual intercourse (UPSI)

Choice of contraceptive

• clearly long-acting reversible contraceptive methods (LARCs) have advantages in


young people as this age group may often be less reliable in remembering to
take medication
• however, there are some concerns about the effect of progesterone-only
injections (Depo-provera) on bone mineral density and the UKMEC category of
the IUS and IUD is 2 for women under the age of 20 years, meaning they may
not be the best choice
• the progesterone-only implant (Nexplanon) is therefore the LARC of choice
is young people

Progestogen only pill: counselling

Women who are considering taking the progestogen only pill (POP) should be
counselled in a number of areas:

Potential adverse effects

• irregular vaginal bleeding is the most common problem

Starting the POP

• if commenced up to and including day 5 of the cycle it provides immediate


protection, otherwise additional contraceptive methods (e.g. condoms) should
be used for the first 2 days
• if switching from a combined oral contraceptive (COC) gives immediate
protection if continued directly from the end of a pill packet (i.e. Day 21)

Taking the POP

• should be taken at same time every day, without a pill-free break (unlike the
COC)

Missed pills

• if < 3 hours* late: continue as normal


• if > 3 hours*: take the missed pill as soon as possible, continue with the rest of
the pack, extra precautions (e.g. condoms) should be used until pill taking has
been re-established for 48 hours

Other potential problems

• diarrhoea and vomiting: continue taking POP but assume pills have been missed
- see above
• antibiotics: have no effect on the POP**
• liver enzyme inducers may reduce the effectiveness

Other information: discussion on STIs

*for Cerazette (desogestrel) a 12 hour period is allowed

**unless the antibiotic alters the P450 enzyme system, for example, rifampicin

*** Women should be advised about the likelihood and types of bleeding patterns
expected with POP use. As a general guide:

•20% of women will be amenorrhoeic

•40% will bleed regularly

•40% will have erratic bleeding.

Between 10% and 25% of women using a POP will discontinue this method within 1
year as a result of these bleeding patterns

83. Recurrent miscarriage


Recurrent miscarriage is defined as 3 or more consecutive spontaneous abortions. It
occurs in around 1% of women
Causes

• antiphospholipid syndrome
• endocrine disorders: poorly controlled diabetes mellitus/thyroid disorders.
Polycystic ovarian syndrome
• uterine abnormality: e.g. uterine septum
• parental chromosomal abnormalities
• smoking

84. Breech presentation

In a breech presentation the caudal end of the fetus occupies the lower segment.
Whilst around 25% of pregnancies at 28 weeks are breech it only occurs in 3% of
babies near term. A frank breech is the most common presentation with the hips
flexed and knees fully extended. A footling breech, where one or both feet come first
with the bottom at a higher position, is rare but carries a higher perinatal morbidity

Risk factors for breech presentation

• uterine malformations, fibroids


• placenta praevia
• polyhydramnios or oligohydramnios
• fetal abnormality (e.g. CNS malformation, chromosomal disorders)
• prematurity (due to increased incidence earlier in gestation)

Cord prolapse is more common in breech presentations

Management

• if < 36 weeks: many fetuses will turn spontaneously


• if still breech at 36 weeks NICE recommend external cephalic version (ECV)- this
has a success rate of around 60%. The RCOG recommend ECV should be offered
from 36 weeks in nulliparous women and from 37 weeks in multiparous women
• if the baby is still breech then delivery options include planned caesarean
section or vaginal delivery

Information to help decision making - the RCOG recommend:


• 'Women should be informed that planned caesarean section carries a reduced
perinatal mortality and early neonatal morbidity for babies with a breech
presentation at term compared with planned vaginal birth.'
• 'Women should be informed that there is no evidence that the long term health
of babies with a breech presentation delivered at term is influenced by how the
baby is born.'

RCOG absolute contraindications to ECV:

• where caesarean delivery is required


• antepartum haemorrhage within the last 7 days
• abnormal cardiotocography
• major uterine anomaly
• ruptured membranes
• multiple pregnancy

85. Contraception for women aged > 40 years

Whilst fertility has usually significantly declined by the age of 40 years women still
require effective contraception until the menopause. The Faculty of Sexual and
Reproductive Healthcare (FSRH) have produced specific guidance looking at this age
group - 'Contraception for Women Aged Over 40 Years' - a link is provided below.

Specific methods

No method of contraception is contraindicated by age alone. All methods are


UKMEC1 except for the combined oral contraceptive pill (UKMEC2 for women >= 40
years) and Depo-Provera (UKMEC2 for women > 45 years). The guidance issued by
the FSRH contained a number of points which should be considered about each
method:

Combined oral contraceptive pill (COCP)

• COCP use in the perimenopausal period may help to maintain bone mineral
density
• COCP use may help reduce menopausal symptoms
• a pill containing < 30 µg ethinylestradiol may be more suitable for women > 40
years

Depo-Provera

• women should be advised there may be a delay in the return of fertility of up to


1 year for women > 40 years
• use is associated with a small loss in bone mineral density which is usually
recovered after discontinuation

Stopping contraception

The FSRH have produced a useful table detailing how the different methods may be
stopped. Please follow the link for the full table.

Method Women < 50 years Women >= 50 years


Non-hormonal (e.g.
Stop contraception
IUD, condoms, Stop contraception after 1 year of
after 2 years of
natural family amenorrhoea
amenorrhoea
planning)
Can be continued to Switch to non-hormonal or progestogen-
COCP
50 years only method
Switch to either a non-hormonal method
Can be continued to and stop after 2 years of amenorrhoea OR
Depo-Provera
50 years switch to a progestogen-only method and
follow advice below
Continue

If amenorrhoeic check FSH and stop after


Can be continued
Implant, POP, IUS 1 year if FSH >= 30u/l or stop at 55 years
beyond 50 years
If not amenorrhoeic consider investigating
abnormal bleeding pattern

Hormone Replacement Therapy and Contraception

As we know hormone replacement therapy (HRT) cannot be relied upon for


contraception so a separate method of contraception is needed. The FSRH advises
that the POP may be be used with in conjunction with HRT as long as the HRT has a
progestogen component (i.e. the POP cannot be relied upon to 'protect' the
endometrium). In contract the IUS is licensed to provide the progestogen component
of HRT.

86. Hormone replacement therapy: adverse effects

Hormone replacement therapy (HRT) involves the use of a small dose of oestrogen
(combined with a progestogen in women with a uterus) to help alleviate menopausal
symptoms.

Side-effects
• nausea
• breast tenderness
• fluid retention and weight gain

Potential complications

• increased risk of breast cancer


o increased by the addition of a progestogen
o in the Women's Health Initiative (WHI) study there was a relative risk of
1.26 at 5 years of developing breast cancer
o the increased risk relates to the duration of use
o the risk of breast cancer begins to decline when HRT is stopped and by 5
years it reaches the same level as in women who have never taken HRT
• increased risk of endometrial cancer
o oestrogen by itself should not be given as HRT to women with a
womb
o reduced by the addition of a progestogen but not eliminated completely
o the BNF states that the additional risk is eliminated if a progestogen is
given continuously
• increased risk of venous thromboembolism
o increased by the addition of a progestogen
o transdermal HRT does not appear to increase the risk of VTE
o NICE state women requesting HRT who are at high risk for VTE should be
referred to haematology before starting any treatment (even transdermal)
• increased risk of stroke
• increased risk of ischaemic heart disease if taken more than 10 years after
menopause

87. Symphysis-fundal height

The symphysis-fundal height (SFH) is measured from the top of the pubic bone to
the top of the uterus in centimetres
It should match the gestational age in weeks to within 2 cm after 20 weeks, e.g. if 24
weeks then the a normal SFH = 22 to 26 cm

88. Placenta praevia

Placenta praevia describes a placenta lying wholly or partly in the lower uterine
segment
Epidemiology

• 5% will have low-lying placenta when scanned at 16-20 weeks gestation


• incidence at delivery is only 0.5%, therefore most placentas rise away from cervix

Associated factors

• multiparity
• multiple pregnancy
• embryos are more likely to implant on a lower segment scar from previous
caesarean section

Clinical features

• shock in proportion to visible loss


• no pain
• uterus not tender
• lie and presentation may be abnormal
• fetal heart usually normal
• coagulation problems rare
• small bleeds before large

Investigations

• placenta praevia is often picked up on the routine 20 week abdominal


ultrasound
• the RCOG recommend the use of transvaginal ultrasound as it improves the
accuracy of placental localisation and is considered safe

Classical grading

• I - placenta reaches lower segment but not the internal os


• II - placenta reaches internal os but doesn't cover it
• III - placenta covers the internal os before dilation but not when dilated
• IV ('major') - placenta completely covers the internal os

89. Combined contraceptive patch

The Evra patch is the only combined contraceptive patch licensed for use in the UK.
The patch cycle lasts 4 weeks. For the first 3 weeks, the patch is worn everyday and
needs to be changed each week. During the 4th week, the patch is not worn and
during this time there will be a withdrawal bleed.
For delays in changing the patch, different rules apply depending what week of the
patch cycle the woman is in.
If the patch change is delayed at the end of week 1 or week 2:
If the delay in changing the patch is less than 48 hours, it should be changed
immediately and no further precautions are needed.
If the delay is greater than 48 hours, the patch should be changed immediately and a
barrier method of contraception used for the next 7 days. If the woman has had
sexual intercourse during this extended patch-free interval or if unprotected sexual
intercourse has occurred in the last 5 days, then emergency contraception needs to
be considered.

If the patch removal is delayed at the end of week 3:

The patch should be removed as soon as possible and the new patch applied on the
usual cycle start day for the next cycle, even if withdrawal bleeding is occurring. No
additional contraception is needed.

If patch application is delayed at the end of a patch-free week, additional barrier


contraception should be used for 7 days following any delay at the start of a new
patch cycle.

** If the contraceptive patch change is delayed greater than 48 hours, the patch
should be changed immediately and a barrier method of contraception used for the
next 7 days

The patch is an alternative to the combined pill and is an effective contraceptive if


used correctly. One should be applied to clean, dry skin on a weekly basis for three
weeks. The fourth week should be patch free, stimulating a withdrawal bleed.

If the contraceptive patch change is delayed greater than 48 hours, the patch should
be changed immediately and a barrier method of contraception used for the next 7
days.

if unprotected intercourse has occurred then emergency contraceptives should be


considered.

90. Hyperemesis gravidarum

Whilst the majority of women experience nausea (previously termed 'morning


sickness') during the early stages of pregnancy it can become problematic in a
minority of cases. The Royal College of Obstetricians and Gynaecologists (RCOG)
now use the term 'nausea and vomiting of pregnancy' (NVP) to describe troublesome
symptoms, with hyperemesis gravidarum being the extreme form of this condition.

It occurs in around 1% of pregnancies and is thought to be related to raised beta


hCG levels. Hyperemesis gravidarum is most common between 8 and 12 weeks but
may persist up to 20 weeks*.
Associations

• multiple pregnancies
• trophoblastic disease
• hyperthyroidism
• nulliparity
• obesity

Smoking is associated with a decreased incidence of hyperemesis.

Referral criteria for nausea and vomiting in pregnancy

NICE Clinical Knowledges Summaries recommend considering admission in the


following situations:

• Continued nausea and vomiting and is unable to keep down liquids or oral
antiemetics
• Continued nausea and vomiting with ketonuria and/or weight loss (greater than
5% of body weight), despite treatment with oral antiemetics
• A confirmed or suspected comorbidity (for example she is unable to tolerate
oral antibiotics for a urinary tract infection)

They also recommend having a lower threshold for admitting to hospital if the
woman has a co-existing condition (for example diabetes) which may be adversely
affected by nausea and vomiting.

Hyperemesis gravidarum

The Royal College of Obstetricians and Gynaecologists (RCOG) recommend that the
following triad is present before diagnosis hyperemesis gravidarum:

• 5% pre-pregnancy weight loss


• dehydration
• electrolyte imbalance

Validated scoring systems such as the Pregnancy-Unique Quantification of Emesis


(PUQE) score can be used to classify the severity of NVP.

Management

• antihistamines should be used first-line (BNF suggests promethazine as first-


line). Cyclizine is also recommended by Clinical Knowledge Summaries (CKS)
• ondansetron and metoclopramide may be used second-line
o metoclopramide may cause extrapyramidal side effects
• ginger and P6 (wrist) acupressure: CKS suggest these can be tried but there is
little evidence of benefit
• admission may be needed for IV hydration

Complications

• Wernicke's encephalopathy
• Mallory-Weiss tear
• central pontine myelinolysis
• acute tubular necrosis
• fetal: small for gestational age, pre-term birth

*and in very rare cases beyond 20 weeks

91. Pregnancy: anaemia

Pregnant women are screened for anaemia at:

• the booking visit (often done at 8-10 weeks), and at 28 weeks

NICE use the following cut-offs to determine whether a woman should receive oral
iron therapy:

Gestation Cut-off
Booking visit < 11 g/dl
28 weeks < 10.5 g/dl
In pregnancy there are a number of physiological changes that take place and many
of these are normal. Ventilation rates are known to increase in pregnancy due to the
increased demand for oxygen and the increased basal metabolic rate. Oxygen
consumption can increase by as much as 20%.

For the cardiovascular system. Plasma volume increases which results in an increase
heart rate, stroke volume and cardiac output. From a haematological point of view
the plasma volume increased by up to 50% and the red blood cell volume increase
by about 20-30%. Due to this discrepancy, the haematocrit can decrease due to the
dilution effect.

92. Menopause

The average women in the UK goes through the menopause when she is 51 years
old. The climacteric is the period prior to the menopause where women may
experience symptoms, as ovarian function starts to fail

It is recommended to use effective contraception until the following time:

• 12 months after the last period in women > 50 years


• 24 months after the last period in women < 50 years

93. Puerperal pyrexia

Puerperal pyrexia may be defined as a temperature of > 38ºC in the first 14 days
following delivery.

Causes:

• endometritis: most common cause


• urinary tract infection
• wound infections (perineal tears + caesarean section)
• mastitis
• venous thromboembolism

Management

• if endometritis is suspected the patient should be referred to hospital for


intravenous antibiotics (clindamycin and gentamicin until afebrile for greater
than 24 hours)

94. Eclampsia

Eclampsia may be defined as the development of seizures in association pre-


eclampsia. To recap, pre-eclampsia is defined as:

• condition seen after 20 weeks gestation


• pregnancy-induced hypertension
• proteinuria

Magnesium sulphate is used to both prevent seizures in patients with severe pre-
eclampsia and treat seizures once they develop. Guidelines on its use suggest the
following:

• should be given once a decision to deliver has been made


• in eclampsia an IV bolus of 4g over 5-10 minutes should be given followed by
an infusion of 1g / hour
• urine output, reflexes, respiratory rate and oxygen saturations should be
monitored during treatment
o respiratory depression can occur: calcium gluconate is the first-line
treatment for magnesium sulphate induced respiratory depression
• treatment should continue for 24 hours after last seizure or delivery (around
40% of seizures occur post-partum)

Other important aspects of treating severe pre-eclampsia/eclampsia include fluid


restriction to avoid the potentially serious consequences of fluid overload

**The history, observations and investigations point towards eclampsia as the cause of this
seizure. Intravenous magnesium sulfate (loading dose followed by infusion) is recommended
for the control of eclamptic seizures.

Intravenous lorazepam is first-line for in-hospital treatment of convulsive status epilepticus,


with phenytoin an option if this fails. Glucose is appropriate where hypoglycaemia is a factor.
Oral medications are not suitable in a patient with a depressed level of consciousness.

95. Chorioamnionitis

Chorioamnionitis (which can affect up to 5% of all pregnancies) is a potentially life-


threatening condition to both mother and foetus and is therefore considered a
medical emergency. It is usually the result of an ascending bacterial infection of the
amniotic fluid / membranes / placenta. The major risk factor in this scenario is the
preterm premature rupture of membranes (however, it can still occur when the
membranes are still intact) which expose the normally sterile environment of the
uterus to potential pathogens. Prompt delivery of the foetus (via cesarean section if
necessary) and administration of intravenous antibiotics is widely considered the
mainstay of initial treatment for this condition.

96. Ovarian enlargement: management

The initial imaging modality for suspected ovarian cysts/tumours is ultrasound. The
report will usually report that the cyst is either:

• simple: unilocular, more likely to be physiological or benign


• complex: multilocular, more likely to be malignant

Management depends on the age of the patient and whether the patient is
symptomatic. It should be remembered that the diagnosis of ovarian cancer is often
delayed due to a vague presentation.

Premenopausal women
• a conservative approach may be taken for younger women (especially if < 35
years) as malignancy is less common. If the cyst is small (e.g. < 5 cm) and
reported as 'simple' then it is highly likely to be benign. A repeat ultrasound
should be arranged for 8-12 weeks and referral considered if it persists.

Postmenopausal women

• by definition physiological cysts are unlikely


• any postmenopausal woman with an ovarian cyst regardless of nature or size
should be referred to gynaecology for assessment

97. Episiotomy

An episiotomy describes an incision in the posterior wall of the vagina and perineum
that is performed in the second stage of labour to facilitate the passage of the fetus.

The procedure performed is an episiotomy. There are different techniques, but this
question outlines the medio-lateral approach (which can be performed at the 7
o'clock or 5 o'clock positions. This procedure serves 2 main purposes in this scenario:

•If the vagina were to tear due to the excessive forces incurred during childbirth
(especially in cephalopelvic disproportion), then the episiotomy is designed to
prevent the tear from going posteriorly where it could affect the anus and
surrounding muscles/sphincters. If this were to occur, the mother may suffer from
long term problems such as faecal incontinence.

•Effectively, the episiotomy creates more space for the foetus, thereby assisting in
delivery.

98. Placental abruption

Placental abruption describes separation of a normally sited placenta from the


uterine wall, resulting in maternal haemorrhage into the intervening space

Epidemiology: occurs in approximately 1/200 pregnancies


Cause –
• proteinuric hypertension
• cocaine use
• multiparity
• maternal trauma
• increasing maternal age

Clinical features
• shock out of keeping with visible loss
• pain constant
• tender, tense uterus
• normal lie and presentation
• fetal heart: absent/distressed
• coagulation problems
• beware pre-eclampsia, DIC, anuria

99. Atrophic vaginitis

Atrophic vaginitis often occurs in women who are post-menopausal women. It


presents with vaginal dryness, dyspareunia and occasional spotting. On examination,
the vagina may appear pale and dry. Treatment is with vaginal lubricants and
moisturisers - if these do not help then topical oestrogen cream can be used

100. Breastfeeding problems

'Minor' breastfeeding problems

• frequent feeding in a breastfed infant is not alone a sign of low milk supply
• nipple pain: may be caused by a poor latch
• blocked duct (‘milk bleb’): causes nipple pain when breastfeeding. Breastfeeding
should continue. Advice should be sought regarding the positioning of the
baby. Breast massage may also be tried
• nipple candidiasis: treatment for nipple candidiasis whilst breastfeeding should
involve miconazole cream for the mother and nystatin suspension for the baby

Treatment of the candidal infection is necessary. In order to fully treat the infection
both the mother and child should be treated, usually with miconazole cream applied
to the nipple post feed and the oral mucosa of the infant. Breast feeding should be
continued during treatment.Education to the mother should also be given: Good
hand hygiene after nappy change, sterilisation of anything that the baby puts in their
mouth (dummies, teats etc)

Mastitis

Mastitis affects around 1 in 10 breastfeeding women. The BNF advises to treat 'if
systemically unwell, if nipple fissure present, if symptoms do not improve after 12-24
hours of effective milk removal of if culture indicates infection'. The first-line
antibiotic is flucloxacillin for 10-14 days. Breastfeeding or expressing should continue
during treatment.
If left untreated, mastitis may develop into a breast abscess. This generally requires
incision and drainage.
Engorgement

Breast engorgement is one of the causes of breast pain in breastfeeding women. It


usually occurs in the first few days after the infant is born and almost always affects
both breasts. The pain or discomfort is typically worse just before a feed. Milk tends
to not flow well from an engorged breast and the infant may find it difficult to attach
and suckle. Fever may be present but usually settles within 24 hours. The breasts may
appear red. Complications include blocked milk ducts, mastitis and difficulties with
breastfeeding and, subsequently, milk supply.

Although it may initially be painful, hand expression of milk may help relieve the
discomfort of engorgement.

Raynaud's disease of the nipple

In Raynaud’s disease of the nipple, pain is often intermittent and present during and
immediately after feeding. Blanching of the nipple may be followed by cyanosis
and/or erythema. Nipple pain resolves when nipples return to normal colour.

Options of treatment for Raynaud's disease of the nipple include advice on


minimising exposure to cold, use of heat packs following a breastfeed, avoiding
caffeine and stopping smoking. If symptoms persist consider specialist referral for a
trial of oral nifedipine (off-license).

Concerns about poor infant weight gain

Around 1 in 10 breastfed babies lose more than the 'cut-off' 10% threshold in the
first week of life. This should prompt consideration of the above breastfeeding
problems. The infant should also be examined to look for any underlying problems.
NICE recommends an 'expert' review of feeding if this occurs (e.g. midwife-led
breastfeeding clinics) and monitoring of weight until weight gain is satisfactory.

101. Combined oral contraceptive pill: special situations

Concurrent antibiotic use

• for many years doctors in the UK have advised that the concurrent use of
antibiotics may interfere with the enterohepatic circulation of oestrogen and
thus make the combined oral contraceptive pill ineffective - 'extra- precautions'
were advised for the duration of antibiotic treatment and for 7 days afterwards
• no such precautions are taken in the US or the majority of mainland Europe
• in 2011 the Faculty of Sexual & Reproductive Healthcare produced new
guidelines abandoning this approach. The latest edition of the BNF has been
updated in line with this guidance
• precautions should still be taken with enzyme inducing antibiotics such as
rifampicin

Switching combined oral contraceptive pills

• the BNF and Faculty of Sexual & Reproductive Healthcare (FSRH) appear to give
contradictory advice. The Clinical Effectiveness Unit of the FSRH have stated in
the Combined Oral Contraception guidelines that the pill free interval does not
need to be omitted (please see link). The BNF however advises missing the pill
free interval if the progesterone changes. Given the uncertainty it is best to
follow the BNF

102. Premenstrual syndrome

Premenstrual syndrome (PMS) describes the emotional and physical symptoms that
women may experience in the luteal phase of the normal menstrual cycle.
PMS only occurs in the presence of ovulatory menstrual cycles - it doesn't occur prior
to puberty, during pregnancy or after the menopause.
Emotional symptoms include:

• anxiety
• stress
• fatigue
• mood swings

Physical symptoms

• bloating
• breast pain

Management

Options depend on the severity of symptoms

• mild symptoms can be managed with lifestyle advice


o apart from the usual advice on sleep, exercise, smoking and alcohol,
specific advice includes regular, frequent (2–3 hourly), small, balanced
meals rich in complex carbohydrates
• moderate symptoms may benefit from a new-generation combined oral
contraceptive pill (COCP)
o examples include Yasmin® (drospirenone 3 mg and ethinylestradiol 0.030
mg)
• severe symptoms may benefit from a selective serotonin reuptake inhibitor
(SSRI)
o this may be taken continuously or just during the luteal phase (for
example days 15–28 of the menstrual cycle, depending on its length)

103. Ovarian hyperstimulation syndrome

Ovarian hyperstimulation syndrome (OHSS) is a complication seen in some forms of


infertility treatment. It is postulated that the presence of multiple luteinized cysts
within the ovaries results in high levels of not only oestrogens and progesterone but
also vasoactive substances such as vascular endothelial growth factor (VEGF). This
results in increased membrane permeability and loss of fluid from the intravascular
compartment

Whilst it is rarely seen with clomifene therapy is more likely to be seen following
gonadotropin or hCG treatment. Up to one third of women who are having IVF may
experience a mild form of OHSS

The RCOG uses the following classification of OHSS

Mild Moderate Severe Critical


• As for moderate • As for severe
• As for mild
• Abdominal • Clinical evidence of • Thromboembolism
• Nausea and
pain ascites • Acute respiratory
vomiting
• Abdominal • Oliguria distress syndrome
• Ultrasound
bloating • Haematocrit > 45% • Anuria
evidence of ascites
• Hypoproteinaemia • Tense ascites

Some important tips:

** Contraceptives - time until effective (if not first day period):

• instant: IUD
• 2 days: POP
• 7 days: COC, injection, implant, IUS

** Calcium channel blockers and ACE inhibitors, including ramipril, are teratogenic. The most
widely used anti-hypertensive for pregnant women is labetalol. NICE recommend labetalol as
first-line treatment in moderate and severe gestational hypertension rather than
methyldopa. NICE CG107

**A simple cyst in a young woman is most likely a physiological cyst such as a follicular cyst.
The follicular cyst is the most common type of ovarian cyst.
Endometrioma is usually filled with old blood, hence the term 'chocolate cyst'.
The dermoid cyst contains dermoid tissue.
Corpus luteum cyst is also a physiological cyst but it is less common than follicular cysts.
Serous cystadenoma is not a simple physiological cyst.
SURGERY

1.Abdominal aortic aneurysm

Abdominal aortic aneurysms occur primarily as a result of the failure of elastic proteins within the
extracellular matrix. Aneurysms typically represent dilation of all layers of the arterial wall. Most
aneurysms are caused by degenerative disease. After the age of 50 years the normal diameter of the
infrarenal aorta is 1.5cm in females and 1.7cm in males. Diameters of 3cm and greater, are
considered aneurysmal. The pathophysiology involved in the development of aneurysms is complex
and the primary event is loss of the intima with loss of elastic fibres from the media. This process is
associated with, and potentiated by, increased proteolytic activity and lymphocytic infiltration.

Major risk factors for the development of aneurysms include smoking and hypertension. Rare but
important causes include syphilis and connective tissues diseases such as Ehlers Danlos type 1 and
Marfan's syndrome.

In the UK, all men aged 65 years are offered aneurysm screening with a single abdominal ultrasound.
Screening has shown to decrease death from abdominal aortic aneurysm by 44% over 4 years.

2. Head injury: NICE guidance on investigation

NICE has strict and clear guidance regarding which adult patients are safe to discharge and which
need further CT head imaging. The latter group are also divided into two further cohorts, those who
require an immediate CT head and those requiring CT head within 8 hours of injury:

CT head immediately

GCS < 13 on initial assessment

GCS < 15 at 2 hours post-injury

suspected open or depressed skull fracture.

any sign of basal skull fracture (haemotympanum, 'panda' eyes, cerebrospinal fluid leakage from the
ear or nose, Battle's sign).

post-traumatic seizure.

focal neurological deficit.

more than 1 episode of vomiting

CT head scan within 8 hours of the head injury - for adults with any of the following risk factors who
have experienced some loss of consciousness or amnesia since the injury:

age 65 years or older

any history of bleeding or clotting disorders

dangerous mechanism of injury (a pedestrian or cyclist struck by a motor vehicle, an occupant


ejected from a motor vehicle or a fall from a height of greater than 1 metre or 5 stairs)

more than 30 minutes' retrograde amnesia of events immediately before the head injury
If a patient is on warfarin who have sustained a head injury with no other indications for a CT head
scan, perform a CT head scan within 8 hours of the injury.

3.Breast cancer: types and classification

The terminology surrounding breast cancer can sometimes be confusing and has changed over
recent years. It is useful to start by considering basic breast anatomy

Image sourced from Wikipedia

Basic breast anatomy

Most breast cancers arise from duct tissue followed by lobular tissue, described as ductal or lobular
carcinoma respectively. These can be further subdivided as to whether the cancer hasn't spread
beyond the local tissue (described as carcinoma-in-situ) or has spread (described as invasive).
Therefore, common breast cancer types include:

• Invasive ductal carcinoma. This is the most common type of breast cancer. To complicate
matters further this has recently been renamed 'No Special Type (NST)'. In contrast, lobular
carcinoma and other rarer types of breast cancer are classified as 'Special Type'

• Invasive lobular carcinoma

• Ductal carcinoma-in-situ (DCIS)

• Lobular carcinoma-in-situ (LCIS)

Rarer types of breast cancer are shown in the following list. These are classed as 'Special Type' but as
noted previously remember that a relatively common type of breast cancer (lobular) is also Special
Type:

• Medullary breast cancer

• Mucinous (mucoid or colloid) breast cancer

• Tubular breast cancer

• Adenoid cystic carcinoma of the breast

• Metaplastic breast cancer

• Lymphoma of the breast

• Basal type breast cancer

• Phyllodes or cystosarcomaphyllodes

• Papillary breast cancer

Other types of breast cancer include the following (although please note they may be associated
with the underlying lesions seen above, rather than completely separate subtypes):

Paget's disease of the nipple is an eczematoid change of the nipple associated with an underlying
breast malignancy and it is present in 1-2% of patients with breast cancer. In half of these patients, it
is associated with an underlying mass lesion and 90% of such patients will have an invasive
carcinoma. 30% of patients without a mass lesion will still be found to have an underlying carcinoma.
The remainder will have carcinoma in situ.

Inflammatory breast cancer where cancerous cells block the lymph drainage resulting in an inflamed
appearance of the breast. This accounts for around 1 in 10,000 cases of breast cancer.

4. Breast disorders

The table below describes some of the features seen in the most common breast disorders:

Disorder Features

Fibroadenoma Common in women under the age of 30 years


Often described as 'breast mice' due as they are discrete, non-tender, highly
mobile lumps

Fibroadenosis (fibrocystic Most common in middle-aged women


disease, benign 'Lumpy' breasts which may be painful. Symptoms may worsen prior to
mammary dysplasia) menstruation

Breast cancer Characteristically a hard, irregular lump. There may be associated nipple
inversion or skin tethering

Paget's disease of the breast - intraductal carcinoma associated with a


Disorder Features

reddening and thickening (may resemble eczematous changes) of the


nipple/areola

Mammary duct ectasia Dilatation of the large breast ducts


Most common around the menopause
May present with a tender lump around the areola +/- a green nipple discharge
If ruptures may cause local inflammation, sometimes referred to as 'plasma cell
mastitis'

Duct papilloma Local areas of epithelial proliferation in large mammary ducts


Hyperplastic lesions rather than malignant or premalignant
May present with blood stained discharge

Fat necrosis More common in obese women with large breasts


May follow trivial or unnoticed trauma
Initial inflammatory response, the lesion is typical firm and round but may
develop into a hard, irregular breast lump
Rare and may mimic breast cancer so further investigation is always warranted

Breast abscess More common in lactating women


Red, hot tender swelling

Lipomas and sebaceous cysts may also develop around the breast tissue.

5.Breast cancer: risk factors

Predisposing factors

• BRCA1, BRCA2 genes - 40% lifetime risk of breast/ovarian cancer

• 1st degree relative premenopausal relative with breast cancer (e.g. mother)

• nulliparity, 1st pregnancy > 30 yrs (twice risk of women having 1st child < 25 yrs)

• early menarche, late menopause

• combined hormone replacement therapy (relative risk increase * 1.023/year of


use), combined oral contraceptive use

• past breast cancer

• not breastfeeding

• ionising radiation

• p53 gene mutations

• obesity
• previous surgery for benign disease (?more follow-up, scar hides lump)

HRT, early menarche, late menopause and COCP all increase the risk of breast cancer whereas
multiple pregnancy and breastfeeding reduce the risk.

6. Brain death

Criteria for brain stem death testing

• Deep coma of known aetiology

• Reversible causes excluded

• No sedation

• Normal electrolytes

Testing for brain death

• Fixed pupils which do not respond to sharp changes in the intensity of incident light

• No corneal reflex

• Absent oculo-vestibular reflexes - no eye movements following the slow injection of at least
50ml of ice-cold water into each ear in turn (the caloric test)

• No response to supraorbital pressure

• No cough reflex to bronchial stimulation or gagging response to pharyngeal stimulation

• No observed respiratory effort in response to disconnection of the ventilator for long


enough (typically 5 minutes) to ensure elevation of the arterial partial pressure of carbon
dioxide to at least 6.0 kPa (6.5 kPa in patients with chronic carbon dioxide retention).
Adequate oxygenation is ensured by pre-oxygenation and diffusion oxygenation during the
disconnection (so the brain stem respiratory centre is not challenged by the ultimate, anoxic,
drive stimulus)

The test should be undertaken by two appropriately experienced doctors on two separate occasions.
Both should be experienced in performing brain stem death testing and have at least 5 years post-
graduate experience. One of them must be a consultant. Neither can be a member of the transplant
team (if organ donation contemplated).

Brain death testing involves consideration of the history, pupil reactions, reflexes, pain response and
observed respiratory effort.

The diagnosis of brain death should be made by two separate senior doctors on separate occasions
to reduce the chance of observer error.
After brain death has occurred life support can be withdrawn. The patient may be suitable for organ
donation which can occur prior to this process.

7. Ascending cholangitis

Ascending cholangitis is a bacterial infection (typically E. coli) of the biliary tree. The most common
predisposing factor is gallstones.

Charcot's triad of right upper quadrant (RUQ) pain, fever and jaundice occurs in about 20-50% of
patients

• fever is the most common feature, seen in 90% of patients

• RUQ pain 70%

• jaundice 60%

• hypotension and confusion are also common (the additional 2 factors in addition to the 3
above make Reynolds' pentad)

Other features

• raised inflammatory markers

Management

• intravenous antibiotics

• endoscopic retrograde cholangiopancreatography (ERCP) after 24-48 hours to relieve any


obstruction

8. Breast cancer: management

The management of breast cancer depends on the staging, tumour type and patient background. It
may involve any of the following:

• surgery

• radiotherapy

• hormone therapy

• biological therapy

• chemotherapy
Surgery

The vast majority of patients who have breast cancer diagnosed will be offered surgery. An
exception may be a very frail, elderly lady with metastatic disease who may be better managed with
hormonal therapy.

Prior to surgery, the presence/absence of axillary lymphadenopathy determines management:

• women with no palpable axillary lymphadenopathy at presentation should have a pre-


operative axillary ultrasound before their primary surgery

o if positive then they should have a sentinel node biopsy to assess the nodal burden

• in patients with breast cancer who present with clinically palpable lymphadenopathy,
axillary node clearance is indicated at primary surgery

o this may lead to arm lymphedema and functional arm impairment

Depending on the characteristics of the tumour women either have a wide-local excision or a
mastectomy. Around two-thirds of tumours can be removed with a wide-local excision. The table
below lists some of the factors determining which operation is offered:

Mastectomy Wide Local Excision

Multifocal tumour Solitary lesion

Central tumour Peripheral tumour

Large lesion in small breast Small lesion in large breast

DCIS > 4cm DCIS < 4cm

Women should be offered breast reconstruction to achieve a cosmetically suitable result regardless
of the type of operation they have. For women who've had a mastectomy this may be done at the
initial operation or at a later date.

Radiotherapy

Whole breast radiotherapy is recommended after a woman has had a wide-local excision as this may
reduce the risk of recurrence by around two-thirds. For women who've had a mastectomy
radiotherapy is offered for T3-T4 tumours and for those with four or more positive axillary nodes
Hormonal therapy

Adjuvant hormonal therapy is offered if tumours are positive for hormone receptors. For many years
this was done using tamoxifen for 5 years after diagnosis. Tamoxifen is still used in pre- and peri-
menopausal women. In post-menopausal women, aromatase inhibitors such as anastrozole are used
for this purpose*. This is important as aromatisation accounts for the majority of oestrogen
production in post-menopausal women and therefore anastrozole is used for ER +ve breast cancer in
this group.

Important side-effects of tamoxifen include an increased risk of endometrial cancer, venous


thromboembolism and menopausal symptoms.

Biological therapy

The most common type of biological therapy used for breast cancer is trastuzumab (Herceptin). It is
only useful in the 20-25% of tumours that are HER2 positive.

Trastuzumab cannot be used in patients with a history of heart disorders.

Chemotherapy

Cytotoxic therapy may be used either prior to surgery ('neoadjuvanant' chemotherapy)


to downstage a primary lesion or after surgery depending on the stage of the tumour, for example, if
there is axillary node disease - FEC-D is used in this situation.

9. Breast cancer: referral

NICE published referral guidelines for suspected breast cancer in 2015 (our emphasis):

Refer people using a suspected cancer pathway referral (for an appointment within 2 weeks) for
breast cancer if they are:

• aged 30 and over and have an unexplained breast lump with or without pain or

• aged 50 and over with any of the following symptoms in one nipple only: discharge,
retraction or other changes of concern

Consider a suspected cancer pathway referral (for an appointment within 2 weeks) for breast cancer
in people:
• with skin changes that suggest breast cancer or

• aged 30 and over with an unexplained lump in the axilla

Consider non-urgent referral in people aged under 30 with an unexplained breast lump with or
without pain.

10. Bilious vomiting in neonates

Incidence and Age at


Disorder causation presentation Diagnosis Treatment

Duodenal 1 in 5000 (higher Few hours after AXR shows Duodenoduodenostomy


atresia in Downs birth double bubble
syndrome) sign, contrast
study may
confirm

Malrotation Usually cause by Usually 3-7 days Upper GI Ladd's procedure


with incomplete after birth, volvulus contrast study
volvulus rotation during with compromised may show DJ
embryogenesis circulation may flexure is more
result in peritoneal medially placed,
signs and USS may show
haemodynamic abnormal
instability orientation of
SMA and SMV

Jejunal/ ileal Usually caused by Usually within 24 AXR will show Laparotomy with primary
atresia vascular hours of birth air-fluid levels resection and anastomosis
insufficiency in
utero, usually 1 in
3000

Meconium Occurs in Typically in first 24- Air - fluid levels Surgical decompression,
ileus between 15 48 hours of life on AXR, sweat serosal damage may require
and20% of those with abdominal test to confirm segmental resection
babies with cystic distension and cystic fibrosis
fibrosis, bilious vomiting
otherwise 1 in
5000
Incidence and Age at
Disorder causation presentation Diagnosis Treatment

Necrotising Up to 2.4 per Usually second Dilated bowel Conservative and supportive
enterocolitis 1000 births, risks week of life loops on AXR, for non perforated cases,
increased in pneumatosis and laparotomy and resection in
prematurity and portal venous air cases of perforation of
inter-current ongoing clinical
illness deterioration

Meconium ileus presents in the first 24-48 hours of life with abdominal distension and bilious
vomiting, more common in cystic fibrosis (the clue is the family history).

Gastroenteritis does not usually cause abdominal distension in newborns.

Necrotising enterocolitis usually presents after the second week of life.

GORD does not cause bilious vomiting.

Cow's milk protein allergy is not correct as the baby has not had contact with cow's milk yet.

11. Abdominal pain

The table below gives characteristic exam question features for conditions causing abdominal pain.
Unusual and 'medical' causes of abdominal pain should also be remembered:

• acute coronary syndrome

• diabetic ketoacidosis

• pneumonia

• acute intermittent porphyria

• lead poisoning

Condition Characteristic exam feature

Peptic ulcer disease Duodenal ulcers: more common than gastric ulcers, epigastric pain relieved by eating
Gastric ulcers: epigastric pain worsened by eating
Condition Characteristic exam feature

Features of upper gastrointestinal haemorrhage may be seen (haematemesis,


melena etc)

Appendicitis Pain initial in the central abdomen before localising to the right iliac fossa
Anorexia is common
Tachycardia, low-grade pyrexia, tenderness in RIF
Rovsing's sign: more pain in RIF than LIF when palpating LIF

Acute pancreatitis Usually due to alcohol or gallstones


Severe epigastric pain
Vomiting is common
Examination may reveal tenderness, ileus and low-grade fever
Periumbilical discolouration (Cullen's sign) and flank discolouration (Grey-Turner's
sign) is described but rare

Biliary colic Pain in the RUQ radiating to the back and interscapular region, may be following a
fatty meal. Slight misnomer as the pain may persist for hours
Obstructive jaundice may cause pale stools and dark urine
It is sometimes taught that patients are female, forties, fat and fair although this is
obviously a generalisation

Acute cholecystitis History of gallstones symptoms (see above)


Continuous RUQ pain
Fever, raised inflammatory markers and white cells
Murphy's sign positive (arrest of inspiration on palpation of the RUQ)

Diverticulitis Colicky pain typically in the LLQ


Fever, raised inflammatory markers and white cells

Abdominal aortic Severe central abdominal pain radiating to the back


aneurysm Presentation may be catastrophic (e.g. Sudden collapse) or sub-acute (persistent
severe central abdominal pain with developing shock)
Patients may have a history of cardiovascular disease

Intestinal History of malignancy/previous operations


obstruction Vomiting
Not opened bowels recently
'Tinkling' bowel sounds
Diagram showing stereotypical areas where particular conditions present. The diagram is not
exhaustive and only lists the more common conditions seen in clinical practice. Note how pain from
renal causes such as renal/ureteric colic and pyelonephritis may radiate and move from the loins
towards the suprapubic area.

12. Abdominal wall hernias

The classical surgical definition of a hernia is the protrusion of an organ or the fascia of an organ
through the wall of the cavity that normally contains it.

Risk factors for abdominal wall hernias include:

• obesity

• ascites

• increasing age

• surgical wounds

Features

• palpable lump

• cough impulse

• pain
• obstruction: more common in femoral hernias

• strangulation: may compromise the bowel blood supply leading to infarction

Types of abdominal wall hernias:

Type of hernia Details

Inguinal hernia Inguinal hernias account for 75% of abdominal wall hernias. Around 95% of patients are
male; men have around a 25% lifetime risk of developing an inguinal hernia.
Above and medial to pubic tubercle
Strangulation is rare

Femoral hernia Below and lateral to the pubic tubercle


More common in women, particularly multiparous ones
High risk of obstruction and strangulation
Surgical repair is required

Umbilical hernia Symmetrical bulge under the umbilicus

Paraumbilical Asymmetrical bulge - half the sac is covered by skin of the abdomen directly above or
hernia below the umbilicus

Epigastric hernia Lump in the midline between umbilicus and the xiphisternum
Most common in men aged 20-30 years

Incisional hernia May occur in up to 10% of abdominal operations

Spigelian hernia Also known as lateral ventral hernia


Rare and seen in older patients
A hernia through the spigelian fascia (the aponeurotic layer between the rectus
abdominis muscle medially and the semilunar line laterally)

Obturator hernia A hernia which passes through the obturator foramen. More common in females and
typical presents with bowel obstruction

Richter hernia A rare type of hernia where only the antimesenteric border of the bowel herniates
through the fascial defect

Richter's hernia can present with strangulation without symptoms of obstruction

Abdominal wall hernias in children:

Type of hernia Details

Congenital inguinal Indirect hernias resulting from a patent processus vaginalis


hernia Occur in around 1% of term babies. More common in premature babies and
boys
Type of hernia Details

60% are right sided, 10% are bilaterally


Should be surgically repaired soon after diagnosis as at risk of incarceration

Infantile umbilical hernia Symmetrical bulge under the umbilicus


More common in premature and Afro-Caribbean babies
The vast majority resolve without intervention before the age of 4-5 years
Complications are rare

Congenital inguinal hernias have a high rate of complications and should be repaired promptly once
identified.

12. Colorectal cancer: referral guidelines

NICE updated their referral guidelines in 2015. The following patients should be referred urgently
(i.e. within 2 weeks) to colorectal services for investigation:

• patients >= 40 years with unexplained weight loss AND abdominal pain

• patients >= 50 years with unexplained rectal bleeding

• patients >= 60 years with iron deficiency anaemia OR change in bowel habit

• tests show occult blood in their faeces (see below)

An urgent referral (within 2 weeks) should be 'considered' if:

• there is a rectal or abdominal mass

• there is an unexplained anal mass or anal ulceration

• patients < 50 years with rectal bleeding AND any of the following unexplained
symptoms/findings:

• -→ abdominal pain

• -→ change in bowel habit

• -→ weight loss

• -→ iron deficiency anaemia

Faecal Occult Blood Testing (FOBT)

This was one of the main changes in 2015. Remember that the NHS now has a national screening
programme offering screening every 2 years to all men and women aged 60 to 74 years. Patients
aged over 74 years may request screening.

In addition FOBT should be offered to:

• patients >= 50 years with unexplained abdominal pain OR weight loss

• patients < 60 years with changes in their bowel habit OR iron deficiency anaemia

• patients >= 60 years who have anaemia even in the absence of iron deficiency

Any patient of this age with an unexplained microcytic anaemia should have a lower gastrointestinal
tract investigation to exclude colorectal cancer.

13. Benign prostatic hyperplasia

Benign prostatic hyperplasia (BPH) is a common condition seen in older men.

Risk factors

• age: around 50% of 50-year-old men will have evidence of BPH and 30% will have symptoms.
Around 80% of 80-year-old men have evidence of BPH

• ethnicity: black > white > Asian

BPH typically presents with lower urinary tract symptoms (LUTS), which may be categorised into:

• voiding symptoms (obstructive): weak or intermittent urinary flow, straining, hesitancy,


terminal dribbling and incomplete emptying

• storage symptoms (irritative) urgency, frequency, urgency incontinence and nocturia

• post-micturition: dribbling

• complications: urinary tract infection, retention, obstructive uropathy

Management options

• watchful waiting

• medication: alpha-1 antagonists, 5 alpha-reductase inhibitors. The use of combination


therapy was supported by the Medical Therapy Of Prostatic Symptoms (MTOPS) trial

• surgery: transurethral resection of prostate (TURP)

Alpha-1 antagonists e.g. tamsulosin, alfuzosin


• decrease smooth muscle tone (prostate and bladder)

• considered first-line, improve symptoms in around 70% of men

• adverse effects: dizziness, postural hypotension, dry mouth, depression

5 alpha-reductase inhibitors e.g. finasteride

• block the conversion of testosterone to dihydrotestosterone (DHT), which is known to


induce BPH

• unlike alpha-1 antagonists causes a reduction in prostate volume and hence may slow
disease progression. This however takes time and symptoms may not improve for 6 months.
They may also decrease PSA concentrations by up to 50%

• adverse effects: erectile dysfunction, reduced libido, ejaculation problems, gynaecomastia

Finasteride treatment of BPH may take 6 months before results are seen

14. Benign breast lesions

Lesion Features Treatment

Fibroadenoma • Develop from a whole lobule If >3cm surgical excision is usual,


Phyllodes tumours should be widely
• Mobile, firm breast lumps
excised (mastectomy if the lesion is
• 12% of all breast masses large)

• Over a 2 year period up to 30%


will get smaller

• No increase in risk of
malignancy

Breast cyst • 7% of all Western females will Cysts should be aspirated, those
present with a breast cyst which are blood stained or
persistently refill should be biopsied
• Usually presents as a smooth
or excised
discrete lump (may be
fluctuant)

• Small increased risk of breast


cancer (especially if younger)

Sclerosingadenosis, (radial • Usually presents as a breast Lesions should be biopsied, excision


scars and complex lump or breast pain is not mandatory
sclerosing lesions)
• Causes mammographic
changes which may mimic
carcinoma
Lesion Features Treatment

• Cause distortion of the distal


lobular unit, without
hyperplasia (complex lesions
will show hyperplasia)

• Considered a disorder of
involution, no increase in
malignancy risk

Epithelial hyperplasia • Variable clinical presentation If no atypical features then


ranging from generalised conservative, those with atypical
lumpiness through to discrete features require either close
lump monitoring or surgical resection

• Disorder consists of increased


cellularity of terminal lobular
unit, atypical features may be
present

• Atypical features and family


history of breast cancer confers
greatly increased risk of
malignancy

Fat necrosis • Up to 40% cases usually have a Imaging and core biopsy
traumatic aetiology

• Physical features usually mimic


carcinoma

• Mass may increase in size


initially

Duct papilloma • Usually present with nipple Microdochectomy


discharge

• Large papillomas may present


with a mass

• The discharge usually originates


from a single duct

• No increase risk of malignancy

Breast cysts should be aspirated as there is a small risk of breast cancer, especially in younger
women.

Monitor with either the ultrasound or mammogram without aspiration is not appropriate.

Wide local excision is not required at this stage.


A short history (e.g. a few days) of pain and a lump would make you consider another diagnosis such
as a breast abscess but the combination of a persistent lump spanning at least one menstrual cycle
and the irregularity point to a diagnosis of cancer.

15. Head injury: NICE guidance on investigation

NICE has strict and clear guidance regarding which adult patients are safe to discharge and which
need further CT head imaging. The latter group are also divided into two further cohorts, those who
require an immediate CT head and those requiring CT head within 8 hours of injury:

CT head immediately

• GCS < 13 on initial assessment

• GCS < 15 at 2 hours post-injury

• suspected open or depressed skull fracture.

• any sign of basal skull fracture (haemotympanum, 'panda' eyes, cerebrospinal fluid leakage
from the ear or nose, Battle's sign).

• post-traumatic seizure.

• focal neurological deficit.

• more than 1 episode of vomiting

CT head scan within 8 hours of the head injury - for adults with any of the following risk factors who
have experienced some loss of consciousness or amnesia since the injury:

• age 65 years or older

• any history of bleeding or clotting disorders

• dangerous mechanism of injury (a pedestrian or cyclist struck by a motor vehicle, an


occupant ejected from a motor vehicle or a fall from a height of greater than 1 metre or 5
stairs)

• more than 30 minutes' retrograde amnesia of events immediately before the head injury

If a patient is on warfarin who have sustained a head injury with no other indications for a CT head
scan, perform a CT head scan within 8 hours of the injury.
16. Abdominal aortic aneurysm:

Abdominal aortic aneurysms occur primarily as a result of the failure of elastic proteins within the
extracellular matrix. Aneurysms typically represent dilation of all layers of the arterial wall. Most
aneurysms are caused by degenerative disease. After the age of 50 years the normal diameter of the
infrarenal aorta is 1.5cm in females and 1.7cm in males. Diameters of 3cm and greater, are
considered aneurysmal. The pathophysiology involved in the development of aneurysms is complex
and the primary event is loss of the intima with loss of elastic fibres from the media. This process is
associated with, and potentiated by, increased proteolytic activity and lymphocytic infiltration.

Major risk factors for the development of aneurysms include smoking and hypertension. Rare but
important causes include syphilis and connective tissues diseases such as Ehlers Danlos type 1 and
Marfan's syndrome.

In the UK, all men aged 65 years are offered aneurysm screening with a single abdominal ultrasound.
Screening has shown to decrease death from abdominal aortic aneurysm by 44% over 4 years.

17. Prostate cancer: PSA testing

Prostate specific antigen (PSA) is a serine protease enzyme produced by normal and malignant
prostate epithelial cells. It has become an important tumour marker but much controversy still exists
regarding its usefulness as a screening tool.

The NHS Prostate Cancer Risk Management Programme (PCRMP) has published updated guidelines
in 2009 on how to handle requests for PSA testing in asymptomatic men. A recent European trial
(ERSPC) showed a statistically significant reduction in the rate of death prostate cancer by 20% in
men aged 55 to 69 years but this was associated with a high risk of over-diagnosis and over-
treatment. Having reviewed this and other data the National Screening Committee have decided not
to introduce a prostate cancer screening programme yet but rather allow men to make an informed
choice.

Age-adjusted upper limits for PSA were recommended by the PCRMP:

Age PSA level (ng/ml)

50-59 years 3.0

60-69 years 4.0

> 70 years 5.0

However, NICE Clinical Knowledge Summaries currently suggest a different cut-off:

• men aged 50-69 years should be referred if the PSA is >= 3.0 ng/ml OR there is an abnormal
DRE

• note this is a lower threshold than the PCRMP 60-69 years limits recommended above
PSA levels may also be raised by*:

• benign prostatic hyperplasia (BPH)

• prostatitis and urinary tract infection (NICE recommend to postpone the PSA test for at least
1 month after treatment)

• ejaculation (ideally not in the previous 48 hours)

• vigorous exercise (ideally not in the previous 48 hours)

• urinary retention

• instrumentation of the urinary tract

Poor specificity and sensitivity

• around 33% of men with a PSA of 4-10 ng/ml will be found to have prostate cancer. With a
PSA of 10-20 ng/ml this rises to 60% of men

• around 20% with prostate cancer have a normal PSA

• various methods are used to try and add greater meaning to a PSA level including age-
adjusted upper limits and monitoring change in PSA level with time (PSA velocity or PSA
doubling time)

*whether digital rectal examination actually causes a rise in PSA levels is a matter of debate.

Prostatitis can raise the prostate specific antigen (PSA) level falsely. It is advised to wait at least one
month prior to testing if a patient has prostatitis.

A digital rectal examination is not a substitute for a PSA test. It is worth noting that if the patient is
asymptomatic the test may result in unnecessary investigation. A normal PSA can also be seen in
prostate cancer.

18. Hearing Loss

Hearing loss may be conductive or sensorineural. To determine which is present patients will often
require a formal assessment with pure tone audiometry. In the clinical setting Webers and Rinnes
tests may be helpful in categorising various types of hearing loss.

Webers and Rinnes Tests


In a normal patient, the Weber tuning fork sound is heard equally loud in both ears with no one ear
hearing the sound louder than the other. A patient with symmetrical hearing loss will hear the
Weber tuning fork sound equally well with diagnostic utility only in asymmetric (one-sided) hearing
losses. In a patient with asymmetrical hearing loss, the Weber tuning fork sound is heard louder in
one ear versus the other. This clinical finding should be confirmed by repeating the procedure and
having the patient occlude one ear with a finger; the sound should be heard best in the occluded
ear.

Weber without
Rinne Test lateralisation Weber lateralises to left Weber lateralises to right

Both ears Normal Sensorineural loss on right Sensorineural loss on left


Air>Bone

Left Bone > Air Conductive loss on left Combined loss on left

Right Bone> Combined loss on right Conductive loss on right


Air

Both Bone > Combined loss on right and Combined loss on left and
Air conductive on left conductive on right

Sudden onset sensorineural hearing loss should be referred (within 24 hours) to ENT, for
investigation and consideration of steroid therapy

Important for meLess important

Sudden onset sensorineural hearing loss should be referred to ENT to be seen within 24 hours, for
urgent audiological assessment and to consider a course of prednisolone.

19. Anti-oestrogen drugs

Selective oEstrogen Receptor Modulators (SERM)

Tamoxifen is a SERM which acts as an oestrogen receptor antagonist and partial agonist. It is used in
the management of oestrogen receptor-positive breast cancer.

Adverse effects

• menstrual disturbance: vaginal bleeding, amenorrhoea

• hot flushes - 3% of patients stop taking tamoxifen due to climacteric side-effects

• venous thromboembolism

• endometrial cancer

Aromatase inhibitors
Anastrozole and letrozole are aromatase inhibitors that reduces peripheral oestrogen synthesis. This
is important as aromatisation accounts for the majority of oestrogen production in postmenopausal
women and therefore anastrozole is used for ER +ve breast cancer in this group.

Adverse effects
• osteoporosis

o NICE recommends a DEXA scan when initiating a patient on aromatase inhibitors for
breast cancer

• hot flushes

• arthralgia, myalgia

• insomnia

20. Vasectomy

Male sterilisation - vasectomy

• failure rate: 1 per 2,000 - male sterilisation is a more effective method of contraception than
female sterilisation

• simple operation, can be done under LA (some GA), go home after a couple of hours

• doesn't work immediately

• semen analysis needs to be performed twice following a vasectomy before a man can have
unprotected sex (usually at 16 and 20 weeks)

• complications: bruising, haematoma, infection, sperm granuloma, chronic testicular pain


(affects between 5-30% men)

• the success rate of vasectomy reversal is up to 55%, if done within 10 years, and
approximately 25% after more than 10 years

21. Breast cancer: screening

The NHS Breast Screening Programme is being expanded to include women aged 47-73 years from
the previous parameter of 50-70 years. Women are offered a mammogram every 3 years. After the
age of 70 years women may still have mammograms but are 'encouraged to make their own
appointments'.

The effectiveness of breast screening is regularly debated although it is currently thought that the
NHS Breast Screening Programme may save around 1,400 lives per year.

Familial breast cancer

NICE published guidelines on the management of familial breast cancer in 2013, giving guidelines on
who needs referral.

If the person concerned only has one first-degree or second-degree relative diagnosed with breast
cancer they do NOT need to be referred unless any of the following are present in the family history:
• age of diagnosis < 40 years

• bilateral breast cancer

• male breast cancer

• ovarian cancer

• Jewish ancestry

• sarcoma in a relative younger than age 45 years

• glioma or childhood adrenal cortical carcinomas

• complicated patterns of multiple cancers at a young age

• paternal history of breast cancer (two or more relatives on the father's side of the family)

Women who are at an increased risk of breast cancer due to their family history may be offered
screening from a younger age. The following patients should be referred to the breast clinic for
further assessment:

• one first-degree female relative diagnosed with breast cancer at younger than age 40 years,
or

• one first-degree male relative diagnosed with breast cancer at any age, or

• one first-degree relative with bilateral breast cancer where the first primary was diagnosed
at younger than age 50 years, or

• two first-degree relatives, or one first-degree and one second-degree relative, diagnosed
with breast cancer at any age, or

• one first-degree or second-degree relative diagnosed with breast cancer at any age and one
first-degree or second-degree relative diagnosed with ovarian cancer at any age (one of
these should be a first-degree relative), or

• three first-degree or second-degree relatives diagnosed with breast cancer at any age

22. Haemorrhoids

Haemorrhoidal tissue is part of the normal anatomy which contributes to anal continence. These
mucosal vascular cushions are found in the left lateral, right posterior and right anterior portions of
the anal canal (3 o'clock, 7'o'clock and 11 o'clock respectively). Haemorrhoids are said to exist when
they become enlarged, congested and symptomatic

Clinical features

• painless rectal bleeding is the most common symptom

• pruritus
• pain: usually not significant unless piles are thrombosed

• soiling may occur with third or forth degree piles

Types of haemorrhoids

External

• originate below the dentate line

• prone to thrombosis, may be painful

Internal

• originate above the dentate line

• do not generally cause pain

Grading of internal haemorrhoids

Grade I Do not prolapse out of the anal canal

Grade II Prolapse on defecation but reduce spontaneously

Grade III Can be manually reduced

Grade IV Cannot be reduced

Management

• soften stools: increase dietary fibre and fluid intake

• topical local anaesthetics and steroids may be used to help symptoms

• outpatient treatments: rubber band ligation is superior to injection sclerotherapy

• surgery is reserved for large symptomatic haemorrhoids which do not respond to outpatient
treatments

• newer treatments: Doppler guided haemorrhoidal artery ligation, stapled


haemorrhoidopexy

Acutely thrombosed external haemorrhoids


• typically present with significant pain

• examination reveals a purplish, oedematous, tender subcutaneous perianal mass

• if patient presents within 72 hours then referral should be considered for excision.
Otherwise patients can usually be managed with stool softeners, ice packs and analgesia.
Symptoms usually settle within 10 days

23.Circumcision

Circumcision has been performed in a variety of cultures for thousands of years. Today it is mainly
people of the Jewish and Islamic faith who undergo circumcision for religious/cultural reasons.
Circumcision for religious or cultural reasons is not available on the NHS.

The medical benefits of routine circumcision remain controversial although some evidence has
emerged that it:

• reduces the risk of penile cancer

• reduces the risk of UTI

• reduces the risk of acquiring sexually transmitted infections including HIV

Medical indications for circumcision

• phimosis

• recurrent balanitis

• balanitis xerotica obliterans

• paraphimosis

It is important to exclude hypospadias prior to circumcision as the foreskin may be used in surgical
repair. Circumcision may be performed under a local or general anaesthetic.

24. Haemorrhoids

Haemorrhoidal tissue is part of the normal anatomy which contributes to anal continence. These
mucosal vascular cushions are found in the left lateral, right posterior and right anterior portions of
the anal canal (3 o'clock, 7'o'clock and 11 o'clock respectively). Haemorrhoids are said to exist when
they become enlarged, congested and symptomatic

Clinical features
• painless rectal bleeding is the most common symptom

• pruritus

• pain: usually not significant unless piles are thrombosed

• soiling may occur with third or forth degree piles

Types of haemorrhoids

External

• originate below the dentate line

• prone to thrombosis, may be painful

Internal

• originate above the dentate line

• do not generally cause pain

Grading of internal haemorrhoids

Grade I Do not prolapse out of the anal canal

Grade II Prolapse on defecation but reduce spontaneously

Grade III Can be manually reduced

Grade IV Cannot be reduced

Management

• soften stools: increase dietary fibre and fluid intake

• topical local anaesthetics and steroids may be used to help symptoms

• outpatient treatments: rubber band ligation is superior to injection sclerotherapy

• surgery is reserved for large symptomatic haemorrhoids which do not respond to outpatient
treatments

• newer treatments: Doppler guided haemorrhoidal artery ligation, stapled


haemorrhoidopexy
Acutely thrombosed external haemorrhoids

• typically present with significant pain

• examination reveals a purplish, oedematous, tender subcutaneous perianal mass

• if patient presents within 72 hours then referral should be considered for excision.
Otherwise patients can usually be managed with stool softeners, ice packs and analgesia.
Symptoms usually settle within 10 days

25.Breast cancer: referral

NICE published referral guidelines for suspected breast cancer in 2015 (our emphasis):

Refer people using a suspected cancer pathway referral (for an appointment within 2 weeks) for
breast cancer if they are:

• aged 30 and over and have an unexplained breast lump with or without pain or

• aged 50 and over with any of the following symptoms in one nipple only: discharge,
retraction or other changes of concern

Consider a suspected cancer pathway referral (for an appointment within 2 weeks) for breast cancer
in people:

• with skin changes that suggest breast cancer or

• aged 30 and over with an unexplained lump in the axilla

Consider non-urgent referral in people aged under 30 with an unexplained breast lump with or
without pain.

26.Breast cancer: screening

The NHS Breast Screening Programme is being expanded to include women aged 47-73 years from
the previous parameter of 50-70 years. Women are offered a mammogram every 3 years. After the
age of 70 years women may still have mammograms but are 'encouraged to make their own
appointments'.

The effectiveness of breast screening is regularly debated although it is currently thought that the
NHS Breast Screening Programme may save around 1,400 lives per year.
Familial breast cancer

NICE published guidelines on the management of familial breast cancer in 2013, giving guidelines on
who needs referral.

If the person concerned only has one first-degree or second-degree relative diagnosed with breast
cancer they do NOT need to be referred unless any of the following are present in the family history:

• age of diagnosis < 40 years

• bilateral breast cancer

• male breast cancer

• ovarian cancer

• Jewish ancestry

• sarcoma in a relative younger than age 45 years

• glioma or childhood adrenal cortical carcinomas

• complicated patterns of multiple cancers at a young age

• paternal history of breast cancer (two or more relatives on the father's side of the family)

Women who are at an increased risk of breast cancer due to their family history may be offered
screening from a younger age. The following patients should be referred to the breast clinic for
further assessment:

• one first-degree female relative diagnosed with breast cancer at younger than age 40 years,
or

• one first-degree male relative diagnosed with breast cancer at any age, or

• one first-degree relative with bilateral breast cancer where the first primary was diagnosed
at younger than age 50 years, or

• two first-degree relatives, or one first-degree and one second-degree relative, diagnosed
with breast cancer at any age, or

• one first-degree or second-degree relative diagnosed with breast cancer at any age and one
first-degree or second-degree relative diagnosed with ovarian cancer at any age (one of
these should be a first-degree relative), or

• three first-degree or second-degree relatives diagnosed with breast cancer at any age
27. Varicocele

A varicocele is an abnormal enlargement of the testicular veins. They are usually asymptomatic but
may be important as they are associated with infertility.

Varicoceles are much more common on the left side (> 80%). Features:

• classically described as a 'bag of worms'

• subfertility

Diagnosis

• ultrasound with Doppler studies

Management

• usually conservative

• occasionally surgery is required if the patient is troubled by pain. There is ongoing debate
regarding the effectiveness of surgery to treat infertility

28. Venous thromboembolism: prophylaxis in patients admitted to hospital

VTEs can cause severe morbidity and mortality, but they are preventable. Current NICE guidelines
(updated for 2018) outline recommendations for assessment and management of patients at risk of
VTE in hospital.

Risk factors

All patients admitted to hospital should be individually assessed to identify risk factors for VTE
development and bleeding risk. For medical and surgical patients the recommended risk proforma is
the department of healths VTE risk assessment tool.

The following inpatients would be deemed at increased risk of developing a VTE:

Medical patients:
• significant reduction in mobility for 3 days or more (or anticipated to have significantly
reduced mobility)

Surgical/trauma patients:

• hip/knee replacement

• hip fracture

• general anaesthetic and a surgical duration of over 90 minutes

• surgery of the pelvis or lower limb with a general anaesthetic and a surgical duration of over
60 minutes

• acute surgical admission with an inflammatory/intra-abdominal condition

• surgery with a significant reduction in mobility

General risk factors:

• active cancer/chemotherapy

• aged over 60

• known blood clotting disorder (e.g. thrombophilia)

• BMI over 35

• dehydration

• one or more significant medical comorbidities (e.g. heart disease; metabolic/endocrine


pathologies; respiratory disease; acute infectious disease and inflammatory conditions)

• critical care admission

• use of hormone replacement therapy (HRT)

• use of the combined oral contraceptive pill

• varicose veins

• pregnant or less than 6 weeks post-partum

After a patients VTE risk has been assessed, this should be compared to their risk of bleeding to
decide whether VTE prophylaxis should be offered. If indicated VTE prophylaxis should be started as
soon as possible.

Types of VTE prophylaxis


Mechanical:

• Correctly fitted anti-embolism (aka compression) stockings (thigh or knee height)

• An Intermittent pneumatic compression device

Pharmacological:

• Fondaparinux sodium (SC injection)

• Low molecular weight heparin (LMWH)

o e.g. enoxaparin

o reduced doses should be used in patients with severe renal impairment

• Unfractionated heparin (UFH)

o used as an alternative to LWMH in patients with chronic kidney disease

Management

In general, all medical patients deemed at risk of VTE after individual assessment are started on
pharmacological VTE prophylaxis. This is providing the risk of VTE outweighs the risk of bleeding (this
is often a clinical judgement) and there are no contraindications. Those at very high risk may be
offered anti-embolic stockings alongside the pharmacological methods.

For surgical patients at low risk of VTE first-line treatment is anti-embolism stockings. If a patient is
at high risk these stockings are used in conjunction with pharmacological prophylaxis.

Advice for patients

Pre-surgical interventions:

• Advise women to stop taking their combined oral contraceptive pill/hormone replacement
therapy 4 weeks before surgery.

Post-surgical interventions:
• Try to mobilise patients as soon as possible after surgery

• Ensure the patient is hydrated

Post procedure prophylaxis

For certain surgical procedures (hip and knee replacements) pharmacological VTE prophylaxis is
recommended for all patients to reduce the risk of a VTE developing post-surgery. NICE make the
following recommendations:

Procedure Prophylaxis

Elective hip LMWH for 10 days followed by aspirin (75 or 150 mg) for a further
28 days

or

LMWH for 28 days combined with anti-embolism stockings until


discharge

or

Rivaroxaban

Elective knee Aspirin (75 or 150 mg) for 14 days

or

LMWH for 14 days combined with anti-embolism stockings until


discharge

or

Rivaroxaban

Fragility fractures of the pelvis, hip The NICE guidance states the following (our bolding):
and proximal femur

Offer VTE prophylaxis for a month to people with fragility fractures


of the pelvis, hip or proximal
femur if the risk of VTE outweighs the risk of bleeding. Choose
either:
Procedure Prophylaxis

• LMWH , starting 6–12 hours after surgery or

• fondaparinux sodium, starting 6 hours after surgery,


providing there is low risk of bleeding.

29. Abdominal wall hernias

The classical surgical definition of a hernia is the protrusion of an organ or the fascia of an organ
through the wall of the cavity that normally contains it.

Risk factors for abdominal wall hernias include:

• obesity

• ascites

• increasing age

• surgical wounds

Features

• palpable lump

• cough impulse

• pain

• obstruction: more common in femoral hernias

• strangulation: may compromise the bowel blood supply leading to infarction

Types of abdominal wall hernias:

Type of hernia Details

Inguinal hernia Inguinal hernias account for 75% of abdominal wall hernias. Around 95% of patients are
male; men have around a 25% lifetime risk of developing an inguinal hernia.
Above and medial to pubic tubercle
Strangulation is rare

Femoral hernia Below and lateral to the pubic tubercle


More common in women, particularly multiparous ones
Type of hernia Details

High risk of obstruction and strangulation


Surgical repair is required

Umbilical hernia Symmetrical bulge under the umbilicus

Paraumbilical Asymmetrical bulge - half the sac is covered by skin of the abdomen directly above or
hernia below the umbilicus

Epigastric hernia Lump in the midline between umbilicus and the xiphisternum
Most common in men aged 20-30 years

Incisional hernia May occur in up to 10% of abdominal operations

Spigelian hernia Also known as lateral ventral hernia


Rare and seen in older patients
A hernia through the spigelian fascia (the aponeurotic layer between the rectus
abdominis muscle medially and the semilunar line laterally)

Obturator hernia A hernia which passes through the obturator foramen. More common in females and
typical presents with bowel obstruction

Richter hernia A rare type of hernia where only the antimesenteric border of the bowel herniates
through the fascial defect

Richter's hernia can present with strangulation without symptoms of obstruction

Abdominal wall hernias in children:

Type of hernia Details

Congenital inguinal Indirect hernias resulting from a patent processus vaginalis


hernia Occur in around 1% of term babies. More common in premature babies and
boys
60% are right sided, 10% are bilaterally
Should be surgically repaired soon after diagnosis as at risk of incarceration

Infantile umbilical hernia Symmetrical bulge under the umbilicus


More common in premature and Afro-Caribbean babies
The vast majority resolve without intervention before the age of 4-5 years
Complications are rare

30.Anal fissure
Anal fissures are longitudinal or elliptical tears of the squamous lining of the distal anal canal. If
present for less than 6 weeks they are defined as acute, and chronic if present for more than 6
weeks.

Risk factors

• constipation

• inflammatory bowel disease

• sexually transmitted infections e.g. HIV, syphilis, herpes

Features

• painful, bright red, rectal bleeding

• around 90% of anal fissures occur on the posterior midline.

o if the fissures are found in alternative locations then other underlying causes should
be considered e.g. Crohn's disease

Management

Management of an acute anal fissure (< 1 week)

• soften stool

o dietary advice: high-fibre diet with high fluid intake

o bulk-forming laxatives are first-line - if not tolerated then lactulose should be tried

• lubricants such as petroleum jelly may be tried before defecation

• topical anaesthetics

• analgesia

Management of a chronic anal fissure

• the above techniques should be continued

• topical glyceryl trinitrate (GTN) is first-line treatment for a chronic anal fissure

• if topical GTN is not effective after 8 weeks then secondary care referral should be
considered for surgery (sphincterotomy) or botulinum toxin

31. Scrotal problems


Epididymal cysts

Epididymal cysts are the most common cause of scrotal swellings seen in primary care.

Features

• separate from the body of the testicle

• found posterior to the testicle

Associated conditions

• polycystic kidney disease

• cystic fibrosis

• von Hippel-Lindau syndrome

Diagnosis may be confirmed by ultrasound.

Management is usually supportive but surgical removal or sclerotherapy may be attempted for
larger or symptomatic cysts.

Hydrocele

A hydrocele describes the accumulation of fluid within the tunica vaginalis. They can be divided into
communicating and non-communicating:

• communicating: caused by patency of the processus vaginalis allowing peritoneal fluid to


drain down into the scrotum. Communicating hydroceles are common in newborn males
(clinically apparent in 5-10%) and usually resolve within the first few months of life

• non-communicating: caused by excessive fluid production within the tunica vaginalis

Hydroceles may develop secondary to:

• epididymo-orchitis

• testicular torsion

• testicular tumours
Features

• soft, non-tender swelling of the hemi-scrotum. Usually anterior to and below the testicle

• the swelling is confined to the scrotum, you can get 'above' the mass on examination

• transilluminates with a pen torch

• the testis may be difficult to palpate if the hydrocele is large

Diagnosis may be clinical but ultrasound is required if there is any doubt about the diagnosis or if the
underlying testis cannot be palpated.

Management

• infantile hydroceles are generally repaired if they do not resolve spontaneously by the age of
1-2 years

• in adults a conservative approach may be taken depending on the severity of the


presentation. Further investigation (e.g. ultrasound) is usually warranted however to exclude
any underlying cause such as a tumour

Varicocele

A varicocele is an abnormal enlargement of the testicular veins. They are usually asymptomatic but
may be important as they are associated with infertility.

Varicoceles are much more common on the left side (> 80%). Features:

• classically described as a 'bag of worms'

• subfertility

Diagnosis

• ultrasound with Doppler studies

Management

• usually conservative
• occasionally surgery is required if the patient is troubled by pain. There is ongoing debate
regarding the effectiveness of surgery to treat infertility

32.Prostate cancer: investigation

The traditional investigation for suspected prostate cancer was a transrectal ultrasound-guided
(TRUS) biopsy. However, recent guidelines from NICE have now advocated the increasing use
of multiparametric MRI as a first-line investigation.

Complications of TRUS biopsy:

• sepsis: 1% of cases

• pain: lasting >= 2 weeks in 15% and severe in 7%

• fever: 5%

• haematuria and rectal bleeding

Multiparametric MRI is now the first-line investigation for people with suspected clinically localised
prostate cancer.

• the results are reported using a 5-point Likert scale

If the Likert scale is >=3 a multiparametric MRI-influenced prostate biopsy is offered

If the Likert scale is 1-2 then NICE recommend discussing with the patient the pros and cons of
having a biopsy.

33.Acute pancreatitis: causes

The vast majority of cases in the UK are caused by gallstones and alcohol.

Popular mnemonic is GET SMASHED

• Gallstones

• Ethanol

• Trauma

• Steroids

• Mumps (other viruses include Coxsackie B)

• Autoimmune (e.g. polyarteritisnodosa), Ascaris infection

• Scorpion venom
• Hypertriglyceridaemia, Hyperchylomicronaemia, Hypercalcaemia, Hypothermia

• ERCP

• Drugs (azathioprine, mesalazine*, didanosine, bendroflumethiazide, furosemide,


pentamidine, steroids, sodium valproate)

34. Renal stones: management

The British Association of Urological Surgeons (BAUS) published guidelines in 2018 on the
management of acute ureteric/renal colic.

Initial management of renal colic

Medication

• the BAUS recommend an NSAID as the analgesia of choice for renal colic

• whilst diclofenac has been traditionally used the increased risk of cardiovascular events with
certain NSAIDs (e.g. diclofenac, ibuprofen) should be considered when prescribing

• the CKS guidelines suggest for patients who require admission: 'Administer a parenteral
analgesic (such as intramuscular diclofenac) for rapid relief of severe pain'

• BAUS no longer endorse the use of alpha-adrenergic blockers to aid ureteric stone passage
routinely. They do however acknowledge a recently published meta-analysis advocates the
use of α-blockers for patients amenable to conservative management, with greatest benefit
amongst those with larger stones

Initial investigations

• urine dipstick and culture

• serum creatinine and electrolytes: check renal function

• FBC / CRP: look for associated infection

• calcium/urate: look for underlying causes

• also: clotting if percutaneous intervention planned and blood cultures if pyrexial or other
signs of sepsis

Imaging

• BAUS now recommend that non-contrast CT KUB should be performed on all patients,
within 14 hours of admission
• if a patient has a fever, a solitary kidney or when the diagnosis is uncertain an immediate CT
KUB should be performed. In the case of an uncertain diagnosis, this is to exclude other
diagnoses such as ruptured abdominal aortic aneurysm

• CT KUB has a sensitivity of 97% for ureteric stones and a specificity of 95%

• ultrasound still has a role but given the wider availability of CT now and greater accurary it is
no longer recommend first-line. The sensitivity of ultrasound for stones is around 45% and
specificity is around 90%

Management of renal stones

Stones < 5 mm will usually pass spontaneously. Lithotripsy and nephrolithotomy may be for severe
cases.

Most renal stones measuring less than 5mm in maximum diameter will typically pass within 4 weeks
of symptom onset. More intensive and urgent treatment is indicated in the presence of ureteric
obstruction, renal developmental abnormality such as horseshoe kidney and previous renal
transplant. Ureteric obstruction due to stones together with infection is a surgical emergency and
the system must be decompressed. Options include nephrostomy tube placement, insertion of
ureteric catheters and ureteric stent placement.

In the non-emergency setting, the preferred options for treatment of stone disease include extra
corporeal shock wave lithotripsy, percutaneous nephrolithotomy, ureteroscopy, open surgery
remains an option for selected cases. However, minimally invasive options are the most popular
first-line treatment.

Shockwave lithotripsy

• A shock wave is generated external to the patient, internally cavitation bubbles and
mechanical stress lead to stone fragmentation. The passage of shock waves can result in the
development of solid organ injury. Fragmentation of larger stones may result in the
development of ureteric obstruction. The procedure is uncomfortable for patients and
analgesia is required during the procedure and afterwards.

Ureteroscopy

• Aureteroscope is passed retrograde through the ureter and into the renal pelvis. It is
indicated in individuals (e.g. pregnant females) where lithotripsy is contraindicated and in
complex stone disease. In most cases a stent is left in situ for 4 weeks after the procedure.

Percutaneous nephrolithotomy
• In this procedure, access is gained to the renal collecting system. Once access is achieved,
intra corporeal lithotripsy or stone fragmentation is performed and stone fragments
removed.

Therapeutic selection

Disease Option

Stone burden of less than 2cm in aggregate Lithotripsy

Stone burden of less than 2cm in pregnant females Ureteroscopy

Complex renal calculi and staghorn calculi Percutaneous nephrolithotomy

Ureteric calculi less than 5mm Manage expectantly

Prevention of renal stones

Calcium stones may be due to hypercalciuria, which is found in up to 5-10% of the general
population.

• high fluid intake

• low animal protein, low salt diet (a low calcium diet has not been shown to be superior to a
normocalcaemic diet)

• thiazides diuretics (increase distal tubular calcium resorption)

Oxalate stones

• cholestyramine reduces urinary oxalate secretion

• pyridoxine reduces urinary oxalate secretion

Uric acid stones

• allopurinol

• urinary alkalinization e.g. oral bicarbonate

35.Abdominal swelling
The table below gives characteristic exam question features for conditions causing abdominal
swelling

Condition Characteristic exam feature

Pregnancy Young female


Amenorrhoea

Intestinal obstruction History of malignancy/previous operations


Vomiting
Not opened bowels recently
'Tinkling' bowel sounds

Ascites History of alcohol excess, cardiac failure

Urinary retention History of prostate problems


Dullness to percussion around suprapubic area

Ovarian cancer Older female


Pelvic pain
Urinary symptoms e.g. urgency
Raised CA125
Early satiety, bloating

36.Anti-oestrogen drugs

Selective oEstrogen Receptor Modulators (SERM)

Tamoxifen is a SERM which acts as an oestrogen receptor antagonist and partial agonist. It is used in
the management of oestrogen receptor-positive breast cancer.

Adverse effects

• menstrual disturbance: vaginal bleeding, amenorrhoea

• hot flushes - 3% of patients stop taking tamoxifen due to climacteric side-effects

• venous thromboembolism

• endometrial cancer

Aromatase inhibitors
Anastrozole and letrozole are aromatase inhibitors that reduces peripheral oestrogen synthesis. This
is important as aromatisation accounts for the majority of oestrogen production in postmenopausal
women and therefore anastrozole is used for ER +ve breast cancer in this group.

Adverse effects

• osteoporosis

o NICE recommends a DEXA scan when initiating a patient on aromatase inhibitors for
breast cancer

• hot flushes

• arthralgia, myalgia

• insomnia

37. cute pancreatitis: management

Acute pancreatitis is a serious condition associated with significant morbidity and mortality. It is
managed in a hospital setting.

Acute pancreatitis may be stratified as follows:

Severity Organ failure Local complications

Mild No No

Moderately severe No or transient (<48 hours) Possible

Severe Persistent (>48 hours) Possible

Key aspects of care

• fluid resuscitation

o aggressive early hydration with crystalloids. In severe cases 3-6 litres of third space
fluid loss may

occur

• aim for a urine output of > 0.5mls/kg/hr

• may also help relieve pain by reducing lactic acidosis

analgesia

• pain may be severe so this is a key priority of care


• intravenous opioids are normally required to adequately control the pain

nutrition

• patients should not routinely be made 'nil-by-mouth' unless there is a clear reason e.g. the
patient is vomiting

• enteral nutrition should be offered to anyone with moderately severe or severe acute
pancreatitis within 72 hours of presentation

• parental nutrition should only be used if enteral nurition has failed or is contraindicated

role of antibiotics

• NICE state the following: 'Do not offer prophylactic antimicrobials to people with acute
pancreatitis'

• potential indications include infected pancreatic necrosis

Role of surgery

• Patients with acute pancreatitis due to gallstones should undergo early cholecystectomy

• Patients with obstructed biliary system due to stones should undergo early ERCP

• Patients who fail to settle with necrosis and have worsening organ dysfunction may require
debridement, fine needle aspiration is still used by some

• Patients with infected necrosis should undergo either radiological drainage or surgical
necrosectomy. The choice of procedure depends upon local expertise

The clinical scenario of epigastric pain, vomiting, a background of alcohol excess and raised serum
amylase are consistent with acute pancreatitis. The flank discolouration is consistent with Grey-
Turner's sign, which although rare in practice, is a common finding in exam questions and it is
classically associated with acute pancreatitis. This discolouration is caused by blood vessel auto-
digestion by pancreatic enzymes, resulting in retroperitoneal haemorrhage, hence the low
haemoglobin in this scenario. The treatment of acute pancreatitis is complex and multifactorial. At
this stage, the treatment should revolve around stabilising the patient which involves rehydration
and pain management. This makes fluids and analgesia the correct answer. Other options to
consider at this stage are enteral feeding and additional oxygen, if oxygen saturations are low.

The current guidelines on the use of antibiotics in acute pancreatitis are that they should not be
given unless evidence of infection, such as positive cultures. There is currently no evidence
suggesting that prophylactic antibiotics against pancreatic necrosis are of benefit. This position may
change.

Gallstones are the most common cause of acute pancreatitis in the UK. If gallstones were the cause
of this patient's pancreatitis, definitive treatment would involve a cholecystectomy to prevent
recurrent attacks. However, at this stage, there is no evidence that this patient has gallstones,
particularly with normal liver function tests (LFT). If this were suspected, this would need further
investigation. Furthermore, the patient should be stabilised prior to considering cholecystectomy,
and so this would not be appropriate at this stage of treatment.

There is no evidence for the use of steroids in acute pancreatitis.

Surgical pancreatic debridement is an option for managing pancreatic necrosis which is a dangerous
complication of acute pancreatitis. This complication is unlikely to have developed at this stage in
this patient based on the normal white cell count and LFTs. Furthermore, aspiration of pancreatic
fluid and imaging (most likely from CT) would be needed to diagnose this complication.

38.Epididymo-orchitis

Epididymo-orchitis describes an infection of the epididymis +/- testes resulting in pain and swelling.
It is most commonly caused by local spread of infections from the genital tract (such as Chlamydia
trachomatis and Neisseria gonorrhoeae) or the bladder.

The most important differential diagnosis is testicular torsion. This needs to be excluded urgently to
prevent ischaemia of the testicle.

Features

• unilateral testicular pain and swelling

• urethral discharge may be present, but urethritis is often asymptomatic

• factors suggesting testicular torsion include patients < 20 years, severe pain and an acute
onset

Management

• the British Association for Sexual Health and HIV (BASHH) produced guidelines in 2010

• if the organism is unknown BASHH recommend: ceftriaxone 500mg intramuscularly single


dose, plus doxycycline 100mg by mouth twice daily for 10-14 days

• further investigations following treatment are recommended to exclude any underlying


structural abnormalities.

his patient (young businessman coming back from Thailand) is at high risk of contracting sexually
transmitted infections (STI). The organisms involved are likely to be Chlamydia trachomatis and
Neisseria gonorrhoeae.
BASHH guidelines recommend that in a suspected case of epididymo-orchitis, if an unknown
organism, empirical treatments are ceftriaxone 500mg intramuscularly single dose, plus oral
doxycycline 100mg twice daily for 10-14 days.

Ciprofloxacin 500mg twice daily for 10-14 days can be used as empirical treatments if the clinical
scenario suggests gram-negative organisms, as in the case of an older man with low risk of STI. In
this scenario, an STI is more likely.

Amoxicillin, metronidazole and clarithromycin are not suitable antibiotics to cover the possible
organisms and are not recommended in the guidelines.

39.Lower urinary tract symptoms in men

Lower urinary tract symptoms (LUTS) in men are very common and are present in the majority of
men aged > 50 years. They are most commonly secondary to benign prostatic hyperplasia but other
causes should be considered including prostate cancer.

It is useful to classify the symptoms into 3 broad groups.

Voiding Storage Post-micturition

Hesitancy Urgency Post-micturition dribbling


Poor or intermittent stream Frequency Sensation of incomplete emptying
Straining Nocturia
Incomplete emptying Urinary incontinence
Terminal dribbling

Examination

• urinalysis: exclude infection, check for haematuria

• digital rectal examination: size and consistency of prostate

• a PSA test may be indicated, but the patient should be properly counselled first

It is useful to get the patient to complete the following to guide management:

• urinary frequency-volume chart: distinguish between urinary frequency, polyuria, nocturia,


and nocturnal polyuria.
• International Prostate Symptom Score (IPSS): assess the impact on the patient's life. This
classifies the symptoms as mild, moderate or severe

Management

Predominately voiding symptoms

• conservative measures include: pelvic floor muscle training, bladder training, prudent fluid
intake and containment products

• if 'moderate' or 'severe' symptoms offer an alpha-blocker

• if the prostate is enlarged and the patient is 'considered at high risk of progression' then a 5-
alpha reductase inhibitor should be offered

• if the patient has an enlarged prostate and 'moderate' or 'severe' symptoms offer both an
alpha-blocker and 5-alpha reductase inhibitor

• if there are mixed symptoms of voiding and storage not responding to an alpha blocker then
a antimuscarinic (anticholinergic) drug may be added

Predominately overactive bladder

• conservative measures include moderating fluid intake

• bladder retraining should be offered

• antimuscarinic drugs should be offered if symptoms persist. NICE recommend oxybutynin


(immediate release), tolterodine (immediate release), or darifenacin (once daily preparation)

• mirabegron may be considered if first-line drugs fail

Nocturia

• advise about moderating fluid intake at night

• furosemide 40mg in late afternoon may be considered

• desmopressin may also be helpful

40. Inguinal hernia

Inguinal hernias account for 75% of abdominal wall hernias. Around 95% of patients are male; men
have around a 25% lifetime risk of developing an inguinal hernia.
Features

• groin lump: disappears on pressure or when the patient lies down

• discomfort and ache: often worse with activity, severe pain is uncommon

• strangulation is rare

Whilst traditional textbooks describe the anatomical differences between indirect (hernia through
the inguinal canal) and direct hernias (through the posterior wall of the inguinal canal) this is of no
relevance to the clinical management.

Management

• the clinical consensus is currently to treat medically fit patients even if they are
asymptomatic

• a hernia truss may be an option for patients not fit for surgery but probably has little role in
other patients

• mesh repair is associated with the lowest recurrence rate

The Department for Work and Pensions recommend that following an open repair patients return to
non-manual work after 2-3 weeks and following laparoscopic repair after 1-2 weeks

Complications

• early: bruising, wound infection

• late: chronic pain, recurrence

41.Colorectal cancer: referral guidelines

NICE updated their referral guidelines in 2015. The following patients should be referred urgently
(i.e. within 2 weeks) to colorectal services for investigation:

• patients >= 40 years with unexplained weight loss AND abdominal pain

• patients >= 50 years with unexplained rectal bleeding

• patients >= 60 years with iron deficiency anaemia OR change in bowel habit

• tests show occult blood in their faeces (see below)


An urgent referral (within 2 weeks) should be 'considered' if:

• there is a rectal or abdominal mass

• there is an unexplained anal mass or anal ulceration

• patients < 50 years with rectal bleeding AND any of the following unexplained
symptoms/findings:

• -→ abdominal pain

• -→ change in bowel habit

• -→ weight loss

• -→ iron deficiency anaemia

Faecal Occult Blood Testing (FOBT)

This was one of the main changes in 2015. Remember that the NHS now has a national screening
programme offering screening every 2 years to all men and women aged 60 to 74 years. Patients
aged over 74 years may request screening.

In addition FOBT should be offered to:

• patients >= 50 years with unexplained abdominal pain OR weight loss

• patients < 60 years with changes in their bowel habit OR iron deficiency anaemia

• patients >= 60 years who have anaemia even in the absence of iron deficiency

The signs and symptoms are suggestive of anaemia, and the blood tests indicate a microcytic
anaemia with low ferritin - therefore iron deficiency anaemia. Previous results were normal, thus
this is a new development.

Under NICE guidelines, any patient over 60-years-old with new-onset iron deficiency anaemia should
be referred urgently (2-week wait) to the colorectal cancer referral pathway.

A - correct answer.

B - faecal occult blood testing is used as a screening tool, from ages 60-74 years-old or in cases less
strongly suggestive of colorectal cancer. This test, however, has started to be replaced by the faecal
immunochemical test (FIT) - proposed to provide greater sensitivity and specificity than the existing
test.
C - the findings in this patient are highly suspicious for colorectal cancer, therefore an urgent, 2-
week referral is indicated, as opposed to a routine, 6-week wait.

D - oral supplementation may be indicated for alleviation of issues caused by the anaemia. However,
the underlying cause may be colorectal cancer which is vital to investigate further, therefore this
answer in isolation is not sufficient.

E - likewise, lifestyle advice would be pertinent but does not investigate the underlying, potentially
sinister, cause and therefore this answer is not sufficient in isolation.

42.Colorectal cancer treatment

Patients diagnosed as having colorectal cancer should be completely staged using CT of the chest/
abdomen and pelvis. Their entire colon should have been evaluated with colonoscopy or CT
colonography. Patients whose tumours lie below the peritoneal reflection should have their
mesorectum evaluated with MRI.

Once their staging is complete patients should be discussed within a dedicated colorectal MDT
meeting and a treatment plan formulated.

Treatment of colonic cancer

Cancer of the colon is nearly always treated with surgery. Stents, surgical bypass and diversion
stomas may all be used as palliative adjuncts. Resectional surgery is the only option for cure in
patients with colon cancer. The procedure is tailored to the patient and the tumour location. The
lymphatic drainage of the colon follows the arterial supply and therefore most resections are
tailored around the resection of particular lymphatic chains (e.g. ileo-colic pedicle for right sided
tumours). Some patients may have confounding factors that will govern the choice of procedure, for
example a tumour in a patient from a HNPCC family may be better served with a
panproctocolectomy rather than segmental resection.

Following resection the decision has to be made regarding restoration of continuity. For an
anastomosis to heal the key technical factors include; adequate blood supply, mucosal apposition
and no tissue tension. Surrounding sepsis, unstable patients and inexperienced surgeons may
compromise these key principles and in such circumstances it may be safer to construct an end
stoma rather than attempting an anastomosis.

When a colonic cancer presents with an obstructing lesion; the options are to either stent it or
resect. In modern practice it is unusual to simply defunction a colonic tumour with a proximal loop
stoma. This differs from the situation in the rectum (see below).
Following resection patients with risk factors for disease recurrence are usually offered
chemotherapy, a combination of 5FU and oxaliplatin is common.

Treatment of rectal cancer

The management of rectal cancer is slightly different to that of colonic cancer. This reflects the
rectum's anatomical location and the challenges posed as a result. Tumours located in the rectum
can be surgically resected with either an anterior resection or an abdomino-perineal excision of
rectum (APER). The technical aspects governing the choice between these two procedures can be
complex to appreciate and the main point to appreciate for the exam is that involvement of the
sphincter complex or very low tumours require APER. In the rectum a 2cm distal clearance margin is
required and this may also impact on the procedure chosen. In addition to excision of the rectal tube
an integral part of the procedure is a meticulous dissection of the mesorectal fat and lymph nodes
(total mesorectal excision/ TME).

In rectal cancer surgery involvement of the cirumferential resection margin carries a high risk of
disease recurrence. Because the rectum is an extraperitoneal structure (until you remove it that is!)
it is possible to irradiate it, something which cannot be offered for colonic tumours. This has a major
impact in rectal cancer treatment and many patients will be offered neoadjuvent radiotherapy (both
long and short course) prior to resectional surgery. Patients with T1 and 2 /N0 disease on imaging do
not require irradiation and should proceed straight to surgery. Patients with T4 disease will typically
have long course chemo radiotherapy. Those with T3 , N0 tumours may be offered short course
radiotherapy prior to surgery. Patients presenting with large bowel obstruction from rectal cancer
should not undergo resectional surgery without staging as primary treatment (very different from
colonic cancer). This is because rectal surgery is more technically demanding, the anastomotic leak
rate is higher and the danger of a positive resection margin in an unstaged patient is high. Therefore
patients with obstructing rectal cancer should have a defunctioning loop colostomy.

Summary of procedures

The operations for cancer are segmental resections based on blood supply and lymphatic drainage.

Site of cancer Type of resection Anastomosis

Caecal, ascending or proximal transverse Right hemicolectomy Ileo-colic


colon

Distal transverse, descending colon Left hemicolectomy Colo-colon

Sigmoid colon High anterior resection Colo-rectal

Upper rectum Anterior resection (TME) Colo-rectal


Site of cancer Type of resection Anastomosis

Low rectum Anterior resection (Low TME) Colo-rectal


(+/- Defunctioning
stoma)

Anal verge Abdomino-perineal excision of None


rectum

In the emergency setting where the bowel has perforated the risk of an anastomosis is much
greater, particularly when the anastomosis is colon-colon. In this situation an end colostomy is often
safer and can be reversed later. When resection of the sigmoid colon is performed and an end
colostomy is fashioned the operation is referred to as a Hartmans procedure. Whilst left sided
resections are more risky, ileo-colic anastomoses are relatively safe even in the emergency setting
and do not need to be defunctioned.

References

A review of the diagnosis and management of colorectal cancer and a summary of the UK National
Institute of Clinical Excellence guidelines is provided in:
Poston G, et al . Diagnosis and management of colorectal cancer:summary of NICE
guidance. BMJ 2011: 343: d 6751.

43.Epididymo-orchitis

Epididymo-orchitis describes an infection of the epididymis +/- testes resulting in pain and swelling.
It is most commonly caused by local spread of infections from the genital tract (such as Chlamydia
trachomatis and Neisseria gonorrhoeae) or the bladder.

The most important differential diagnosis is testicular torsion. This needs to be excluded urgently to
prevent ischaemia of the testicle.

Features

• unilateral testicular pain and swelling

• urethral discharge may be present, but urethritis is often asymptomatic

• factors suggesting testicular torsion include patients < 20 years, severe pain and an acute
onset
Management

• the British Association for Sexual Health and HIV (BASHH) produced guidelines in 2010

• if the organism is unknown BASHH recommend: ceftriaxone 500mg intramuscularly single


dose, plus doxycycline 100mg by mouth twice daily for 10-14 days

• further investigations following treatment are recommended to exclude any underlying


structural abnormalities

44. Head injury: types of traumatic brain injury

Basics

• primary brain injury may be focal (contusion/haematoma) or diffuse (diffuse axonal injury)

• diffuse axonal injury occurs as a result of mechanical shearing following deceleration,


causing disruption and tearing of axons

• intra-cranial haematomas can be extradural, subdural or intracerebral, while contusions may


occur adjacent to (coup) or contralateral (contre-coup) to the side of impact

• secondary brain injury occurs when cerebral oedema, ischaemia, infection, tonsillar or
tentorial herniation exacerbates the original injury. The normal cerebral auto regulatory
processes are disrupted following trauma rendering the brain more susceptible to blood
flow changes and hypoxia

• the Cushings reflex (hypertension and bradycardia) often occurs late and is usually a pre
terminal event

Type of injury Notes

Extradural Bleeding into the space between the dura mater and the skull. Often results from
(epidural) acceleration-deceleration trauma or a blow to the side of the head. The majority of
haematoma epidural haematomas occur in the temporal region where skull fractures cause a
rupture of the middle meningeal artery.

Features

• features of raised intracranial pressure

• some patients may exhibit a lucid interval

Subdural Bleeding into the outermost meningeal layer. Most commonly occur around the
haematoma frontal and parietal lobes.

Risk factors include old age, alcoholism and anticoagulation.


Type of injury Notes

Slower onset of symptoms than a epidural haematoma. There may be fluctuating


confusion/consciousness

Subarachnoid Classically causes a sudden occipital headache. Usually occurs spontaneously in the
haemorrhage context of a ruptured cerebral aneurysm but may be seen in association with other
injuries when a patient has sustained a traumatic brain injury

Intracerebral An intracerebral (or intraparenchymal) haemorrhage is a collection of blood within the


haematoma substance of the brain.

Causes / risk factors include: hypertension, vascular lesion (e.g. aneurysm or


arteriovenous malformation), cerebral amyloid angiopathy, trauma, brain tumour or
infarct (particularly in stroke patients undergoing thrombolysis).

Patients will present similarly to an ischaemic stroke (which is why it is crucial to


obtain a CT in head in all stroke patients prior to thrombolysis) or with a decrease in
consciousness.

CT imaging will show a hyperdensity (bright lesion) within the substance of the brain.

Treatment is often conservative under the care of stroke physicians, but large clots in
patients with impaired consciousness may warrant surgical evacuation.

45.Abdominal pain

The table below gives characteristic exam question features for conditions causing abdominal pain.
Unusual and 'medical' causes of abdominal pain should also be remembered:

• acute coronary syndrome

• diabetic ketoacidosis

• pneumonia

• acute intermittent porphyria

• lead poisoning

Condition Characteristic exam feature

Peptic ulcer disease Duodenal ulcers: more common than gastric ulcers, epigastric pain relieved by eating
Gastric ulcers: epigastric pain worsened by eating
Condition Characteristic exam feature

Features of upper gastrointestinal haemorrhage may be seen (haematemesis,


melena etc)

Appendicitis Pain initial in the central abdomen before localising to the right iliac fossa
Anorexia is common
Tachycardia, low-grade pyrexia, tenderness in RIF
Rovsing's sign: more pain in RIF than LIF when palpating LIF

Acute pancreatitis Usually due to alcohol or gallstones


Severe epigastric pain
Vomiting is common
Examination may reveal tenderness, ileus and low-grade fever
Periumbilical discolouration (Cullen's sign) and flank discolouration (Grey-Turner's
sign) is described but rare

Biliary colic Pain in the RUQ radiating to the back and interscapular region, may be following a
fatty meal. Slight misnomer as the pain may persist for hours
Obstructive jaundice may cause pale stools and dark urine
It is sometimes taught that patients are female, forties, fat and fair although this is
obviously a generalisation

Acute cholecystitis History of gallstones symptoms (see above)


Continuous RUQ pain
Fever, raised inflammatory markers and white cells
Murphy's sign positive (arrest of inspiration on palpation of the RUQ)

Diverticulitis Colicky pain typically in the LLQ


Fever, raised inflammatory markers and white cells

Abdominal aortic Severe central abdominal pain radiating to the back


aneurysm Presentation may be catastrophic (e.g. Sudden collapse) or sub-acute (persistent
severe central abdominal pain with developing shock)
Patients may have a history of cardiovascular disease

Intestinal History of malignancy/previous operations


obstruction Vomiting
Not opened bowels recently
'Tinkling' bowel sounds
Diagram showing stereotypical areas where particular conditions present. The diagram is not
exhaustive and only lists the more common conditions seen in clinical practice. Note how pain from
renal causes such as renal/ureteric colic and pyelonephritis may radiate and move from the loins
towards the suprapubic area.

46.Head injury

Patients who suffer head injuries should be managed according to ATLS principles and extracranial
injuries should be managed alongside cranial trauma. An inadequate cardiac output will compromise
CNS perfusion irrespective of the nature of the cranial injury.

Types of traumatic brain injury

Extradural Bleeding into the space between the dura mater and the skull. Often results from
haematoma acceleration-deceleration trauma or a blow to the side of the head. The majority of
extradural haematomas occur in the temporal region where skull fractures cause a
rupture of the middle meningeal artery.

Features

• Raised intracranial pressure

• Some patients may exhibit a lucid interval


Subdural Bleeding into the outermost meningeal layer. Most commonly occur around the frontal
haematoma and parietal lobes. May be either acute or chronic.

Risk factors include old age and alcoholism.

Slower onset of symptoms than a extradural haematoma.

Subarachnoid Usually occurs spontaneously in the context of a ruptured cerebral aneurysm, but may
haemorrhage be seen in association with other injuries when a patient has sustained a traumatic
brain injury.

Pathophysiology

• Primary brain injury may be focal (contusion/ haematoma) or diffuse (diffuse axonal injury)

• Diffuse axonal injury occurs as a result of mechanical shearing following deceleration,


causing disruption and tearing of axons

• Intra-cranial haematomas can be extradural, subdural or intracerebral, while contusions may


occur adjacent to (coup) or contralateral (contre-coup) to the side of impact

• Secondary brain injury occurs when cerebral oedema, ischaemia, infection, tonsillar or
tentorial herniation exacerbates the original injury. The normal cerebral auto regulatory
processes are disrupted following trauma rendering the brain more susceptible to blood
flow changes and hypoxia

• The Cushings reflex (hypertension and bradycardia) often occurs late and is usually a pre
terminal event

Management

• Where there is life threatening rising ICP such as in extradural haematoma and whilst
theatre is prepared or transfer arranged use of IV mannitol/ frusemide may be required.

• Diffuse cerebral oedema may require decompressive craniotomy

• Exploratory Burr Holes have little management in modern practice except where scanning
may be unavailable and to thus facilitate creation of formal craniotomy flap

• Depressed skull fractures that are open require formal surgical reduction and debridement,
closed injuries may be managed nonoperatively if there is minimal displacement.

• ICP monitoring is appropriate in those who have GCS 3-8 and normal CT scan.

• ICP monitoring is mandatory in those who have GCS 3-8 and abnormal CT scan.

• Hyponatraemia is most likely to be due to the syndrome of inappropriate ADH secretion.

• Minimum of cerebral perfusion pressure of 70mmHg in adults.

• Minimum cerebral perfusion pressure of between 40 and 70 mmHg in children.


Interpretation of pupillary findings in head injuries

Pupil size Light response Interpretation

Unilaterally dilated Sluggish or fixed 3rd nerve compression secondary to tentorial


herniation

Bilaterally dilated Sluggish or fixed • Poor CNS perfusion

• Bilateral 3rd nerve palsy

Unilaterally dilated or Cross reactive (Marcus - Optic nerve injury


equal Gunn)

Bilaterally constricted May be difficult to assess • Opiates

• Pontine lesions

• Metabolic encephalopathy

Unilaterally constricted Preserved Sympathetic pathway disruption

47.Subarachnoid haemorrhage

A subarachnoid haemorrhage (SAH) is an intracranial haemorrhage that is defined as the presence of


blood within the subarachnoid space, i.e. deep to the subarachnoid layer of the meninges.

The most common cause of SAH is head injury and this is called traumatic SAH . In the absence of
trauma, SAH is termed spontaneous SAH . The rest of this note focuses on spontaneous SAH.

Causes of spontaneous SAH include:

• Intracranial aneurysm* (saccular ‘berry’ aneurysms): this accounts for around 85% of cases.
Conditions associated with berry aneurysms include adult polycystic kidney disease, Ehlers-
Danlos syndrome and coarctation of the aorta

• Arteriovenous malformation

• Pituitary apoplexy

• Arterial dissection

• Mycotic (infective) aneurysms

• Perimesencephalic (an idiopathic venous bleed)


Classical presenting features include:

• Headache: typically sudden-onset (‘thunderclap’ or ‘baseball bat’), severe (‘worst of my life’)


and occipital

• Nausea and vomiting

• Meningism (photophobia, neck stiffness)

• Coma

• Seizures

• Sudden death

• ECG changes including ST elevation may be seen

Confirmation of SAH:

• Computed tomography (CT) head

o Acute blood (hyperdense/bright on CT) is typically distributed in the basal cisterns,


sulci and in severe cases the ventricular system.

o CT is negative for SAH (no blood seen) in 7% of cases.

• Lumbar puncture (LP)

o Used to confirm SAH if CT is negative.

o LP is performed at least 12 hours following the onset of symptoms to allow the


development of xanthochromia (the result of red blood cell breakdown).

o Xanthochromia helps to distinguish true SAH from a ‘traumatic tap’ (blood


introduced by the LP procedure).

o As well as xanthochromia, CSF findings consistent with subarachnoid haemorrhage


include a normal or raised opening pressure

• Referral to neurosurgery to be made as soon as SAH is confirmed

After spontaneous SAH is confirmed, the aim of investigation is to identify a causative pathology that
needs urgent treatment:

• CT intracranial angiogram (to identify a vascular lesion e.g. aneurysm or AVM)

• +/- digital subtraction angiogram (catheter angiogram)

Treatment
• The treatment in spontaneous SAH is in accordance with the causative pathology

• Intracranial aneurysms are at risk of rebleeding and therefore require prompt intervention,
preferably within 24 hours

• Most intracranial aneurysms are now treated with a coil by interventional neuroradiologists,
but a minority require a craniotomy and clipping by a neurosurgeon

• Until the aneurysm is treated, the patient should be kept on strict bed rest, well-controlled
blood pressure and should avoid straining in order to prevent a re-bleed of the aneurysm

• Vasospasm is prevented using a 21-day course of nimodipine (a calcium channel inhibitor


targeting the brain vasculature) and treated with hypervolaemia, induced-hypertension and
haemodilution**

• Hydrocephalus is temporarily treated with an external ventricular drain (CSF diverted into a
bag at the bedside) or, if required, a long-term ventriculo-peritoneal shunt

Complications of aneurysmal SAH:

• Re-bleeding

o happens in around 10% of cases and most common in the first 12 hours

o if rebleeding is suspected (e.g. sudden worsening of neurological symptoms) then


a repeat CT should be arranged

o associated with a high mortality (up to 70%)

• Vasospasm (also termed delayed cerebral ischaemia), typically 7-14 days after onset

• Hyponatraemia (most typically due to syndrome inappropriate anti-diuretic hormone


(SIADH))

• Seizures

• Hydrocephalus

• Death

Important predictive factors in SAH:

• conscious level on admission

• age

• amount of blood visible on CT head


© Image used on license from Radiopaedia

CT image shows diffuse subarachnoid haemorrhage in all basal cisterns, bilateral sylvian fissures and
the inter-hemispheric fissure. This case demonstrates the typical distribution that takes the blood
into the subarachnoid space in a subarachnoid hemorrhage.

*this may be secondary to either autonomic neural stimulation from the hypothalamus or elevated
levels of circulating catecholamines

**the way nimodipine works in subarachnoid haemorrhage is not fully understood. It has been
previously postulated that it reduces cerebral vasospasm (hence maintaining cerebral perfusion) but
this has not been demonstrated in studies

48. Chronic pancreatitis

Chronic pancreatitis is an inflammatory condition which can ultimately affect both the exocrine and
endocrine functions of the pancreas. Around 80% of cases are due to alcohol excess with up to 20%
of cases being unexplained.
Other than alcohol, causes include:

• genetic: cystic fibrosis, haemochromatosis

• ductal obstruction: tumours, stones, structural abnormalities including pancreas divisum and
annular pancreas

Features

• pain is typically worse 15 to 30 minutes following a meal

• steatorrhoea: symptoms of pancreatic insufficiency usually develop between 5 and 25 years


after the onset of pain

• diabetes mellitus develops in the majority of patients. It typically occurs more than 20
years after symptom begin

Investigation

• abdominal x-ray shows pancreatic calcification in 30% of cases

• CT is more sensitive at detecting pancreatic calcification. Sensitivity is 80%, specificity is 85%

• functional tests: faecal elastase may be used to assess exocrine function if imaging
inconclusive

Management

• pancreatic enzyme supplements

• analgesia

• antioxidants: limited evidence base - one study suggests benefit in early disease.

ExplanThis patient is likely suffering from chronic pancreatitis due to his long-term
alcohol consumption. Chronic pancreatitis can cause a number of complications,
including diabetes (which this patient has been diagnosed with recently). As such, it is
important to assess the function of the pancreas. Faecal elastase is the test used to assess
the exocrine function of the pancreas.

Faecal calprotectin is used in the diagnosis of inflammatory bowel diseases such as


Crohn's disease. It has no role in the assessment of chronic pancreatitis.

Serum amylase is used in the assessment of acute pancreatitis - a level more than 3 times
the upper limit of normal is suggestive of acute disease. Patients with chronic pancreatitis
may have a normal amylase due to loss of pancreatic function.
Serum calcium is not used in assessing the function of the pancreas in chronic
pancreatitis. It is used as part of the Glasgow score in acute pancreatitis, in which low
calcium is one of the scoring criteria.

Lipase is one of the hormones that a patient may be deficient in if they have chronic
pancreatitis, leading to loose, greasy stools (steatorrhoea). It is not normally tested to
assess exocrine function.
• DiscussImprove

49. Anti-oestrogen drugs

Selective oEstrogen Receptor Modulators (SERM)

Tamoxifen is a SERM which acts as an oestrogen receptor antagonist and partial agonist. It is
used in the management of oestrogen receptor-positive breast cancer.

Adverse effects

• menstrual disturbance: vaginal bleeding, amenorrhoea


• hot flushes - 3% of patients stop taking tamoxifen due to climacteric side-effects
• venous thromboembolism
• endometrial cancer

Aromatase inhibitors

Anastrozole and letrozole are aromatase inhibitors that reduces peripheral oestrogen
synthesis. This is important as aromatisation accounts for the majority of oestrogen
production in postmenopausal women and therefore anastrozole is used for ER +ve breast
cancer in this group.

Adverse effects

• osteoporosis
o NICE recommends a DEXA scan when initiating a patient on aromatase
inhibitors for breast cancer
• hot flushes
• arthralgia, myalgia
• insomnia

Explanation: Anastrozole is an aromatase inhibitor used for the treatment of oestrogen receptor-
positive breast cancer in postmenopausal women, such as this patient. Aromatase inhibitors reduce
peripheral oestrogen synthesis, which accounts for the majority of oestrogen production in
postmenopausal women. Patients taking these medications are at an increased risk of osteoporosis.

Amenorrhoea, endometrial cancer, vaginal bleeding and venous thromboembolism are all potential
side effects of selective oestrogen receptor modulators (SERM) such as tamoxifen. Tamoxifen is used
to treat oestrogen receptor-positive breast cancer in pre-menopausal women.

50.
RHEUMATOLOGY
1.Ankylosing spondylitis: features

Ankylosing spondylitis is a HLA-B27 associated spondyloarthropathy (HLA-B27 is


positive in 90% of patients). It typically presents in males (sex ratio 3:1) aged 20-30
years old.

• typically a young man who presents with lower back pain and stiffness of
insidious onset
• stiffness is usually worse in the morning and improves with exercise
• the patient may experience pain at night which improves on getting up

Clinical examination

• reduced lateral flexion of the lumbar spine


• reduced forward flexion - Schober's test - a line is drawn 10 cm above and 5 cm
below the back dimples (dimples of Venus). The distance between the two lines
should increase by more than 5 cm when the patient bends as far forward as
possible
• reduced chest expansion

Other features - the 'A's

• Apical fibrosis
• Anterior uveitis
• Aortic regurgitation
• Achilles tendonitis
• AV node block
• Amyloidosis
• and cauda equina syndrome
• peripheral arthritis (25%, more common if female)

Ankylosing spondylitis: investigation and management

Ankylosing spondylitis is a HLA-B27 associated spondyloarthropathy. It typically


presents in males (sex ratio 3:1) aged 20-30 years old.

Investigation

Inflammatory markers (ESR, CRP) are typically raised although normal levels do
not exclude ankylosing spondylitis.
HLA-B27 is of little use in making the diagnosis as it is positive in:

• 90% of patients with ankylosing spondylitis


• 10% of normal patients

Plain x-ray of the sacroiliac joints is the most useful investigation in establishing
the diagnosis. Radiographs may be normal early in disease, later changes include:

• sacroiliitis: subchondral erosions, sclerosis


• squaring of lumbar vertebrae
• 'bamboo spine' (late & uncommon)
• syndesmophytes: due to ossification of outer fibers of annulus fibrosus
• chest x-ray: apical fibrosis

X ray image shows the typical appearance of bamboo spine with a single central
radiodense line related to ossification of supraspinous and interspinous ligaments
which is called dagger sign. Ankylosing is detectable in both sacroiliac joints.

If the x-ray is negative for sacroiliac joint involvement in ankylosing spondylitis


but suspicion for AS remains high, the next step in the evaluation should be
obtaining an MRI. Signs of early inflammation involving sacroiliac joints (bone
marrow oedema) confirm the diagnosis of AS and prompt further treatment.

Spirometry may show a restrictive defect due to a combination of pulmonary fibrosis,


kyphosis and ankylosis of the costovertebral joints.

Management

The following is partly based on the 2010 EULAR guidelines (please see the link for
more details):

• encourage regular exercise such as swimming


• NSAIDs are the first-line treatment
• physiotherapy
• the disease-modifying drugs which are used to treat rheumatoid arthritis
(such as sulphasalazine) are only really useful if there is peripheral joint
involvement
• the 2010 EULAR guidelines suggest: 'Anti-TNF therapy should be given to
patients with persistently high disease activity despite conventional treatments'
• research is ongoing to see whether anti-TNF therapies such as etanercept and
adalimumab should be used earlier in the course of the disease
2. Gout: drug causes

Gout is a form of microcrystal synovitis caused by the deposition of monosodium


urate monohydrate in the synovium. It is caused by chronic hyperuricaemia (uric acid
> 0.45 mmol/l).

Drug causes

• diuretics: thiazides, furosemide


• ciclosporin
• alcohol
• cytotoxic agents
• pyrazinamide
• aspirin: it was previously thought that only high-dose aspirin could precipitate
gout. However, a systematic review (see link) showed that low-dose (e.g. 75mg)
also increases the risk of gout attacks. This obviously needs to be balanced
against the cardiovascular benefits of aspirin and the study showed patients
coprescribed allopurinol were not at an increased risk

3. Gout: management

Gout is a form of microcrystal synovitis caused by the deposition of monosodium


urate monohydrate in the synovium. It is caused by chronic hyperuricaemia (uric acid
> 450 µmol/l)

Acute management

• NSAIDs or colchicine are first-line


• the maximum dose of NSAID should be prescribed until 1-2 days after the
symptoms have settled. Gastroprotection (e.g. a proton pump inhibitor) may
also be indicated
• colchicine* has a slower onset of action. The main side-effect is diarrhoea
• oral steroids may be considered if NSAIDs and colchicine are
contraindicated. A dose of prednisolone 15mg/day is usually used
• another option is intra-articular steroid injection
• if the patient is already taking allopurinol it should be continued

*** NSAIDs should be avoided in elderly patients taking warfarin due to the risk
of a life-threatening gastrointestinal haemorrhage. Oral steroids are an option but
would upset his diabetic control.
*** Diclofenac and indomethacin are contraindicated because of his duodenal
ulcer. Colchicine is a suitable alternative. Allopurinol should not be given in the acute
phase, but is good for preventing recurrent attacks.

Indications for urate-lowering therapy (ULT)

• the British Society of Rheumatology Guidelines now advocate offering urate-


lowering therapy to all patients after their first attack of gout
• ULT is particularly recommended if:
o >= 2 attacks in 12 months
o tophi
o renal disease
o uric acid renal stones
o prophylaxis if on cytotoxics or diuretics

Urate-lowering therapy

• allopurinol is first-line
• it has traditionally been taught that urate-lowering therapy should not be
started until 2 weeks after an acute attack, as starting too early may precipitate a
further attack. The evidence base to support this however looks weak
• in 2017 the BSR updated their guidelines. They still support a delay in starting
urate-lowering therapy because it is better for a patient to make long-term drug
decisions whilst not in pain
o the key passage is: 'Commencement of ULT is best delayed until
inflammation has settled as ULT is better discussed when the patient
is not in pain'
o In circumstances where attacks are so frequent that this is not
possible, the initiation of allopurinol can be considered before
inflammation has completely settled.
• initial dose of 100 mg od, with the dose titrated every few weeks to aim for a
serum uric acid of < 300 µmol/l. Lower initial doses should be given if the
patient has a reduced eGFR
• colchicine cover should be considered when starting allopurinol. NSAIDs
can be used if colchicine cannot be tolerated. The BSR guidelines suggest
this may need to be continued for 6 months
• the second-line agent when allopurinol is not tolerated or ineffective is
febuxostat (also a xanthine oxidase inhibitor)
• in refractory cases other agents may be tried:
o uricase (urate oxidase) is an enzyme that catalyzes the conversion of
urate to the degradation product allantoin. It is present in certain
mammals but not humans
o in patients who have persistent symptomatic and severe gout despite the
adequate use of urate-lowering therapy, pegloticase (polyethylene glycol
modified mammalian uricase) can achieve rapid control of hyperuricemia.
It is given as an infusion once every two weeks

Lifestyle modifications

• reduce alcohol intake and avoid during an acute attack


• lose weight if obese
• avoid food high in purines e.g. Liver, kidneys, seafood, oily fish (mackerel,
sardines) and yeast products

Other points

• consideration should be given to stopping precipitating drugs (such as


thiazides)
• losartan has a specific uricosuric action and may be particularly suitable for the
many patients who have coexistent hypertension
• increased vitamin C intake (either supplements or through normal diet) may also
decrease serum uric acid levels

*inhibits microtubule polymerization by binding to tubulin, interfering with mitosis.


Also inhibits neutrophil motility and activity

4. Septic arthritis in adults

• most common organism overall is Staphylococcus aureus


o in young adults who are sexually active, Neisseria gonorrhoeae is the most
common organism (disseminated gonococcal infection)
• the most common cause is hematogenous spread
o this may be from distant bacterial infections e.g. abscesses
• in adults, the most common location is the knee

Features
• acute, swollen joint
o restricted movement in 80% of patients
o examination findings: warm to touch/fluctuant
• fever: present in the majority of patients
Investigations
• synovial fluid sampling is obligatory
o this should be done prior to the administration of antibiotics if necessary
o may need to be done under radiographic guidance
• blood cultures: the most common cause of septic arthritis is hematogenous
spread
• joint imaging
Management

• intravenous antibiotics which cover Gram-positive cocci are indicated. The BNF
currently recommends flucloxacillin or clindamycin if penicillin allergic
o antibiotic treatment is normally be given for several weeks (BNF states 6-
12 weeks)
• needle aspiration should be used to decompress the joint
• arthroscopic lavage may be required

5. Hypersensitivity

The Gell and Coombs classification divides hypersensitivity traditionally divides


reactions into 4 types:

Type Mechanism Examples


Type I - Antigen reacts with IgE • Anaphylaxis
Anaphylactic bound to mast cells • Atopy (e.g. asthma, eczema and hayfever)
• Autoimmune haemolytic anaemia
• ITP
• Goodpasture's syndrome
Type II – IgG or IgM binds to
• Pernicious anaemia
Cell bound antigen on cell surface
• Acute haemolytic transfusion reactions
• Rheumatic fever
• Pemphigus vulgaris / bullous pemphigoid
• Serum sickness
Type III - Free antigen and • Systemic lupus erythematosus
Immune antibody (IgG, IgA) • Post-streptococcal glomerulonephritis
complex combine • Extrinsic allergic alveolitis (especially
acute phase)
• Tuberculosis / tuberculin skin reaction
• Graft versus host disease
• Allergic contact dermatitis
Type IV -
• Scabies
Delayed T-cell mediated
• Extrinsic allergic alveolitis (especially
hypersensitivity
chronic phase)
• Multiple sclerosis
• Guillain-Barre syndrome
In recent times a further category has been added:

Type Mechanism Examples


Antibodies that recognise and bind to the cell surface • Graves' disease
Type V receptors. • Myasthenia
This either stimulating them or blocking ligand binding gravis

6. Osteoporosis: causes

Advancing age and female sex are significant risk factors for osteoporosis. Prevalence
of osteoporosis increases from 2% at 50 years to more than 25% at 80 years in
women.

There are many other risk factors and secondary causes of osteoporosis. We'll start
by looking at the most 'important' ones - these are risk factors that are used by
major risk assessment tools such as FRAX:

• history of glucocorticoid use


• rheumatoid arthritis
• alcohol excess
• history of parental hip fracture
• low body mass index
• current smoking

Other risk factors

• sedentary lifestyle
• premature menopause
• Caucasians and Asians
• endocrine disorders: hyperthyroidism, hypogonadism (e.g. Turner's, testosterone
deficiency), growth hormone deficiency, hyperparathyroidism, diabetes mellitus
• multiple myeloma, lymphoma
• gastrointestinal disorders: inflammatory bowel disease, malabsorption (e.g.
Coeliac's), gastrectomy, liver disease
• chronic kidney disease
• osteogenesis imperfecta, homocystinuria

Medications that may worsen osteoporosis (other than glucocorticoids):


• SSRIs
• antiepileptics
• proton pump inhibitors
• glitazones
• long term heparin therapy
• aromatase inhibitors e.g. anastrozole

Investigations for secondary causes

If a patient is diagnosed with osteoporosis or has a fragility fracture further


investigations may be warranted. NOGG recommend testing for the following
reasons:

• exclude diseases that mimic osteoporosis (e.g. osteomalacia, myeloma);


• identify the cause of osteoporosis and contributory factors;
• assess the risk of subsequent fractures;
• select the most appropriate form of treatment

The following investigations are recommended by NOGG:

• History and physical examination


• Blood cell count, sedimentation rate or C-reactive protein, serum calcium,

albumin, creatinine, phosphate, alkaline phosphatase and liver transaminases

• Thyroid function tests


• Bone densitometry ( DXA)

Other procedures, if indicated

• Lateral radiographs of lumbar and thoracic spine/DXA-based vertebral imaging


• Protein immunoelectrophoresis and urinary Bence-Jones proteins
• 25OHD
• PTH
• Serum testosterone, SHBG, FSH, LH (in men),
• Serum prolactin
• 24 hour urinary cortisol/dexamethasone suppression test
• Endomysial and/or tissue transglutaminase antibodies (coeliac disease)
• Isotope bone scan
• Markers of bone turnover, when available
• Urinary calcium excretion
So from the first list we should order the following bloods as a minimum for all
patients:

• full blood count


• urea and electrolytes
• liver function tests
• bone profile
• CRP
• thyroid function tests

Osteoporosis: glucocorticoid-induced

We know that one of the most important risk factors for osteoporosis is the use of
corticosteroids. As these drugs are so widely used in clinical practice it is important
we manage this risk appropriately.

The most widely followed guidelines are based around the 2002 Royal College of
Physicians (RCP) 'Glucocorticoid-induced osteoporosis: A concise guide to prevention
and treatment'.

The risk of osteoporosis is thought to rise significantly once a patient is taking


the equivalent of prednisolone 7.5mg a day for 3 or more months. It is
important to note that we should manage patients in an anticipatory, i.e. if it likely
that the patient will have to take steroids for at least 3 months then we should
start bone protection straight away, rather than waiting until 3 months has
elapsed. A good example is a patient with newly diagnosed polymyalgia rheumatica.
As it is very likely they will be on a significant dose of prednisolone for greater than 3
months bone protection should be commenced immediately.

Management of patients at risk of corticosteroid-induced osteoporosis

The RCP guidelines essentially divide patients into two groups.

1. Patients over the age of 65 years or those who've previously had a fragility fracture
should be offered bone protection.

2. Patients under the age of 65 years should be offered a bone density scan, with
further management dependent:
T score Management
Greater than 0 Reassure
Between 0 and -1.5 Repeat bone density scan in 1-3 years
Less than -1.5 Offer bone protection
The first-line treatment is alendronate. Patients should also be calcium and vitamin D
replete.

Osteoporosis: Assessing patients following a fragility fracture

The management of patients following a fragility fracture depends on age.

Patients >= 75 years of age

Patients who've had a fragility fracture and are >= 75 years of age are presumed to
have underlying osteoporosis and should be started on first-line therapy (an oral
bisphosphonate), without the need for a DEXA scan.

It should be noted that the 2014 NOGG guidelines have a different threshold,
suggesting treatment is started in all women over the age of 50 years who've had a
fragility fracture - 'although BMD measurement may sometimes be appropriate,
particularly in younger postmenopausal women.'

Patients < 75 years of age

If a patient is under the age of 75 years a DEXA scan should be arranged. These
results can then be entered into a FRAX assessment (along with the fact that they've
had a fracture) to determine the patients ongoing fracture risk.

For example, a 79-year-old woman falls over on to an outstretched hand and


sustains a Colles' fracture (fracture of the distal radius). Given her age she is
presumed to have osteoporosis and therefore started on oral alendronate 70mg
once weekly. No DEXA scan is arranged.

Osteoporosis: management

NICE guidelines were updated in 2008 on the secondary prevention of osteoporotic


fractures in postmenopausal women.

Key points include

• treatment is indicated following osteoporotic fragility fractures in


postmenopausal women who are confirmed to have osteoporosis (a T-score of -
2.5 SD or below). In women aged 75 years or older, a DEXA scan may not be
required 'if the responsible clinician considers it to be clinically
inappropriate or unfeasible'
• vitamin D and calcium supplementation should be offered to all women
unless the clinician is confident they have adequate calcium intake and are
vitamin D replete
• alendronate is first-line
• around 25% of patients cannot tolerate alendronate, usually due to upper
gastrointestinal problems. These patients should be offered risedronate or
etidronate (see treatment criteria below)
• strontium ranelate and raloxifene are recommended if patients cannot tolerate
bisphosphonates (see treatment criteria below)

Treatment criteria for patients not taking alendronate

Unfortunately, a number of complicated treatment cut-off tables have been


produced in the latest guidelines for patients who do not tolerate alendronate

These take into account a patients age, their T-score and the number of risk factors
they have from the following list:

• parental history of hip fracture


• alcohol intake of 4 or more units per day
• rheumatoid arthritis

It is very unlikely that examiners would expect you to have memorised these risk
tables so we've not included them in the revision notes but they may be found by
following the NICE link. The most important thing to remember is:

• the T-score criteria for risedronate or etidronate are less than the others
implying that these are the second line drugs
• if alendronate, risedronate or etidronate cannot be taken then strontium
ranelate or raloxifene may be given based on quite strict T-scores (e.g. a 60-
year-old woman would need a T-score < -3.5)
• the strictest criteria are for denosumab
Supplementary notes on treatment

Bisphosphonates

• alendronate, risedronate and etidronate are all licensed for the prevention and
treatment of post-menopausal and glucocorticoid-induced osteoporosis
• all three have been shown to reduce the risk of both vertebral and non-vertebral
fractures although alendronate, risedronate may be superior to etidronate in
preventing hip fractures
• ibandronate is a once-monthly oral bisphosphonate

Vitamin D and calcium

• poor evidence base to suggest reduced fracture rates in the general population
at risk of osteoporotic fractures - may reduce rates in frail, housebound patients

Raloxifene - selective oestrogen receptor modulator (SERM)


• has been shown to prevent bone loss and to reduce the risk of vertebral
fractures, but has not yet been shown to reduce the risk of non-vertebral
fractures
• has been shown to increase bone density in the spine and proximal femur
• may worsen menopausal symptoms
• increased risk of thromboembolic events
• may decrease risk of breast cancer

Strontium ranelate
• 'dual action bone agent' - increases deposition of new bone by osteoblasts
(promotes differentiation of pre-osteoblast to osteoblast) and reduces the
resorption of bone by inhibiting osteoclasts
• concerns regarding the safety profile of strontium have been raised recently. It
should only be prescribed by a specialist in secondary care
• due to these concerns the European Medicines Agency in 2014 said it should
only be used by people for whom there are no other treatments for
osteoporosis
• increased risk of cardiovascular events: any history of cardiovascular disease or
significant risk of cardiovascular disease is a contraindication
• increased risk of thromboembolic events: a Drug Safety Update in 2012
recommended it is not used in patients with a history of venous
thromboembolism
• may cause serious skin reactions such as Stevens Johnson syndrome
Denosumab
• human monoclonal antibody that inhibits RANK ligand, which in turn inhibits the
maturation of osteoclasts
• given as a single subcutaneous injection every 6 months
• initial trial data suggests that it is effective and well tolerated

Teriparatide
• recombinant form of parathyroid hormone
• very effective at increasing bone mineral density but role in the management of
osteoporosis yet to be clearly defined

Hormone replacement therapy

• has been shown to reduce the incidence of vertebral fracture and non-vertebral
fractures
• due to concerns about increased rates of cardiovascular disease and breast
cancer it is no longer recommended for primary or secondary prevention of
osteoporosis unless the woman is suffering from vasomotor symptoms

Hip protectors

• evidence to suggest significantly reduce hip fractures in nursing home patients


• compliance is a problem

Falls risk assessment

• no evidence to suggest reduced fracture rates


• however, do reduce rate of falls and should be considered in management of
high risk patients

***

Bisphosphonates

Bisphosphonates are analogues of pyrophosphate, a molecule which decreases


demineralisation in bone. They inhibit osteoclasts by reducing recruitment and
promoting apoptosis.

Clinical uses

• prevention and treatment of osteoporosis


• hypercalcaemia
• Paget's disease
• pain from bone metatases
Adverse effects

• oesophageal reactions: oesophagitis, oesophageal ulcers (especially


alendronate)-- NICE guidance recommends that if patients suffer significant
upper gastrointestinal side effects from the use of alendronate, then this
should first be changed to risedronate or etidronate
• osteonecrosis of the jaw
• increased risk of atypical stress fractures of the proximal femoral shaft in
patients taking alendronate
• acute phase response: fever, myalgia and arthralgia may occur following
administration
• hypocalcaemia: due to reduced calcium efflux from bone. Usually clinically
unimportant

The BNF suggests the following counselling for patients taking oral bisphosphonates

• 'Tablets should be swallowed whole with plenty of water while sitting or


standing; to be given on an empty stomach at least 30 minutes before
breakfast (or another oral medication); patient should stand or sit
upright for at least 30 minutes after taking tablet'

Hypocalcemia/vitamin D deficiency should be corrected before giving


bisphosphonates. However, when starting bisphosphonate treatment for
osteoporosis, calcium should only be prescribed if dietary intake is inadequate.
Vitamin D supplements are normally given.

The duration of bisphosphonate treatment varies according to the level of risk. Some
authorities recommend stopping bisphosphonates at 5 years if the following apply:

• patient is < 75-years-old


• femoral neck T-score of > -2.5
• low risk according to FRAX/NOGG

*** This lady needs bone protection - her T score (-4.2) is well below the cut off and
she has already broken her hip.

Irrespective of her problems taking alendronate, bisphosphonates are not the most
appropriate choice as the BNF states them to be contraindicated if the eGFR is less
than 35 mL/minute/1.73m(2). The fact she has chronic kidney disease stage IV
therefore contraindicates this and so the option of re-introducing a trial of
alendronate is not appropriate, and neither is an IV zoledronate infusion.
There is nothing to be added by repeating her DEXA scan this early so this is not the
correct answer. Although calcium and vitamin D supplementation should be
continued, she really needs additional bone protection if possible therefore the
option of continuing this alone is not the best option.

This leaves denosumab as the best option. This would need to be given in secondary
care and her calcium would need to be closely monitored due to her renal disease.

***Bisphosphonate 'holidays' is a recent hot topic, addressed by recent evidence


from the National Osteoporosis Guideline Group (NOGG) in January 2016.

After a five year period for oral bisphosphonates (three years for IV zoledronate),
treatment should be re-assessed for ongoing treatment, with an updated FRAX score
and DEXA scan.

This guidance separates patients into high and low risk groups. To fall into the high
risk group, one of the following must be true:

• Age >75
• Glucocorticoid therapy
• Previous hip/vertebral fractures
• Further fractures on treatment
• High risk on FRAX scoring
• T score <-2.5 after treatment

If any of the high risk criteria apply, treatment should be continued indefinitely, or
until the criteria no longer apply. If they are in the low risk group however, treatment
may be discontinued and re-assessed after two years, or if a further fracture occurs.

In the case of this patient, she has no risk factors which put her into the high risk
group, but we do not have a recent DEXA scan. The best option would therefore be
to re-scan her now, and consider a two year break if her T score is >-2.5

Vitamin D supplementation

Vitamin D supplementation has been a hot topic for a number of years now. The
muddied waters are now slightly clearer following the release of the following:

• 2012: letter by the Chief Medical Officer regarding vitamin D supplementation


• 2013: National Osteoporosis Society (NOS) release UK Vitamin D guideline
The following groups should be advised to take vitamin D supplementation:

• all pregnant and breastfeeding women should take a daily supplement


containing 10µg of vitamin D
• all children aged 6 months - 5 years. Babies fed with formula milk do not need
to take a supplement if they are taking more than 500ml of milk a day, as
formula milk is fortified with vitamin D
• adults > 65 years
• 'people who are not exposed to much sun should also take a daily
supplement' e.g. housebound patients

Testing for vitamin D deficiency

The key message is that not many people warrant a vitamin D test. The NOS
guidelines specify that testing may be appropriate in the following situtations:

• patients with bone diseases that may be improved with vitamin D treatment
e.g. known osteomalacia or Paget's disease
• patients with bone diseases, prior to specific treatment where correcting
vitamin deficiency is appropriate e,g, prior to intravenous zolendronate or
denosumab
• patients with musculoskeletal symptoms that could be attributed to vitamin D
deficiency e.g. bone pain ?osteomalacia

Patients with osteoporosis should always be given calcium/vitamin D supplements so


testing is not considered necessary. People who are at higher risk of vitamin D
deficiency (see above) should be treated anyway so again testing is not necessary.

7. Dermatomyositis

• an inflammatory disorder causing symmetrical, proximal muscle weakness and


characteristic skin lesions
• May be idiopathic or associated with connective tissue disorders or
underlying malignancy (typically ovarian, breast and lung cancer, found in
20-25% - more if patient older). Screening for an underlying malignancy is
usually performed following a diagnosis of dermatomyositis
• polymyositis is a variant of the disease where skin manifestations are not
prominent
Skin features

• photosensitive
• macular rash over back and shoulder
• heliotrope rash in the periorbital region
• Gottron's papules - roughened red papules over extensor surfaces of
fingers
• 'mechanic's hands': extremely dry and scaly hands with linear 'cracks' on
the palmar and lateral aspects of the fingers
• nail fold capillary dilatation

***(Heberden's and Bouchard's nodes are seen in osteoarthritis. Aschoff nodules are
pathognomonic of rheumatic fever whilst Muehrcke's lines are white, transverse lines
of the fingernail seen in hypoalbuminaemia)

Other features

• proximal muscle weakness +/- tenderness


• Raynaud's
• respiratory muscle weakness
• interstitial lung disease: e.g. Fibrosing alveolitis or organising pneumonia
• dysphagia, dysphonia

Investigations

• the majority of patients (around 80%) are ANA positive


• around 30% of patients have antibodies to aminoacyl-tRNA synthetases (anti-
synthetase antibodies), including:
o antibodies against histidine-tRNA ligase (also called Jo-1)
o antibodies to signal recognition particle (SRP)
o anti-Mi-2 antibodies

8. Rheumatoid arthritis: x-ray changes

Early x-ray findings

• loss of joint space (common in both RA and Osteoarthritis)


• juxta-articular osteoporosis (only in RA, differentiating feature)
• soft-tissue swelling

Late x-ray findings

• periarticular erosions (only seen in RA)


• subluxation
Rheumatoid arthritis: initial investigations

Anti-cyclic citrullinated peptide antibody

Anti-cyclic citrullinated peptide antibody may be detectable up to 10 years before


the development of rheumatoid arthritis. It has a key role in the diagnosis of
rheumatoid arthritis, allowing early detection of patients suitable for aggressive anti-
TNF therapy. It has a sensitivity similar to rheumatoid factor (around 70%) with a
much higher specificity of 90-95%.

NICE recommends that patients with suspected rheumatoid arthritis who are
rheumatoid factor negative should be tested for anti-CCP antibodies.

Anti-CCP has the highest specificity for Rheumatoid Arthritis. It may be


detectable in patients up to 10 years prior to the diagnosis and is recommended in
all patients with suspected Rheumatoid Arthritis who test negative for rheumatoid
factor.

Rheumatoid arthritis: management

The management of rheumatoid arthritis (RA) has been revolutionised by the


introduction of disease-modifying therapies in the past decade.

Patients with evidence of joint inflammation should start a combination of disease-


modifying drugs (DMARD) as soon as possible. Other important treatment options
include analgesia, physiotherapy and surgery.

Initial therapy
• In 2018 NICE updated their rheumatoid arthritis guidelines. They now
recommend DMARD monotherapy +/- a short-course of bridging
prednisolone. In the past dual DMARD therapy was advocated as the initial step.
Monitoring response to treatment
• NICE recommends using a combination of CRP and disease activity (using a
composite score such as DAS28) to assess response to treatment

Flares
• flares of RA are often managed with corticosteroids - oral or intramuscular

DMARDs
• methotrexate is the most widely used DMARD. Monitoring of FBC & LFTs is
essential due to the risk of myelosuppression and liver cirrhosis. Other important
side-effects include pneumonitis
• sulfasalazine (Patients who are allergic to aspirin may also react to
sulfasalazine)
• leflunomide
• hydroxychloroquine

TNF-inhibitors
• the current indication for a TNF-inhibitor is an inadequate response to at least
two DMARDs including methotrexate
• etanercept: recombinant human protein, acts as a decoy receptor for TNF-α,
subcutaneous administration, can cause demyelination, risks include reactivation
of tuberculosis
• infliximab: monoclonal antibody, binds to TNF-α and prevents it from binding
with TNF receptors, intravenous administration, risks include reactivation of
tuberculosis
• adalimumab: monoclonal antibody, subcutaneous administration. Biologics such
as adalimumab can cause reactivation of tuberculosis (TB). It is recommended
that all patients being considered for biologics should be evaluated for both
active and TB infection, which usually involves a chest x-ray as well as a
tuberculin skin test or interferon-gamma release assay.

Rituximab
• anti-CD20 monoclonal antibody, results in B-cell depletion
• two 1g intravenous infusions are given two weeks apart
• infusion reactions are common

Abatacept
• fusion protein that modulates a key signal required for activation of T
lymphocytes
• leads to decreased T-cell proliferation and cytokine production
• given as an infusion
• not currently recommend by NICE

Rheumatoid arthritis: prognostic features

A number of features have been shown to predict a poor prognosis in patients with
rheumatoid arthritis, as listed below

Poor prognostic features

• rheumatoid factor positive


• anti-CCP antibodies
• poor functional status at presentation
• X-ray: early erosions (e.g. after < 2 years)
• extra articular features e.g. nodules
• HLA DR4
• insidious onset

In terms of gender there seems to be a split in what the established sources state is
associated with a poor prognosis. However both the American College of
Rheumatology and the recent NICE guidelines (which looked at a huge number of
prognosis studies) seem to conclude that female gender is associated with a poor
prognosis.

Rheumatoid arthritis: drug side-effects

The table below lists some of the characteristic (if not common) side-effects of drugs
used to treat rheumatoid arthritis:

Drug Side-effects
Myelosuppression
Methotrexate Liver cirrhosis
Pneumonitis
Rashes
Oligospermia
Sulfasalazine
Heinz body anaemia
Interstitial lung disease
Liver impairment
Leflunomide Interstitial lung disease
Hypertension
Retinopathy- bull’s eye
Hydroxychloroquine
Corneal deposits
Cushingoid features
Osteoporosis
Prednisolone Impaired glucose tolerance
Hypertension
Cataracts
Gold Proteinuria
Proteinuria
Penicillamine
Exacerbation of myasthenia gravis
Demyelination
Etanercept
Reactivation of tuberculosis
Infliximab Reactivation of tuberculosis
Adalimumab Reactivation of tuberculosis
Drug Side-effects
Rituximab Infusion reactions are common
Bronchospasm in asthmatics
NSAIDs (e.g. naproxen, ibuprofen)
Dyspepsia/peptic ulceration

Sulfasalazine

Sulfasalazine is a disease modifying anti-rheumatic drug (DMARDs) used in the


management of inflammatory arthritis, especially rheumatoid arthritis. It is also used
in the management of inflammatory bowel disease.

Sulfasalazine is a prodrug for 5-ASA which works through decreasing neutrophil


chemotaxis alongside suppressing proliferation of lymphocytes and pro-
inflammatory cytokines.

Cautions
• G6PD deficiency
• allergy to aspirin or sulphonamides (cross-sensitivity)

Adverse effects

• oligospermia
• Stevens-Johnson syndrome
• pneumonitis / lung fibrosis
• myelosuppression, Heinz body anaemia, megaloblastic anaemia
• may colour tears → stained contact lenses

In contrast to other DMARDs, sulfasalazine is considered safe to use in both


pregnancy and breastfeeding.

9. Osteoarthritis: diagnosis

NICE recommend that we can diagnose osteoarthritis clinically without the need for
investigations if:

• patient is > 45 years


• has exercise related pain
• no morning stiffness or morning stiffness lasting > 30 minutes

Osteoarthritis of the hands usually occurs as part of nodal osteoarthritis (a form of


osteoarthritis that is usually genetic).
It results in the formation of:

Heberden's nodes - swelling of the distal interphalangeal joints.


Bouchard's nodes - swelling of proximal interphalangeal joints

No treatment is needed for these.

Osteoarthritis: x-ray changes

X-ray changes of osteoarthritis

• decrease of joint space


• subchondral sclerosis
• subchondral cysts
• osteophytes forming at joint margins

Osteoarthritis: management

NICE published guidelines on the management of osteoarthritis (OA) in 2014

• all patients should be offered help with weight loss, given advice about local
muscle strengthening exercises and general aerobic fitness
• paracetamol and topical NSAIDs are first-line analgesics. Topical NSAIDs
are indicated only for OA of the knee or hand
• second-line treatment is oral NSAIDs/COX-2 inhibitors, opioids, capsaicin cream
and intra-articular corticosteroids. A proton pump inhibitor should be co-
prescribed with NSAIDs and COX-2 inhibitors. These drugs should be
avoided if the patient takes aspirin
• non-pharmacological treatment options include supports and braces, TENS and
shock-absorbing insoles or shoes
• if conservative methods fail then refer for consideration of joint replacement

What is the role of glucosamine?

• normal constituent of glycosaminoglycans in cartilage and synovial fluid


• a systematic review of several double-blind RCTs of glucosamine in knee
osteoarthritis reported significant short-term symptomatic benefits including
significantly reduced joint space narrowing and improved pain scores
• more recent studies have however been mixed
• the 2008 NICE guidelines suggest it is not recommended
• a 2008 Drug and Therapeutics Bulletin review advised that whilst glucosamine
provides modest pain relief in knee osteoarthritis it should not be prescribed on
the NHS due to limited evidence of cost-effectiveness

10. Antiphospholipid syndrome

Antiphospholipid syndrome is an acquired disorder characterised by a predisposition


to both venous and arterial thromboses, recurrent fetal loss and thrombocytopenia.
It may occur as a primary disorder or secondary to other conditions, most commonly
systemic lupus erythematosus (SLE)

A key point for the exam is to appreciate that antiphospholipid syndrome causes a
paradoxical rise in the APTT. This is due to an ex-vivo reaction of the lupus
anticoagulant autoantibodies with phospholipids involved in the coagulation cascade

Features

• venous/arterial thrombosis
• recurrent fetal loss
• livedo reticularis (Livedo reticularis is the skin rash most commonly
associated with antiphospholipid syndrome. Lupus vulgaris is seen in
tuberculosis)
• thrombocytopenia
• prolonged APTT, normal PT
• other features: pre-eclampsia, pulmonary hypertension

Associations other than SLE

• other autoimmune disorders


• lymphoproliferative disorders
• phenothiazines (rare)

Management - based on EULAR guidelines

• primary thromboprophylaxis
o low-dose aspirin
• secondary thromboprophylaxis
o initial venous thromboembolic events: lifelong warfarin with a target INR
of 2-3
o recurrent venous thromboembolic events: lifelong warfarin; if
occurred whilst taking warfarin then consider adding low-dose
aspirin, increase target INR to 3-4
o arterial thrombosis should be treated with lifelong warfarin with target
INR 2-3

11. Drug-induced lupus

In drug-induced lupus not all the typical features of systemic lupus erythematosus
are seen, with renal and nervous system involvement being unusual. It usually
resolves on stopping the drug.

Features

• arthralgia
• myalgia
• skin (e.g. malar rash) and pulmonary involvement (e.g. pleurisy) are common
• ANA positive in 100%, dsDNA negative
• anti-histone antibodies are found in 80-90%
• anti-Ro, anti-Smith positive in around 5%

Most common causes


• procainamide
• hydralazine
Less common causes

• isoniazid
• minocycline
• phenytoin

12. Reactive arthritis: features

Reactive arthritis is one of the HLA-B27 associated seronegative


spondyloarthropathies. It encompasses what was formerly called Reiter's
syndrome*, a term which described a classic triad of urethritis, conjunctivitis and
arthritis following a dysenteric illness during the Second World War. Later studies
identified patients who developed symptoms following a sexually transmitted
infection (post-STI, now sometimes referred to as sexually acquired reactive arthritis,
SARA).
***
The rash on the soles is keratoderma blenorrhagica. His reactive arthritis may be
secondary to either gastrointestinal infection or Chlamydia.
Reactive arthritis is defined as an arthritis that develops following an infection where
the organism cannot be recovered from the joint.

Features

• time course
o typically develops within 4 weeks of initial infection - symptoms generally
last around 4-6 months
o around 25% of patients have recurrent episodes whilst 10% of patients
develop chronic disease
• arthritis is typically an asymmetrical oligoarthritis of lower limbs
• dactylitis
• symptoms of urethritis
• eye
o conjunctivitis (seen in 10-30%)
o anterior uveitis
• skin
o circinate balanitis (painless vesicles on the coronal margin of the
prepuce)
o keratoderma blenorrhagica (waxy yellow/brown papules on palms and
soles)

'Can't see, pee or climb a tree'

Keratoderma blenorrhagica

*as Reiter was a member of the Nazi party the term is no longer used#

Epidemiology
• post-STI form much more common in men (e.g. 10:1)
• post-dysenteric form equal sex incidence

The table below shows the organisms that are most commonly associated with
reactive arthritis:

Post-dysenteric form Post-STI form


Shigella flexneri
Salmonella typhimurium
Salmonella enteritidis Chlamydia trachomatis
Yersinia enterocolitica
Campylobacter

Management

• symptomatic: analgesia, NSAIDS, intra-articular steroids


• sulfasalazine and methotrexate are sometimes used for persistent disease
• symptoms rarely last more than 12 months

*as Reiter was a member of the Nazi party the term is no longer used

13. Osteomalacia

Osteomalacia describes softening of the bones secondary to low vitamin D levels


that in turn lead to decreased bone mineral content. If this occurs in growing
children it is referred to as rickets, with the term osteomalacia preferred for adults.
The low calcium and phosphate combined with the raised alkaline phosphatase
point towards osteomalacia.
Causes
• vitamin D deficiency
o malabsorption
o lack of sunlight
o diet
• chronic kidney disease
• drug induced e.g. anticonvulsants
• inherited: hypophosphatemic rickets (previously called vitamin D-resistant
rickets)
• liver disease: e.g. cirrhosis

Features
• bone pain
• bone/muscle tenderness
• fractures: especially femoral neck
• proximal myopathy: may lead to a waddling gait

Investigation
• bloods
o low vitamin D levels
o low calcium, phosphate (in around 30%)
o raised alkaline phosphatase (in 95-100% of patients)
• x-ray
o translucent bands (Looser's zones or pseudofractures)

Treatment
• vitamin D supplmentation
o a loading dose is often needed initially
• calcium supplementation if dietary calcium is inadequate

14. Azathioprine (induces remission and long term maintenance of Crohn’s


disease )

Azathioprine is metabolised to the active compound mercaptopurine, a purine


analogue that inhibits purine synthesis. A thiopurine methyltransferase (TPMT)
test may be needed to look for individuals prone to azathioprine toxicity.

Adverse effects include


• bone marrow depression
• nausea/vomiting
• pancreatitis
• increased risk of non-melanoma skin cancer

A significant interaction may occur with allopurinol and hence lower doses of
azathioprine should be used.

Azathioprine is generally considered safe to use in pregnancy

15. Chronic fatigue syndrome

Diagnosed after at least 4 months of disabling fatigue affecting mental and


physical function more than 50% of the time in the absence of other disease which
may explain symptoms

Epidemiology
• more common in females, better prognosis in children
• past psychiatric history has not been shown to be a risk factor

Fatigue is the central feature, other recognised features include

• sleep problems, such as insomnia, hypersomnia, unrefreshing sleep, a disturbed


sleep-wake cycle
• muscle and/or joint pains
• headaches
• painful lymph nodes without enlargement
• sore throat
• cognitive dysfunction, such as difficulty thinking, inability to concentrate,
impairment of short-term memory, and difficulties with word-finding
• physical or mental exertion makes symptoms worse
• general malaise or 'flu-like' symptoms
• dizziness
• nausea
• palpitations

Investigation

• NICE guidelines suggest carrying out a large number of screening blood tests
to exclude other pathology e.g. FBC, U&E, LFT, glucose, TFT, ESR, CRP, calcium,
CK, ferritin (*children and young people only)*, coeliac screening and also
urinalysis

Management
• cognitive behaviour therapy - very effective, number needed to treat = 2
• graded exercise therapy - a formal supervised program, not advice to go to the
gym
• 'pacing' - organising activities to avoid tiring
• low-dose amitriptyline may be useful for poor sleep
• referral to a pain management clinic if pain is a predominant feature
16. Psoriatic arthropathy

Psoriatic arthropathy is an inflammatory arthritis that is associated with psoriasis and


is classed as one of the seronegative spondyloarthropathies. It correlates poorly with
cutaneous psoriasis and often precedes the development of skin lesions. Around 10-
20% of patients with skin lesions develop an arthropathy with males and females
being equally affected.
Presenation

Patterns

• symmetric polyarthritis
o very similar to rheumatoid arthritis
o 30-40% of cases, most common type
• asymmetrical oligoarthritis: typically affects hands and feet (20-30%)
o until recently it was thought asymmetrical oligoarthritis was the most
common type, based on data from the original 1973 Moll and Wright
paper. Please see the link for a comparison of more recent studies
• sacroilitis
• DIP joint disease (10%)
• arthritis mutilans (severe deformity fingers/hand, 'telescoping fingers')

Other signs
• psoriatic skin lesions
• periarticular disease - tenosynovitis and soft tissue inflammation resulting in:
o enthesitis: inflammation at the site of tendon and ligament insertion e.g.
Achilles tendonitis, plantar fascitis
o tenosynovitis: typically of the flexor tendons of the hands
o dactylitis: diffuse swelling of a finger or toe
• nail changes
o pitting
o onycholysis

Investigation and management

X-ray

• often have the unusual combination of coexistence of erosive changes and new
bone formation
• periostitis
• 'pencil-in-cup' appearance

Management

• should be managed by a rheumatologist


• treat as rheumatoid arthritis but better prognosis
17. Lateral epicondylitis

Lateral epicondylitis typically follows unaccustomed activity such as house painting


or playing tennis ('tennis elbow'). It is most common in people aged 45-55 years
and typically affects the dominant arm.

Features
• pain and tenderness localised to the lateral epicondyle
• pain worse on wrist extension against resistance with the elbow extended or
supination of the forearm with the elbow extended
• episodes typically last between 6 months and 2 years. Patients tend to have
acute pain for 6-12 weeks

Management options
• advice on avoiding muscle overload
• simple analgesia
• steroid injection
• physiotherapy

***Radial tunnel syndrome can be difficult to distinguish from lateral epicondylitis


as both conditions present with lateral elbow pain. Radial tunnel syndrome however
typically presents with tenderness distal to the common extensor origin in
comparison to lateral epicondylitis where there is pain over the common extensor
origin. It is most common in gymnasts, racquet players and golfers who frequently
hyperextend at the wrist or carry out frequent supination/pronation. Patients can also
complain of hand paraesthesia or aching at the wrist. Sometimes feels like an
unusual sensation in hand a bit like pins and needles.

** You see a 48-year-old woman who presents with increasing pain whilst writing
notes in her new job as a secretary. She describes a pain in her upper forearm which
develops whilst she is writing. This is only relieved when she stops writing and it
progresses through the working day. On examination, she has elbow pain with wrist
dorsiflexion and middle finger extension. There is no weakness. What is the most
likely diagnosis?
Answer: tennis elbow/lateral epicondylitis

18. Systemic lupus erythematosus: features

Systemic lupus erythematosus (SLE) is a multisystem, autoimmune disorder. It


typically presents in early adulthood and is more common in women and people of
Afro-Caribbean origin.
General features
• fatigue
• fever
• mouth ulcers
• lymphadenopathy

Skin
• malar (butterfly) rash: spares nasolabial folds
• discoid rash: scaly, erythematous, well demarcated rash in sun-exposed areas.
Lesions may progress to become pigmented and hyperkeratotic before
becoming atrophic
• photosensitivity
• Raynaud's phenomenon
• livedo reticularis
• non-scarring alopecia
Musculoskeletal
• arthralgia
• non-erosive arthritis
Cardiovascular
• pericarditis: the most common cardiac manifestation
• myocarditis
Respiratory
• pleurisy
• fibrosing alveolitis
Renal
• proteinuria
• glomerulonephritis (diffuse proliferative glomerulonephritis is the most common
type)
Neuropsychiatric
• anxiety and depression
• psychosis
• seizures

19. Pseudogout

Pseudogout is a form of microcrystal synovitis caused by the deposition of calcium


pyrophosphate dihydrate crystals in the synovium. For this reason, it is now more
correctly termed acute calcium pyrophosphate crystal deposition disease.

Pseudogout is strongly associated with increasing age. Patients who develop


pseudogout at a younger age (e.g. < 60 years) usually have some underlying risk
factor, such as:

• haemochromatosis
• hyperparathyroidism
• low magnesium, low phosphate
• acromegaly, Wilson's disease

Features
• knee, wrist and shoulders most commonly affected
• joint aspiration: weakly-positively birefringent rhomboid-shaped crystals
• x-ray: chondrocalcinosis
o in the knee this can be seen as linear calcifications of the meniscus
and articular cartilage. This is seen in pseudogout, but not
particularly associated with gout, and so it can be used to
distinguish between the two.
Management
• aspiration of joint fluid, to exclude septic arthritis
• NSAIDs or intra-articular, intra-muscular or oral steroids as for gout
20. Polymyalgia rheumatica

Polymyalgia rheumatica (PMR) is a relatively common condition seen in older people


characterized by muscle stiffness and raised inflammatory markers. Whilst it appears
to be closely related to temporal arteritis the underlying cause is not fully
understood and it does not appear to be a vasculitic process.

Features

• typically patient > 60 years old


• usually rapid onset (e.g. < 1 month)
• aching, morning stiffness in proximal limb muscles
o weakness is not considered a symptom of polymyalgia rheumatica
• also mild polyarthralgia, lethargy, depression, low-grade fever, anorexia, night
sweats

Investigations

• raised inflammatory markers e.g. ESR > 40 mm/hr (sometimes the only
clue, age may not be above 60 years)
• note creatine kinase and EMG normal

Treatment
• prednisolone e.g. 15mg/od
o patients typically respond dramatically to steroids, failure to do so
should prompt consideration of an alternative diagnosis

21. Systemic sclerosis

Systemic sclerosis is a condition of unknown aetiology characterised by hardened,


sclerotic skin and other connective tissues. It is four times more common in females.

There are three patterns of disease:

Limited cutaneous systemic sclerosis

• Raynaud's may be first sign


• scleroderma affects face and distal limbs predominately
• associated with anti-centromere antibodies
• a subtype of limited systemic sclerosis is CREST syndrome: Calcinosis,
Raynaud's phenomenon, oEsophageal dysmotility, Sclerodactyly,
Telangiectasia

Diffuse cutaneous systemic sclerosis

• scleroderma affects trunk and proximal limbs predominately


• associated with scl-70 antibodies
• the most common cause of death is now respiratory involvement, which is seen
in around 80%: interstitial lung disease (ILD) and pulmonary arterial
hypertension (PAH)
• other complications include renal disease and hypertension
• poor prognosis

Scleroderma (without internal organ involvement)

• tightening and fibrosis of skin


• may be manifest as plaques (morphoea) or linear

Antibodies

• ANA positive in 90%


• RF positive in 30%
• anti-scl-70 antibodies associated with diffuse cutaneous systemic sclerosis
• anti-centromere antibodies associated with limited cutaneous systemic sclerosis
22. Sjogren's syndrome

Sjogren's syndrome is an autoimmune disorder affecting exocrine glands resulting in


dry mucosal surfaces. It may be primary (PSS) or secondary to rheumatoid arthritis or
other connective tissue disorders, where it usually develops around 10 years after the
initial onset. Sjogren's syndrome is much more common in females (ratio 9:1). There
is a marked increased risk of lymphoid malignancy (40-60 fold).

Features

• dry eyes: keratoconjunctivitis sicca


• dry mouth
• vaginal dryness
• arthralgia
• Raynaud's, myalgia
• sensory polyneuropathy
• recurrent episodes of parotitis
• renal tubular acidosis (usually subclinical)

Investigation

• rheumatoid factor (RF) positive in nearly 50% of patients


• ANA positive in 70%
• anti-Ro (SSA) antibodies in 70% of patients with PSS
• anti-La (SSB) antibodies in 30% of patients with PSS
• Schirmer's test: filter paper near conjunctival sac to measure tear formation
• histology: focal lymphocytic infiltration
• also: hypergammaglobulinaemia, low C4

Management
• artificial saliva and tears
• pilocarpine may stimulate saliva production

23. Temporal arteritis

Temporal arteritis is large vessel vasculitis which overlaps with polymyalgia


rheumatica (PMR). Histology shows changes that characteristically 'skips' certain
sections of the affected artery whilst damaging others.
Features

• typically patient > 60 years old


• usually rapid onset (e.g. < 1 month)
• headache (found in 85%)
• jaw claudication (65%)
• visual disturbances
o amaurosis fugax
o blurring
o double vision
o vision testing is a key investigation in patients with suspected temporal
arteritis
o secondary to anterior ischemic optic neuropathy
• tender, palpable temporal artery
• around 50% have features of PMR: aching, morning stiffness in proximal limb
muscles (not weakness)
• also lethargy, depression, low-grade fever, anorexia, night sweats

Investigations

• raised inflammatory markers


o ESR > 50 mm/hr (note ESR < 30 in 10% of patients)
o CRP may also be elevated
• temporal artery biopsy
o skip lesions may be present
• note creatine kinase and EMG normal

Treatment

• urgent high-dose glucocorticoids should be given as soon as the diagnosis


is suspected and before the temporal artery biopsy
o if there is no visual loss then high-dose prednisolone is used
o if there is evolving visual loss IV methylprednisolone is usually given
prior to starting high-dose prednisolone
o there should be a dramatic response, if not the diagnosis should be
reconsidered
• urgent ophthalmology review
o patients with visual symptoms should be seen the same-day by an
ophthalmologist
o visual damage is often irreversible
• other treatments
o bone protection with bisphosphonates is required as long, tapering
course of steroids is required
o low-dose aspirin is sometimes given to patients as well, although the
evidence base supporting this is weak

24. Hydroxychloroquine

Hydroxychloroquine is used in the management of rheumatoid arthritis and


systemic/discoid lupus erythematosus. It is pharmacologically very similar to
chloroquine which is used to treat certain types of malaria.

Adverse effects

• bull's eye retinopathy - may result in severe and permanent visual loss
o recent data suggest that retinopathy caused by hydroxychloroquine is
more common than previously thought and the most recent RCOphth
guidelines (March 2018) suggest colour retinal photography and spectral
domain optical coherence tomography scanning of the macula
o baseline ophthalmological examination and annual screening is generally
recommened

A contrast to many drugs used in rheumatology, hydroxychloroquine may be used if


needed in pregnant women.

Monitoring

• the BNF advises: 'Ask patient about visual symptoms and monitor visual
acuity annually using the standard reading chart'

25. Hip pain in adults

The table below provides a brief summary of the potential causes of hip pain in
adults

Condition Features
Pain exacerbated by exercise and relieved by rest
Osteoarthritis Reduction in internal rotation is often the first sign
Age, obesity and previous joint problems are risk factors
Pain in the morning
Inflammatory
Systemic features
arthritis
Raised inflammatory markers
Condition Features
Femoral nerve compression may cause referred pain in the
hip
Referred lumbar Femoral nerve stretch test may be positive - lie the patient
spine pain prone. Extend the hip joint with a straight leg then bend the
knee. This stretches the femoral nerve and will cause pain if it
is trapped
Greater trochanteric
Due to repeated movement of the fibroelastic iliotibial band
pain syndrome
Pain and tenderness over the lateral side of thigh (jogger)
(Trochanteric
Most common in women aged 50-70 years
bursitis)
Caused by compression of lateral cutaneous nerve of thigh
Meralgia
Typically burning sensation over antero-lateral aspect of
paraesthetica
thigh
Symptoms may be of gradual or sudden onset
May follow high dose steroid therapy or previous hip
Avascular necrosis
fracture of dislocation
Hx of asthma
Common in pregnancy
Ligament laxity increases in response to hormonal changes of
Pubic symphysis pregnancy
dysfunction Pain over the pubic symphysis with radiation to the groins
and the medial aspects of the thighs. A waddling gait may be
seen
An uncommon condition sometimes seen in the third
trimester of pregnancy
Transient idiopathic Groin pain associated with a limited range of movement in
osteoporosis the hip
Patients may be unable to weight bear
ESR may be elevated

*** Femeroacetabular impingement is a cause of anterior groin pain in active young


adults. Femoroacetabular impingement (FAI) is one of the most common causes of
persistent hip pain in active young adults. It commonly presents with hip/groin
pain worse on prolonged sitting and associated with snapping, clicking or
locking of the hip. There is an association between FAI and prior hip pathology eg
Perthes in childhood. It is caused by a variant in hip anatomy leading to abnormal
contact between the femur and acetabulum rim. Over time this can lead to soft tissue
damage including labral tears.

In snapping hip syndrome patients experience a snapping or clunking as tendons e.g


the iliotibial band, move over the hip bones during flexion and extension. The
snapping sound occurs either due to subluxation of a tendon or catching of a
thickened tendon on the greater trochanter. In most cases, this is painless unless
there is associated labral tear.

26. Marfan's syndrome

Marfan's syndrome is an autosomal dominant connective tissue disorder. It is caused


by a defect in the FBN1 gene on chromosome 15 that codes for the protein fibrillin-
1. It affects around 1 in 3,000 people.

Features
• tall stature with arm span to height ratio > 1.05
• high-arched palate
• arachnodactyly
• pectus excavatum
• pes planus
• scoliosis of > 20 degrees
• heart: dilation of the aortic sinuses (seen in 90%) which may lead to aortic
aneurysm, aortic dissection, aortic regurgitation, mitral valve prolapse
(75%),
• lungs: repeated pneumothoraces
• eyes: upwards lens dislocation (superotemporal ectopia lentis), blue sclera,
myopia
• dural ectasia (ballooning of the dural sac at the lumbosacral level)

The life expectancy of patients used to be around 40-50 years. With the advent of
regular echocardiography monitoring and beta-blocker/ACE-inhibitor therapy this
has improved significantly over recent years. Aortic dissection and other
cardiovascular problems remain the leading cause of death however.

27. Paget's disease of the bone -

old man, bone pain, raised ALP

Paget's disease is a disease of increased but uncontrolled bone turnover. It is


thought to be primarily a disorder of osteoclasts, with excessive osteoclastic
resorption followed by increased osteoblastic activity. Paget's disease is common (UK
prevalence 5%) but symptomatic in only 1 in 20 patients. The skull, spine/pelvis, and
long bones of the lower extremities are most commonly affected.

Predisposing factors
• increasing age
• male sex
• northern latitude
• family history

Clinical features - only 5% of patients are symptomatic

• the stereotypical presentation is an older male with bone pain and an


isolated raised ALP
• bone pain (e.g. pelvis, lumbar spine, femur)
• classical, untreated features: bowing of tibia, bossing of skull
• raised alkaline phosphatase (ALP) - calcium* and phosphate are typically
normal
• other markers of bone turnover include: procollagen type I N-terminal
propeptide (PINP), serum C-telopeptide (CTx), urinary N-telopeptide (NTx),
and urinary hydroxyproline
• skull x-ray: thickened vault, osteoporosis circumscripta

Indications for treatment include bone pain, skull or long bone deformity, fracture,
periarticular Paget's

• bisphosphonate (either oral risedronate or IV zoledronate)


• calcitonin is less commonly used now

Complications

• deafness (cranial nerve entrapment)


• bone sarcoma (1% if affected for > 10 years)
• fractures
• skull thickening
• high-output cardiac failure

28. Raynaud's phenomenon

Raynaud's phenomenon is characterised by an exaggerated vasoconstrictive


response of the digital arteries and cutaneous arteriole to the cold or emotional
stress. It may be primary (Raynaud's disease) or secondary (Raynaud's phenomenon).

Raynaud's disease typically presents in young women (e.g. 30 years old) with bilateral
symptoms.

Secondary causes of Raynaud's phenomenon


• connective tissue disorders
o scleroderma (most common)
o rheumatoid arthritis
o systemic lupus erythematosus
• leukaemia
• type I cryoglobulinaemia, cold agglutinins
• use of vibrating tools
• drugs: oral contraceptive pill, ergot
• cervical rib

Factors suggesting underlying connective tissue disease

• onset after 40 years


• unilateral symptoms
• rashes
• presence of autoantibodies
• features which may suggest rheumatoid arthritis or SLE, for example arthritis
or recurrent miscarriages
• digital ulcers, calcinosis
• very rarely: chilblains

Management

• all patients with suspected secondary Raynaud's phenomenon should be


referred to secondary care
• first-line: calcium channel blockers e.g. nifedipine
• IV prostacyclin (epoprostenol) infusions: effects may last several
weeks/months
PHARMACOLOGY
1.Calcium channel blockers: Ankle swelling

Calcium channel blockers are primarily used in the management of cardiovascular disease.
Voltage-gated calcium channels are present in myocardial cells, cells of the conduction
system and those of the vascular smooth muscle. The various types of calcium channel
blockers have varying effects on these three areas and it is therefore important to differentiate
their uses and actions.

Side-effects and
Examples Indications & notes
cautions
Angina, hypertension, arrhythmias
Heart failure,
Highly negatively inotropic constipation,
Verapamil
hypotension,
Should not be given with beta-blockers as bradycardia, flushing
may cause heart block
Angina, hypertension
Hypotension,
Less negatively inotropic than verapamil but
Diltiazem bradycardia, heart
caution should still be exercised when
failure, ankle swelling
patients have heart failure or are taking beta-
blockers
Hypertension, angina, Raynaud's
Nifedipine,
Affects the peripheral vascular smooth
amlodipine, Flushing, headache,
muscle more than the myocardium and
felodipine ankle swelling
therefore do not result in worsening of heart
(dihydropyridines)
failure but may therefore cause ankle
swelling

** Bendroflumethiazide does not cause ankle oedema, and it's most likely side-effects are
postural hypotension and electrolyte disturbances, especially hypokalaemia.

Beta blockers like bisoprolol are not associated with ankle oedema. They can cause
peripheral coldness due to vasoconstriction, hypotension and bronchospasm.

Clopidogrel likewise isn't associated with ankle oedema. It can cause gastrointestinal
symptoms or occasionally bleeding disorders.

ACE inhibitors like ramipril are known to cause hypotension, renal dysfunction and a dry
cough.They are not known to cause ankle oedema.

Amlodipine is a calcium channel blocker which commonly causes ankle oedema which tends
to be only partially responsive to diuretics. They can also cause other side-effects associated
with vasodilatation such as flushing and headaches.
**Ramipril is an ACE inhibitor and candesartan is angiotensin 2 receptor blocker both of
which require regular monitoring of renal function and electrolytes.

Amlodipine is a calcium channel blocker and does not require any blood test monitoring.

Indapamide and bendroflumethiazide are diuretics which require regular monitoring of renal
function and electrolytes.

2. Tuberculosis: drug side-effects and mechanism of action

Rifampicin: mechanism of action: inhibits bacterial DNA dependent RNA polymerase


preventing transcription of DNA into mRNA

• potent liver enzyme inducer


• hepatitis, orange secretions
• flu-like symptoms

Isoniazid: mechanism of action: inhibits mycolic acid synthesis

• peripheral neuropathy: prevent with pyridoxine (Vitamin B6)


• hepatitis, agranulocytosis
• liver enzyme inhibitor

Pyrazinamide: mechanism of action: converted by pyrazinamidase into pyrazinoic


acid which in turn inhibits fatty acid synthase (FAS)

• hyperuricaemia causing gout


• arthralgia, myalgia
• hepatitis

Ethambutol: mechanism of action: inhibits the enzyme arabinosyl transferase which


polymerizes arabinose into arabinan

• optic neuritis: check visual acuity before and during treatment


• dose needs adjusting in patients with renal impairment

**Peripheral neuropathy is a commonly recognised side effect of isoniazid. Although


paraesthesia is listed under the side effects for amlodipine in the BNF, it is
uncommon. In this case isoniazid is the most likely answer.

Drug Most common side effects


Rifampicin Orange bodily fluids, rash, hepatotoxicity, drug interactions
Isoniazid Peripheral neuropathy, psychosis, hepatotoxicity
Pyrazinamide Arthralgia, gout, hepatotoxicity, nausea
Drug Most common side effects
Ethambutol Optic neuritis, rash

3. Drugs causing ocular problems

Cataracts
• steroids
Corneal opacities
• amiodarone
• indomethacin
Optic neuritis
• ethambutol
• amiodarone
• metronidazole
Retinopathy
• chloroquine, quinine
Sildenafil can cause both blue discolouration and non-arteritic anterior
ischaemic neuropathy

Drugs causing lung fibrosis

Causes

• amiodarone
• cytotoxic agents: busulphan, bleomycin
• anti-rheumatoid drugs: methotrexate, sulfasalazine, gold
• nitrofurantoin
• ergot-derived dopamine receptor agonists (bromocriptine, cabergoline,
pergolide

Drugs causing photosensitivity

Causes of drug-induced photosensitivity

• thiazides
• tetracyclines, sulphonamides, ciprofloxacin
• amiodarone
• NSAIDs e.g. piroxicam
• psoralens
• sulphonylureas
4. St John's Wort

• shown to be as effective as tricyclic antidepressants in the treatment of mild-


moderate depression
• mechanism: thought to be similar to SSRIs (although noradrenaline uptake
inhibition has also been demonstrated)
• NICE advise 'may be of benefit in mild or moderate depression, but its use
should not be prescribed or advised because of uncertainty about appropriate
doses, variation in the nature of preparations, and potential serious interactions
with other drugs'

Adverse effects

• profile in trials similar to placebo


• can cause serotonin syndrome
• inducer of P450 system, therefore decreased levels of drugs such as warfarin,
ciclosporin. The effectiveness of the combined oral contraceptive pill may also
be reduced

5. Phosphodiesterase type V inhibitors

Phosphodiesterase type V (PDE5) inhibitors are used in the treatment of erectile


dysfunction. They are also used in the management of pulmonary hypertension.
PDE5 inhibitors cause vasodilation through an increase in cGMP leading to smooth
muscle relaxation in blood vessels supplying the corpus cavernosum.

Examples

• sildenafil (Viagra) - this was the first phosphodiesterase type V inhibitor


• tadalafil (Cialis)
• vardenafil (Levitra)

Contraindications

• patients taking nitrates and related drugs such as nicorandil


• hypotension
• recent stroke or myocardial infarction (NICE recommend waiting 6 months)

Side-effects

• visual disturbances e.g. blue discolouration, non-arteritic anterior ischaemic


neuropathy
• nasal congestion
• flushing
• gastrointestinal side-effects
• headache (most common)

The blue pill, Viagra (sildenafil), causes blue discolouration of vision

**Sildenafil, brand name Viagra, is a medication used to treat erectile dysfunction in adults.
However in neonates it is used to treat pulmonary hypertension.

The approach to settling distress in the neonate depends upon the cause and is commonly due
to pain or respiratory distress.

Gaviscon and ranitidine are used, off licence, to treat gastro-oesophageal reflux.

Caffeine is used to facilitate weening a neonate off a ventilator.

**Patients taking nitrates cannot take sildenafil concurrently as this may potentiate the
vasodilating effects of such drugs.

** Visual changes secondary to drugs

• blue vision: Viagra ('the blue pill')


• yellow-green vision: digoxin

6. Ecstasy poisoning

Ecstasy (MDMA, 3,4-Methylenedioxymethamphetamine) use became popular in the


1990's during the emergence of dance music culture

Clinical features

• neurological: agitation, anxiety, confusion, ataxia


• cardiovascular: tachycardia, hypertension
• hyponatraemia
• hyperthermia
• rhabdomyolysis

Management

• supportive
• dantrolene may be used for hyperthermia if simple measures fail

7. P450 enzyme system


Induction usually requires prolonged exposure to the inducing drug, as opposed to
P450 inhibitors, where effects are often seen rapidly

Inducers of the P450 system include

• antiepileptics: phenytoin, carbamazepine


• barbiturates: phenobarbitone
• rifampicin
• St John's Wort
• chronic alcohol intake
• griseofulvin
• smoking (affects CYP1A2, reason why smokers require more aminophylline)

**Concurrent use of Carbamazepine would make combined oral contraceptive pill less
effective

Inhibitors of the P450 system include

• antibiotics: ciprofloxacin, erythromycin


• isoniazid
• cimetidine,omeprazole
• amiodarone
• allopurinol
• imidazoles: ketoconazole, fluconazole
• SSRIs: fluoxetine, sertraline
• ritonavir
• sodium valproate
• acute alcohol intake
• quinupristin

A raised INR is a result of inhibited liver enzymes. A 44-year-old Bangladeshi man with a
history of mitral stenosis and atrial fibrillation is diagnosed with tuberculosis. He is
commenced on anti-tuberculosis therapy. Three weeks after starting treatment his INR has
increased to 5.6. Which one of the following medications is most likely to be responsible for
this increase?—Isoniazid

A 66-year-old female is admitted to hospital after suffering a pulmonary embolism. She


currently suffers from hypertension, type 2 diabetes mellitus, epilepsy and schizophrenia. Her
current medications include ramipril, olanzapine, metformin, gliclazide and sodium valproate.
She needs to be started on warfarin but the junior doctors are concerned about an
interaction which may alter dosing. Which medication is responsible for this interaction?—
Sodium Valproate (This question requires knowledge of the cytochrome P450 system.
Cytochrome P450 is responsible for metabolising many drugs, including warfarin. Sodium
valproate inhibits cytochrome P450 which would lead to diminished warfarin levels in the
blood. Therefore the dose of warfarin would most likely have to be adjusted.)
8. Hormone replacement therapy: indications and types

Hormone replacement therapy (HRT) involves the use of a small dose of oestrogen,
combined with a progestogen (in women with a uterus), to help alleviate menopausal
symptoms.

The indications for HRT have changed significantly over the past ten years as the
long-term risks became apparent, primarily as a result of the Women's Health
Initiative (WHI) study.

Indications

• vasomotor symptoms such as flushing, insomnia and headaches


o this is considered the most important factor in choosing whether to start
HRT, rather than other possible health benefits such as increased bone
mineral density
o other indications such as reversal of vaginal atrophy should be treated
with other agents as first-line therapies
• premature menopause
o should be continued until the age of 50 years
o the most important reason in giving HRT to younger women is
preventing the development of osteoporosis

Other benefits include a reduced incidence of colorectal cancer.

Types of HRT

HRT generally consists of an oestrogenic compound, which replaces the diminished


levels that occur in the perimenopausal period. This is normally combined with a
progestogen if a woman has a uterus to reduce the risk of endometrial cancer.

Choice of hormone

• oestrogens
o 'natural' oestrogens such as estradiol, estrone and conjugated oestrogen
are generally used rather than synthetic oestrogens such as
ethinylestradiol (which is used in the combined oral contraceptive pill)
• progestogens
o 'synthetic' progestogens such as medroxyprogesterone, norethisterone,
levonorgestrel, and drospirenone are usually used
o a levonorgestrel-releasing intrauterine system (e.g. Mirena) may be used
as the progestogen component of HRT, i.e. a woman could take an oral
oestrogen and have endometrial protection using a Mirena coil
• tibolone
o synthetic compound with both oestrogenic, progestogenic, and
androgenic activity

Route

• HRT can be taken orally or transdermally (via a patch or gel)


• transdermal is preferred if the woman is at risk of venous thromboembolism
(VTE), as the rates of VTE do not appear to rise with transdermal preparations

**When a woman wishes to discuss commencing HRT, a detailed history is important to


determine the menopausal status. This includes the date of her LMP and bleeding patterns
prior to this. In this scenario, the woman would be considered to be peri-menopausal (as she
has not been amenorrhoeic for 1year).

The main clinical indication for starting HRT is for the relief of vasomotor symptoms.
However, it can also be used to prevent osteoporosis only in women diagnosed with
premature menopause.

In women with a uterus, HRT should comprise an oestrogen combined with a progesterone.
The progesterone is paramount in reducing the risk of endometrial cancer, that can occur with
the use of unopposed oestrogen. In women who have undergone a hysterectomy continuous
oestrogen-only therapy is given.

Women should be prescribed cyclical combined HRT if their LMP was less than 1 year ago
and continuous combined HRT if they have:
•taken cyclical combined for at least 1 year or
•it has been at least 1 year since their LMP or
•it has been at least 2 years since their LMP, if they had premature menopause (menopause
below the age of 40)

There are many non-hormonal treatments for vasomotor symptoms in women who are unable
or unwilling to take HRT. These treatments include some selective serotonin and
noradrenaline reuptake inhibitors, clonidine and gabapentin. Sertraline is not helpful in
treating hot flushes.

The National Institute for Health and Clinical Excellence (NICE) do not recommend the use
of herbal and complementary therapies for managing hot flushes and night sweats. Evening
primrose oil has no proven benefit in reducing vasomotor symptoms.

9. Metformin

Metformin is a biguanide used mainly in the treatment of type 2 diabetes mellitus. It


has a number of actions which improves glucose tolerance (see below). Unlike
sulphonylureas it does not cause hypoglycaemia and weight gain and is therefore
first-line, particularly if the patient is overweight. Metformin is also used in polycystic
ovarian syndrome and non-alcoholic fatty liver disease
• acts by activation of the AMP-activated protein kinase (AMPK)
• increases insulin sensitivity
• decreases hepatic gluconeogenesis
• may also reduce gastrointestinal absorption of carbohydrates

Adverse effects
• gastrointestinal upsets are common (nausea, anorexia, diarrhoea), intolerable in
20%
• reduced vitamin B12 absorption - rarely a clinical problem
• lactic acidosis with severe liver disease or renal failure
o it is now increasingly recognised that lactic acidosis secondary to
metformin is rare, although it remains important in the context of exams

Contraindications

• chronic kidney disease: NICE recommend that the dose should be reviewed if
the creatinine is > 130 µmol/l (or eGFR < 45 ml/min) and stopped if the
creatinine is > 150 µmol/l (or eGFR < 30 ml/min)
• metformin may cause lactic acidosis if taken during a period where there is
tissue hypoxia. Examples include a recent myocardial infarction, sepsis, acute
kidney injury and severe dehydration
• iodine-containing x-ray contrast media: examples include peripheral arterial
angiography, coronary angiography, intravenous pyelography (IVP); there is
an increasing risk of provoking renal impairment due to contrast nephropathy;
metformin should be discontinued on the day of the procedure and for 48
hours thereafter
• alcohol abuse is a relative contraindication

Starting metformin
• metformin should be titrated up slowly to reduce the incidence of
gastrointestinal side-effects
• if patients develop unacceptable side-effects then modified-release metformin
should be considered; like if the patient has developed an acute kidney injury
metformin should be withheld due to the risk of lactic acidosis. In the long term,
paroxetine may also need to be stopped as SSRIs can contribute to urinary retention.

Gastrointestinal side-effects are commonly associated with metformin, particularly at high


doses. Modified-release metformin may be better tolerated compared to immediate-
release formulations and should be tried before switching to a second-line agent. Patients
taking up to 2 g daily of the standard-release metformin may start with the same daily dose
of metformin modified-release.

If a trial of modified-release metformin failed, switching to gliclazide would be a reasonable


choice. It should be noted that gliclazide can cause hypoglycaemia so would be unsuitable
for some patients.

Pioglitazone would be contraindicated with a history of bladder cancer and a modified-


release metformin should be trialled first.

If a trial of modified-release metformin failed, switching to sitagliptin would be a reasonable


choice. The dose of sitagliptin should be reduced in chronic kidney disease.

Insulin is not indicated at this stage. HbA1c should be checked at 3-6 monthly intervals and
insulin should be considered if blood glucose levels are inadequately controlled despite dual
therapy with metformin plus another oral hypoglycaemic drug, or other antidiabetic drugs
are not tolerated.

**Switching to a modified-release formulation of metformin is the most appropriate answer.


Modified-release formulations typically result in fewer GI side-effects than other
formulations. As metformin is working for this patient it would be appropriate to keep him
on it rather than trying another medication that might not work as effectively.

Keeping the patient on the same formulation of metformin is not the best solution. He is
having significant GI side-effects and is unlikely to tolerate these for much longer. Modified-
release formulations should be able to control his diabetes while limiting these side effects.

Stopping all antidiabetic medications is not the best option. This patient has failed to make
any lifestyle changes so is unlikely to be able to control his diabetes without medical
support.

While switching to a second-line agent like a sulphonlyurea may be effective at managing his
diabetes and reducing GI side-effects there is a risk that these will not effectively control his
condition and introduce troublesome side-effects of their own. These are better saved for
when, or if, he requires combination therapy.

GLP-1 receptor antagonists are effective antidiabetic medications, however they are
expensive and inconvenient. These medications are delivered by injection which is not
preferable for the patient, so other medications should be tried before these are indicated.

**Metformin should be discontinued for 48 hours following a contrast CT due to the risk of
renal impairment. There is no need to discontinue statins prior to or following a contrast CT
scan.

Metformin should also be stopped in chronic kidney disease if the creatinine is > 150 mmol/l
(or eGFR < 30 ml/min).

**Bob, 73, has chronic obstructive pulmonary disease (COPD) and type 2 diabetes mellitus.
He has been experiencing diarrhoea and vomiting for the past 3 days and his family have
brought him to the emergency department as they are worried. History, examination and
blood tests reveal Bob to be dehydrated and to have developed an acute kidney injury with
an estimated glomerular filtration rate (eGFR) of 29ml/min/1.73m². According to NICE
guidelines, which one of the following medications should be stopped?
>> Metformin is the drug that needs to be stopped in this case. NICE
recommendations on the use of metformin in the treatment of diabetes mellitus
specify that:

• A review of the dose of metformin be undertaken if the serum creatinine


exceeds 130 micromol/litre or the estimated glomerular filtration rate (eGFR) is
below 45 ml/minute/1.73m²
• Stop Metformin if the serum creatinine exceeds 150 micromol/litre or the eGFR
is below 30 ml/minute/1.73m²

10. Digoxin and digoxin toxicity

Digoxin is a cardiac glycoside now mainly used for rate control in the management
of atrial fibrillation. As it has positive inotropic properties it is sometimes used for
improving symptoms (but not mortality) in patients with heart failure.

Mechanism of action

• decreases conduction through the atrioventricular node which slows the


ventricular rate in atrial fibrillation and flutter
• increases the force of cardiac muscle contraction due to inhibition of the Na+/K+
ATPase pump. Also stimulates vagus nerve
• digoxin has a narrow therapeutic index

Monitoring

• digoxin level is not monitored routinely, except in suspected toxicity


• if toxicity is suspected, digoxin concentrations should be measured within 8 to
12 hours of the last dose

Digoxin toxicity

Plasma concentration alone does not determine whether a patient has developed
digoxin toxicity. Toxicity may occur even when the concentration is within the
therapeutic range. The BNF advises that the likelihood of toxicity increases
progressively from 1.5 to 3 mcg/l.

Features

• generally unwell, lethargy, nausea & vomiting, anorexia, confusion, yellow-green


vision
• arrhythmias (e.g. AV block, bradycardia)
• gynaecomastia

Precipitating factors

• classically: hypokalaemia
o digoxin normally binds to the ATPase pump on the same site as
potassium. Hypokalaemia → digoxin more easily bind to the ATPase
pump → increased inhibitory effects
• increasing age
• renal failure
• myocardial ischaemia
• hypomagnesaemia, hypercalcaemia, hypernatraemia, acidosis
• hypoalbuminaemia
• hypothermia
• hypothyroidism
• drugs: amiodarone, quinidine, verapamil, diltiazem, spironolactone (competes
for secretion in distal convoluted tubule therefore reduce excretion), ciclosporin.
Also drugs which cause hypokalaemia e.g. thiazides and loop diuretics

Management
• Digibind
• correct arrhythmias
• monitor potassium

**Yellow vision, bradycardia, nausea and vomiting are symptoms suggestive of digoxin
toxicity. Digoxin toxicity can be precipitated by numerous drugs and other factors,
particularly hypokalemia. As thiazide diuretics increase the likelihood of hypokalemia, they
may precipitate digoxin toxicity.

Bisoprolol may cause bradycardia but would not explain the other symptoms.

Atorvastatin and ramipril are not recognised precipitants of digoxin toxicity.

11. Overdose and poisoning: management

The table below outlines the main management for common overdoses:

Toxin Treatment
• activated charcoal if ingested < 1 hour ago
• N-acetylcysteine (NAC)
Paracetamol
• liver transplantation
Toxin Treatment
• urinary alkalinization with IV bicarbonate
Salicylate • haemodialysis

Opioid/opiates Naloxone
Flumazenil
The majority of overdoses are managed with supportive care
Benzodiazepines
only due to the risk of seizures with flumazenil. It is generally
only used with severe or iatrogenic overdoses.
• IV bicarbonate may reduce the risk of seizures and
arrhythmias in severe toxicity
• arrhythmias: class 1a (e.g. Quinidine) and class Ic
antiarrhythmics (e.g. Flecainide) are contraindicated as
they prolong depolarisation. Class III drugs such as
Tricyclic
amiodarone should also be avoided as they prolong the
antidepressants
QT interval. Response to lignocaine is variable and it
should be emphasized that correction of acidosis is the
first line in management of tricyclic induced arrhythmias
• dialysis is ineffective in removing tricyclics

• mild-moderate toxicity may respond to volume


resuscitation with normal saline
• haemodialysis may be needed in severe toxicity
Lithium • sodium bicarbonate is sometimes used but there is
limited evidence to support this. By increasing the
alkalinity of the urine it promotes lithium excretion

Warfarin Vitamin K, prothrombin complex


Heparin Protamine sulphate
• if bradycardic then atropine
Beta-blockers • in resistant cases glucagon may be used

• ethanol has been used for many years


• works by competing with ethylene glycol for the enzyme
alcohol dehydrogenase
• this limits the formation of toxic metabolites (e.g.
Glycoaldehyde and glycolic acid) which are responsible
Ethylene glycol
for the haemodynamic/metabolic features of poisoning
• fomepizole, an inhibitor of alcohol dehydrogenase, is
now used first-line in preference to ethanol
• haemodialysis also has a role in refractory cases
Toxin Treatment
• fomepizole or ethanol
Methanol
• haemodialysis
poisoning

• atropine
Organophosphate • the role of pralidoxime is still unclear - meta-analyses to
insecticides date have failed to show any clear benefit

Digoxin Digoxin-specific antibody fragments


Iron Desferrioxamine, a chelating agent
Lead Dimercaprol, calcium edetate
• 100% oxygen
Carbon monoxide • hyperbaric oxygen

Hydroxocobalamin; also combination of amyl nitrite, sodium


Cyanide
nitrite, and sodium thiosulfate
The typical signs of acute opioid toxicity - bradypnoea, miosis and altered mental status.
Naloxone is administered to patients with suspected opiate overdose, with the main aim of
restoring adequate ventilation. In apnoeic patients, ventilation and oxygenation with a bag-
valve mask is recommended to decrease the risk of acute lung injury in hypercapnoeic
subjects.

Most cases of benzodiazepine overdose are managed expectantly with supportive care.
Flumazenil is a competitive antagonist of the benzodiazepine receptor and its use is usually
reserved for iatrogenic cases (e.g. sedation following general anaesthesia). Flumazenil can
precipitate withdrawal seizures in patients with chronic benzodiazepine use and is therefore
contraindicated in this patient group.
**An alcoholic man is brought to the Emergency Department. His friend says he has drunk
two bottles of antifreeze --The correct answer is: Fomepizole
12. Prescribing in pregnant patients

Very few drugs are known to be completely safe in pregnancy. The list below largely
comprises of those known to be harmful. Some countries have developed a grading
system - see the link.

Antibiotics
• tetracyclines
• aminoglycosides
• sulphonamides and trimethoprim
• quinolones: the BNF advises to avoid due to arthropathy in some animal studies
Other drugs
• ACE inhibitors, angiotensin II receptor antagonists
• statins
• warfarin
• sulfonylureas
• retinoids (including topical)
• cytotoxic agents
The majority of antiepileptics including valproate, carbamazepine and phenytoin are
known to be potentially harmful. The decision to stop such treatments however is
difficult as uncontrolled epilepsy is also a risk

13. Prescribing in patients with heart failure

The following medications may exacerbate heart failure:

• thiazolidinediones
o pioglitazone is contraindicated as it causes fluid retention
• verapamil
o negative inotropic effect
• NSAIDs/glucocorticoids
o should be used with caution as they cause fluid retention
o low-dose aspirin is an exception - many patients will have coexistent
cardiovascular disease and the benefits of taking aspirin easily outweigh
the risks
• class I antiarrhythmics
o flecainide (negative inotropic and proarrhythmic effect)

Non-steroidal anti-inflammatories, such as diclofenac, are important to avoid in heart


failure as they can promote fluid retention, which can exacerbate heart failure.

Sildenafil is occasionally used for management of pulmonary hypertension secondary


to congestive heart failure.

Some anti-diabetic medications are important to avoid in patients with heart failures,
such as pioglitazone, which can also cause fluid retention. However, metformin is
considered safe to use unless the patient is in acute heart failure.

Gliclazide and exenatide are considered safe to use in patients with chronic heart
failure.

14. Prescribing in patients with renal failure

Questions regarding which drugs to avoid in renal failure are common

Drugs to avoid in renal failure

• antibiotics: tetracycline, nitrofurantoin


• NSAIDs
• lithium
• metformin

Drugs likely to accumulate in chronic kidney disease - need dose adjustment

• most antibiotics including penicillins, cephalosporins, vancomycin, gentamicin,


streptomycin
• digoxin, atenolol
• methotrexate
• sulphonylureas
• furosemide
• opioids

Drugs relatively safe - can sometimes use normal dose depending on the degree of
chronic kidney disease

• antibiotics: erythromycin, rifampicin


• diazepam
• warfarin

antibiotic prescribing in chronic kidney disease (CKD):


Many drugs need dose adjustment in renal disease due to changes in drug metabolism and
also pharmacokinetics. Often this dose adjustment is made on the level of the estimated
glomerular filtration rate (eGFR) which is a calculated surrogate of renal function using the
serum creatinine. Stages of chronic kidney disease are classified according to the eGFR; stage
3 CKD equates to an eGFR of 30-59ml/min.

Nitrofurantoin is a relatively old and unique antibiotic which has enjoyed a new lease of life
with increasing antibiotic resistance. It is actually an inactive pro-drug which is reduced in
vivo to active forms by the bacterial flavoprotein nitrofuran reductase, and it is these reduced
forms of the drug which exert their antibiotic properties by damaging bacterial proteins. In
order to be effective at treating urinary tract infections, nitrofurantoin needs to be
concentrated in the urine and an adequate glomerular filtration is required for this to occur.
An eGFR of less than 40-60ml/min means that the drug is wholly ineffective as a bactericidal
agent and is not recommended in patients with CKD stage 3 or worse due to the likelihood of
treatment failure. Coupled with this is the risk of drug toxicity in the patient. Without
adequate renal filtration, the drug is likely to accumulate. Although bacterial flavoproteins
activate nitrofurantoin more readily, human enzymes can reduce this drug to generate many
highly active radical species, which can cause side effects including peripheral neuropathy,
which may not be reversible, hepatotoxicity and acute and chronic pulmonary reactions and
fibrosis.

Patients taking nitrofurantoin should be advised that this drug will discolour the urine. It is
also a safe drug to use in pregnancy except at full term when there is a risk of haemolysis in
the neonate.
Amoxicillin and co-amoxiclav are widely used antibiotics in the treatment of urinary tract
infections and are relatively safe in renal impairment. Dose reduction is recommended in
severe chronic renal disease, i.e. an eGFR <15-30ml/min to avoid the risk of crystalluria.
Similarly, a reduction in dose is necessary for ciprofloxacin in CKD to avoid crystalluria
although this is recommended from an eGFR of 30-60ml/min.

Trimethoprim is an antibiotic which is entirely safe to use in all but the most severe forms of
chronic kidney disease where a modest dose adjustment is required. It should be noted
however that use of trimethoprim is likely to affect the results of renal function tests since the
drug inhibits tubular secretion of creatinine leading to a rise in serum levels in all patients,
including those with previously normal renal function. This is without any effect on the
glomerular filtration rate.

15. Quinolones

Quinolones are a group of antibiotics which work by inhibiting DNA synthesis and
are bactericidal in nature. Examples include:

• ciprofloxacin
• levofloxacin

Mechanism of action: inhibit topoisomerase II (DNA gyrase) and topoisomerase IV


Mechanism of resistance: mutations to DNA gyrase, efflux pumps which reduce
intracellular quinolone concentration

Adverse effects

• lower seizure threshold in patients with epilepsy


• tendon damage (including rupture) - the risk is increased in patients also taking
steroids. (This patient has classical signs of Achilles tendon rupture. Tendon damage is
a well-documented complication of quinolone therapy. It appears to be an idiosyncratic
reaction, with the actual median duration of treatment being 8 days before problems
occur)
• cartilage damage has been demonstrated in animal models and for this reason
quinolones are generally avoided (but not necessarily contraindicated) in
children
• lengthens QT interval

Contraindications

• Quinolones should generally be avoided in women who are pregnant or


breastfeeding
• avoid in G6PD

16. Carbon monoxide poisoning


Carbon monoxide has a high affinity for haemoglobin and myoglobin resulting in a
left-shift of the oxygen dissociation curve and tissue hypoxia. There are
approximately 50 per year deaths from accidental carbon monoxide poisoning in the
UK.

Pathophysiology

• in carbon monoxide poisoning the oxygen saturation of haemoglobin decreases


leading to an early plateau in the oxygen dissociation curve

Questions may hint at badly maintained housing e.g. student houses. Confusion and
pink mucosae are typical features of carbon monoxide poisoning. A low-grade pyrexia is
seen in a minority of cases.

Features of carbon monoxide toxicity

• headache: 90% of cases


• nausea and vomiting: 50%
• vertigo: 50%
• confusion: 30%
• subjective weakness: 20%
• severe toxicity: 'pink' skin and mucosae, hyperpyrexia, arrhythmias,
extrapyramidal features, coma, death

Investigations

• pulse oximetry may be falsely high due to similarities between oxyhaemoglobin


and carboxyhaemoglobin
• therefore a venous or arterial blood gas should be taken
• typical carboxyhaemoglobin levels
o < 3% non-smokers
o < 10% smokers
o 10 - 30% symptomatic: headache, vomiting
o > 30% severe toxicity
• an ECG is a useful supplementary investgation to look for cardiac ischaemia

Management

• patients with suspected carbon monoxide poisoning should be assessed in the


emergency department
• 100% high-flow oxygen via a non-rebreather mask
o from a physiological perspective, this decreases the half-life of
carboxyhemoglobin (COHb)
o should be administered as soon as possible, with treatment continuing
for a minimum of six hours
o target oxygen saturations are 100%
o treatment is generally continued until all symptoms have resolved, rather
than monitoring CO levels
• hyperbaric oxygen
o due to the small number of cases the evidence base is limited, but there
is some evidence that long-term outcomes may be better than standard
oxygen therapy for more severe cases
o therefore, discussion with a specialist should be considered for more
severe cases (e.g. levels > 25%)
o in 2008, the Department of Health publication 'Recognising Carbon
Monoxide Poisoning' also listed loss of consciousness at any point,
neurological signs other than headache, myocardial ischaemia or
arrhythmia and pregnancy as indications for hyperbaric oxygen

17. Finasteride

Finasteride is an inhibitor of 5 alpha-reductase, an enzyme which metabolises


testosterone into dihydrotestosterone.

Indications

• benign prostatic hyperplasia


• male-pattern baldness

Adverse effects

• impotence
• decrease libido
• ejaculation disorders
• gynaecomastia and breast tenderness

Finasteride causes decreased levels of serum prostate-specific antigen

**There are a number of causes of gynaecomastia in males and it is important to rule out
sinister ones such as kidney failure, endocrine disturbances, liver failure or malignancy.

Another key cause is medication-related, in this case, the finasteride taken by this patient can
cause gynaecomastia.

Finasteride works by blocking 5-alpha-reductase thus reducing the production of


dihydrotestosterone and therefore shrinking the prostate. However, side effects can include
gynaecomastia and sexual dysfunction.
** Finasteride is a 5-alpha reductase inhibitor given to patients suffering from benign
prostatic hyperplasia. By blocking 5-alpha reductase it stops the conversion of testosterone
to dihydrotestosterone, thereby reducing the size of the prostate.

Trimethoprim works by binding to dihydrofolate reductase, therefore, interfering with


bacterial DNA synthesis.

Tamsulosin is an alpha blocker which selectively blocks the alpha 1 receptors in the bladder
neck and prostate causing a relaxation of the smooth muscle.

Oxybutynin has a direct spasmolytic effect on the bladder smooth muscle by competitively
antagonising the muscarinic receptors on the bladder.

Finally sildenafil inhibits cGMP specific phosphodiesterase type 5 therefore helping to


improve blood flow to the penis to help with erectile dysfunction.

18. Tuberculosis: drug side-effects and mechanism of action

Rifampicin

• mechanism of action: inhibits bacterial DNA dependent RNA polymerase


preventing transcription of DNA into mRNA
• potent liver enzyme inducer
• hepatitis, orange secretions
• flu-like symptoms

Isoniazid

• mechanism of action: inhibits mycolic acid synthesis


• peripheral neuropathy: prevent with pyridoxine (Vitamin B6)
• hepatitis, agranulocytosis
• liver enzyme inhibitor

Pyrazinamide

• mechanism of action: converted by pyrazinamidase into pyrazinoic acid which in


turn inhibits fatty acid synthase (FAS) I
• hyperuricaemia causing gout
• arthralgia, myalgia
• hepatitis

Ethambutol:Visual acuity and renal function should also be checked prior to starting
• mechanism of action: inhibits the enzyme arabinosyl transferase which
polymerizes arabinose into arabinan
• optic neuritis: check visual acuity before and during treatment
• dose needs adjusting in patients with renal impairment

19. Alcohol - problem drinking: management

Nutritional support

• SIGN recommends alcoholic patients should receive oral thiamine if their 'diet
may be deficient'

Drugs used

• benzodiazepines for acute withdrawal


• disulfram: promotes abstinence - alcohol intake causes severe reaction due to
inhibition of acetaldehyde dehydrogenase. Patients should be aware that even
small amounts of alcohol (e.g. In perfumes, foods, mouthwashes) can produce
severe symptoms. Contraindications include ischaemic heart disease and
psychosis
• acamprosate: reduces craving, known to be a weak antagonist of NMDA
receptors, improves abstinence in placebo controlled trials

** Whilst vitamin B compound is widely prescribed it is not recommended in recent


guidelines, for example SIGN.

20. Side-effects of common drugs: diabetes drugs

The table below summarises characteristic (if not necessarily the most common) side-
effects of drugs used to treat diabetes mellitus

Drug Side-effect
Gastrointestinal side-effects
Metformin
Lactic acidosis
Hypoglycaemic episodes
Increased appetite and weight gain
Sulfonylureas
Syndrome of inappropriate ADH secretion
Liver dysfunction (cholestatic)
Weight gain
Fluid retention
Glitazones
Liver dysfunction
Fractures
Drug Side-effect
Gliptins Pancreatitis

21. Side-effects of common drugs: anti-hypertensives

The table below summarises characteristic (if not necessarily the most common) side-
effects of drugs used to treat hypertension

Drug Side-effect
• Cough
ACE inhibitors
• Hyperkalaemia
• Gout
• Hypokalaemia
Bendroflumethiazide
• Hyponatraemia
• Impaired glucose tolerance
• Headache
Calcium channel blockers • Flushing
• Ankle oedema
• Bronchospasm (especially in asthmatics)
Beta-blockers • Fatigue
• Cold peripheries
Doxazosin • Postural hypotension

22. Side-effects of common drugs: antibiotics

The table below summarises characteristic (if not necessarily the most common) side-
effects of drugs used antibiotics

Drug Side-effect
Amoxicillin Rash with infectious mononucleosis
Co-amoxiclav Cholestasis
Flucloxacillin Cholestasis (usually develops several weeks after use)
Gastrointestinal upset
Erythromycin
Prolongs QT interval
Lowers seizure threshold
Ciprofloxacin
Tendonitis
Metronidazole Reaction following alcohol ingestion
Doxycycline Photosensitivity
Rashes, including photosensitivity
Trimethoprim Pruritus
Suppression of haematopoiesis
23. Lithium toxicity

Lithium is a mood stabilising drug used most commonly prophylactically in bipolar


disorder but also as an adjunct in refractory depression. It has a very narrow
therapeutic range (0.4-1.0 mmol/L) and a long plasma half-life being excreted
primarily by the kidneys. Lithium toxicity generally occurs following concentrations >
1.5 mmol/L.

Toxicity may be precipitated by:

• dehydration
• renal failure
• drugs: diuretics (especially thiazides), ACE inhibitors/angiotensin II receptor
blockers, NSAIDs and metronidazole.

Features of toxicity

• coarse tremor (a fine tremor is seen in therapeutic levels)


• hyperreflexia
• acute confusion
• polyuria
• seizure
• coma

Management

• mild-moderate toxicity may respond to volume resuscitation with normal saline


• haemodialysis may be needed in severe toxicity
• sodium bicarbonate is sometimes used but there is limited evidence to support
this. By increasing the alkalinity of the urine it promotes lithium excretion

This patient is displaying signs of lithium toxicity. It is likely that he has become dehydrated
due to fasting and this has led to high serum lithium levels. The normal range is 0.4-1.0,
toxicity symptoms are seen when levels are >1.5. Non-steroidal anti-inflammatory
medications such as ibuprofen can also decrease lithium excretion so should not be used.

Common side effects of lithium include a fine tremor, a metallic taste in the mouth, thirst,
polyuria and weight gain.

Side effects of sodium valproate include hair loss, nausea, diarrhoea, weight gain, drowsiness
and a small increased risk of suicidal thoughts. Symptoms seen in sodium valproate toxicity
include central nervous system depression up to and including coma, ataxia, tachycardia and
electrolyte disturbances.

Whilst ibuprofen can cause GI upset, it does not cause the majority of this patient's symptoms
therefore is an incorrect answer.
** Lithium toxicity can be precipitated by thiazides- Indapamide

Thiazide diuretics can cause a rapid increase in serum lithium levels (7–10 days) by
reducing the clearance of lithium. The increase in lithium levels varies from 25–400%.
Indapamide is a thiazide-like diuretic and therefore, the correct answer.

Amlodipine would not result in lithium toxicity. Angiotensin-converting enzyme (ACE)


inhibitors decrease the excretion of lithium and can also potentially precipitate renal
failure which could result in lithium toxicity.

Paracetamol would also not result in lithium toxicity. Non-steroidal anti-inflammatory


(NSAID) medications such as oral ibuprofen or naproxen may increase lithium levels.
This would not be the case with topical NSAIDs or oral paracetamol for analgesia.

There is not a clear interaction between omeprazole and lithium toxicity. Omeprazole
can result in hypomagnesemia and theoretically, this electrolyte imbalance could
lead to issues but this would be much less common.

24. Paracetamol overdose: management

The following is based on 2012 Commission on Human Medicines (CHM) review of


paracetamol overdose management. The big change in these guidelines was the
removal of the 'high-risk' treatment line on the normogram. All patients are therefore
treated the same regardless of risk factors for hepatotoxicity. The National Poisons
Information Service/TOXBASE should always be consulted for situations outside of
the normal parameters.

The minority of patients who present within 1 hour may benefit from activated
charcoal to reduce absorption of the drug.

Acetylcysteine should be given if:

• there is a staggered overdose (an overdose is considered staggered if all the


tablets were not taken within 1 hour)or there is doubt over the time of
paracetamol ingestion, regardless of the plasma paracetamol concentration;
or
• the plasma paracetamol concentration is on or above a single treatment line
joining points of 100 mg/L at 4 hours and 15 mg/L at 15 hours, regardless of risk
factors of hepatotoxicity

Acetylcysteine is now infused over 1 hour (rather than the previous 15 minutes) to
reduce the number of adverse effects. Acetylcysteine commonly causes an
anaphylactoid reaction (non-IgE mediated mast cell release). Anaphylactoid reactions
to IV acetylcysteine are generally treated by stopping the infusion, then restarting at
a slower rate.

King's College Hospital criteria for liver transplantation (paracetamol liver failure)
Arterial pH < 7.3, 24 hours after ingestion
or all of the following:
 prothrombin time > 100 seconds
 creatinine > 300 µmol/l
 grade III or IV encephalopathy

25. Drug-induced urinary retention

The following drugs may cause urinary retention:

• tricyclic antidepressants e.g. amitriptyline


• anticholinergics
• opioids
• NSAIDs
• disopyramide

Codeine is an opioid, and a common side effect of this class of medication is urinary
retention. Other drugs that can cause this are tricyclic antidepressants, anticholinergics, and
NSAIDs.
Doxazosin is an alpha-blocker and is used as a first-line treatment for improving the
symptoms of benign prostatic hyperplasia. It works by reducing the resistance to bladder
outflow, hence is used to treat retention.
Duloxetine is an SSRI used to treat diabetic neuropathy and does not include urinary
retention in its side effect profile.
Finasteride is a 5-alpha reductase inhibitor, used as a second-line treatment for improving
the symptoms of benign prostatic hyperplasia where alpha-blockers are not effective. It
works by reducing prostatic bulk.
Loratadine is a non-sedating antihistamine used for symptomatic relief of allergy. It does not
cause urinary retention, but keep in mind that some older antihistamines can.

Drug-induced thrombocytopenia

Drug-induced thrombocytopenia (probable immune-mediated)


• quinine (nocturnal leg cramp)
• abciximab
• NSAIDs
• diuretics: furosemide
• antibiotics: penicillins, sulphonamides, rifampicin
• anticonvulsants: carbamazepine, valproate
• heparin
26. Opioid misuse

Opioids are substances which bind to opioid receptors. This includes both naturally
occurring opiates such as morphine and synthetic opioids such as buprenorphine
and methadone.

Features of opioid misuse


• rhinorrhoea
• needle track marks
• pinpoint pupils
• drowsiness
• watering eyes
• yawning
Complications of opioid misuse

• viral infection secondary to sharing needles: HIV, hepatitis B & C


• bacterial infection secondary to injection: infective endocarditis, septic arthritis,
septicaemia, necrotising fasciitis
• venous thromboembolism
• overdose may lead to respiratory depression and death
• psychological problems: craving
• social problems: crime, prostitution, homelessness

Emergency management of opioid overdose

• IV or IM naloxone: has a rapid onset and relatively short duration of action

Harm reduction interventions may include

• needle exchange
• offering testing for HIV, hepatitis B & C

Management of opioid dependence

• patients are usually managed by specialist drug dependence clinics although


some GPs with a specialist interest offer similar services
• patients may be offered maintenance therapy or detoxification
• NICE recommend methadone or buprenorphine as the first-line treatment in
opioid detoxification
• compliance is monitored using urinalysis
• detoxification should normally last up to 4 weeks in an inpatient/residential
setting and up to 12 weeks in the community
27. Ecstasy poisoning

Ecstasy (MDMA, 3,4-Methylenedioxymethamphetamine) use became popular in the


1990's during the emergence of dance music culture

Clinical features

• neurological: agitation, anxiety, confusion, ataxia


• cardiovascular: tachycardia, hypertension
• hyponatraemia
• hyperthermia
• rhabdomyolysis

Management

• supportive
• dantrolene may be used for hyperthermia if simple measures fail

** In many cases of intoxication (both drug and alcohol related) it may be difficult to
ascertain details from the patient themselves and as such you may need to question friends
and family. In this case the friends give a clue that being on a night out, there could be the
involvement of alcohol or drugs.

The clinical picture then gives you a better idea of what could have been taken. MDMA (the
active ingredient in ecstasy) often leads to feelings of euphoria, increased sociability and
empathy, enhanced senses and mild hallucinations. It can also have some quite serious side
effects including dehydration, hyperthermia, insomnia, tachycardia and profuse sweating.
This patient displays many of these signs.

It is important to note that many users of Ecstasy will be aware of dehydration as a side
effect and to compensate may often drink too much water which can put then in a state of
hyponatraemia which in extreme circumstances can be fatal, therefore it is important to
measure electrolytes before rapidly correcting with fluids (especially sodium chloride).

28. Amiodarone and the thyroid gland

Around 1 in 6 patients taking amiodarone develop thyroid dysfunction

Amiodarone-induced hypothyroidism

The pathophysiology of amiodarone-induced hypothyroidism (AIH) is thought to be


due to the high iodine content of amiodarone causing a Wolff-Chaikoff effect*

Amiodarone may be continued if this is desirable


Amiodarone-induced thyrotoxicosis

Amiodarone-induced thyrotoxicosis (AIT) may be divided into two types:


AIT type 1 AIT type 2
Excess iodine-induced thyroid Amiodarone-related destructive
Pathophysiology
hormone synthesis thyroiditis
Goitre Present Absent
Carbimazole or potassium
Management Corticosteroids
perchlorate

Unlike in AIH, amiodarone should be stopped if possible in patients who develop AIT

Wolff-Chaikoff effect : an autoregulatory phenomenon where thyroxine formation is


inhibited due to high levels of circulating iodide.

Amiodarone frequently causes abnormalities in thyroid function tests and may cause both
hypothyroidism and hyperthyroidism.

It may cause the former by interfering with the conversion of thyroxine (T4) to tri-
iodothyronine (T3) and it may produce the latter either through thyroiditis or donation of
iodine (amiodarone contains a large quantity of iodine).

Other side effects of amiodarone include pulmonary fibrosis, corneal deposits,


photosensitivity reactions and derangement in liver function tests.

Amiodarone: adverse effects

Adverse effects of amiodarone use


• thyroid dysfunction: both hypothyroidism and hyper-thyroidism
• corneal deposits
• pulmonary fibrosis/pneumonitis
• liver fibrosis/hepatitis
• peripheral neuropathy, myopathy
• photosensitivity
• 'slate-grey' appearance
• thrombophlebitis and injection site reactions
• bradycardia
• lengths QT interval
Important drug interactions of amiodarone include:
• decreased metabolism of warfarin, therefore increased INR
• increased digoxin levels
29. Drug monitoring
The tables below show the monitoring requirements of common drugs. It should be
noted these are basic guidelines and do not relate to monitoring effectiveness of
treatment (e.g. Checking lipids for patients taking a statin)

Cardiovascular drugs

Main monitoring
Drug Details of monitoring
parameters
LFTs at baseline, 3 months and 12
Statins LFT
months
U&E prior to treatment
ACE
U&E U&E after increasing dose
inhibitors
U&E at least annually
TFT, LFT, U&E, CXR prior to treatment
Amiodarone TFT, LFT
TFT, LFT every 6 months

Rheumatology drugs

Main
Drug monitoring Details of monitoring
parameters
The Committee on Safety of Medicines recommend
'FBC and renal and LFTs before starting treatment and
Methotrexate FBC, LFT, U&E
repeated weekly until therapy stabilised, thereafter
patients should be monitored every 2-3 months'
FBC, LFT before treatment
Azathioprine FBC, LFT FBC weekly for the first 4 weeks
FBC, LFT every 3 months

Neuropsychiatric drugs

Main monitoring
Drug Details of monitoring
parameters
TFT, U&E prior to treatment
Lithium levels weekly until stabilised then
Lithium Lithium level, TFT, U&E
every 3 months
TFT, U&E every 6 months
Sodium LFT, FBC before treatment
LFT
valproate LFT 'periodically' during first 6 months

Endocrine drugs
Drug Main monitoring parameters Details of monitoring
LFT before treatment
Glitazones LFT
LFT 'regularly' during treatment

30. Tamoxifen

Tamoxifen is a Selective oEstrogen Receptor Modulator (SERM) which acts as an


oestrogen receptor antagonist and partial agonist. It is used in the management of
oestrogen receptor positive breast cancer

Adverse effects

• menstrual disturbance: vaginal bleeding, amenorrhoea


• hot flushes - 3% of patients stop taking tamoxifen due to climacteric side-effects
• venous thromboembolism
• endometrial cancer

Tamoxifen is typically used for 5 years following removal of the tumour.

Raloxifene is a pure oestrogen receptor antagonist, and carries a lower risk of


endometrial cancer

31. Aspirin

Aspirin works by blocking the action of both cyclooxygenase-1 and 2.


Cyclooxygenase is responsible for prostaglandin, prostacyclin and thromboxane
synthesis. The blocking of thromboxane A2 formation in platelets reduces the ability
of platelets to aggregate which has lead to the widespread use of low-dose aspirin in
cardiovascular disease. Until recent guidelines changed all patients with established
cardiovascular disease took aspirin if there was no contraindication. Following the
2010 technology appraisal of clopidogrel this is no longer the case*.

Two recent trials (the Aspirin for Asymptomatic Atherosclerosis and the
Antithrombotic Trialists Collaboration) have cast doubt on the use of aspirin in
primary prevention of cardiovascular disease. Guidelines have not yet changed to
reflect this. However the Medicines and Healthcare products Regulatory Agency
(MHRA) issued a drug safety update in January 2010 reminding prescribers that
aspirin is not licensed for primary prevention.

What do the current guidelines recommend?

• first-line for patients with ischaemic heart disease


Potentiates

• oral hypoglycaemics
• warfarin
• steroids

Aspirin should not be used in children under 16 due to the risk of Reye's syndrome.
An exception is Kawasaki disease, where the benefits are thought to outweigh the
risks.

*NICE now recommend clopidogrel first-line following an ischaemic stroke and for
peripheral arterial disease. For TIAs the situation is more complex. Recent Royal
College of Physician (RCP) guidelines support the use of clopidogrel in TIAs. However
the older NICE guidelines still recommend aspirin + dipyridamole - a position the
RCP state is 'illogical'
In the BNF section 'Prescribing in dental practice' it advises that patients in this
situation should continue taking anti-platelets as normal

32. Gentamicin

Gentamicin is a type of aminoglycoside antibiotic. It is poorly lipid-soluble and is


therefore given parentally (e.g. for infective endocarditis) or topically (e.g. for otitis
externa).

Adverse effects

• ototoxicity
o due to auditory or vestibular nerve damage
o irreversible
• nephrotoxicity
o accumulates in renal failure
o the toxicity is secondary to acute tubular necrosis
o concomitant use of furosemide increases the risk
o lower doses and more frequent monitoring is required

Contraindications: myasthenia gravis

Dosing

• due to the significant ototoxic and nephrotoxic potential of gentamicin it is


important to monitor plasma concentrations
• both peak (1 hour after administration) and trough levels (just before the next
dose) are measured
• if the trough (pre-dose) level is high the interval between the doses should be
increased
• if the peak (post-dose) level is high the dose should be decreased

33. Cocaine

Cocaine is an alkaloid derived from the coca plant. It is widely used as a recreational
stimulant. The price of cocaine has fallen sharply in the past decade resulting in
cocaine toxicity becoming a much more frequent clinical problem. This increase has
made cocaine a favourite topic of question writers.

Mechanism of action: cocaine blocks the uptake of dopamine, noradrenaline and


serotonin

The use of cocaine is associated with a wide variety of adverse effects:

Cardiovascular effects include:


• coronary artery spasm → myocardial ischaemia/infarction
• both tachycardia and bradycardia may occur
• hypertension
• QRS widening and QT prolongation
• aortic dissection

Neurological effects
• seizures
• mydriasis
• hypertonia
• hyperreflexia

Psychiatric effects
• agitation
• psychosis
• hallucinations

Others
• ischaemic colitis is recognised in patients following cocaine ingestion. This
should be considered if patients complain of abdominal pain or rectal bleeding
• hyperthermia
• metabolic acidosis
• rhabdomyolysis

Management of cocaine toxicity


• in general, benzodiazepines are generally first-line for most cocaine-related
problems
• chest pain: benzodiazepines + glyceryl trinitrate. If myocardial infarction
develops then primary percutaneous coronary intervention
• hypertension: benzodiazepines + sodium nitroprusside
• the use of beta-blockers in cocaine-induced cardiovascular problems is a
controversial issue. The American Heart Association issued a statement in 2008
warning against the use of beta-blockers (due to the risk of unopposed alpha-
mediated coronary vasospasm) but many cardiologists since have questioned
whether this is valid. If a reasonable alternative is given in an exam it is probably
wise to choose it

34. Beta-blocker overdose

Features ●bradycardia ●hypotension ● Heart failure ●Syncope


Management
• if bradycardic then atropine and in resistant cases glucagon may be used
Haemodialysis is not effective in beta-blocker overdose and Gastric lavage should only
be attempted (if at all) if patients present within 1-2 hours of taking the overdose.

35. Macrolides

Erythromycin was the first macrolide used clinically. Newer examples include
clarithromycin and azithromycin.

Macrolides act by inhibiting bacterial protein synthesis by blocking translocation. If


pushed to give an answer they are bacteriostatic in nature, but in reality this depends
on the dose and type of organism being treated.

Mechanism of resistance

• post-transcriptional methylation of the 23S bacterial ribosomal RNA

Adverse effects

• prolongation of the QT interval


• gastrointestinal side-effects are common. Nausea is less common with
clarithromycin than erythromycin
• cholestatic jaundice: risk may be reduced if erythromycin stearate is used
• P450 inhibitor (see below)
• azithromycin is associated with hearing loss and tinnitus

Common interactions
• statins should be stopped whilst taking a course of macrolides. Macrolides
inhibit the cytochrome P450 isoenzyme CYP3A4 that metabolises statins. Taking
macrolides concurrently with statins significantly increases the risk of myopathy
and rhabdomyolysis.

**A 40-year-old female presents with symptoms of a productive cough, fever and mild
shortness of breath. She describes the phlegm as green in colour and feels generally run
down and 'under the weather.' On examination she is found to have oxygen saturations of
97% on air and a respiratory rate of 18 breaths per minute. Upon auscultation you can hear
crepitations at the left lung base. Her GP record mentions 'raised QTC on previous ECG' and
she is allergic to penicillin.
Answer: The answer is doxycycline. This is a good choice for treating a lower respiratory tract
infection though it is not recommended first line.

First line is amoxicillin but she can't have this because she is penicillin allergic. It is worth
clarifying with patients what the nature of their allergy is; penicillin can be a lifesaving
treatment and often patients are documented as 'allergic' when they had a mild side effect
such as stomach upset. In these cases, if possible, it's worth arranging a trial dose in a
medical setting to establish whether there is any evidence of actual allergy.

Clarithromycin, along with other macrolides, has a potential side effect of QT prolongation
so should be avoided here in view of the patient's previous ECG. When prescribing a
macrolide antibiotic it's important to tell patients to stop taking their statin for the duration
of their treatment.

Co-amoxiclav is broad-spectrum and thus its use can increase the patient's risk of
developing Clostridium difficile, for this reason, it should be avoided as a first-line treatment
and should usually be initiated only after discussing with a microbiologist.

Metronidazole covers anaerobic bacteria and is not an appropriate treatment for lower
respiratory tract infection.
**Erythromycin may potentially interact with amiodarone, warfarin and simvastatin.
Levofloxacin reacts to a lesser extent with both amiodarone and warfarin.

36. Organophosphate insecticide poisoning

One of the effects of organophosphate poisoning is inhibition of acetylcholinesterase


leading to upregulation of nicotinic and muscarinic cholinergic neurotransmission. In
warfare, sarin gas is a highly toxic synthetic organophosphorus compound that has
similar effects.

Features can be predicted by the accumulation of acetylcholine (mnemonic = SLUD)


• Salivation
• Lacrimation
• Urination
• Defecation/diarrhoea
• cardiovascular: hypotension, bradycardia
• also: small pupils, muscle fasciculation
Management
• atropine
• the role of pralidoxime is still unclear - meta-analyses to date have failed to
show any clear benefit
37. Mefloquine

Mefloquine (brand name Lariam) is used for both the prophylaxis and treatment of
certain types of malaria. There has long been a concern about the neuropsychiatric
side-effects of mefloquine. A recent review has however led to 'strengthened
warnings' about the potential risks.

The following advice is therefore given:

• certain side-effects such nightmares or anxiety may be 'prodromal' of a more


serious neuropsychiatric event
• suicide and deliberate self harm have been reported in patients taking
mefloquine
• adverse reactions may continue for several months due to the long half-life or
mefloquine
• mefloquine should not be used in patients with a history of anxiety, depression
schizophrenia or other psychiatric disorders
• patients who experience neuropsychiatric sife-effects should stop mefloquine
and seek medical advice

38. Adrenaline

Adrenaline is a sympathomimetic amine with both alpha and beta adrenergic


stimulating properties

Indications
• anaphylaxis
• cardiac arrest
Recommend Adult Life Support (ALS) adrenaline doses

• anaphylaxis: 0.5ml 1:1,000 IM


• cardiac arrest: 10ml 1:10,000 IV or 1ml of 1:1000 IV

Management of accidental injection: local infiltration of phentolamine


Background

• responsible for the fight or flight response


• released by the adrenal glands
• acts on α 1 and 2, β 1 and 2 receptors
• acts on β 2 receptors in skeletal muscle vessels-causing vasodilation
• increases cardiac output and total peripheral resistance
• causes vasoconstriction in the skin and kidneys causing a narrow pulse
pressure

Actions on α adrenergic receptors:

• inhibits insulin secretion by the pancreas


• stimulates glycogenolysis in the liver and muscle
• stimulates glycolysis in muscle

Actions onβ adrenergic receptors:

• stimulates glucagon secretion in the pancreas


• stimulates ACTH
• stimulates lipolysis by adipose tissue

Recommend Adult Life Support (ALS) adrenaline doses


•anaphylaxis: 0.5mg - 0.5ml 1:1,000 IM
•cardiac arrest: 1mg - 10ml 1:10,000 IV or 1ml of 1:1000 IV

39.Botulinum toxin

As well as the well-publicised cosmetic uses of Botulinum toxin ('Botox') there are
also a number of licensed indications:

• blepharospasm
• hemifacial spasm
• focal spasticity including cerebral palsy patients, hand and wrist disability
associated with stroke
• spasmodic torticollis
• severe hyperhidrosis of the axillae
• achalasia
40. Diclofenac

The MHRA updated it's guidance on diclofenac in June 2013 after a Europe-wide of
review of cardiovascular safety.

Whilst it has long been known that NSAIDs may be linked to an increased risk of
cardiovascular events the evidence base has now become much clearer. Diclofenac
appears to be associated with a significantly increased risk of cardiovascular events
compared with other NSAIDs.

It is therefore advised that diclofenac is contraindicated in patients with the


following:

• ischaemic heart disease


• peripheral arterial disease
• cerebrovascular disease
• congestive heart failure (New York Heart Association classification II-IV)

Patients should be switched from diclofenac to other NSAIDs such as naproxen or


ibuprofen. This advice does not apply to topical diclofenac.

Studies have shown that naproxen and low-dose ibuprofen have the best
cardiovascular risk profiles of the NSAIDs
Paediatrics

1.Developmental milestones: gross motor

The table below summarises the major gross motor developmental milestones

Age Milestone
Little or no head lag on being pulled to sit
3 months Lying on abdomen, good head control
Held sitting, lumbar curve
Lying on abdomen, arms extended
Lying on back, lifts and grasps feet
6 months Pulls self to sitting
Held sitting, back straight
Rolls front to back
7-8 months Sits without support (Refer at 12 months)
Pulls to standing
9 months
Crawls
Cruises
12 months
Walks with one hand held
13-15 months Walks unsupported (Refer at 18 months)
18 months Squats to pick up a toy
Runs
2 years
Walks upstairs and downstairs holding on to rail
Rides a tricycle using pedals
3 years
Walks up stairs without holding on to rail
4 years Hops on one leg

Notes

• the majority of children crawl on all fours before walking but some children
'bottom-shuffle'. This is a normal variant and runs in families

Developmental milestones: speech and hearing

The table below summarises the major speech and hearing developmental milestones
Age Milestone
Quietens to parents voice
3 months Turns towards sound
Squeals
6 months Double syllables 'adah', 'erleh'
Says 'mama' and 'dada'
9 months
Understands 'no'
12 months Knows and responds to own name
Knows about 2-6 words (Refer at 18 months)
12-15 months
Understands simple commands - 'give it to mummy'
Combine two words
2 years
Points to parts of the body
2½ years Vocabulary of 200 words
Talks in short sentences (e.g. 3-5 words)
Asks 'what' and 'who' questions
3 years
Identifies colours
Counts to 10 (little appreciation of numbers though)
4 years Asks 'why', 'when' and 'how' questions

Developmental milestones: fine motor and vision

The tables below summarises the major fine motor and vision developmental
milestones

Age Milestone
Reaches for object
Holds rattle briefly if given to hand
3 months
Visually alert, particularly human faces
Fixes and follows to 180 degrees
Holds in palmar grasp
6 months Pass objects from one hand to another
Visually insatiable, looking around in every direction
Points with finger
9 months
Early pincer
Good pincer grip
12 months
Bangs toys together

Bricks

Age Milestone
15 months Tower of 2
Age Milestone
18 months Tower of 3
2 years Tower of 6
3 years Tower of 9

Drawing

Age Milestone
18 months Circular scribble
2 years Copies vertical line
3 years Copies circle
4 years Copies cross
5 years Copies square and triangle

Book

Age Milestone
15 months Looks at book, pats page
18 months Turns pages, several at time
2 years Turns pages, one at time

Notes

• hand preference before 12 months is abnormal and may indicate cerebral


palsy

Development problems

Referral points
• doesn't smile at 10 weeks
• cannot sit unsupported at 12 months
• cannot walk at 18 months
Fine motor skill problems
• hand preference before 12 months is abnormal and may indicate cerebral palsy
Gross motor problems
• most common causes of problems: variant of normal, cerebral palsy and
neuromuscular disorders (e.g. Duchenne muscular dystrophy)
Speech and language problems
• always check hearing
• other causes include environmental deprivation and general development delay
Head banging

Head banging is normal behaviour for a 2-year-old. However, if this behaviour


persists beyond 3 years it could be a sign of autism.

2. Cyanosis in the neonatal period

Peripheral cyanosis, for example of the feet and hands, is very common in the first 24 hours
of life and may occur when the child is crying or unwell from any cause
Central cyanosis can be recognised clinically when the concentration of reduced
haemoglobin in the blood exceeds 5g/dl

The nitrogen washout test (also known as the hyperoxia test) may be used to differentiate
cardiac from non-cardiac causes. The infant is given 100% oxygen for ten minutes after
which arterial blood gases are taken. A pO2 of less than 15 kPa indicates cyanotic congenital
heart disease

Causes of cyanotic congenital heart disease

• tetralogy of Fallot (TOF)


• transposition of the great arteries (TGA)
• tricuspid atresia

Initial management of suspected cyanotic congenital heart disease

• supportive care
• prostaglandin E1
o used to maintain a patent ductus arteriosus in ductal-dependent congenital
heart defect

Acrocyanosis

Acrocyanosis is often seen in healthy newborns and refers to the peripheral cyanosis around
the mouth and the extremities (hands and feet) (picture 1). It is caused by benign vasomotor
changes that result in peripheral vasoconstriction and increased tissue oxygen extraction and
is a benign condition [4]. Acrocyanosis is differentiated from other causes of peripheral
cyanosis with significant pathology (eg, septic shock) as it occurs immediately after birth in
healthy infants. It is a common finding and may persist for 24 to 48 hours.

3. X-linked recessive

In X-linked recessive inheritance only males are affected. An exception to this seen in
examinations are patients with Turner's syndrome, who are affected due to only having one
X chromosome. X-linked recessive disorders are transmitted by heterozygote females
(carriers) and male-to-male transmission is not seen. Affected males can only have
unaffected sons and carrier daughters.

Each male child of a heterozygous female carrier has a 50% chance of being affected whilst
each female child of a heterozygous female carrier has a 50% chance of being a carrier.

The possibility of an affected father having children with a heterozygous female carrier is
generally speaking extremely rare. However, in certain Afro-Caribbean communities G6PD
deficiency is relatively common and homozygous females with clinical manifestations of the
enzyme defect are seen.

4. Croup

Croup is a form of upper respiratory tract infection seen in infants and toddlers. It is
characterised by stridor which is caused by a combination of laryngeal oedema and
secretions. Parainfluenza viruses account for the majority of cases.

Epidemiology

• peak incidence at 6 months - 3 years


• more common in autumn

Features

• stridor
• barking cough (worse at night)
• fever
• coryzal symptoms

Clinical Knowledge Summaries (CKS) suggest using the following criteria to grade the
severity:

Mild Moderate Severe

• Occasional barking
cough • Frequent barking
• Frequent barking
• No audible stridor cough
cough
at rest • Prominent inspiratory
• Easily audible stridor
• No or mild (and occasionally,
at rest
suprasternal expiratory) stridor at
• Suprasternal and
and/or intercostal rest
sternal wall
recession • Marked sternal wall
retraction at rest
retractions
Mild Moderate Severe
• The child is happy • No or little distress • Significant distress and
and is prepared to or agitation agitation, or lethargy or
eat, drink, and play • The child can be restlessness (a sign of
placated and is hypoxaemia)
interested in its • Tachycardia occurs with
surroundings more severe
obstructive symptoms
and hypoxaemia

CKS suggest admitting any child with moderate or severe croup. Other features
which should prompt admission include:

• < 6 months of age


• known upper airway abnormalities (e.g. Laryngomalacia, Down's syndrome)
• uncertainty about diagnosis (important differentials include acute epiglottitis,
bacterial tracheitis, peritonsillar abscess and foreign body inhalation)

Investigations

• the vast majority of children are diagnosed clinically


• however, if a chest x-ray is done:
o a posterior-anterior view will show subglottic narrowing, commonly called
the 'steeple sign'
o in contrast, a lateral view in acute epiglottis will show swelling of the
epiglottis - the 'thumb sign'

Management

• CKS recommend giving a single dose of oral dexamethasone (0.15mg/kg)


to all children regardless of severity
• prednisolone is an alternative if dexamethasone is not available

Emergency treatment when sats are low and there is Intercostal recession

• high-flow oxygen
• nebulised adrenaline

5. Sotos syndrome

Sotos syndrome is a rare genetic disorder characterised by excessive physical growth


during the first 2 to 3 years of life
It is caused by a mutation in the NSD1 (Nuclear receptor-binding SET domain
containing protein) gene and is inherited with in an autosomal dominant fashion.

*** Sotos syndrome: a rare genetic disorder characterised by excessive physical growth and
learning disabilities. Dysmorphic features include: macrodolichocephaly, down-slanting
palpebral fissures and a pointed chin. Patients have a normal life expectancy.

Edwards syndrome or trisomy 18 gives: microcephaly, prominent occiput, low set ears,
micrognathia, cleft lip and palate.

Down syndrome or trisomy 21 gives: a small chin, slanted eyes, flattened nasal bridge, single
palmar crease and macroglossia.

Patau syndrome or trisomy 13 gives: microcephaly, cleft lip/palate, polydactyly and survival
beyond the neonatal period is rare.

Cystic fibrosis is a genetic disorder characterised by: recurrent lung infections, pancreatic
insufficiency and poor growth.

6. Paediatric umbilical disorders

Embryology

During development the umbilicus has two umbilical arteries and one umbilical vein.
The arteries are continuous with the internal iliac arteries and the vein is continuous
with the falciform ligament (ductus venosus). After birth, the cord dessicates and
separates and the umbilical ring closes.

Umbilical hernia

Up to 20% of neonates may have an umbilical hernia, it is more common in


premature infants. The majority of these hernias will close spontaneously (may take
between 12 months and three years). Strangulation is rare.

Umbilical hernia are relatively common in children and may be found during the
newborn exam. Usually no treatment is required as they typically resolve by 3 years
of age

Associations

• Afro-Caribbean infants
• Down's syndrome
• mucopolysaccharide storage diseases

Paraumbilical hernia
These are due to defects in the linea alba that are in close proximity to the umbilicus.
The edges of a paraumbilical hernia are more clearly defined than those of an
umbilical hernia. They are less likely to resolve spontaneously than an umbilical
hernia.

Omphalitis

This condition consists of an infection of the umbilicus. Infection with Staphylococcus


aureus is the commonest cause. The condition is potentially serious as infection may
spread rapidly through the umbilical vessels in neonates with a risk of portal
pyaemia, and portal vein thrombosis. Treatment is usually with a combination of
topical and systemic antibiotics.

Umbilical granuloma

These consist of cherry red lesions surrounding the umbilicus, they may bleed on
contact and be a site of seropurulent discharge. Infection is unusual and they will
often respond favourably to chemical cautery with topically applied silver nitrate.

An umbilical granuloma is an overgrowth of tissue which occurs during the healing process
of the umbilicus. It is most common in the first few weeks of life. On examination, a small,
red growth of tissue is seen in the centre of the umbilicus. It is usually wet and leaks small
amounts of clear or yellow fluid. It is treated by regular application of salt to the wound, if
this does not help then the granuloma can be cauterised with silver nitrate.

Persistent urachus

This is characterised by urinary discharge from the umbilicus. It is caused by


persistence of the urachus which attaches to the bladder. They are associated with
other urogenital abnormalities.

Persistent vitello-intestinal duct

This will typically present as an umbilical discharge that discharges small bowel
content. Complete persistence of the duct is a rare condition. Much more common is
the persistence of part of the duct (Meckel's diverticulum). Persistent vitello-intestinal
ducts are best imaged using a contrast study to delineate the anatomy and are
managed by laparotomy and surgical closure.

*** Omphalitis or umbilical cellulitis is a bacterial infection of the umbilical stump which
presents as a superficial cellulitis, usually a few days after birth.

Umbilical hernias occur in 1 in 5 newborn children and usually resolves by 2 years.

Gastroschisis is a congenital condition which is characterised by a defect in the anterior


abdominal wall through which the abdominal contents protrude.
Umbilical hernia in children

Umbilical hernia are relatively common in children and may be found during the
newborn exam. Usually no treatment is required as they typically resolve by 3 years
of age
Associations

• Afro-Caribbean infants
• Down's syndrome
• mucopolysaccharide storage diseases

** Small umbilical hernias are common in babies and tend to resolve by 12 months of
age. Parents should be reassured no treatment is usually required but to be aware of
the signs of obstruction or strangulation such as vomiting, pain and being unable to
push the hernia in - this is rare in infants. Advise the parents to present the child at
around 2 years of age if the hernia is still present to arrange referral to a surgeon.
Attempts to treat the hernia by strapping or taping things over the area are not
helpful and can irritate the skin.

7. Headache in children

Epidemiology

• up to 50 per cent of 7-year-olds and up to 80 per cent of 15-year-old have


experienced at least one headache
• equally as common in boys/girls until puberty then strong (3:1) female
preponderance

Migraine

Migraine without aura is the most common cause of primary headache in children.
The International Headache Society (IHS) have produced criteria for paediatric
migraine without aura:

A >= 5 attacks fulfilling features B to D


B Headache attack lasting 4-72 hours
Headache has at least two of the following four features:

C • bilateral or unilateral (frontal/temporal) location


• pulsating quality
• moderate to severe intensity
A >= 5 attacks fulfilling features B to D
• aggravated by routine physical activity

At least one of the following accompanies headache:

D • nausea and/or vomiting


• photophobia and phonophobia (may be inferred from behaviour)

Acute management

• ibuprofen is thought to be more effective than paracetamol for paediatric


migraine
• NICE CKS recommends that triptans may be used in children >= 12 years
but follow-up is required
• sumatriptan nasal spay (licensed) is the only triptan that has proven
efficacy but it is poorly tolerated by young people who don't like the taste in
the back of the throat
• it should be noted that oral triptans are not currently licensed in people < 18
years
• side-effects of triptans include tingling, heat and heaviness/pressure sensations

Oral Ibuprofen is the first-line treatment for paediatric migraines.


Sumatriptan nasal spray is only licensed in children >12 years of
age. Oral sumatriptan is not licensed in people <18 years old.

Prophylaxis

• the evidence base is limited and no clear consensus guidelines exist


• the GOSH website states: 'in practice, pizotifen and propranolol should be used
as first line preventatives in children. Second line preventatives are valproate,
topiramate and amitryptiline'

Tension-type headache (TTH)

Tension-type headache is the second most common cause of headache in children.


The IHS diagnostic criteria for TTH in children is reproduced below:

A At least 10 previous headache episodes fulfilling features B to D


B Headache lasting from 30 minutes to 7 days
C At least two of the following pain characteristics:
A At least 10 previous headache episodes fulfilling features B to D

• pressing/tightening (non/pulsating) quality


• mild or moderate intensity (may inhibit but does not prohibit activity)
• bilateral location
• no aggravation by routine physical activity

Both of the following:

D • no nausea or vomiting
• photophobia and phonophobia, or one, but not the other is present

8. Cystic fibrosis

Cystic fibrosis (CF) is an autosomal recessive disorder causing increased viscosity of


secretions (e.g. lungs and pancreas). It is due to a defect in the cystic fibrosis
transmembrane conductance regulator gene (CFTR), which codes a cAMP-
regulated chloride channel

In the UK 80% of CF cases are due to delta F508 on the long arm of chromosome 7.
Cystic fibrosis affects 1 per 2500 births, and the carrier rate is c. 1 in 25

Organisms which may colonise CF patients

• Staphylococcus aureus
• Pseudomonas aeruginosa
• Burkholderia cepacia (*previously known as Pseudomonas cepacia)*
• Aspergillus

9. Gynaecological problems in children

In general vaginal examinations and vaginal swabs should not be performed -


referral to a paediatric gynaecologist is appropriate for persistent problems

Most newborn girls have some mucoid white vaginal discharge. This usually
disappears by 3 months of age

Vulvovaginitis

• commonest gynaecological disorder in girls


• risk factors include poor hygiene, tight clothing, lack of labial fat pads protecting
vaginal orifice and lack of protective acid secretion found in the reproductive
years
• bacterial (such as Gardnerella and Bacteroides) or fungal organisms may be
responsible
• sexual abuse may occasionally present as vulvovaginitis
• if bloody discharge consider foreign body

Management

• advise about hygiene


• soothing creams may be useful
• topical antibiotics/antifungals
• oestrogen cream in resistant cases

10. Nocturnal enuresis

The majority of children achieve day and night time continence by 3 or 4 years
of age. Enuresis may be defined as the 'involuntary discharge of urine by day or
night or both, in a child aged 5 years or older, in the absence of congenital or
acquired defects of the nervous system or urinary tract'

Nocturnal enuresis can be defined as either primary (the child has never achieved
continence) or secondary (the child has been dry for at least 6 months before)

NICE issued guidance in 2010. Management:

• look for possible underlying causes/triggers (e.g. Constipation, diabetes mellitus,


UTI if recent onset)
• advise on fluid intake, diet and toileting behaviour
• reward systems (e.g. Star charts). NICE recommend these 'should be given for
agreed behaviour rather than dry nights' e.g. Using the toilet to pass urine
before sleep
• NICE advises: 'Consider whether an alarm or drug treatment is appropriate,
depending on the age, maturity and abilities of the child or young person, the
frequency of bedwetting and the motivation and needs of the family'. Generally:
• an enuresis alarm is first-line for children under the age of 7 years
• desmopressin may be used first-line for children over the age 7 years,
particularly if short-term control is needed or an enuresis alarm has been
ineffective/is not acceptable to the family

11. Developmental dysplasia of the hip


Developmental dysplasia of the hip (DDH) is gradually replacing the old term
'congenital dislocation of the hip' (CDH). It affects around 1-3% of newborns.

Risk factors

• female sex: 6 times greater risk


• breech presentation
• positive family history
• firstborn children
• oligohydramnios
• birth weight > 5 kg
• congenital calcaneovalgus foot deformity

DDH is slightly more common in the left hip. Around 20% of cases are bilateral.

Screening for DDH

• the following infants require a routine ultrasound examination


o first-degree family history of hip problems in early life
o breech presentation at or after 36 weeks gestation, irrespective of
presentation at birth or mode of delivery
o multiple pregnancy
• all infants are screened at both the newborn check and also the six-week baby
check using the Barlow and Ortolani tests

Clinical examination

• Barlow test: attempts to dislocate an articulated femoral head


• Ortolani test: attempts to relocate a dislocated femoral head
• other important factors include:
o symmetry of leg length
o level of knees when hips and knees are bilaterally flexed
o restricted abduction of the hip in flexion

Imaging

• ultrasound is generally used to confirm the diagnosis if clinically suspected


• however, if the infant is > 4.5 months then x-ray is the first line investigation

Management

• most unstable hips will spontaneously stabilise by 3-6 weeks of age


• Pavlik harness (dynamic flexion-abduction orthosis) in children younger than 4-5
months
• older children may require surgery

Q: You are a foundation doctor on the postnatal ward. You perform a newborn
examination on a one-day-old baby boy. He was born in good condition by normal
vaginal delivery. When examining the hips you are concerned that you are able to
relocate a dislocated left hip. What is the appropriate management step?

A: This baby has a clearly abnormal ortolani manoeuvre which requires urgent
assessment to identify potential developmental dysplasia of the hip. Ultrasound is
the most appropriate imaging, avoiding radiation and giving the best visualisation of
the developing hip.

Limping child

Causes of a limping child may vary according to age

Acute onset
Usually accompanies viral infections, but the child
Transient synovitis
is well or has a mild fever
More common in boys, aged 2-12 years
Septic
Unwell child, high fever
arthritis/osteomyelitis
Juvenile idiopathic arthritis Limp may be painless
Trauma History is usually diagnostic
Usually detected in neonates
Development dysplasia of 6 times more common in girls
the hip noted to have a discrepancy between the skin
creases behind the right and left hips
More common at 4-8 years
Perthes disease
Due to avascular necrosis of the femoral head
Slipped upper femoral 10-15 years –obese- Displacement of the femoral
epiphysis head epiphysis postero-inferiorly

Transient synovitis

Transient synovitis is sometimes referred to as irritable hip. It generally presents as


acute hip pain associated with a viral infection. The typical age group is 2-10 years.

A low-grade fever is present in a minority of patients but high fever should raise the
suspicion of other causes such as septic arthritis.

Transient synovitis of the hip is the commonest cause of hip pain in children
and presents acutely often post viral infection

It is most common in children between age 2 and 8 years old and occurs in equal
incidence in boys and girls. It is thought to have a link with recent viral infection or
autoimmune disease. It often presents with pain and limp and restricted range of
motion due to pain. Symptoms usually last a few days to 1 week and management is
conservative with rest and analgesia.

Transient synovitis is self-limiting, requiring only rest and analgesia.

12. Asthma: management in children

NICE released guidance on the management of asthma in 2017. These followed on


quickly from the 2016 British Thoracic Society (BTS) guidelines. Given previous
precedents where specialist societies or Royal colleges eventually default/contribute
to NICE, we have followed the NICE guidance for the notes and questions.

Children aged 5-16 with asthma are now managed in a very similar way to adults

Children and young people aged 5 to 16

NICE do not follow the stepwise approach of the previous BTS guidelines. However,
to try to make the guidelines easier to follow we've added our own steps:
Step Notes
1
Short-acting beta agonist (SABA)
Newly-diagnosed asthma
2

Not controlled on previous step


SABA + paediatric low-dose inhaled
OR
corticosteroid (ICS)
Newly-diagnosed asthma with
symptoms >= 3 / week or night-
time waking
SABA + paediatric low-dose ICS + leukotriene
3
receptor antagonist (LTRA)
SABA + paediatric low-dose ICS + long-acting
beta agonist (LABA)
4
In contrast to the adult guidance, NICE
Step Notes
recommend stopping the LTRA at this point if it
hasn't helped
SABA + switch ICS/LABA for a maintenance and
5 reliever therapy (MART), that includes a paediatric
low-dose ICS
SABA + paediatric moderate-dose ICS MART
6
OR consider changing back to a fixed-dose of a
moderate-dose ICS and a separate LABA
SABA + one of the following options:

• increase ICS to paediatric high-dose, either


as part of a fixed-dose regime or as a
MART
7
• a trial of an additional drug (for example
theophylline)
• seeking advice from a healthcare
professional with expertise in asthma

Children aged less than 5 years

Clearly, it can be difficult to definitively diagnose asthma in young children. NICE


acknowledge the greater role for clinical judgement in this age group.

Again, the stepwise approach is our own rather than NICE's:

Step Notes
1

Short-acting beta agonist (SABA)


Newly-diagnosed asthma
2 SABA + an 8-week trial of paediatric MODERATE-dose
inhaled corticosteroid (ICS)

Not controlled on previous After 8-weeks stop the ICS and monitor the child's
step symptoms:
OR
Newly-diagnosed asthma • if symptoms did not resolve during the trial
with symptoms >= 3 / period, review whether an alternative diagnosis is
week or night-time waking likely
Step Notes
• if symptoms resolved then reoccurred within 4
weeks of stopping ICS treatment, restart the ICS
at a paediatric low dose as first-line maintenance
therapy
• if symptoms resolved but reoccurred beyond 4
weeks after stopping ICS treatment, repeat the
8-week trial of a paediatric moderate dose of ICS

SABA + paediatric low-dose ICS + leukotriene receptor


3
antagonist (LTRA)
Stop the LTRA and refer to an paediatric asthma
4
specialist

Other points

Maintenance and reliever therapy (MART)

• a form of combined ICS and LABA treatment in which a single inhaler,


containing both ICS and a fast-acting LABA, is used for both daily maintenance
therapy and the relief of symptoms as required
• MART is only available for ICS and LABA combinations in which the LABA has a
fast-acting component (for example, formoterol)

It should be noted that NICE does not advocate changing treatment in patients who
have well-controlled asthma simply to adhere to the latest guidance.

Table showing examples of inhaled corticosteroid doses

Frustratingly, the definitions of what constitutes a low, moderate or high-dose ICS


have also changed. In contrast to the BTS guidelines NICE also have different
definitions for adults and children. For children:

• <= 200 micrograms budesonide or equivalent = paediatric low dose


• 200 micrograms - 400 micrograms budesonide or equivalent = paediatric
moderate dose
• > 400 micrograms budesonide or equivalent= paediatric high dose.

Asthma in children: assessment of acute attacks

The 2016 BTS/SIGN guidelines suggest the following criteria are used to assess the
severity of asthma in general practice:
Severe attack Life-threatening attack
SpO2 < 92%
PEF 33-50% best or predicted
Too breathless to talk or feed
Heart rate SpO2 <92%
PEF <33% best or predicted
• >125 (>5 years)
Silent chest
• >140 (1-5 years)
Poor respiratory effort
Agitation
Respiratory rate
Altered consciousness
• >30 breaths/min (>5 years)
Cyanosis
• >40 (1-5 years)

Use of accessory neck muscles

Asthma in children: management of acute attacks

Children with severe or life threatening asthma should be transferred immediately to


hospital.

Children between 2 and 5 years of age

Life-threatening
Moderate attack Severe attack
attack
SpO2 < 92%
SpO2 <92%
Too breathless to talk or
Silent chest
SpO2 > 92% feed
Poor respiratory effort
No clinical features of severe Heart rate > 140/min
Agitation
asthma Respiratory rate > 40/min
Altered consciousness
Use of accessory neck
Cyanosis
muscles

Children greater than 5 years of age

Attempt to measure PEF in all children aged > 5 years.


Life-threatening
Moderate attack Severe attack
attack
SpO2 > 92% SpO2 < 92% SpO2 < 92%
PEF > 50% best or PEF 33-50% best or predicted PEF < 33% best or
predicted Can't complete sentences in one breath or predicted
Life-threatening
Moderate attack Severe attack
attack
No clinical features too breathless to talk or feed Silent chest
of Heart rate > 125/min Poor respiratory
severe asthma Respiratory rate > 30/min effort
Use of accessory neck muscles Altered
consciousness
Cyanosis

For children with mild to moderate acute asthma:

Bronchodilator therapy

• give a beta-2 agonist via a spacer (for a child < 3 years use a close-fitting mask)
• give 1 puff every 30-60 seconds up to a maximum of 10 puffs
• if symptoms are not controlled repeat beta-2 agonist and refer to hospital

Steroid therapy

• should be given to all children with an asthma exacerbation


• treatment should be given for 3-5 days

Usual prednisolone dose

Age Dose as per BTS Dose as per cBNF


2 - 5 years 20 mg od 1-2 mg/kg od (max 40mg)
> 5 years 30 - 40 mg od 1-2 mg/kg od (max 40mg)

** Quiet breath sounds in a child with asthma is a worrying feature. Children


with asthma normally have an obvious bilateral wheeze - the absence of this
may suggest a life-threatening asthma attack

14. Chickenpox

Chickenpox is caused by primary infection with varicella zoster virus. Shingles is a


reactivation of the dormant virus in dorsal root ganglion

Chickenpox is highly infectious

• spread via the respiratory route


• can be caught from someone with shingles
• infectivity = 4 days before rash, until 5 days after the rash first appeared*
• incubation period = 10-21 days

Clinical features (tend to be more severe in older children/adults)

• fever initially
• itchy, rash starting on head/trunk before spreading. Initially macular then
papular then vesicular
• systemic upset is usually mild

Management is supportive

• keep cool, trim nails


• calamine lotion
• school exclusion: NICE Clinical Knowledge Summaries state the following: Advise
that the most infectious period is 1–2 days before the rash appears, but infectivity
continues until all the lesions are dry and have crusted over (usually about 5 days
after the onset of the rash).
• immunocompromised patients and newborns with peripartum exposure should
receive varicella zoster immunoglobulin (VZIG). If chickenpox develops then IV
aciclovir should be considered

A common complication is secondary bacterial infection of the lesions

• NSAIDs may increase this risk


• whilst this commonly may manifest as a single infected lesion/small area of
cellulitis, in a small number of patients invasive group A streptococcal soft tissue
infections may occur resulting in necrotizing fasciitis

Rare complications include

• pneumonia
• encephalitis (cerebellar involvement may be seen)
• disseminated haemorrhagic chickenpox
• arthritis, nephritis and pancreatitis may very rarely be seen

15. Growing pains

A common presentation in General Practice is a child complaining of pain in the legs


with no obvious cause. Such presentations, in the absence of any worrying features,
are often attributed to 'growing pains'. This is a misnomer as the pains are often not
related to growth - the current term used in rheumatology is 'benign idiopathic
nocturnal limb pains of childhood'
Growing pains are equally common in boys and girls and occur in the age range of
3-12 years.

Features of growing pains

• never present at the start of the day after the child has woken
• no limp
• no limitation of physical activity
• systemically well
• normal physical examination
• motor milestones normal
• symptoms are often intermittent and worse after a day of vigorous activity

16. Precocious puberty

Definition

• 'development of secondary sexual characteristics before 8 years in females and 9


years in males'
• more common in females

Some other terms

• thelarche (the first stage of breast development)


• adrenarche (the first stage of pubic hair development)

May be classified into:

1. Gonadotrophin dependent ('central', 'true')

• due to premature activation of the hypothalamic-pituitary-gonadal axis


• FSH & LH raised

2. Gonadotrophin independent ('pseudo', 'false')

• due to excess sex hormones


• FSH & LH low

Males - uncommon and usually has an organic cause

Testes

• bilateral enlargement = gonadotrophin release from intracranial lesion


• unilateral enlargement = gonadal tumour
• small testes = adrenal cause (tumour or adrenal hyperplasia)

Females - usually idiopathic or familial and follows normal sequence of puberty

Organic causes

• are rare, associated with rapid onset, neurological symptoms and signs and
dissonance
• e.g. McCune Albright syndrome

17. Autosomal dominant conditions

Autosomal recessive conditions are often thought to be 'metabolic' as opposed


to autosomal dominant conditions being 'structural', notable exceptions:

• some 'metabolic' conditions such as Hunter's and G6PD are X-linked recessive
whilst others such as hyperlipidaemia type II and hypokalaemic periodic
paralysis are autosomal dominant
• some 'structural' conditions such as ataxia telangiectasia and Friedreich's ataxia
are autosomal recessive

The following conditions are autosomal dominant:

• Achondroplasia
• Acute intermittent porphyria
• Adult polycystic disease
• Antithrombin III deficiency
• Ehlers-Danlos syndrome
• Familial adenomatous polyposis
• Hereditary haemorrhagic telangiectasia
• Hereditary spherocytosis
• Hereditary non-polyposis colorectal carcinoma
• Huntington's disease
• Hyperlipidaemia type II
• Hypokalaemic periodic paralysis
• Malignant hyperthermia
• Marfan's syndromes
• Myotonic dystrophy
• Neurofibromatosis
• Noonan syndrome
• Osteogenesis imperfecta
• Peutz-Jeghers syndrome
• Retinoblastoma
• Romano-Ward syndrome
• tuberous sclerosis
• Von Hippel-Lindau syndrome
• Von Willebrand's disease*

*type 3 von Willebrand's disease (most severe form) is inherited as an autosomal


recessive trait. Around 80% of patients have type 1 disease
*** Conditions with low genetic penetrance can have a normal or mild phenotype
(few clinical signs and symptoms) despite an abnormal genetic profile

Penetrance describes the severity of a particular genotype's presenting phenotype. If


a condition is highly penetrant, the phenotype is more likely to be expressed (and so
demonstrates more signs/symptoms). Conversely, a condition with low penetrance is
less likely to express the phenotype (and so a patient may appear normal despite an
abnormal genetic profile)

Autosomal recessive conditions

Autosomal recessive conditions are often thought to be 'metabolic' as opposed to


autosomal dominant conditions being 'structural', notable exceptions:

• some 'metabolic' conditions such as Hunter's and G6PD are X-linked recessive
whilst others such as hyperlipidemia type II and hypokalemic periodic paralysis
are autosomal dominant
• some 'structural' conditions such as ataxia telangiectasia and Friedreich's ataxia
are autosomal recessive

The following conditions are autosomal recessive:

• Albinism
• Ataxic telangiectasia
• Congenital adrenal hyperplasia
• Cystic fibrosis
• Cystinuria
• Familial Mediterranean Fever
• Fanconi anaemia
• Friedreich's ataxia
• Gilbert's syndrome*
• Glycogen storage disease
• Haemochromatosis
• Homocystinuria
• Lipid storage disease: Tay-Sach's, Gaucher, Niemann-Pick
• Mucopolysaccharidoses: Hurler's
• PKU
• Sickle cell anaemia
• Thalassaemias
• Wilson's disease

*this is still a matter of debate and many textbooks will list Gilbert's as autosomal
dominant

Autosomal recessive

In autosomal recessive inheritance

• only homozygotes are affected


• males and females are equally likely to be affected
• not manifest in every generation - may 'skip a generation'

If two heterozygote parents

• 25% chance of having an affected (homozygote) child


• 50% chance of having a carrier (heterozygote) child
• 25% chance of having an unaffected (i.e. genotypical) child

If one affected parent (i.e. homozygote for gene) and one unaffected (i.e. not a
carrier or affected)

• all the children will be carriers

Autosomal recessive disorders are often metabolic in nature and are generally more
life-threatening compared to autosomal dominant conditions.

Bartter's syndrome:
It is an inherited cause (usually autosomal recessive) of severe hypokalaemia due to
defective chloride absorption at the Na+ K+ 2Cl- cotransporter in the ascending loop
of Henle. It should be noted that it is associated with normotension (unlike other
endocrine causes of hypokalaemia such as Conn's, Cushing's and Liddle's syndrome
which are associated with hypertension)

Features
•usually presents in childhood, e.g. Failure to thrive
•polyuria, polydipsia
•hypokalaemia
•normotension
•weakness
X-linked recessive

In X-linked recessive inheritance only males are affected. An exception to this seen in
examinations are patients with Turner's syndrome, who are affected due to only
having one X chromosome. X-linked recessive disorders are transmitted by
heterozygote females (carriers) and male-to-male transmission is not seen. Affected
males can only have unaffected sons and carrier daughters.

Each male child of a heterozygous female carrier has a 50% chance of being affected
whilst each female child of a heterozygous female carrier has a 50% chance of being
a carrier.

The possibility of an affected father having children with a heterozygous female


carrier is generally speaking extremely rare. However, in certain Afro-Caribbean
communities G6PD deficiency is relatively common and homozygous females with
clinical manifestations of the enzyme defect are seen.

18. Down's syndrome: features

Clinical features

• face: upslanting palpebral fissures, epicanthic folds, Brushfield spots in iris,


protruding tongue, small low-set ears, round/flat face
• flat occiput
• single palmar crease, pronounced 'sandal gap' between big and first toe
• hypotonia
• congenital heart defects (40-50%, see below)
• duodenal atresia
• Hirschsprung's disease

Cardiac complications

• multiple cardiac problems may be present


• endocardial cushion defect (most common, 40%, also known as atrioventricular
septal canal defects)
• ventricular septal defect (c. 30%)
• secundum atrial septal defect (c. 10%)
• tetralogy of Fallot (c. 5%)
• isolated patent ductus arteriosus (c. 5%)

Later complications
• subfertility: males are almost always infertile due to impaired spermatogenesis.
Females are usually subfertile, and have an increased incidence of problems with
pregnancy and labour
• learning difficulties
• short stature
• repeated respiratory infections (+hearing impairment from glue ear)
• acute lymphoblastic leukaemia
• hypothyroidism
• Alzheimer's disease
• atlantoaxial instability

Down's syndrome: epidemiology and genetics

Risk of Down's syndrome with increasing maternal age

Age (years) Risk


20 1 in 1,500
30 1 in 800
35 1 in 270
40 1 in 100
45 1 in 50 or greater

One way of remembering this is by starting at 1/1,000 at 30 years and then dividing
the denominator by 3 (i.e. 3 times more common) for every extra 5 years of age

Cytogenetics

Mode % of cases Risk of recurrence


Nondisjunction 94% 1 in 100 if under mother < 35 years
Robertsonian translocation 10-15% if mother is translocation carrier
5%
(usually onto 14) 2.5% if father is translocation carrier
Mosaicism* 1%

The chance of a further child with Down's syndrome is approximately 1 in 100 if the
mother is less than 35 years old. If the trisomy 21 is a result of a translocation the risk
is much higher

*Mosaicism is defined as the presence of two genetically different populations of


cells in the body

Down's syndrome: vision and hearing problems


Individuals with Down's syndrome are more likely to suffer from vision and hearing
problems, as detailed below:

Vision

• refractive errors are more common


• strabismus: seen in around 20-40%
• cataracts: congenital and acquired are both more common
• recurrent blepharitis
• glaucoma

Hearing

• otitis media and glue ear are very common resulting in hearing problems

** Otitis media with effusion usually presents with a sensation of fullness in the ear
and retracted tympanic membrane(not bulging).

19. Innocent murmurs

Innocent murmurs heard in children include

Ejection
Due to turbulent blood flow at the outflow tract of the heart
murmurs
Due to the turbulent blood flow in the great veins returning to the
Venous
heart. Heard as a continuous blowing noise heard just below the
hums
clavicles
Still's
Low-pitched sound heard at the lower left sternal edge
murmur

Characteristics of an innocent ejection murmur include:

• soft-blowing murmur in the pulmonary area or short buzzing murmur in the


aortic area
• may vary with posture
• localised with no radiation
• no diastolic component
• no thrill
• no added sounds (e.g. clicks)
• asymptomatic child
• no other abnormality

20. Headache in children


Some of the following is based on an excellent review article on the Great Ormond
Street Hospital website.

Epidemiology

• up to 50 per cent of 7-year-olds and up to 80 per cent of 15-year-old have


experienced at least one headache
• equally as common in boys/girls until puberty then strong (3:1) female
preponderance

Migraine

Migraine without aura is the most common cause of primary headache in children.
The International Headache Society (IHS) have produced criteria for paediatric
migraine without aura:

A >= 5 attacks fulfilling features B to D


B Headache attack lasting 4-72 hours
Headache has at least two of the following four features:

• bilateral or unilateral (frontal/temporal) location


C • pulsating quality
• moderate to severe intensity
• aggravated by routine physical activity

At least one of the following accompanies headache:

D • nausea and/or vomiting


• photophobia and phonophobia (may be inferred from behaviour)

Acute management

• ibuprofen is thought to be more effective than paracetamol for paediatric


migraine
• NICE CKS recommends that triptans may be used in children >= 12 years but
follow-up is required
• sumatriptan nasal spay (licensed) is the only triptan that has proven efficacy but
it is poorly tolerated by young people who don't like the taste in the back of the
throat
• it should be noted that oral triptans are not currently licensed in people < 18
years
• side-effects of triptans include tingling, heat and heaviness/pressure sensations
Prophylaxis

• the evidence base is limited and no clear consensus guidelines exist


• the GOSH website states: 'in practice, pizotifen and propranolol should be used
as first line preventatives in children. Second line preventatives are valproate,
topiramate and amitryptiline'

Tension-type headache (TTH)

Tension-type headache is the second most common cause of headache in children.


The IHS diagnostic criteria for TTH in children is reproduced below:

A At least 10 previous headache episodes fulfilling features B to D


B Headache lasting from 30 minutes to 7 days
At least two of the following pain characteristics:

• pressing/tightening (non/pulsating) quality


C • mild or moderate intensity (may inhibit but does not prohibit activity)
• bilateral location
• no aggravation by routine physical activity

Both of the following:

D • no nausea or vomiting
• photophobia and phonophobia, or one, but not the other is present

21. Meningitis in children: organisms

Neonatal to 3 months

• Group B Streptococcus: usually acquired from the mother at birth. More common
in low birth weight babies and following prolonged rupture of the membranes
• E. coli and other Gram -ve organisms
• Listeria monocytogenes

1 month to 6 years

• Neisseria meningitidis (meningococcus)


• Streptococcus pneumoniae (pneumococcus)
• Haemophilus influenzae
Greater than 6 years

• Neisseria meningitidis (meningococcus)


• Streptococcus pneumoniae (pneumococcus)

Meningitis in children: investigation and management

Investigations

Contraindication to lumbar puncture (any signs of raised ICP)

• focal neurological signs


• papilloedema
• significant bulging of the fontanelle
• disseminated intravascular coagulation
• signs of cerebral herniation

For patients with meningococcal septicaemia a lumbar puncture is contraindicated -


blood cultures and PCR for meningococcus should be obtained.

Management

1. Antibiotics

• < 3 months: IV amoxicillin (or ampicillin) + IV cefotaxime


• > 3 months: IV cefotaxime (or ceftriaxone)

2. Steroids

• NICE advise against giving corticosteroids in children younger than 3


months
• dexamethsone should be considered if the lumbar puncture reveals any of the
following:
o frankly purulent CSF
o CSF white blood cell count greater than 1000/microlitre
o raised CSF white blood cell count with protein concentration greater than
1 g/litre
o bacteria on Gram stain

3. Fluids

• treat any shock, e.g. with colloid

4. Cerebral monitoring
• mechanical ventilation if respiratory impairment

5. Public health notification and antibiotic prophylaxis of contacts

• ciprofloxacin is now preferred over rifampicin

Paediatric drug doses: emergency

The current BNF should always be consulted prior to prescribing drugs you are unfamiliar
with, the following is just a guide

IM benzylpenicillin for suspected meningococcal septicaemia in the community

Age Dose
< 1 year 300 mg
1 - 10 years 600 mg
> 10 years 1200 mg

Meningitis B vaccine

Children in the UK have been routinely immunised against serotypes A & C of


meningococcus for many years. As a result meningococcal B became the most
common cause of bacterial meningitis in the UK. A vaccination against
meningococcal B (Bexsero) has recently been developed and introduced to the UK
market.

The Joint Committee on Vaccination and Immunisation (JCVI) initially rejected the
use of Bexsero after doing a cost-benefit analysis. This descision was eventually
reversed and meningitis B has now been added to the routine NHS immunisation.

Three doses are now given at:

• 2 months
• 4 months
• 12-13 months

Bexsero will also be available on the NHS for patients at high risk of meningococcal
disease, such as people with asplenia, splenic dysfunction or complement disorder.

22. Snoring in children

Causes
• obesity- most common
• nasal problems: polyps, deviated septum, hypertrophic nasal turbinates
• recurrent tonsillitis
• Down's syndrome
• hypothyroidism

Obesity in children

Defining obesity is more difficult in children than adults as body mass index (BMI)
varies with age. BMI percentile charts are therefore needed to make an accurate
assessment. Recent NICE guidelines suggest to use 'UK 1990 BMI charts to give age-
and gender-specific information'

NICE recommend

• consider tailored clinical intervention if BMI at 91st centile or above.


• consider assessing for comorbidities if BMI at 98th centile or above

By far the most common cause of obesity in childhood is lifestyle factors. Other
associations of obesity in children include:

• Asian children: four times more likely to be obese than white children
• female children
• taller children: children with obesity are often above the 50th percentile in
height

Cause of obesity in children

• growth hormone deficiency


• hypothyroidism
• Down's syndrome
• Cushing's syndrome
• Prader-Willi syndrome

Consequences of obesity in children

• orthopaedic problems: slipped upper femoral epiphyses, Blount's disease (a


development abnormality of the tibia resulting in bowing of the legs),
musculoskeletal pains
• psychological consequences: poor self-esteem, bullying
• sleep apnoea
• benign intracranial hypertension
• long-term consequences: increased incidence of type 2 diabetes mellitus,
hypertension and ischaemic heart disease
23. Perthes' disease

Perthes' disease is a degenerative condition affecting the hip joints of children,


typically between the ages of 4-8 years. It is due to avascular necrosis of the
femoral head, specifically the femoral epiphysis. Impaired blood supply to the
femoral head causes bone infarction.

Perthes' disease is 5 times more common in boys. Around 10% of cases are bilateral

** hip pain: develops progressively over a few weeks

• limp
• stiffness and reduced range of hip movement
• x-ray: early changes include widening of joint space, later changes include
decreased femoral head size/flattening

Diagnosis

• plain x-ray
• technetium bone scan or magnetic resonance imaging if normal x-ray and
symptoms persist

Complications

• osteoarthritis
• premature fusion of the growth plates

Catterall staging
Stage Features
Stage 1 Clinical and histological features only
Stage 2 Sclerosis with or without cystic changes and preservation of the articular surface
Stage 3 Loss of structural integrity of the femoral head
Stage 4 Loss of acetabular integrity

Management

• To keep the femoral head within the acetabulum: cast, braces


• If less than 6 years: observation
• Older: surgical management with moderate results
• Operate on severe deformities

Prognosis
• Most cases will resolve with conservative management. Early diagnosis improves
outcomes.

24. Necrotising enterocolitis

Necrotising enterocolitis is one of the leading causes of death among premature


infants. Initial symptoms can include feeding intolerance, abdominal distension and
bloody stools, which can quickly progress to abdominal discolouration, perforation
and peritonitis.

Abdominal x-rays are useful when diagnosing necrotising enterocolitis, as they can
show:

• dilated bowel loops (often asymmetrical in distribution)


• bowel wall oedema
• pneumatosis intestinalis (intramural gas)
• portal venous gas
• pneumoperitoneum resulting from perforation
• air both inside and outside of the bowel wall (Rigler sign)
• air outlining the falciform ligament (football sign)

25. Childhood infections

The table below summarises the main characteristics of childhood infections

Infection Features
Fever initially
Itchy, rash starting on head/trunk before spreading. Initially
Chickenpox
macular then papular then vesicular
Systemic upset is usually mild
Prodrome: irritable, conjunctivitis, fever
Koplik spots: white spots ('grain of salt') on buccal mucosa
Measles
Rash: starts behind ears then to whole body, discrete
maculopapular rash becoming blotchy & confluent
Fever, malaise, muscular pain
Mumps Parotitis ('earache', 'pain on eating'): unilateral initially then
becomes bilateral in 70%
Rash: pink maculopapular, initially on face before spreading
Rubella to whole body, usually fades by the 3-5 day
Lymphadenopathy: suboccipital and postauricular
Infection Features
Also known as fifth disease or 'slapped-cheek syndrome'
Caused by parvovirus B19
Erythema
Lethargy, fever, headache
infectiosum
'Slapped-cheek' rash spreading to proximal arms and
extensor surfaces
Reaction to erythrogenic toxins produced by Group A
haemolytic streptococci
Fever, malaise, tonsillitis
Scarlet fever
'Strawberry' tongue
Rash - fine punctate erythema sparing the area around the
mouth (circumoral pallor)
Caused by the coxsackie A16 virus
Hand, foot and
Mild systemic upset: sore throat, fever
mouth disease
Vesicles in the mouth and on the palms and soles of the feet

Kawasaki disease

Kawasaki disease is a type of vasculitis which is predominately seen in children.


Whilst Kawasaki disease is uncommon it is important to recognise as it may cause
potentially serious complications, including coronary artery aneurysms.
High fever lasting >5 days, red palms with desquamation and strawberry tongue are
indicative of Kawasaki disease

Features

• high-grade fever which lasts for > 5 days. Fever is characteristically


resistant to antipyretics
• conjunctival injection
• bright red, cracked lips
• strawberry tongue
• cervical lymphadenopathy
• red palms of the hands and the soles of the feet which later peel

Kawasaki disease is a clinical diagnosis as there is no specific diagnostic test.

Management

• high-dose aspirin
o Kawasaki disease is one of the few indications for the use of aspirin in
children. Due to the risk of Reye's syndrome aspirin is normally
contraindicated in children
• intravenous immunoglobulin
• echocardiogram (rather than angiography) is used as the initial screening test
for coronary artery aneurysms

Complications

• coronary artery aneurysm

Childhood syndromes

Below is a list of common features of selected childhood syndromes

Syndrome Key features


Microcephalic, small eyes
Cleft lip/palate
Patau syndrome (trisomy 13)
Polydactyly
Scalp lesions
Micrognathia
Low-set ears
Edward's syndrome (trisomy 18)
Rocker bottom feet
Overlapping of fingers
Learning difficulties
Macrocephaly
Fragile X Long face
Large ears
Macro-orchidism
Webbed neck
Pectus excavatum
Noonan syndrome
Short stature
Pulmonary stenosis
Micrognathia
Posterior displacement of the tongue (may result
Pierre-Robin syndrome*
in upper airway obstruction)
Cleft palate
Hypotonia
Prader-Willi syndrome Hypogonadism
Obesity
Short stature
Learning difficulties
William's syndrome Friendly, extrovert personality
Transient neonatal hypercalcaemia
Supravalvular aortic stenosis
Cri du chat syndrome (chromosome Characteristic cry (hence the name) due to larynx and
5p deletion syndrome) neurological problems
Syndrome Key features
Feeding difficulties and poor weight gain
Learning difficulties
Microcephaly and micrognathism
Hypertelorism

*this condition has many similarities with Treacher-Collins syndrome. One of the key
differences is that Treacher-Collins syndrome is autosomal dominant so there is usually a
family history of similar problems

26. Fragile X syndrome

Fragile X syndrome is a trinucleotide repeat disorder.

Features in males

• learning difficulties
• large head, macrocephaly, low set ears, long thin face, high arched palate
• macroorchidism
• hypotonia
• autism is more common
• mitral valve prolapse

Features in females (who have one fragile chromosome and one normal X
chromosome) range from normal to mild

Diagnosis

• can be made antenatally by chorionic villus sampling or amniocentesis


• analysis of the number of CGG repeats using restriction endonuclease digestion
and Southern blot analysis

Microcephaly

Microcephaly may be defined as an occipital-frontal circumference < 2nd centile

Causes include

• normal variation e.g. small child with small head


• familial e.g. parents with small head
• congenital infection
• perinatal brain injury e.g. hypoxic ischaemic encephalopathy
• fetal alcohol syndrome
• syndromes: Patau
• craniosynostosis

27. Tetralogy of Fallot

Tetralogy of Fallot (TOF) is the most common cause of cyanotic congenital heart
disease*. It typically presents at around 1-2 months, although may not be picked up
until the baby is 6 months old

TOF is a result of anterior malalignment of the aorticopulmonary septum. The four


characteristic features are:

• ventricular septal defect (VSD)


• right ventricular hypertrophy
• right ventricular outflow tract obstruction, pulmonary stenosis
• overriding aorta

The severity of the right ventricular outflow tract obstruction determines the degree
of cyanosis and clinical severity

Other features

• cyanosis
• causes a right-to-left shunt
• ejection systolic murmur due to pulmonary stenosis (the VSD doesn't usually
cause a murmur)
• a right-sided aortic arch is seen in 25% of patients
• chest x-ray shows a 'boot-shaped' heart, ECG shows right ventricular
hypertrophy

Management

• surgical repair is often undertaken in two parts


• cyanotic episodes may be helped by beta-blockers to reduce infundibular spasm

*however, at birth transposition of the great arteries is the more common lesion
as patients with TOF generally present at around 1-2 months

29. Cow's milk protein intolerance/allergy


Cow's milk protein intolerance/allergy (CMPI/CMPA) occurs in around 3-6% of all
children and typically presents in the first 3 months of life in formula-fed infants,
although rarely it is seen in exclusively breastfed infants.

Both immediate (IgE mediated) and delayed (non-IgE mediated) reactions are seen.
The term CMPA is usually used for immediate reactions and CMPI for mild-moderate
delayed reactions.

Features

• regurgitation and vomiting


• diarrhoea
• urticaria, atopic eczema
• 'colic' symptoms: irritability, crying
• wheeze, chronic cough
• rarely angioedema and anaphylaxis may occur

Diagnosis is often clinical (e.g. improvement with cow's milk protein elimination).
Investigations include:

• skin prick/patch testing


• total IgE and specific IgE (RAST) for cow's milk protein

Management

If the symptoms are severe (e.g. failure to thrive) refer to a paediatrician.

Management if formula-fed

• extensive hydrolysed formula (eHF) milk is the first-line replacement formula for
infants with mild-moderate symptoms
• amino acid-based formula (AAF) in infants with severe CMPA or if no response
to eHF
• around 10% of infants are also intolerant to soya milk

Management if breastfed

• continue breastfeeding
• eliminate cow's milk protein from maternal diet. Consider prescribing calcium
supplements for breastfeeding mothers whose babies have, or are suspected to
have, CMPI, to prevent deficiency whilst they exclude dairy from their diet
• use eHF milk when breastfeeding stops, until 12 months of age and at least for 6
months
CMPI usually resolves in most children

• in children with IgE mediated intolerance around 55% will be milk tolerant by
the age of 5 years
• in children with non-IgE mediated intolerance most children will be milk tolerant
by the age of 3 years
• a challenge is often performed in the hospital setting as anaphylaxis can occur.

30. Skull problems in children

Plagiocephaly

• parallelogram shaped head


• the incidence of plagiocephaly has increased over the past decade. This may be
due to the success of the 'Back to Sleep' campaign

Craniosynostosis

• premature fusion of skull bones

In the newborn positional head molding is a normal finding, this can be left alone
but should be documented adequately for review by the general practitioner at the
six to eight week baby check.

Cranial orthosis (head helmets) can be used for persistent head shape deformities
but are unlikely to be required. Physiotherapy can be used if there is also torticollis.

Surgical intervention would be grossly inappropriate as this is a normal finding.

31. Wilms' tumour

Wilms' nephroblastoma is one of the most common childhood malignancies. It


typically presents in children under 5 years of age, with a median age of 3 years old.

Associations

• Beckwith-Wiedemann syndrome
• as part of WAGR syndrome with Aniridia, Genitourinary malformations, mental
Retardation
• hemihypertrophy
• around one-third of cases are associated with a loss-of-function mutation in the
WT1 gene on chromosome 11

Features
• abdominal mass (most common presenting feature)
• painless haematuria
• flank pain
• other features: anorexia, fever
• unilateral in 95% of cases
• metastases are found in 20% of patients (most commonly lung)

Referral
• children with an unexplained enlarged abdominal mass in children - possible
Wilm's tumour - arrange paediatric review with 48 hours

Management
• nephrectomy
• chemotherapy
• radiotherapy if advanced disease
• prognosis: good, 80% cure rate

32. Congenital heart disease: types

Acyanotic - most common causes

• ventricular septal defects (VSD) - most common, accounts for 30%


• atrial septal defect (ASD)
• patent ductus arteriosus (PDA)
• coarctation of the aorta
• aortic valve stenosis

VSDs are more common than ASDs. However, in adult patients ASDs are the more
common new diagnosis as they generally presents later.

Cyanotic - most common causes

• tetralogy of Fallot (1-2 months)


• transposition of the great arteries (TGA) (within 24 hours)
• tricuspid atresia

Fallot's is more common than TGA. However, at birth TGA is the more common
lesion as patients with Fallot's generally presenting at around 1-2 months

The presence of cyanosis in pulmonary valve stenosis depends very much on the
severity and any other coexistent defects.

33. Sudden infant death syndrome


Sudden infant death syndrome is the commonest cause of death in the first year of
life. It is most common at 3 months of age.

Major risk factors

• putting the baby to sleep prone: the relative risk or odds ratio varies from 3.5 -
9.3. If not accustomed to prone sleeping (i.e. the baby usually sleeps on their
back) the odds ratio increases to 8.7-45.4
• parental smoking: studies suggest this increases the risk up to 5 fold
• prematurity: 4-fold increased risk
• bed sharing: odds ratio 5.1
• hyperthermia (e.g. over-wrapping) or head covering (e.g. blanket accidentally
moves)

Other risk factors

• male sex
• multiple births
• social classes IV and V
• maternal drug use
• incidence increases in winter

It is important to remember that these odds are additive. For example, if both
parents smoke, the mother had consumed two units of alcohol and they then shared
a bed with the infant the adjusted odds ratio was shown to be 89.7.

Protective factors

• breastfeeding
• room sharing (but not bed sharing, which is a significant risk factor)
• the use of dummies (pacifiers)

Following a cot death siblings should be screened for potential sepsis and inborn
errors of metabolism.

The major risk factors for SIDS are:

• prone sleeping
• parental smoking
• bed sharing
• hyperthermia and head covering
• prematurity
Of the options listed above, studies have shown that co-sleeping is by far the
most significant risk factor for SIDS.

34. Necrotising enterocolitis

Necrotising enterocolitis is one of the leading causes of death among premature


infants. Initial symptoms can include feeding intolerance, abdominal distension and
bloody stools, which can quickly progress to abdominal discolouration, perforation
and peritonitis.

Abdominal x-rays are useful when diagnosing necrotising enterocolitis, as they can
show:

• dilated bowel loops (often asymmetrical in distribution)


• bowel wall oedema
• pneumatosis intestinalis (intramural gas)
• portal venous gas
• pneumoperitoneum resulting from perforation
• air both inside and outside of the bowel wall (Rigler sign)
• air outlining the falciform ligament (football sign)

** The characteristic hallmarks of intussusception on physical examination of a sausage-


shaped mass in the right hypochondrium and emptiness in the right lower quadrant
(Dance's sign) where not present in this patient.
Inflammatory bowel disease is unlikely to affect an infant
Infants with pyloric stenosis generally present with increasing episodes of vomiting
following feeding, with these episodes often being described as projectile due to their
forceful nature.
Hirschsprung's disease could be a possibility. Necrotising enterocolitis is more prevalent
in premature infants

35. Neonatal blood spot screening

Neonatal blood spot screening (previously called the Guthrie test or 'heel-prick
test') is performed at 5-9 days of life

The following conditions are currently screened for:

• congenital hypothyroidism
• cystic fibrosis
• sickle cell disease
• phenylketonuria
• medium chain acyl-CoA dehydrogenase deficiency (MCADD)
• maple syrup urine disease (MSUD)
• isovaleric acidaemia (IVA)
• glutaric aciduria type 1 (GA1)
• homocystinuria (pyridoxine unresponsive) (HCU)

36. Aortic stenosis in children

Aortic stenosis accounts for 5% of congenital heart disease

Associations include:

• William's syndrome (causes supravalvular aortic stenosis)


• coarctation of the aorta
• Turner's syndrome

Management

• aim is to avoid or delay valve replacement if possible


• if gradient across valve is > 60 mmHg then balloon valvotomy may be indicated

**William's syndrome - The syndromic features include the small upturned nose, long
philtrum (upper lip length), wide mouth, full lips, small chin, and puffiness around the eyes.

Down's syndrome is most commonly associated with atrioventricular septal defects.

Klinefelter syndrome is associated with hypogonadism.

Angelman syndrome is not usually associated with aortic stenosis.

Prader-Willi syndrome is associated with obesity.

37. Whooping cough (pertussis)

If admission is not needed, prescribe an antibiotic if the onset of cough is within


the previous 21 days. A macrolide antibiotic is recommended first-line:
•Prescribe clarithromycin for infants less than 1 month of age.
•Prescribe azithromycin or clarithromycin for children aged 1 month or older,
and non-pregnant adults.
•Prescribe erythromycin for pregnant women.
Whooping cough (pertussis) is an infectious disease caused by the Gram-negative
bacterium Bordetella pertussis. It typically presents in children. There are around
1,000 cases are reported each year in the UK.

Immunisation

• infants are routinely immunised at 2, 3, 4 months and 3-5 years. Newborn


infants are particularly vulnerable, which is why the vaccination campaign for
pregnant women was introduced
• neither infection nor immunisation results in lifelong protection - hence
adolescents and adults may develop whooping cough despite having had their
routine immunisations

Features, 2-3 days of coryza precede onset of:

• coughing bouts: usually worse at night and after feeding, may be ended by
vomiting & associated central cyanosis
• inspiratory whoop: not always present (caused by forced inspiration against a
closed glottis)
• infants may have spells of apnoea
• persistent coughing may cause subconjunctival haemorrhages or even anoxia
leading to syncope & seizures
• symptoms may last 10-14 weeks* and tend to be more severe in infants
• marked lymphocytosis

Diagnostic criteria
Whooping cough should be suspected if a person has an acute cough that has lasted
for 14 days or more without another apparent cause, and has one or more of the
following features:

• Paroxysmal cough.
• Inspiratory whoop.
• Post-tussive vomiting.
• Undiagnosed apnoeic attacks in young infants.

Diagnosis

• per nasal swab culture for Bordetella pertussis - may take several days or weeks
to come back
• PCR and serology are now increasingly used as their availability becomes more
widespread

Management
• infants under 6 months with suspect pertussis should be admitted
• in the UK pertussis is a notifiable disease
• an oral macrolide (e.g. clarithromycin, azithromycin or erythromycin) is
indicated if the onset of the cough is within the previous 21 days to eradicate
the organism and reduce the spread
• household contacts should be offered antibiotic prophylaxis
• antibiotic therapy has not been shown to alter the course of the illness
• school exclusion: 48 hours after commencing antibiotics (or 21 days from onset
of symptoms if no antibiotics )

Complications

• subconjunctival haemorrhage
• pneumonia
• bronchiectasis
• seizures

Vaccination of pregnant women

In 2012 there was an outbreak of whooping cough (pertussis) which resulted in the
death of 14 newborn children. As a temporary measure, a vaccination programme
was introduced in 2012 for pregnant women. This has successfully reduced the
number of cases of whooping cough (the vaccine is thought to be more than 90%
effective in preventing newborns developing whooping cough). It was however
decided in 2014 to extend the whooping cough vaccination programme for pregnant
women. This decision was taken as there was a 'great deal of uncertainty' about the
timing of future outbreaks.

Women who are between 16-32 weeks pregnant will be offered the vaccine.

*weeks, not days

38. Normal lower limb variants in children

Apparent abnormalities in the lower limbs of developing children can cause parental
concern and is a common reason for presentation to primary care, often resulting in
referral to secondary care.

The table below lists some of the common presentations:


Typical age
Description of Signs Notes
presentation
Absent Typically resolves between the ages of 4-8
Flat feet (pes medial years
All ages
planus) arch on Orthotics are not recommended
standing Parental reassurance appropriate
Possible causes:

• metatarsus adductus: abnormal heel


bisector line. 90% of cases resolve
spontaneously, severe/persistent cases
may require serial casting
• internal tibial torsion: difference the
In toeing 1st year
thigh and foot ankle: resolves in the
vast majority
• femoral anteversion: 'W' sign resolves
in around 80% by adolescence, surgical
intervention in the remaining not
usually advised

Common in early infancy and usually resolves


by the age of 2 years. Usually due to external
Out toeing All ages tibial torsion. Intervention may be appropriate
if doesn't resolve as increases risk of
patellofemoral pain
Increased
Bow legs intercond
1st-2nd year Typically resolves by the age of 4-5 years
(genu varum) ylar
distance
Increased
Knock knees
intermalle
(genu 3rd-4th year Typically resolves spontaneously
olar
valgum)
distance

39. Bronchiolitis

Bronchiolitis is a condition characterised by acute bronchiolar inflammation.


Respiratory syncytial virus (RSV) is the pathogen in 75-80% of cases. NICE released
guidelines on bronchiolitis in 2015. Please see the link for more details.
Epidemiology

• most common cause of a serious lower respiratory tract infection in < 1yr olds
(90% are 1-9 months, with a peak incidence of 3-6 months). Maternal IgG
provides protection to newborns against RSV
• higher incidence in winter

Basics

• respiratory syncytial virus (RSV) is the pathogen in 75-80% of cases


• other causes: mycoplasma, adenoviruses
• may be secondary bacterial infection
• more serious if bronchopulmonary dysplasia (e.g. Premature), congenital heart
disease or cystic fibrosis

Features

• coryzal symptoms (including mild fever) precede:


• dry cough
• increasing breathlessness
• wheezing, fine inspiratory crackles (not always present)
• feeding difficulties associated with increasing dyspnoea are often the reason for
hospital admission

NICE recommend immediate referral (usually by 999 ambulance) if they have any of
the following:

• apnoea (observed or reported)


• child looks seriously unwell to a healthcare professional
• severe respiratory distress, for example grunting, marked chest recession, or a
respiratory rate of over 70 breaths/minute
• central cyanosis
• persistent oxygen saturation of less than 92% when breathing air.

NICE recommend that clinicians 'consider' referring to hospital if any of the


following apply:

• a respiratory rate of over 60 breaths/minute


• difficulty with breastfeeding or inadequate oral fluid intake (50-75% of usual
volume 'taking account of risk factors and using clinical judgement')
• clinical dehydration.
Consider a diagnosis of pneumonia if the child has:

• high fever (over 39°C) and/or


• persistently focal crackles.

Investigation

• immunofluorescence of nasopharyngeal secretions may show RSV

Management is largely supportive

• humidified oxygen is given via a head box and is typically recommended if the
oxygen saturations are persistently < 92%
• nasogastric feeding may be needed if children cannot take enough fluid/feed by
mouth
• suction is sometimes used for excessive upper airway secretions

40. Knee problems: children and young adults

The table below summarises the key features of common knee problems:

Condition Key features


Softening of the cartilage of the patella
Common in teenage girls
Chondromalacia
Characteristically anterior knee pain on walking up and
patellae
down stairs and rising from prolonged sitting
Usually responds to physiotherapy
Osgood-Schlatter
Seen in sporty teenagers
disease
Pain, tenderness and swelling over the tibial tubercle
(tibial apophysitis)
Osteochondritis Pain after exercise
dissecans Intermittent swelling and locking
Medial knee pain due to lateral subluxation of the patella
Patellar subluxation
Knee may give way
More common in athletic teenage boys
Patellar tendonitis Chronic anterior knee pain that worsens after running
Tender below the patella on examination

Referred pain may come from hip problems such as slipped upper femoral epiphysis
41. Pyloric stenosis

Pyloric stenosis typically presents in the second to fourth weeks of life with vomiting,
although rarely may present later at up to four months. It is caused by hypertrophy
of the circular muscles of the pylorus.

Epidemiology

• incidence of 4 per 1,000 live births


• 4 times more common in males
• 10-15% of infants have a positive family history
• first-borns are more commonly affected

Features

• 'projectile' vomiting, typically 30 minutes after a feed


• constipation and dehydration may also be present
• a palpable mass may be present in the upper abdomen
• hypochloraemic, hypokalaemic alkalosis due to persistent vomiting

Diagnosis is most commonly made by ultrasound.

Management is with Ramstedt pyloromyotomy.

42. Trinucleotide repeat disorders


Trinucleotide repeat disorders are genetic conditions caused by an abnormal number
of repeats (expansions) of a repetitive sequence of three nucleotides. These
expansions are unstable and may enlarge which may lead to an earlier age of onset
in successive generations - a phenomenon known as anticipation*. In most cases, an
increase in the severity of symptoms is also noted

Examples - note dominance of neurological disorders

• Fragile X (CGG)
• Huntington's (CAG)
• myotonic dystrophy (CTG)
• Friedreich's ataxia* (GAA)
• spinocerebellar ataxia
• spinobulbar muscular atrophy
• dentatorubral pallidoluysian atrophy

*Friedreich's ataxia is unusual in not demonstrating anticipation

43. Seborrhoeic dermatitis in children

Seborrhoeic dermatitis is a relatively common skin disorder seen in children. It


typically affects the scalp ('Cradle cap'), nappy area, face and limb flexures.

Cradle cap is an early sign which may develop in the first few weeks of life. It is
characterised by an erythematous rash with coarse yellow scales.

Management depends on severity

• mild-moderate: baby shampoo and baby oils


• severe: mild topical steroids e.g. 1% hydrocortisone

Seborrhoeic dermatitis in children tends to resolve spontaneously by around 8


months of age

44. Gastro-oesophageal reflux in children

Gastro-oesophageal reflux is the commonest cause of vomiting in infancy. Around


40% of infants regurgitate their feeds to a certain extent so there is a degree of
overlap with normal physiological processes.

Risk factors

• preterm delivery
• neurological disorders

Features

• typically develops before 8 weeks


• vomiting/regurgitation following feeds

Diagnosis is usually made clinically

Management (partly based on the 2015 NICE guidelines)

• advise regarding position during feeds - 30 degree head-up


• infants should sleep on their backs as per standard guidance to reduce the risk
of cot death
• ensure infant is not being overfed (as per their weight) and consider a trial of
smaller and more frequent feeds
• a trial of thickened formula (for example, containing rice starch, cornstarch,
locust bean gum or carob bean gum)
• a trial of alginate therapy e.g. Gaviscon. Alginates should not be used at the
same time as thickening agents
• NICE do not recommend a proton pump inhibitor (PPI) to treat overt
regurgitation in infants and children occurring as an isolated symptom. A trial of
one of these agents should be considered if 1 or more of the following apply:
o unexplained feeding difficulties (for example, refusing feeds, gagging or
choking)
o distressed behaviour
o faltering growth
• ranitidine was previously used as an alternative to a PPI but was withdrawn from
the market in 2020 as small amounts of the carcinogen N-nitrosodimethylamine
(NDMA) were discovered in products from a number of manufacturers.
• prokinetic agents e.g. metoclopramide should only be used with specialist
advice

Complications

• distress
• failure to thrive
• aspiration
• frequent otitis media
• in older children dental erosion may occur

If there are severe complications (e.g. failure to thrive) and medical treatment is
ineffective then fundoplication may be considered.
PPI should be trialled in infants with GORD who do not respond to
alginates/thickened feeds and who have 1. feeding difficulties, 2. distressed
behaviour or 3. faltering growth

**NICE advise that breastfed infants who have persisting symptoms of frequent
regurgitation associated with marked distress should be given a 1-2 week trial of
alginate therapy (eg. Gaviscon) following every feed.

If symptoms persist following this then a 4-week trial of a proton pump inhibitor (such
as oral omeprazole) or a histamine-2 receptor antagonist (such as oral ranitidine)
should be given.

If symptoms still persist despite this then a referral to paediatrics should be considered.

Thickened feeds should be given to bottle-fed infants with reflux.

45. Jaundice in the newborn period

Jaundice in the first 24 hrs is always pathological

Causes of jaundice in the first 24 hrs

• rhesus haemolytic disease


• ABO haemolytic disease
• hereditary spherocytosis
• glucose-6-phosphodehydrogenase

Jaundice in the neonate from the c. 2-14 days is common (up to 40%) and usually
physiological. It is more commonly seen in breastfed babies

If there are still signs of jaundice after 14 days a prolonged jaundice screen is
performed, including:

• conjugated and unconjugated bilirubin: the most important test as a raised


conjugated bilirubin could indicate biliary atresia which requires urgent surgical
intervention
• direct antiglobulin test (Coombs' test)
• TFTs
• FBC and blood film
• urine for MC&S and reducing sugars
• U&Es and LFTs

Causes of prolonged jaundice


• biliary atresia
• hypothyroidism
• galactosaemia
• urinary tract infection
• breast milk jaundice
• congenital infections e.g. CMV, toxoplasmosis

Measure and record the serum bilirubin level urgently (within 2 hours) in all
babies with suspected or obvious jaundice in the first 24 hours of life since this
is likely to be pathological rather than physiological jaundice

Haemorrhagic disease of the newborn

Newborn babies are relatively deficient in vitamin K. This may result in impaired
production of clotting factors which in turn can lead to haemorrhagic disease of the
newborn (HDN). Bleeding may range from minor brushing to intracranial
haemorrhages

Breast-fed babies are particularly at risk as breast milk is a poor source of


vitamin K. Maternal use of antiepileptics also increases the risk

Because of this all newborns in the UK are offered vitamin K, either intramuscularly or
orally

46. Threadworms

Infestation with threadworms (Enterobius vermicularis, sometimes called pinworms) is


extremely common amongst children in the UK. Infestation occurs after swallowing
eggs that are present in the environment.

Threadworm infestation is asymptomatic in around 90% of cases, possible features


include:

• perianal itching, particularly at night


• girls may have vulval symptoms

Diagnosis may be made by the applying Sellotape to the perianal area and sending it
to the laboratory for microscopy to see the eggs. However, most patients are treated
empirically and this approach is supported in the CKS guidelines.

Management
• CKS recommend a combination of anthelmintic with hygiene measures for all
members of the household
• mebendazole is used first-line for children > 6 months old. A single dose is
given unless infestation persists

Mebendazole is first-line therapy for treatment of threadworm. All household


members should be treated simultaneously with strict hygiene measures in place,
such as hand washing with soap and water, regularly cutting fingernails, daily
morning shower, regular bed linen and nightwear changes with laundry washed on a
hot cycle and thorough vacuuming/dusting, for a minimum of 2 weeks until all eggs
are removed.

Ivermectin is an antiparasitic used for the treatment of head lice, scabies,


strongyloides, river blindness and others. It is not used in the treatment of
threadworm.

Albendazole is active against threadworm but is not first-line treatment.

47. Hypertension in children

Measuring blood pressure in children

• correct cuff size is approximately 2/3 the length of the upper arm
• the 4th Korotkoff sound is used to measure the diastolic blood pressure until
adolescence, when the 5th Korotkoff sound can be used
• results should be compared with a graph of normal values for age

In younger children secondary hypertension is the most common cause, with renal
parenchymal disease accounting for up to 80%

Causes of hypertension in children

• renal parenchymal disease


• renal vascular disease
• coarctation of the aorta
• phaeochromocytoma
• congenital adrenal hyperplasia
• essential or primary hypertension (becomes more common as children become
older)

48. Cow's milk protein intolerance/allergy


Cow's milk protein intolerance/allergy (CMPI/CMPA) occurs in around 3-6% of all
children and typically presents in the first 3 months of life in formula-fed infants,
although rarely it is seen in exclusively breastfed infants.

Both immediate (IgE mediated) and delayed (non-IgE mediated) reactions are seen.
The term CMPA is usually used for immediate reactions and CMPI for mild-moderate
delayed reactions.

Features

• regurgitation and vomiting


• diarrhoea
• urticaria, atopic eczema
• 'colic' symptoms: irritability, crying
• wheeze, chronic cough
• rarely angioedema and anaphylaxis may occur

Diagnosis is often clinical (e.g. improvement with cow's milk protein elimination).
Investigations include:

• skin prick/patch testing


• total IgE and specific IgE (RAST) for cow's milk protein

Management

If the symptoms are severe (e.g. failure to thrive) refer to a paediatrician.

Management if formula-fed

• extensive hydrolysed formula (eHF) milk is the first-line replacement formula for
infants with mild-moderate symptoms
• amino acid-based formula (AAF) in infants with severe CMPA or if no response
to eHF
• around 10% of infants are also intolerant to soya milk

Management if breastfed

• continue breastfeeding
• eliminate cow's milk protein from maternal diet. Consider prescribing calcium
supplements for breastfeeding mothers whose babies have, or are suspected to
have, CMPI, to prevent deficiency whilst they exclude dairy from their diet
• use eHF milk when breastfeeding stops, until 12 months of age and at least for 6
months

CMPI usually resolves in most children

• in children with IgE mediated intolerance around 55% will be milk tolerant by
the age of 5 years
• in children with non-IgE mediated intolerance most children will be milk tolerant
by the age of 3 years
• a challenge is often performed in the hospital setting as anaphylaxis can occur.

The following clues in the history would suggest the diagnosis of cows' milk
protein intolerance:

•Multi-system involvement
•7 months would suggest the new introduction of top up feeds which correlates
with the symptoms
•Faltering growth along with the multi-system involvement would suggest
cows' milk protein intolerance

49. Immunisation

The Department of Health published guidance in 2006 on the safe administration of


vaccines in its publication 'Immunisation against infectious disease'

General contraindications to immunisation

• confirmed anaphylactic reaction to a previous dose of a vaccine containing the


same antigens
• confirmed anaphylactic reaction to another component contained in the
relevant vaccine (e.g. egg protein)

Situations where vaccines should be delayed

• febrile illness/intercurrent infection

Contraindications to live vaccines

• pregnancy
• immunosuppression
Specific vaccines

• DTP: vaccination should be deferred in children with an evolving or unstable


neurological condition

Not contraindications to immunisation

• asthma or eczema
• history of seizures (if associated with fever then advice should be given
regarding antipyretics)
• breastfed child
• previous history of natural pertussis, measles, mumps or rubella infection
• history of neonatal jaundice
• family history of autism
• neurological conditions such as Down's or cerebral palsy
• low birth weight or prematurity
• patients on replacement steroids e.g. (CAH)

Babies who were born prematurely should receive their routine vaccinations according to
chronological age; there should be no correcting for gestational age. Babies who were
born prior to 28 weeks gestation should receive their first set of immunisations at hospital
due to risk of apnoea.

Immunisation schedule

The current UK immunisation schedule is as follows.

Age Recommended immunisations


At birth BCG if risk factors (see below)
'6-1 vaccine' (diphtheria, tetanus, whooping cough, polio, Hib and
hepatitis B)
2 months
Oral rotavirus vaccine
Men B
'6-1 vaccine' (diphtheria, tetanus, whooping cough, polio, Hib and
hepatitis B)
3 months*
Oral rotavirus vaccine
PCV
'6-1 vaccine' (diphtheria, tetanus, whooping cough, polio, Hib and
4 months hepatitis B)
Men B
12-13 Hib/Men C
months MMR
Age Recommended immunisations
PCV
Men B
2-8 years Flu vaccine (annual)
'4-in-1 pre-school booster' (diphtheria, tetanus, whooping cough and
3-4 years polio)
MMR
12-13 years HPV vaccination
'3-in-1 teenage booster' (tetanus, diphtheria and polio)
13-18 years
Men ACWY

At birth the BCG vaccine should be given if the baby is deemed at risk of tuberculosis
(e.g. Tuberculosis in the family in the past 6 months).

Meningitis ACWY vaccine

Note that the meningitis ACWY vaccine has replaced meningitis C for 13-18 year-
olds. This is due to an increased incidence of meningitis W disease in recent years.
The ACWY vaccine will also be offered to new students (up to the age of 25 years) at
university. With respect to getting the vaccine, the NHS give the following advice to
patients:

'GP practices will automatically send letters inviting 17-and 18-year-olds in school year
13 to have the Men ACWY vaccine.

Students going to university or college for the first time as freshers, including overseas
and mature students up to the age of 25, should contact their GP to have the Men
ACWY vaccine, ideally before the start of the academic year'

Key

• Hib = Haemophilus influenzae B vaccine


• PCV = Pneumococcal Conjugate Vaccine
• Men B = Meningococcal B vaccine
• Men C = Meningococcal C vaccine
• Men ACWY = Meningococcal vaccine covering A, C, W and Y serotypes
• MMR = Measles, Mumps, Rubella vaccine
• HPV = Human Papilloma Vaccine

All children will continue to be offered the Hib/Men C vaccine at one year of
age, and the Men ACWY vaccine at 14 years of age to provide protection across
all age groups.
MMR vaccine

Children in the UK receive two doses of the Measles, Mumps and Rubella (MMR)
vaccine before entry to primary school. This currently occurs at 12-15 months and 3-
4 years as part of the routine immunisation schedule

Contraindications to MMR

• severe immunosuppression
• allergy to neomycin
• children who have received another live vaccine by injection within 4 weeks
• pregnancy should be avoided for at least 1 month following vaccination
• immunoglobulin therapy within the past 3 months (there may be no immune
response to the measles vaccine if antibodies are present)

Adverse effects

• malaise, fever and rash may occur after the first dose of MMR. This typically
occurs after 5-10 days and lasts around 2-3 days

***The Green Book recommends allowing 3 months between doses to maximise


the response rate. A period of 1 month is considered adequate if the child is greater
than 10 years of age. In an urgent situation (e.g. an outbreak at the child's school)
then a shorter period of 1 month can be used in younger children.

50. Stridor in children

Causes of stridor in children include:

Cause Notes
Croup is a form of upper respiratory tract infection seen in infants
and toddlers. It is characterised by stridor which is caused by a
combination of laryngeal oedema and secretions. Parainfluenza
viruses account for the majority of cases.

Epidemiology
Croup
• peak incidence at 6 months - 3 years
• more common in autumn

Features
Cause Notes

• stridor
• barking cough (worse at night)
• fever
• coryzal symptoms

Acute epiglottitis is rare but serious infection caused by


Haemophilus influenzae type B. Prompt recognition and treatment
is essential as airway obstruction may develop. Epiglottitis
generally occurs in children between the ages of 2 and 6 years.
The incidence of epiglottitis has decreased since the introduction
of the Hib vaccine
Acute
epiglottitis Features

• rapid onset
• unwell, toxic child
• stridor
• drooling of saliva

Symptoms depend on the site of impaction

Features are of sudden onset


Inhaled foreign
• coughing
body
• choking
• vomiting
• stridor

Congenital abnormality of the larynx.

Infants typical present at 4 weeks of age with:


Laryngomalacia

• stridor

Acute epiglottitis

Acute epiglottitis is rare but serious infection caused by Haemophilus influenzae


type B. Prompt recognition and treatment is essential as airway obstruction may
develop. Epiglottitis was generally considered a disease of childhood but in the UK it
is now more common in adults due to the immunisation programme. The incidence
of epiglottitis has decreased since the introduction of the Hib vaccine.

Acute epiglottitis is characterised by rapid onset fever, stridor and drooling.


There is usually dysphagia and patient is typically sitting in the tripod position.

Features

• rapid onset
• high temperature, generally unwell
• stridor
• drooling of saliva
• 'tripod' position: the patient finds it easier to breathe if they are leaning forward
and extending their neck in a seated position

Diagnosis is made by direct visualisation (only by senior/airway trained staff, see


below). However, x-rays may be done, particularly if there is concern about a foreign
body:

• a lateral view in acute epiglottis will show swelling of the epiglottis - the
'thumb sign'
• in contrast, a posterior-anterior view in croup will show subglottic
narrowing, commonly called the 'steeple sign'

Management

• immediate senior involvement, including those able to provide emergency


airway support (e.g. anaesthetics, ENT)
o endotracheal intubation may be necessary to protect the airway
• if suspected do NOT examine the throat due to the risk of acute airway
obstruction
o the diagnosis is made by direct visualisation but this should only be done
by senior staff who are able to intubate if necessary
• oxygen
• intravenous antibiotics

51. Consent in children

The law around consent in children is complicated. However, as a rough guide:

• patients less than 16 years old may consent to treatment if they are deemed to
be competent (an example is the Fraser guidelines, previously termed Gillick
competence), but cannot refuse treatment which may be deemed in their best
interest
• between the ages of 16-18 years it is presumed patients are competent to give
consent to treatment
• patients 18 years or older may consent to treatment or refuse treatment

With regards to the provision of contraceptives to patients under 16 years of age the
Fraser Guidelines state that all the following requirements should be fulfilled:

• the young person understands the professional's advice


• the young person cannot be persuaded to inform their parents
• the young person is likely to begin, or to continue having, sexual intercourse
with or without contraceptive treatment
• unless the young person receives contraceptive treatment, their physical or
mental health, or both, are likely to suffer
• the young person's best interests require them to receive contraceptive advice
or treatment with or without parental consent

Gillick or Fraser?

Some doctors use the term Fraser competency when referring to contraception and Gillick
competency when referring to general issues of consent in children. The (widespread)
rumours that Victoria Gillick removed her permission to use her name or applied copyright
have recently been debunked.

Children under the age of 13 years are not able to consent to sexual
intercourse and hence any sexual activity would be regarded as rape
under the law. This is one situation under the GMC guidelines where
you are compelled to break confidentiality

52. Pneumonia in children

Pathophysiology

• S .pneumoniae is the most likely causative agent of a bacterial pneumonia in


children

The British Thoracic Society published guidelines in 2011 on the management of


community acquired pneumonia in childhood. Key points are summarised below:

Treatment
• Amoxicillin is first-line for all children with pneumonia who are not allergic
to penicillin
• Macrolides may be added if there is no response to first line therapy
• Macrolides should be used if mycoplasma or chlamydia is suspected
• In pneumonia associated with influenza, co-amoxiclav is recommended

53. Hand, foot and mouth disease

Hand, foot and mouth disease is a self-limiting condition affecting children. It is


caused by the intestinal viruses of the Picornaviridae family (most commonly
coxsackie A16 and enterovirus 71). It is very contagious and typically occurs in
outbreaks at nursery

Clinical features

• mild systemic upset: sore throat, fever


• oral ulcers
• followed later by vesicles on the palms and soles of the feet

Management

• symptomatic treatment only: general advice about hydration and analgesia


• reassurance no link to disease in cattle
• children do not need to be excluded from school
o the HPA recommends that children who are unwell should be kept off
school until they feel better
o they also advise that you contact them if you suspect that there may be a
large outbreak.

54. Roseola infantum:- fever followed later by rash

Roseola infantum (also known as exanthem subitum, occasionally sixth disease) is a


common disease of infancy caused by the human herpes virus 6 (HHV6). It has an
incubation period of 5-15 days and typically affects children aged 6 months to 2
years.

Features

• high fever: lasting a few days, followed later by a


• maculopapular rash
• Nagayama spots: papular enanthem on the uvula and soft palate
• febrile convulsions occur in around 10-15%
• diarrhoea and cough are also commonly seen
Other possible consequences of HHV6 infection

• aseptic meningitis
• hepatitis

School exclusion is not needed.

55. Transient tachypnoea of the newborn

Transient tachypnoea of the newborn (TTN) is the commonest cause of respiratory


distress in the newborn period. It is caused by delayed resorption of fluid in the
lungs

It is more common following Caesarean sections, possibly due to the lung fluid
not being 'squeezed out' during the passage through the birth canal

Chest x-ray may show hyperinflation of the lungs and fluid in the horizontal
fissure

Supplementary oxygen may be required to maintain oxygen saturations. Transient


tachypnoea of the newborn usually settles within 1-2 days

**It is important to note that even if you suspect transient tachypnoea of the newborn the
baby should be screened and treated for sepsis. Transient tachypnoea of the newborn is
the commonest cause of respiratory distress in a neonate and will resolve in 24-48 hours.

A pneumothorax is associated with mechanical ventilation but should be carefully


looked for in all chest x-rays.
X-ray will show hyperinflation and fluid in the horizontal fissure.
Ground glass appearance and low volume lungs are seen in respiratory distress
syndrome.
Asymmetric patchy opacities are seen in meconium aspiration syndrome.

Meconium aspiration syndrome

Meconium aspiration syndrome refers to respiratory distress in the newborn as a result of


meconium in the trachea. It occurs in the immediate neonatal period. It is more common in
post-term deliveries, with rates of up to 44% reported in babies born after 42 weeks. It
causes respiratory distress, which can be severe. Higher rates occur where there is a history
of maternal hypertension, pre-eclampsia, chorioamnionitis, smoking or substance abuse.

56. Labial adhesions


Labial adhesions describe the fusion of the labia minora in the midline. It is usually
seen in girls between the ages of 3 months and 3 years and can generally be treated
conservatively. Spontaneous resolution tends to occur around puberty. It should be
noted that the condition is different from an imperforate hymen.

The majority of cases are symptomatic. Features may include:

• problems with micturition including pooling in the vagina


• on examination thin semitranslucent adhesions covering the vaginal opening
between the labia minora are seen, which sometimes cover the vaginal opening
completely

Management

• conservative management is appropriate in the majority of cases


• if there are associated problems such as recurrent urinary tract infections
oestrogen cream may be tried
• if this fails surgical intervention may be warranted

57. Retinoblastoma

Retinoblastoma is the most common ocular malignancy found in children. The


average age of diagnosis is 18 months.

Pathophysiology

• autosomal dominant
• caused by a loss of function of the retinoblastoma tumour suppressor gene on
chromosome 13
• around 10% of cases are hereditary

Possible features

• absence of red-reflex, replaced by a white pupil (leukocoria) - the most


common presenting symptom
• strabismus
• visual problems

There are a number of causes for a loss of red reflex in young patients, the most
concerning of which is retinoblastoma. This patient should be urgently reviewed by
ophthalmology and undergo a full, dilated ocular exam.
Management

• enucleation is not the only option


• depending on how advanced the tumour is other options include external beam
radiation therapy, chemotherapy and photocoagulation

Prognosis

• excellent, with > 90% surviving into adulthood

58. Child health surveillance

The following table gives a basic outline of child health surveillance in the UK
Ensure intrauterine growth
Check for maternal infections e.g. HIV
Antenatal
Ultrasound scan for fetal abnormalities
Blood tests for Neural Tube Defects
Clinical examination of newborn
Newborn Hearing Screening Programme e.g. oto-acoustic emissions
Newborn
test
Give mother Personal Child Health Record
Heel-prick test day 5-9 - hypothyroidism, PKU, metabolic diseases,
cystic fibrosis, medium-chain acyl Co-A dehydrogenase deficiency
First month
(MCADD)
Midwife visit up to 4 weeks*
Health visitor input
Following
GP examination at 6-8 weeks
months
Routine immunisations
National orthoptist-led programme for pre-school vision screening to
Pre school
be introduced
Monitoring of growth, vision, hearing
Ongoing Health professionals advice on immunisations, diet, accident
prevention

*this doesn't seem to happen in practice with health visitors usually taking over at 2
weeks

59. Cephalohaematoma
A cephalohaematoma is seen as a swelling on the newborns head. It typically
develops several hours after delivery and is due to bleeding between the periosteum
and skull. The most common site affected is the parietal region

Jaundice may develop as a complication.

A cephalohaematoma up to 3 months to resolve.

Comparison of caput succedaneum and cephalohaematoma

A cephalohaematoma appears as a swelling due to bleeding between the


periosteum and the skull. It is most commonly noted in the parietal region and is
associated with instrumental deliveries. The swelling usually appears 2-3 days
following delivery and does not cross suture lines. It gradually resolves over a
number of weeks.

Caput succadeneum is commonly seen in newborns immediately after birth. It


occurs due to generalised superficial scalp oedema, which crosses suture lines. It is
associated with prolonged labour and will rapidly resolve over a couple of days.

Subaponeurotic haematoma is a rare condition where bleeding occurs that is not


bound by the periosteum. It can be life threatening and presents as a fluctuant scalp
swelling, which is not limited by suture lines.

Craniosynostosis is uncommon and is where there is premature closure of cranial


sutures, causing deformity of the skull. It can be evident at birth and may be
associated with genetic syndromes. The shape of the skull will depend upon which
sutures are involved. Other clinical features include early closure of the anterior
fontanelle and raised ridge along the fused suture.

60. Patent ductus arteriosus


Overview

• a form of congenital heart defect


• generally classed as 'acyanotic'. However, uncorrected can eventually result in
late cyanosis in the lower extremities, termed differential cyanosis
• connection between the pulmonary trunk and descending aorta
• usually, the ductus arteriosus closes with the first breaths due to increased
pulmonary flow which enhances prostaglandins clearance
• more common in premature babies, born at high altitude or maternal rubella
infection in the first trimester

Features

• left subclavicular thrill


• continuous 'machinery' murmur
• large volume, bounding, collapsing pulse
• wide pulse pressure
• heaving apex beat

Management

• indomethacin or ibuprofen
o given to the neonate
o inhibits prostaglandin synthesis
o closes the connection in the majority of cases
• if associated with another congenital heart defect amenable to surgery then
prostaglandin E1 is useful to keep the duct open until after surgical repair

*** A patent ductus arteriosis can be kept open with prostaglandins. This may be
useful in duct dependent cardiac lesions

Duct dependent cardiac lesions include: tetralogy of fallot, ebstein’s anomaly,


pulmonary atresia and pulmonary stenosis. They are often diagnosed antenatally but
may present at birth with cyanosis, tachypnoea and poor peripheral pulses. Often the
definitive treatment is surgery, however keeping the duct open can buy time for the
management to be appropriately planned.

Prostaglandins are used to keep the duct open. This is desirable in cases of duct
dependent cardiac lesions where symptoms only become apparent when the duct
begins to close in the first week of life.

Aspirin is avoided in children due to the risk of reyes syndrome. Although rare reyes
syndrome can be fatal due to oedema of the liver and brain. It would not keep the
duct open.

Both Indomethacin and Ibuprofen would close the duct therefore worsening
symptoms.

Congenital heart disease: presentation

Congenital heart disease may present in a number of ways:

• during the antenatal period: imaging of the heart is part of the 18-20 week fetal
anomaly scan
• murmur: may be detected during the routine newborn examine
• cyanosis
• heart failure: features may include poor feeding, shortness of breath,
sweating, hepatomegaly

61. Food allergy in children and young people

The 2011 NICE guidelines differentiate between IgE mediated and non-IgE mediated
allergies. It should be noted that the guidance does not govern food intolerance,
which is not caused by immune system dysfunction.

The first step is to identify possible food allergy and differentiate the possible causes:
IgE-mediated Non-IgE-mediated
Skin

• pruritus
• erythema Skin
• urticaria
• angioedema • pruritus
• erythema
• atopic eczema
Gastrointestinal system

• nausea Gastrointestinal system


• colicky abdominal pain
• vomiting • gastro-oesophageal reflux disease
• diarrhoea • loose or frequent stools
• blood and/or mucus in stools
• abdominal pain
• infantile colic
Respiratory system • food refusal or aversion
• constipation
• upper respiratory tract symptoms - • perianal redness
nasal itching,sneezing, rhinorrhoea • pallor and tiredness
or congestion (with or without • faltering growth plus one or more
conjunctivitis) gastrointestinal symptoms above
• lower respiratory tract symptoms - (with or without significant atopic
cough, chest tightness, wheezing or eczema)
shortness of breath

Symptoms of anaphylaxis

If the history is suggestive of an IgE-mediated allergy

• offer a skin prick test or blood tests for specific IgE antibodies to the suspected
foods and likely co-allergens

If the history is suggestive of an non-IgE-mediated allergy

• eliminate the suspected allergen for 2-6 weeks, then reintroduce. NICE advise to
'consult a dietitian with appropriate competencies about nutritional adequacies,
timings and follow-up'

62. Surfactant deficient lung disease


Surfactant deficient lung disease (SDLD, also known as respiratory distress syndrome
and previously as hyaline membrane disease) is a condition seen in premature
infants. It is caused by insufficient surfactant production and structural immaturity of
the lungs

The risk of SDLD decreases with gestation

• 50% of infants born at 26-28 weeks


• 25% of infants born at 30-31 weeks

Other risk factors for SDLD include

• male sex
• diabetic mothers
• Caesarean section
• second born of premature twins

Clinical features are those common to respiratory distress in the newborn, i.e.
tachypnoea, intercostal recession, expiratory grunting and cyanosis

Chest x-ray characteristically shows 'ground-glass' appearance with an indistinct


heart border

Management

• prevention during pregnancy: maternal corticosteroids to induce fetal lung


maturation
• oxygen
• assisted ventilation
• exogenous surfactant given via endotracheal tube

***Caffeine is used to facilitate weening a neonate off a ventilator, who is born


prematurely.

Sildenafil, brand name Viagra, is a medication used to treat erectile dysfunction in adults.
However in neonates it is used to treat pulmonary hypertension.

The approach to settling distress in the neonate depends upon the cause and is commonly
due to pain or respiratory distress.

Gaviscon and ranitidine are used, off licence, to treat gastro-oesophageal reflux.
Necrotising enterocolitis is much more common in premature babies. Treatment is either
medical (by resting the gut) or surgical (resection).

63. Nephrotic syndrome in children

Nephrotic syndrome is classically defined as a triad of

• proteinuria (> 1 g/m^2 per 24 hours)


• hypoalbuminaemia (< 25 g/l)
• oedema

In children the peak incidence is between 2 and 5 years of age. Around 80% of
cases in children are due to a condition called minimal change
glomerulonephritis. The condition generally carries a good prognosis with around
90% of cases responding to high-dose oral steroids.

Other features include hyperlipidaemia, a hypercoagulable state (due to loss of


antithrombin III) and a predisposition to infection (due to loss of immunoglobulins)

64. Jaundice in the newborn period

Jaundice in the first 24 hrs is always pathological

Causes of jaundice in the first 24 hrs

• rhesus haemolytic disease


• ABO haemolytic disease
• hereditary spherocytosis
• glucose-6-phosphodehydrogenase

Jaundice in the neonate from the c. 2-14 days is common (up to 40%) and usually
physiological. It is more commonly seen in breastfed babies

If there are still signs of jaundice after 14 days a prolonged jaundice screen is
performed, including:

• conjugated and unconjugated bilirubin: the most important test as a raised


conjugated bilirubin could indicate biliary atresia which requires urgent surgical
intervention
• direct antiglobulin test (Coombs' test)
• TFTs
• FBC and blood film
• urine for MC&S and reducing sugars
• U&Es and LFTs

Causes of prolonged jaundice

• biliary atresia
• hypothyroidism
• galactosaemia
• urinary tract infection
• breast milk jaundice
• congenital infections e.g. CMV, toxoplasmosis

Jaundice which develops within the first 24 hours post-delivery is considered always
pathological. Physiological jaundice develops 2-3 days post-delivery and should resolve
by 14 days. Being pre-term increases the risk of developing jaundice. In physiological
jaundice, levels of bilirubin usually do not reach higher than 200 μmol/L.

65. Coeliac disease in children

Coeliac disease is caused by sensitivity to the protein gluten. Repeated exposure


leads to villous atrophy which in turn causes malabsorption. Children normally
present before the age of 3 years, following the introduction of cereals into the diet

Genetics

• incidence of around 1:100


• it is strongly associated with HLA-DQ2 (95% of patients) and HLA-DQ8 (80%)

Features may coincide with the introduction of cereals (i.e. gluten)

• failure to thrive
• diarrhoea
• abdominal distension
• older children may present with anaemia
• many cases are not diagnosed to adulthood

Diagnosis

• jejunal biopsy showing subtotal villous atrophy


• anti-endomysial and anti-gliadin antibodies are useful screening tests
***The most likely diagnosis here is coeliac disease, diagnosed using IgA TTG
antibodies, as explained below.

A stool sample would be diagnostic for gastroenteritis, in order to dictate which


antibiotic should be used.

The hydrogen breath test is used to diagnose irritable bowel syndrome or some food
intolerances.

Endoscopy is more commonly used in adults where cancer is suspected.

An abdominal X-ray may be useful where obstruction is suspected.

Coeliac disease is a digestive condition which is becoming increasingly common, and


describes an adverse reaction to gluten. gluten is a protein found in wheat, barley
and rye.

66. Apgar score

The Apgar score is used to assess the health of a newborn baby

Respiratory
Score Pulse Colour Muscle tone Reflex irritability
effort
Cries on
Active
2 > 100 Strong, crying Pink stimulation/sneezes,
movement
coughs
Weak, Body pink,
1 < 100 Limb flexion Grimace
irregular extremities blue
0 Absent Nil Blue all over Flaccid Nil

A score of 0-3 is very low score, between 4-6 is moderate low and between 7 - 10
means the baby is in a good state
Airway suction should not be performed unless there is obviously thick meconium
causing obstruction, as it can cause reflex bradycardia in babies. Chest compressions
are not indicated, as the HR in this case is >100bpm. CPR should only be commenced
at a HR < 60bpm. In cases where there are no signs of breathing and this is thought to
be due to fluid in the lungs, five breaths should be given via a 250ml bag via face
mask. This is a more effective and more hygienic method than using mouth-to-mouth
in a hospital setting

67. Gastroschisis and exomphalos

Gastroschisis and exomphalos are both examples of congenital visceral


malformations.
Babies with gastroschisis experience a significant loss of fluid and heat from the
exposed bowel. This can be life-threatening.

Gastroschisis tends to be a stand-alone medical condition. It is exomphalos that is


associated with cardiac and renal problems.

Gastroschisis

Gastroschisis describes a congenital defect in the anterior abdominal wall just lateral
to the umbilical cord.

Management
• vaginal delivery may be attempted
• newborns should go to theatre as soon as possible after delivery, e.g. within 4
hours

Exomphalos (omphalocoele)

In exomphalos (also known as an omphalocoele) the abdominal contents protrude


through the anterior abdominal wall but are covered in an amniotic sac formed by
amniotic membrane and peritoneum.

Associations

• Beckwith-Wiedemann syndrome
• Down's syndrome
• cardiac and kidney malformations

Management

• caesarean section is indicated to reduce the risk of sac rupture


• a staged repair may be undertaken as primary closure may be difficult due to
lack of space/high intra-abdominal pressure
o if this occurs the sacs is allowed to granulate and epithelialise over the
coming weeks/months
o this forms a 'shell'
o as the infant grows a point will be reached when the sac contents can fit
within the abdominal cavity. At this point the shell will be removed and
the abdomen closed

68. Acute lymphoblastic leukaemia

Acute lymphoblastic leukaemia (ALL) is the most common malignancy affecting


children and accounts for 80% of childhood leukaemias. The peak incidence is at
around 2-5 years of age and boys are affected slightly more commonly than girls

Features may be divided into those predictable by bone marrow failure:

• anaemia: lethargy and pallor


• neutropaenia: frequent or severe infections
• thrombocytopenia: easy bruising, petechiae (on abdomen)

And other features

• bone pain (secondary to bone marrow infiltration)


• splenomegaly
• hepatomegaly
• fever is present in up to 50% of new cases (representing infection or
constitutional symptom)
• testicular swelling

Types

• common ALL (75%), CD10 present, pre-B phenotype


• T-cell ALL (20%)
• B-cell ALL (5%)

Poor prognostic factors

• age < 2 years or > 10 years


• WBC > 20 * 109/l at diagnosis
• T or B cell surface markers
• non-Caucasian
• male sex

***The lesions of Henoch-Schonlein purpura are normally confined to buttocks, extensor


surfaces of legs and arms. Other symptoms such as haematuria or swollen painful knees and
ankles.

69. Undescended testis

Undescended testis occurs in around 2-3% of term male infants, but is much more
common if the baby is preterm. Around 25% of cases are bilateral.

Complications of undescended testis

• infertility
• torsion
• testicular cancer
• psychological

Management

• Unilateral undescended testis


o NICE CKS now recommend referral should be considered from around 3
months of age, with the baby ideally seeing a urological surgeon before 6
months of age
o Orchidopexy: Surgical practices vary although the majority of procedures
are performed at around 1 year of age
• Bilateral undescended testes
o Should be reviewed by a senior paediatrician within 24hours as the child
may need urgent endocrine or genetic investigation

70. Attention Deficit Hyperactivity Disorder

March 2018 saw NICE issue new guidance around recognising and managing attention
deficit hyperactivity disorder (ADHD). This condition can inflict significant morbidity on a
child's life and thus has consequences into adulthood, making good diagnosis and treatment
vital.

DSM-V defines ADHD as a condition incorporating features relating to inattention and/or


hyperactivity/impulsivity that are persistent. Like many paediatric conditions, there has to be
an element of developmental delay. For children up to the age of 16 years, six of these
features have to be present; in those aged 17 or over, the threshold is five features (Table
below).

Epidemiology

• ADHD has a UK prevalence of 2.4%, about twice that of autism, and is more common
in boys than in girls (M:F 4:1);
• Most children are diagnosed between the ages of 3 and 7;
• There is a possible genetic component.

Diagnostic Features

Inattention Hyperactivity/Impulsivity
Does not follow through on instructions Unable to play quietly
Reluctant to engage in mentally-intense
Talks excessively
tasks
Easily distracted Does not wait their turn easily
Will spontaneously leave their seat when expected
Finds it difficult to sustain tasks
to sit
Finds it difficult to organise tasks or
Is often 'on the go'
activities
Often forgetful in daily activities Often interruptive or intrusive to others
Often loses things necessary for tasks or Will answer prematurely, before a question has
activities been finished
Often does not seem to listen when WIll run and climb in situations where it is not
spoken to directly appropriate

Management

NICE stipulates a holistic approach to treating ADHD that isn't entirely reliant on
therapeutics. Following presentation, a ten-week 'watch and wait' period should follow to
observe whether symptoms change or resolve. If they persist then referral to secondary care
is required. This is normally to a paediatrician with a special interest in behavioural disorders,
or to the local Child and Adolescent Mental Health Service (CAMHS). Here, the needs and
wants of the patient, as well as how their condition affects their lives should be taken into
account, to offer a tailored plan of action.

Drug therapy should be seen as a last resort and is only available to those aged 5 years or
more. Patients with mild/moderate symptoms can usually benefit from their parents
attending education and training programmes. For those who fail to respond, or whose
symptoms are severe, pharmacotherapy can be considered:

• Methylphenidate is first line in children and should initially be given on a six-week


trial basis. It is a CNS stimulant which primarily acts as a dopamine/norepinephrine
reuptake inhibitor. Side-effects include abdominal pain, nausea and dyspepsia. In
children, weight and height should be monitored every 6 months
• If there is inadequate response, switch to lisdexamfetamine;
• Dexamfetamine should be started in those who have benefited from
lisdexamfetamine, but who can't tolerate its side effects.

In adults:

• Methylphenidate or lisdexamfetamine are first-line options;


• Switch between these drugs if no benefit is seen after a trial of the other.

All of these drugs are potentially cardiotoxic. Perform a baseline ECG before starting
treatment, and refer to a cardiologist if there is any significant past medical history or family
history, or any doubt or ambiguity.

Like most psychiatric conditions, whether adult or paediatric, a thorough history and
clinical examination are key, especially given the overlap of ADHD with many other
psychiatric and physical conditions.

71. Eczema in children

Eczema occurs in around 15-20% of children and is becoming more common. It


typically presents before 6 months but clears in around 50% of children by 5 years of
age and in 75% of children by 10 years of age

Features

• in infants the face and trunk are often affected


• in younger children eczema often occurs on the extensor surfaces
• in older children a more typical distribution is seen, with flexor surfaces affected
and the creases of the face and neck
Management

• avoid irritants
• simple emollients: large quantities should be prescribed (e.g. 250g / week),
roughly in a ratio of with topical steroids of 10:1. If a topical steroid is also
being used the emollient should be applied first followed by waiting at
least 30 minutes before applying the topical steroid thinly. Creams soak into
the skin faster than ointments. Emollients can become contaminated with
bacteria - fingers should not be inserted into pots (many brands have pump
dispensers)
• topical steroids hydrocortisone 1% ointment thinly twice a day
• in severe cases wet wraps and oral ciclosporin may be used

72. Epstein's pearl

A congenital cyst found in the mouth. They are common on the hard palate, but may
also be seen on the gums where the parents may mistake it for an erupting tooth. No
treatment is generally required as they tend to spontaneously resolve over the
course of a few weeks.

**Neonatal teeth is rare and not usually found on the hard palate.
Bohn's nodules are mucous gland cysts, frequently located on the buccal or lingual
aspects of the alveolar ridges and rarely on the palate.
A congenital ranula is a translucent, firm papule or nodule found on the anterior
floor of mouth, lateral to lingual frenulum.
Alveolar lymphangioma is bluish fluid-filled lesions on the alveolar ridges, not seen
on the palate.

73. Seborrhoeic dermatitis in children

Seborrhoeic dermatitis is a relatively common skin disorder seen in children. It


typically affects the scalp ('Cradle cap'), nappy area, face and limb flexures.

Cradle cap is an early sign which may develop in the first few weeks of life. It is
characterised by an erythematous rash with coarse yellow scales.

Management depends on severity

• mild-moderate: baby shampoo and baby oils


• severe: mild topical steroids e.g. 1% hydrocortisone
Seborrhoeic dermatitis in children tends to resolve spontaneously by around 8
months of age.

74. Paediatric basic life support

The 2015 Resuscitation Council guidelines made the following changes to paediatric
basic life support

• compression:ventilation ratio: lay rescuers should use a ratio of 30:2. If there are
two or more rescuers with a duty to respond then a ratio of 15:2 should be used
• age definitions: an infant is a child under 1 year, a child is between 1 year and
puberty

Key points of algorithm (please see link attached for more details)

• unresponsive?
• shout for help
• open airway
• look, listen, feel for breathing
• give 5 rescue breaths
• check for signs of circulation
o infants use brachial or femoral pulse, children use femoral pulse
• 15 chest compressions: 2 rescue breaths (see above)

75. Feverish illness in children

The 2007 NICE Feverish illness in children guidelines introduced a 'traffic light'
system for risk stratification of children under the age of 5 years presenting with a
fever. These guidelines were later modified in a 2013 update.

It should be noted that these guidelines only apply 'until a clinical diagnosis of the
underlying condition has been made'. A link to the guidelines is provided but some
key points are listed below.

Assessment

The following should be recorded in all febrile children:

• temperature
• heart rate
• respiratory rate
• capillary refill time

Signs of dehydration (reduced skin turgor, cool extremities etc) should also be
looked for

Measuring temperature should be done with an electronic thermometer in the axilla


if the child is < 4 weeks or with an electronic/chemical dot thermometer in the axilla
or an infra-red tympanic thermometer.

Risk stratification

Please see the link for the complete table, below is a modified version

Green - low Amber - intermediate


Red - high risk
risk risk
• Pallor reported by
Colour • Normal colour • Pale/mottled/ashen/blue
parent/carer
• No response to social
• Responds
cues
normally to • Not responding
• Appears ill to a
social cues normally to social cues
healthcare professional
• Content/smiles • No smile
Activity • Does not wake or if
• Stays awake or • Wakes only with
roused does not stay
awakens quickly prolonged stimulation
awake
• Strong normal • Decreased activity
• Weak, high-pitched or
cry/not crying
continuous cry
• Nasal flaring
• Tachypnoea:
respiratory rate

• >50
• Grunting
breaths/minute,
• Tachypnoea: respiratory
age 6-12 months;
Respiratory rate >60 breaths/minute
• >40
• Moderate or severe chest
breaths/minute,
indrawing
age >12 months

• Oxygen saturation
<=95% in air
• Crackles in the chest
Circulation
• Normal skin
and • Tachycardia: • Reduced skin turgor
and eyes
hydration
Green - low Amber - intermediate
Red - high risk
risk risk
• Moist mucous
membranes • >160
beats/minute,
age <12 months
• >150
beats/minute,
age 12-24
months
• >140
beats/minute,
age 2-5 years

• Capillary refill time


>=3 seconds
• Dry mucous
membranes
• Poor feeding in infants
• Reduced urine output
• Age 3-6 months,
• Age <3 months,
temperature >=39ºC
temperature >=38°C
• Fever for >=5 days
• Non-blanching rash
• Rigors
No amber or • Bulging fontanelle
Other • Swelling of a limb or
red signs • Neck stiffness
joint
• Status epilepticus
• Non-weight bearing
• Focal neurological signs
limb/not using an
• Focal seizures
extremity

Management

If green:

• Child can be managed at home with appropriate care advice, including when to
seek further help

If amber:

• provide parents with a safety net or refer to a paediatric specialist for further
assessment
• a safety net includes verbal or written information on warning symptoms and
how further healthcare can be accessed, a follow-up appointment, liaison with
other healthcare professionals, e.g. out-of-hours providers, for further follow-up

If red:

• refer child urgently to a paediatric specialist

Other key points include

• oral antibiotics should not be prescribed to children with fever without apparent
source
• if a pneumonia is suspected but the child is not going to be referred to hospital
then a chest x-ray does not need to be routinely performed

76. Measles

Measles is now rarely seen in the developed world following the adoption of
immunisation programmes. Outbreaks are occasionally seen, particularly when
vaccinations rates drop, for example after the MMR controversy of the early 2000s.

Overview

• RNA paramyxovirus
• spread by droplets
• infective from prodrome until 4 days after rash starts
• incubation period = 10-14 days

Features

• prodrome: irritable, conjunctivitis, fever


• Koplik spots (before rash): white spots ('grain of salt') on buccal mucosa
• rash: starts behind ears then to whole body, discrete maculopapular rash
becoming blotchy & confluent
• diarrhoea occurs in around 10% of patients

Investigations

• IgM antibodies can be detected within a few days of rash onset

Management

• mainly supportive
• admission may be considered in immunosuppressed or pregnant patients
• notifiable disease → inform public health

Complications

• otitis media: the most common complication


• pneumonia: the most common cause of death
• encephalitis: typically occurs 1-2 weeks following the onset of the illness)
• subacute sclerosing panencephalitis: very rare, may present 5-10 years following
the illness
• febrile convulsions
• keratoconjunctivitis, corneal ulceration
• diarrhoea
• increased incidence of appendicitis
• myocarditis

Management of contacts

• if a child not immunized against measles comes into contact with measles then
MMR should be offered (vaccine-induced measles antibody develops more
rapidly than that following natural infection)
• this should be given within 72 hours

77. Hypospadias

Hypospadias is a congenital abnormality of the penis which occurs in approximately


3/1,000 male infants. There appears to be a significant genetic element, with further
male children having a risk of around 5-15%.

It is usually identified on the newborn baby check. If missed, parents may notice an
abnormal urine stream.

Hypospadias is characterised by

• a ventral urethral meatus


• a hooded prepuce
• chordee (ventral curvature of the penis) in more severe forms
• the urethral meatus may open more proximally in the more severe variants.
However, 75% of the openings are distally located.
Hypospadias most commonly occurs as an isolated disorder. However, associated
conditions include cryptorchidism (present in 10%) and inguinal hernia.

Management

• once hypospadias has been identified, infants should be referred to specialist


services
• corrective surgery is typically performed when the child is around 12
months of age
• it is essential that the child is not circumcised prior to the surgery as the
foreskin may be used in the corrective procedure
• in boys with very distal disease, no treatment may be needed.

78. Hearing testing in children

The table below summarises the hearing tests which may be performed on children

Age Test Comments


All newborns should be tested as part of the
Newborn Hearing Screening Programme. A
Otoacoustic
Newborn computer generated click is played through a small
emission test
earpiece. The presence of a soft echo indicates a
healthy cochlea
Auditory
Newborn &
Brainstem May be done if otoacoustic emission test is abnormal
infants
Response test
6-9 months Distraction test Performed by health visitor, requires two trained staff
Age Test Comments
18 months Recognition of Uses familiar objects e.g. teddy, cup. Ask child simple
- 2.5 years familiar objects questions - e.g. 'where is the teddy?'
Performance
> 2.5 years -
testing
Speech
Uses similar sounding objects e.g. Kendall Toy test,
> 2.5 years discrimination
McCormick Toy Test
tests
Pure tone
> 3 years Done at school entry in most areas of the UK
audiometry

As well as the above test there is a questionnaire for parents in the Personal Child
Health Records - 'Can your baby hear you?'

79. Hypotonia

Hypotonia, or floppiness, may be central in origin or related to nerve and muscle


problems. An acutely ill child (e.g. septicaemic) may be hypotonic on
examination. Hypotonia associated with encephalopathy in the newborn period is
most likely caused by hypoxic ischaemic encephalopathy

Central causes

• Down's syndrome
• Prader-Willi syndrome
• hypothyroidism
• cerebral palsy (hypotonia may precede the development of spasticity)

Neurological and muscular problems

• spinal muscular atrophy


• spina bifida
• Guillain-Barre syndrome
• myasthenia gravis
• muscular dystrophy
• myotonic dystrophy

Poor tone in a neonate is never normal. It can indicate serious infection therefore this
baby should receive a septic screen (C-reactive protein, full blood count, blood gas
and blood culture) and antibiotics immediately.
Hypoglycaemia can present with hypotonia so it important to check this baby's
blood sugar. However, hypoglycaemia is more likely to present with jitteriness and
lethargy on a background of poor intake.

80. Benign rolandic epilepsy

Benign rolandic epilepsy is a form of childhood epilepsy which typically occurs


between the age of 4 and 12 years.

Features

• seizures characteristically occur at night


• seizures are typically partial (e.g. paraesthesia affecting face) but
secondary generalisation may occur (i.e. parents may only report tonic-
clonic movements)
• child is otherwise normal

EEG characteristically shows centro-temporal spikes


Prognosis is excellent, with seizures stopping by adolescence

***
Rolandic seizures are a form of epilepsy that is often seen in children but will
disappear as they reach adolescence. The seizures start around the central sulcus of
the brain (located near the rolandic fissure) and usually present with facial
symptoms such as hypersalivation, drooling, loss of speech, facial twitching and
numbness of the face or tongue. The seizures can also spread to other parts of the
brain and present with symptoms in other areas. Consciousness usually remains
during the seizures but it is possible for the patient to not have any recollection in
the post-ictal period.
Reflex anoxic seizures are syncope and subsequent convulsions caused by anoxia,
typically occur in very young children.

Panayiotopoulos syndrome is characterised by seizures, often prolonged, with


predominantly autonomic symptoms such as nausea and retching.

Idiopathic childhood occipital epilepsy of Gastaut is mainly characterised by visual


hallucinations.

The night terror is a sleep disorder causing feelings of dread or terror typically during
REM sleep.

81. Napkin rashes


Causes of a napkin ('nappy') rash include the following:

Type Notes
The most common cause, due to irritant effect of urinary
Irritant dermatitis ammonia and faeces
Creases are characteristically spared
Candida Typically an erythematous rash which involve the flexures and
dermatitis has characteristic satellite lesions
Seborrhoeic
Erythematous rash with flakes. May be coexistent scalp rash
dermatitis
A less common cause characterised by an erythematous scaly
Psoriasis
rash also present elsewhere on the skin
Atopic eczema Other areas of the skin will also be affected

General management points

• disposable nappies are preferable to towel nappies


• expose napkin area to air when possible
• apply barrier cream (e.g. Zinc and castor oil)
• mild steroid cream (e.g. 1% hydrocortisone) in severe cases
• management of suspected candidal nappy rash is with a topical imidazole.
Cease the use of a barrier cream until the candida has settled

82. Puberty

Males

• first sign is testicular growth at around 12 years of age (range = 10-15 years)
• testicular volume > 4 ml indicates onset of puberty
• maximum height spurt at 14

Females

• first sign is breast development at around 11.5 years of age (range = 9-13 years)
• height spurt reaches its maximum early in puberty (at 12) , before menarche
• menarche at 13 (11-15)
• there is an increase of only about 4% of height following menarche

Normal changes in puberty

• gynaecomastia may develop in boys


• asymmetrical breast growth may occur in girls
• diffuse enlargement of the thyroid gland may be seen
83. Urinary tract infection in children: investigation

In contrast to adults, the development of a urinary tract infection (UTI) in childhood


should prompt consideration of a possible underlying causes and damage to the
kidneys (renal scarring)

NICE guidelines for imaging the urinary tract

• infants < 6 months who present with a first UTI which responds to treatment
should have an ultrasound within 6 weeks
• children > 6 months who present with a first UTI which responds to treatment
do not require imaging unless there are features suggestive of an atypical
infection (see below) or recurrent infection

Features of suggestive of an atypical infection

• seriously ill
• poor urine flow
• abdominal or bladder mass
• raised creatinine
• septicaemia
• failure to respond to treatment with suitable antibiotics within 48 hours
• infection with non-E. coli organisms

Possible further investigations

• urine for microscopy and culture: urine should be sent for culture as only 50% of
children with a UTI have pyuria. Microscopy or dipstick of the urine is therefore
inadequate for diagnosis
• static radioisotope scan (e.g. DMSA): identifies renal scars. Should be done 4-6
months after initial infection
• micturating cystourethrography (MCUG): identifies vesicoureteric reflux. Only
recommended for infants younger than 6 months who present with atypical or
recurrent infections

*The most common organism causing a UTI in both children and adults is E. coli

84. School exclusion

The table below summarises Health Protection Agency guidance on school exclusion
Advice Condition(s)
Conjunctivitis
Fifth disease (slapped
cheek)
Roseola
No exclusion Infectious
mononucleosis
Head lice
Threadworms
Hand, foot and mouth
24 hours after commencing antibiotics Scarlet fever
2 days after commencing antibiotics (or 21 days from onset
Whooping cough
of symptoms if no antibiotics )
4 days from onset of rash Measles
5 days from onset of rash Rubella
All lesions crusted over Chickenpox*
5 days from onset of swollen glands Mumps
Until symptoms have settled for 48 hours Diarrhoea & vomiting
Until lesions are crusted and healed, or 48 hours
Impetigo
after commencing antibiotic treatment
Until treated Scabies
Until recovered Influenza

Cases of chickenpox are generally infectious from 2 days before the rash appears to 5
days after the onset of rash.

Although the usual exclusion period is 5 days, all lesions should be crusted over before
children return to nursery or school.

However, Clinical Knowledge Summaries are a bit vaguer:

Advise that the most infectious period is 1-2 days before the rash appears, but
infectivity continues until all the lesions are dry and have crusted over (usually about 5
days after the onset of the rash).

85. Constipation in children

The frequency at which children open their bowels varies widely but generally
decreases with age from a mean of 3 times per day for infants under 6 months old to
once a day after 3 years of age.
NICE produced guidelines in 2010 on the diagnosis and management of constipation
in children. A diagnosis of constipation is suggested by 2 or more of the following:

Child < 1 year Child > 1 year


Fewer than 3 complete
stools per week (type 3 or
Fewer than 3 complete stools per week
4)
(type 3 or 4 on Bristol Stool Form Scale)
Overflow soiling
(this does not apply to exclusively
(commonly very loose,
Stool pattern breastfed babies after 6 weeks
very smelly, stool passed
of age)
without sensation)
Hard large stool
'Rabbit droppings' (type 1)
'Rabbit droppings' (type 1)
Large, infrequent stools
that can block the toilet
Poor appetite that
improves with passage of
large stool
Waxing and waning of
abdominal pain with
Symptoms Distress on passing stool passage of stool
associated with Bleeding associated with hard stool Evidence of retentive
defecation Straining posturing: typical straight-
legged, tiptoed, back
arching
posture
Straining
Anal pain
Previous episode(s) of
constipation
Previous or current anal
Previous episode(s) of constipation
History fissure
Previous or current anal fissure
Painful bowel movements
and bleeding associated
with hard stools

The vast majority of children have no identifiable cause - idiopathic constipation.


Other causes of constipation in children include:

• dehydration
• low-fibre diet
• medications: e.g. Opiates
• anal fissure
• over-enthusiastic potty training
• hypothyroidism
• Hirschsprung's disease
• hypercalcaemia
• learning disabilities

After making a diagnosis of constipation NICE then suggesting excluding secondary


causes. If no red or amber flags are present then a diagnosis of idiopathic
constipation can be made:

'Red flag' suggesting


Indicates idiopathic constipation
underlying disorder
Starts after a few weeks of life
Obvious precipitating factors coinciding
with the start of symptoms: fissure,
Reported from birth
change of diet, timing of potty/toilet
Timing or first few weeks of
training or acute events such as infections,
life
moving house, starting nursery/school,
fears and phobias, major change in family,
taking medicines
Passage of
< 48 hours > 48 hours
meconium
Stool pattern 'Ribbon' stools
Generally well, weight and height within Faltering growth is an
Growth
normal limits, fit and active amber flag
Previously unknown
No neurological problems in legs, normal or undiagnosed
Neuro/locomotor
locomotor development weakness in legs,
locomotor delay
Abdomen Distension
Changes in infant formula, weaning,
Diet
insufficient fluid intake or poor diet
Amber flag:
Disclosure or evidence
Other that raises concerns
over possibility of
child maltreatment

Prior to starting treatment, the child needs to be assessed for faecal impaction.
Factors which suggest faecal impaction include:

• symptoms of severe constipation


• overflow soiling
• faecal mass palpable in the abdomen (digital rectal examination should only be
carried out by a specialist)

NICE guidelines on management

If faecal impaction is present

• polyethylene glycol 3350 + electrolytes (Movicol Paediatric Plain) using an


escalating dose regimen as the first-line treatment
• add a stimulant laxative if Movicol Paediatric Plain does not lead to disimpaction
after 2 weeks
• substitute a stimulant laxative singly or in combination with an osmotic laxative
such as lactulose if Movicol Paediatric Plain is not tolerated
• inform families that disimpaction treatment can initially increase symptoms of
soiling and abdominal pain

Maintenance therapy

• very similar to the above regime, with obvious adjustments to the starting dose,
i.e.
• first-line: Movicol Paediatric Plain
• add a stimulant laxative if no response
• substitute a stimulant laxative if Movicol Paediatric Plain is not tolerated. Add
another laxative such as lactulose or docusate if stools are hard
• continue medication at maintenance dose for several weeks after regular bowel
habit is established, then reduce the dose gradually

General points

• do not use dietary interventions alone as first-line treatment although ensure


the child is having adequate fluid and fibre intake
• consider regular toileting and non-punitive behavioural interventions
• for all children consider asking the Health Visitor or Paediatric Continence
Advisor to help support the parents.

The NICE guidelines do not specifically discuss the management of a very young
child. The following recommendations are largely based on the old Clinical
Knowledge Summaries recommendations.

Infants not yet weaned (usually < 6 months)


• bottle-fed infants: give extra water in between feeds. Try gentle abdominal
massage and bicycling the infant's legs
• breast-fed infants: constipation is unusual and organic causes should be
considered

Infants who have or are being weaned

• offer extra water, diluted fruit juice and fruits


• if not effective consider adding lactulose
ORTHOPAEDICS
1.Lower back pain

Lower back pain (LBP) is one of the most common presentations seen in practice.
Whilst the majority of presentations will be of a non-specific muscular nature it is
worth keeping in mind possible causes which may need specific treatment.

Red flags for lower back pain


• age < 20 years or > 50 years
• history of previous malignancy
• night pain
• history of trauma
• systemically unwell e.g. weight loss, fever

The table below indicates some specific causes of LBP:


May be acute or chronic
Pain worse in the morning and on standing
Facet joint
On examination there may be pain over the facets. The pain is
typically worse on extension of the back
Usually gradual onset
Unilateral or bilateral leg pain (with or without back pain),
numbness, and weakness which is worse on walking. Resolves when
Spinal stenosis sits down. Pain may be described as 'aching', 'crawling'.
Relieved by sitting down, leaning forwards and crouching down
Clinical examination is often normal
Requires MRI to confirm diagnosis
Typically a young man who presents with lower back pain and
Ankylosing stiffness
spondylitis Stiffness is usually worse in morning and improves with activity
Peripheral arthritis (25%, more common if female)
Pain on walking, relieved by rest
Peripheral
Absent or weak foot pulses and other signs of limb ischaemia
arterial disease
Past history may include smoking and other vascular diseases

**A positive straight leg raise is performed by raising the leg whilst it is straight if this causes
pain in the distribution of the sciatic nerve then the test is positive.

Ankylosing spondylitis is a chronic inflammatory rheumatic disease which results in lower


back pain that is worse in the mornings and typically improved by exercise. The condition
usually begins in early adulthood and is associated with human leukocyte antigen B27 (HLA-
B27).
Spinal stenosis presents with back and/or leg pain that is associated with numbness, and
weakness that worsens on walking but is relieved by rest. Facet joint pain is back pain that is
felt over the facets (superior and inferior processes of each vertebra). It most commonly
occurs in the cervical and lumbar vertebrae and is typically worsened by stress on the facet
joints, e.g. by extending the back. A vertebral compression fracture is usually due to
osteoporosis, trauma or malignancy and presents with acute or chronic back pain and can be
diagnosed by an x-ray.

LOWER BACK PAIN INVESTIGATION AND MANAGEMENT:

NICE updated their guidelines on the management of lower back pain in 2016. They
apply to patients with non-specific lower back pain (i.e. not due to malignancy,
infection, trauma etc)

NSAIDs are now first-line for back pain

Investigation
• lumbar spine x-ray should not be offered
• MRI
o should only be offered to patients with non-specific back pain 'only if the
result is likely to change management' and to patients where malignancy,
infection, fracture, cauda equina or ankylosing spondylitis is suspected
o it is the most useful imaging modality as no other imaging can see
neurological / soft tissue structures
Advice to people with low back pain
• try to encourage self-management
• stay physically active and exercise

Analgesia

• NSAIDS are now recommended first-line for patients with back pain. This follows
studies that show paracetamol monotherapy is relatively ineffective for back
pain
• proton pump inhibitors should be co-prescribed for patients over the age of 45
years who are given NSAIDs
• NICE guidelines on neuropathic pain should be followed for patients with
sciatica

Other possible treatments

• exercise programme: 'Consider a group exercise programme (biomechanical,


aerobic, mindbody or a combination of approaches) within the NHS for people '
• manual therapy (spinal manipulation, mobilisation or soft tissue techniques such
as massage) 'but only as part of a treatment package including exercise, with or
without psychological therapy.'
• radiofrequency denervation
• epidural injections of local anaesthetic and steroid for acute and severe sciatica

**NSAIDs are now first-line management of lower back pain following updated NICE
guidelines in 2016. Recommended NSAIDs include ibuprofen or naproxen and consideration
should be given to co-administration of PPI.
Paracetamol alone is not recommended for lower back pain and for patients unable to
tolerate NSAIDs co-codamol should be considered.

**A 65-year-old man presents with bilateral leg pain that is brought on by walking. His past
medical history includes peptic ulcer disease and osteoarthritis. He can typically walk for
around 5 minutes before it develops. The pain subsides when he sits down. He has also
noticed that leaning forwards or crouching improves the pain. Musculoskeletal and vascular
examination of his lower limbs is unremarkable. What is the most likely diagnosis?
Answer: This is a classic presentation of spinal stenosis. Whilst peripheral arterial disease is an
obvious differential the characteristic relieving factors of the pain and normal vascular
examination point away from this diagnosis.

LOWER BACK PAIN: PROLAPSED DISC:

A prolapsed lumbar disc usually produces clear dermatomal leg pain associated with
neurological deficits.

Features
• leg pain usually worse than back
• pain often worse when sitting

The table below demonstrates the expected features according to the level of
compression:

Site of compression Features


Sensory loss over anterior thigh
L3 nerve root Weak quadriceps
compression Reduced knee reflex
Positive femoral stretch test
Sensory loss anterior aspect of knee
L4 nerve root Weak quadriceps
compression Reduced knee reflex
Positive femoral stretch test
L5 nerve root Sensory loss dorsum of foot
compression Weakness in foot and big toe dorsiflexion
Site of compression Features
Reflexes intact
Positive sciatic nerve stretch test
Sensory loss posterolateral aspect of leg and lateral
aspect of foot
S1 nerve root
Weakness in plantar flexion of foot
compression
Reduced ankle reflex
Positive sciatic nerve stretch test

Management

• similar to that of other musculoskeletal lower back pain: analgesia,


physiotherapy, exercises
• if symptoms persist (e.g. after 4-6 weeks) then referral for consideration of MRI
is appropriate.
• A referral for sciatica is appropriate after 4-6 months of conservative treatment
with analgesia and physiotherapy.

**This man is presenting with sciatica along the L5 distribution caused by a small disc
prolapse. There are no features in the clinical history or on the MRI scan which
suggest a need for an urgent referral (no bladder/bowel symptoms, no weakness and
no cauda equina compression seen on imaging). The duration of symptoms is less
than 4 weeks and conservative management has not been attempted yet so this
should be the first-line management at this point in time. This comprises of a trial of
an anti-neuropathic pain agent (such as gabapentin, pregabalin or amitriptyline) and
physiotherapy. Approximately 90% of cases of sciatica settle within 3 months with
conservative management and it is rare to actually need a referral to specialist
services.

If the pain has failed to settle after 4-6 weeks of physiotherapy and anti-neuropathic
agent treatment then it would be sensible to consider referring routinely to spinal
surgery. Usually if the pain has not settled within this period, the patient will need an
intervention to treat their pain such as an injection or an operation.

Reassurance and discharge would be inappropriate given the small risk of worsening
of symptoms and progression to cauda equina syndrome. All patients with nerve
root compression should be given red-flag advice for cauda equina syndrome and
advised to attend the emergency department if they develop any of the following:

• Peri-anal or saddle sensory change


• Difficulty passing urine
• Symptoms affecting both legs
Treatment with opiates is inappropriate and unlikely to be of any benefit in
neuropathic pain and will simply give the patient a dependence issue. Given his age a
more appropriate and effective general analgesic would be a non-steroidal anti-
inflammatory drug (NSAID) such as naproxen.

Lower back pain: Lumbar spinal stenosis

Lumbar spinal stenosis is a condition in which the central canal is narrowed by tumour, disk
prolapse or other similar degenerative changes.

Patients may present with a combination of back pain, neuropathic pain and symptoms
mimicking claudication. One of the main features that may help to differentiate it from true
claudication in the history is the positional element to the pain. Sitting is better than
standing and patients may find it easier to walk uphill rather than downhill. The
neurogenic claudication type history makes lumbar spinal stenosis a likely underlying
diagnosis, the absence of such symptoms makes it far less likely.

Pathology

Degenerative disease is the commonest underlying cause. Degeneration is believed to begin


in the intervertebral disk where biochemical changes such as cell death and loss of
proteoglycan and water content lead to progressive disk bulging and collapse. This process
leads to an increased stress transfer to the posterior facet joints, which accelerates
cartilaginous degeneration, hypertrophy, and osteophyte formation; this is associated with
thickening and distortion of the ligamentum flavum. The combination of the ventral disk
bulging, osteophyte formation at the dorsal facet, and ligamentum flavum hyptertrophy
combine to circumferentially narrow the spinal canal and the space available for the neural
elements. The compression of the nerve roots of the cauda equina leads to the characteristic
clinical signs and symptoms of lumbar spinal stenosis.

Diagnosis

MRI scanning is the best modality for demonstrating the canal narrowing. Historically a
bicycle test was used as true vascular claudicants could not complete the test.

Treatment: Laminectomy

2. Knee problems: older adults

**A meniscal tear is usually caused by twisting of the knee and on examination, McMurrays
test will be positive. To perform McMurrays test, the knee is held in one hand, which is
placed along the joint line, and flexed while the sole of the foot is held with the other hand.
One hand is placed on the medial side of the knee to pull the knee towards a varus position.
The other hand is used to rotate the leg internally whilst extending the knee. If pain or a
'click' is felt, this constitutes a 'positive McMurray test'.
An anterior cruciate ligament tear is usually also caused by twisting of the knee and would
be exemplified by a positive draw test

The table below summarises the key features of common knee problems:

Condition Key features


Patient is typically > 50 years, often overweight
Osteoarthritis of Pain may be severe
the knee Intermittent swelling, crepitus and limitation of movement may
occur
Associated with kneeling.
Infrapatellar
Inflammation of the infrapatellar bursae is due to similar wear
bursitis
between the skin, patellar tendon and tibial tuberosity
(Clergyman's knee)
Occupation: priest
Associated with more upright kneeling.
Prepatellar bursitis The prepatellar bursa lies anterior to the patella, and inflammation can
(Housemaid's knee) be caused by excessive skin/patella friction, e.g. by working on one's
knees. For this reason, it has been known as 'Housemaid's knee'.
May be caused by twisting of the knee - 'popping' noise may
Anterior cruciate have been noted
ligament Rapid onset of knee effusion
Positive draw test
Posterior cruciate May be caused by anterior force applied to the proximal tibia
ligament (e.g. knee hitting dashboard during car accident)
Tenderness over the affected ligament
Collateral ligament
Knee effusion may be seen
May be caused by twisting of the knee
Meniscal lesion Locking and giving-way are common feature
Tender joint line
**Baker's cyst:
They are more likely to develop in patients with arthritis or gout and following a minor
trauma to the knee. Foucher's sign describes the increase in tension of the Baker's cyst on
extension of the knee.

A DVT (deep vein thrombosis) needs to be considered because it can mimic a Baker's cyst. A
DVT can also co-exist with a Baker's cyst and a low threshold for ultrasound should be
considered.

Knee pain: meniscal tear

Typically result from twisting injuries.

• pain worse on straightening the knee


• knee may 'give way'
• displaced meniscal tears may cause knee locking
• tenderness along the joint line
• Thessaly's test - weight bearing at 20 degrees of knee flexion, patient supported
by doctor, postive if pain on twisting knee

***
Knee locking and giving-way are common features of the meniscal lesions. A positive
Thessaly's test confirms the diagnosis.

ACL injury typically presents more acutely with a history of popping, immediate pain and
swelling and instability of the knee.
LCL injury typically occurs from direct impact on the LCL e.g. contact sport. There is usually
pain and swelling particularly at the lateral side of the knee joint.

Fat pad impingement typically has a history of knee hyperextension.

Osteochondritis dissecans is typically a condition in adolescents.

3. Cubital tunnel syndrome

Cubital tunnel syndrome occurs due to compression of the ulnar nerve as it passes
through the cubital tunnel.

Clincial features

• Tingling and numbness of the 4th and 5th finger which starts off intermittent
and then becomes constant.
• Over time patients may also develop weakness and muscle wasting
• Pain worse on leaning on the affected elbow
• Often a history of osteoarthritis or prior trauma to the area.

Investigations

• the diagnosis is usually clinical


• however, in selected cases nerve conduction studies may be used

Management

• Avoid aggravating activity


• Physiotherapy
• Steroid injections
• Surgery in resistant cases

**The answer here is cubital tunnel syndrome which occurs due to compression of the ulnar
nerve as it passes through the cubital tunnel. Initial symptoms include tingling and
numbness of the 4th and 5th finger. Often pain is worse on leaning on the affected elbow
and there is a history of osteoarthritis or prior trauma to the area.

Medial epicondylitis would not cause distal symptoms in the hand but pain of the medial
elbow.

Radial tunnel syndrome causes aching and paraesthesia of the hand with forearm pain distal
to the lateral epicondyle

Carpal tunnel syndrome is caused by compression of the median nerve and presents with
pain and tingling of the thumb, index, middle and medial half of the ring finger

De Quervain's tenosynovitis typically causes symptoms affecting the base of the thumb.

4. Carpal tunnel syndrome

Carpal tunnel syndrome is caused by compression of median nerve in the carpal


tunnel.

History

• pain/pins and needles in thumb, index, middle finger


• unusually the symptoms may 'ascend' proximally
• patient shakes his hand to obtain relief, classically at night

Examination

• weakness of thumb abduction (abductor pollicis brevis)


• wasting of thenar eminence (NOT hypothenar)
• Tinel's sign: tapping causes paraesthesia
• Phalen's sign: flexion of wrist causes symptoms

Causes

• idiopathic
• pregnancy
• oedema e.g. heart failure
• lunate fracture
• rheumatoid arthritis
• hypothyroidism

Electrophysiology

• motor + sensory: prolongation of the action potential

Treatment

• corticosteroid injection
• wrist splints at night
• surgical decompression (flexor retinaculum division)

**This patient has symptoms which are in keeping with a median nerve palsy, and
pregnancy is a known risk factor for carpal tunnel syndrome. The symptoms of a
median nerve palsy in this context can be reproduced by tapping the area of the
flexor retinaculum (Tinel's sign), or by holding the wrist in flexion (Phalen's sign) or
extension (reverse Phalen's), both of which increase the pressure within the carpal
tunnel. Froment's sign is associated with ulnar nerve palsies and Westphal's sign is
the absence of a patellar jerk.

5. Discitis

Discitis is an infection in the intervertebral disc space. It can lead to serious


complications such as sepsis or an epidural abscess.
Signs of systemic sepsis with changing lower limb neurology = possible epidural
abscess
Features
• Back pain
• General features
o pyrexia,
o rigors
o sepsis
• Neurological features
o e.g. changing lower limb neurology
o if epidural abscess develops
Causes
• Bacterial
o Staphylococcus aureus is the most common cause of discitis
• Viral
• TB
• Aseptic

Diagnosis

• Imaging: MRI has the highest sensitivity


• CT guided biopsy may be required to guide antimicrobial treatment

Treatment

• The standard therapy requires six to eight weeks of intravenous antibiotic


therapy
• Choice of antibiotic is dependent on a variety of factors. The most important
factor is to identify the organism with a positive culture (e.g. blood culture, or CT
guided biopsy)

Complications

• sepsis
• epidural abscess
Further investigation:

• Assess the patient for endocarditis e.g. with transthoracic echo or


transesophageal echo. Discitis is usually due to haematogenous seeding of the
vertebrae implying that the patient has had a bacteraemia and seeding could
have occurred elsewhere

**The patient here is clearly showing signs of sepsis with an evolving lower limb
neurological deficit. The most likely explanation would be an epidural abscess
causing compression of the cauda equina. Apart from the clinical signs he also has a
history of diabetes which is a known risk factor for developing an epidural abscess.

A diabetic foot can present with a neurological deficit and sepsis but the neurological
distribution is peripheral and distal to proximal. A left-sided diabetic gangrene would
not cause right-sided neurological symptoms such as sensory loss and paraesthesia,
especially not in the proximal limb.

Discitis with a pathological fracture can also cause a similar presentation of sepsis
with neurological deficit but the key factor is the absence of pain. Fractures,
especially pathological fractures with instability, are painful and the lack of back pain
makes this far less likely than an epidural abscess.

Intracranial pathology is unlikely given the focal neurological deficit confined to the
lower limbs. Both an intracranial abscess and meningitis are likely to cause systemic
neurological symptoms such as meningism, depressed consciousness and weakness
affecting upper and lower limbs. As these features are all absent it makes an
intracranial pathology very unlikely.

**A normally fit and well 46-year-old woman is admitted to hospital with T11/12 discitis
complicated by a T4 to L1 epidural abscess and a left psoas abscess. Surgery to drain them is
carried out and intra-operative samples are cultured in the laboratory and grow
Staphylococcus aureus. Additionally blood cultures on admission grow Staphylococcus aureus
after 72 hours.

Which of the following investigations is most appropriate in the first instance to look for a
source of the infection given the pathogenic organism found?
Answer: Ecocardiogram

In discitis due to Staphylococcus an echo is needed to look for endocarditis

The bacteria isolated in this case, Staphylococcus aureus is an aggressive pathogen and can
disseminate around the body widely when it enters the blood stream. A primary psoas abscess
in a fit and well patient is highly unlikely and a haematogenous source of spread should be
looked for in all deep abscesses caused by Staphylococcus aureus. The most common cause
is septic emboli from endocarditis and systemic bacteraemia with Staphylococcus aureus has
a high rate of development of endocarditis due to the bacterial ability to attach to structures
and form biofilms. Therefore an echocardiogram must be performed to look for evidence of
valvular injury or vegetations in all cases of Staphylococcus aureus-positive cultures.

CT imaging is helpful in looking for other abscesses and other foci of infection but is unlikely
to demonstrate a primary source of haematogenous spread. CT imaging would be much more
helpful in looking for sources of Gram-negative organisms as these usually originate from the
gastrointestinal or urinary tracts within the abdomen.

Although Staphylococcus aureus sepsis is associated with immunosuppression and HIV


testing is definitely indicated, this would not identify a primary source of the bacteraemia.

Staphylococcus aureus rarely causes urinary tract infections and a midstream urine is a poor
test for a primary source in this case. It would be more helpful in Gram-negative bacteraemia.

Duke’s Criteria for Endocarditis


Given the positive blood culture for a ‘typical’ organism, this patient would score one major
criterion at present. Any endocardial involvement demonstrated on an echocardiogram
(valvular lesions) would be a second and give a firm diagnosis of bacterial endocarditis. The
treatment of complicated discitis usually lasts for 6-12 weeks with only the first two needed
as intravenous treatment. If endocarditis is diagnosed this will also change the length of time
intravenous therapy is offered and may also alter the total length of time treatment is required
although practice does vary from unit to unit.

6. Adhesive capsulitis

Adhesive capsulitis (frozen shoulder) is a common cause of shoulder pain. It is most


common in middle-aged females. The aetiology of frozen shoulder is not fully
understood.

Associations
• diabetes mellitus: up to 20% of diabetics may have an episode of frozen
shoulder
Features typically develop over days
• external rotation is affected more than internal rotation or abduction
• both active and passive movement are affected
• patients typically have a painful freezing phase, an adhesive phase and a
recovery phase
• bilateral in up to 20% of patients
• the episode typically lasts between 6 months and 2 years

The diagnosis is usually clinical although imaging may be required for atypical or
persistent symptoms.

Management

• no single intervention has been shown to improve outcome in the long-term


• treatment options include NSAIDs, physiotherapy, oral corticosteroids and intra-
articular corticosteroids

**Diabetic amyotrophy affects the lower limbs

***
Adhesive capsulitis presents as a painful stiff shoulder with restriction of active and passive
range of motion in abduction, internal and external rotation. However external rotation often
shows the most marked restriction and is the first movement to show impairment. The stem
describes difficulty dressing and doing up her bra as well as weakness of external rotation
suggesting a globally impaired range of motion. Patients often report difficulty sleeping on
the affected side. Other indications that the answer is adhesive capsulitis, include coexisting
diabetes, female gender and symptoms in the non-dominant hand, all of which are common
findings in this condition

Acromioclavicular degeneration is often associated with popping, swelling, clicking or


grindings and a positive scarf test not reported in the stem

Subacromial impingement patients often complain of pain on overhead activities and


demonstrate a painful arc of abduction on examination - worse between 90 and 120
degrees. There may also be popping, snapping or grinding.

Rotator cuff tears can occur either due to specific trauma or chronic impingement. Patients
will normally describe weakness as well as pain and there may be muscle wasting and
tenderness on palpation. There may be a painful arc of movement and weakness of the
affected muscle.

Patients with calcific tendinopathy would normally have tenderness on palpation of the
affected area and be reluctant to move the arm. There may be overlap with symptoms of
impingement syndrome making this a less likely answer.

7. Osteoporosis: assessing risk

We worry about osteoporosis because of the increased risk of fragility fractures. So


how do we assess which patients are at risk and need further investigation?

NICE produced guidelines in 2012: Osteoporosis: assessing the risk of fragility


fracture. The following is based on those guidelines.

They advise that all women aged >= 65 years and all men aged >= 75 years should
be assessed. Younger patients should be assessed in the presence of risk factors,
such as:
• previous fragility fracture
• current use or frequent recent use of oral or systemic glucocorticoid
• history of falls
• family history of hip fracture
• other causes of secondary osteoporosis
• low body mass index (BMI) (less than 18.5 kg/m²)
• smoking
• alcohol intake of more than 14 units per week for women and more than 14
units per week for men.

Methods of risk assessment

NICE recommend using a clinical prediction tool such as FRAX or QFracture to assess
a patients 10 year risk of developing a fracture. This is analogous to the
cardiovascular risk tools such as QRISK.

FRAX
• estimates the 10-year risk of fragility fracture
• valid for patients aged 40-90 years
• based on international data so use not limited to UK patients
• assesses the following factors: age, sex, weight, height, previous fracture,
parental fracture, current smoking, glucocorticoids, rheumatoid arthritis,
secondary osteoporosis, alcohol intake
• bone mineral density (BMD) is optional, but clearly improves the accuracy of the
results. NICE recommend arranging a DEXA scan if FRAX (without BMD) shows
an intermediate result
QFracture
• estimates the 10-year risk of fragility fracture
• developed in 2009 based on UK primary care dataset
• can be used for patients aged 30-99 years (this is stated on the QFracture
website, but other sources give a figure of 30-85 years)
• includes a larger group of risk factors e.g. cardiovascular disease, history of falls,
chronic liver disease, rheumatoid arthritis, type 2 diabetes and tricyclic
antidepressants

There are some situations where NICE recommend arranging BMD assessment (i.e. a
DEXA scan) rather than using one of the clinical prediction tools:

• before starting treatments that may have a rapid adverse effect on bone density
(for example, sex hormone deprivation for treatment for breast or prostate
cancer).
• in people aged under 40 years who have a major risk factor, such as history of
multiple fragility fracture, major osteoporotic fracture, or current or recent use of
high-dose oral or high-dose systemic glucocorticoids (more than 7.5 mg
prednisolone or equivalent per day for 3 months or longer).
Interpreting the results of FRAX

Once we've decided that we need to do a risk assessment using FRAX and have
entered all the data we are left with results to interpret.

If the FRAX assessment was done without a bone mineral density (BMD)
measurement the results (10-year risk of a fragility fracture) will be given and
categorised automatically into one of the following:
• low risk: reassure and give lifestyle advice
• intermediate risk: offer BMD test
• high risk: offer bone protection treatment
Therefore, with intermediate risk results FRAX will recommend that you arrange a
BMD test to enable you to more accurately determine whether the patient needs
treatment

If the FRAX assessment was done with a bone mineral density (BMD)
measurement the results (10-year risk of a fragility fracture) will be given and
categorised automatically into one of the following:
• reassure
• consider treatment
• strongly recommend treatment
If you use QFracture instead patients are not automatically categorised into low,
intermediate or high risk. Instead the 'raw data' relating to the 10-year risk of any
sustaining an osteoporotic fracture. This data then needs to be interpreted alongside
either local or national guidelines, taking into account certain factors such as the
patient's age.

WHEN SHOULD WE REASSESS A PATIENT’S RISK?


NICE recommend that we recalculate a patient's risk (i.e. repeat the FRAX/QFracture):

• if the original calculated risk was in the region of the intervention threshold for a
proposed treatment and only after a minimum of 2 years, or
• when there has been a change in the person's risk factors

8. Patellofemoral pain syndrome

Patellofemoral pain syndrome is now the preferred term for chondromalacia patellae
Softening of the cartilage of the patella.
Common in teenage girls.
Characteristically anterior knee pain on walking up and down stairs and rising from
prolonged sitting.
Usually responds to physiotherapy.
**This patient is a runner and as such is at risk of chondromalacia patellae, where the
posterior cartilaginous aspect of the patella becomes softened and worn with overuse, e.g.
from athletics, causing pain anteriorly and medially over the knee joint.
Pre-patellar bursitis can also cause pain in the patellar region, although tends to present in
older patients, with a history of frequent kneeling. In the absence of a trauma, a patellar
fracture is unlikely. Juvenile arthritis would generally present earlier and with more
widespread symptoms. Iliotibial band syndrome can affect runners, but pain tends to
present more laterally.

9. De Quervain's tenosynovitis

De Quervain's tenosynovitis is a common condition in which the sheath containing


the extensor pollicis brevis and abductor pollicis longus tendons is inflamed. It
typically affects females aged 30 - 50 years old.

Features

• pain on the radial side of the wrist


• tenderness over the radial styloid process
• abduction of the thumb against resistance is painful
• Finkelstein's test: the examiner pulls the thumb of the patient in ulnar deviation
and longitudinal traction. In a patient with tenosynovitis this action causes pain
over the radial styloid process and along the length of extensor pollisis brevis
and abductor pollicis longus

Management

• analgesia
• steroid injection
• immobilisation with a thumb splint (spica) may be effective
• surgical treatment is sometimes required

**Finkelstein's test: the examiner pulls the thumb of the patient in ulnar deviation
and longitudinal traction. In a patient with tenosynovitis this action causes pain over
the radial styloid process and along the length of extensor pollisis brevis and
abductor pollicis longus
***De Quervain's tenosynovitis is the correct answer. This condition typically affects
women between 30-50 and is particularly associated with picking up young children.
Clinical examination findings of tenderness near the radial styloid and a positive Finkelstein's
test are suggestive.
**This patient likely has De Quervain's tenosynovitis therefore Finkelstein's test would be
appropriate. It can be falsely positive in osteoarthritis however this is unlikely in a young
patient.

Both Phalen's (pushing the dorsal sides of the hands together) and Tinel's (percussion over
the median nerve) test are seen in carpel tunnel. Carpel tunnel is common in office workers
causing pain, numbness and tingling. This patient is more likely to have De Quervain's
tenosynovitis because the pain is associated with movement of the thumb with minimal
involvement of the fingers.

Mill's test identifies lateral epicondylitis or tennis elbow. It is performed by flexing the
patients wrist whilst the arm is pronated and elbow extended. If positive, the test
reproduces pain over the lateral epicondyle.

Forced extension of the wrist with a supinated and extended forearm is the test for medial
epicondylitis or golfers elbow. There is no eponymous name. If positive, the test reproduces
pain over the medial epicondyle.

10. Osteoarthritis of the hip

Osteoarthritis (OA) of the hip is the second most common presentation of OA after
the knee. It accounts for significant morbidity and total hip replacement is now one
of the most common operations performed in the developed world.

Risk factors
• increasing age
• female gender (twice as common)
• obesity
• developmental dysplasia of the hip

Features
• chronic history of groin ache following exercise and relieved by rest
• red flag features suggesting an alternative cause include rest pain, night
pain and morning stiffness > 2 hours
• the Oxford Hip Score is widely used to assess severity

Investigations
• NICE recommends that if the features are typical then a clinical diagnosis can be
made
• otherwise plain x-rays are the first-line investigation

Management

• oral analgesia
• intra-articular injections: provide short-term benefit
• total hip replacement remains the definitive treatment

Complications of total hip replacement

• venous thromboembolism
• intraoperative fracture
• nerve injury

Reasons for revision of total hip replacement

• aseptic loosening (most common reason)


• pain
• dislocation
• infection

**Morning stiffness lasting > 2 hours may be an indication of inflammatory arthritis.


This would warrant further investigations

11. Dupuytren's contracture

Dupuytren's contracture has a prevalence of about 5%. It is more common in older


male patients and around 60-70% have a positive family history.

Specific causes include:


• manual labour
• phenytoin treatment
• alcoholic liver disease
• diabetes mellitus
• trauma to the hand

Features
• the ring finger and little finger are the fingers most commonly affected
Management

• consider surgical treatment of Dupuytren's contracture when the


metacarpophalangeal joints cannot be straightened and thus the hand cannot
be placed flat on the table

12. Elbow pain

The table below details some of the characteristic features of conditions causing
elbow pain:
Condition Notes
• pain and tenderness localised to the lateral epicondyle
• pain worse on resisted wrist extension with the elbow
extended or supination of the forearm with the elbow
Lateral epicondylitis
extended
(tennis elbow)
• episodes typically last between 6 months and 2 years.
Patients tend to have acute pain for 6-12 weeks

• pain and tenderness localised to the medial epicondyle


• pain is aggravated by wrist flexion and pronation
Medial epicondylitis • symptoms may be accompanied by numbness /
(golfer's elbow) tingling in the 4th and 5th finger due to ulnar nerve
involvement

Most commonly due to compression of the posterior


interosseous branch of the radial nerve. It is thought to be a
result of overuse.

• symptoms are similar to lateral epicondylitis making it


Radial tunnel
difficult to diagnose
syndrome
• however, the pain tends to be around 4-5 cm distal to
the lateral epicondyle
• symptoms may be worsened by extending the elbow
and pronating the forearm

Due to the compression of the ulnar nerve.

• initially intermittent tingling in the 4th and 5th finger


Cubital tunnel • may be worse when the elbow is resting on a firm
syndrome surface or flexed for extended periods
• later numbness in the 4th and 5th finger with
associated weakness

Swelling over the posterior aspect of the elbow. There may be


Olecranon bursitis associated pain, warmth and erythema. It typically affects
middle-aged male patients.

13. Hip problems in children

The table below provides a brief summary of the potential causes of hip problems in
children
Condition Notes
Often picked up on newborn examination
Development
Barlow's test, Ortolani's test are positive
dysplasia of the
Unequal skin folds/leg length
hip

Transient Typical age group = 2-10 years


synovitis Acute hip pain associated with viral infection
(irritable hip) Commonest cause of hip pain in children
Perthes disease is a degenerative condition affecting the hip joints
of children, typically between the ages of 4-8 years. It is due to
avascular necrosis of the femoral head

Perthes disease is 5 times more common in boys. Around 10% of


cases are bilateral
Perthes disease
• hip pain: develops progressively over a few weeks
• limp
• stiffness and reduced range of hip movement
• x-ray: early changes include widening of joint space, later
changes include decreased femoral head size/flattening

Typical age group = 10-15 years


More common in obese children and boys
Displacement of the femoral head epiphysis postero-inferiorly
Bilateral slip in 20% of cases
Slipped upper
May present acutely following trauma or more commonly with
femoral
chronic, persistent symptoms
epiphysis
• knee or distal thigh pain is common
• loss of internal rotation of the leg in flexion

Preferred to the older term juvenile chronic arthritis, describes


arthritis occurring in someone who is less than 16 years old that
lasts for more than three months. Pauciarticular JIA refers to cases
where 4 or less joints are affected. It accounts for around 60% of
Juvenile
cases of JIA
idiopathic
Features of pauciarticular JIA
arthritis (JIA)
• joint pain and swelling: usually medium sized joints e.g.
knees, ankles, elbows
• limp
Condition Notes
• ANA may be positive in JIA - associated with anterior
uveitis

Acute hip pain associated with systemic upset e.g. pyrexia.


Septic arthritis
Inability/severe limitation of affected joint

14. Upper limb fractures

Colles' fracture

• Fall onto extended outstretched hands


• Described as a dinner fork type deformity
• Classical Colles' fractures have the following 3 features:
• Features of the injury
o 1. Transverse fracture of the radius
o 2. 1 inch proximal to the radio-carpal joint
o 3. Dorsal displacement and angulation

Smith's fracture (reverse Colles' fracture)

• Volar angulation of distal radius fragment (Garden spade deformity)


• Caused by falling backwards onto the palm of an outstretched hand or falling
with wrists flexed

Bennett's fracture

• Intra-articular fracture at the base of the thumb metacarpal


• Impact on flexed metacarpal, caused by fist fights
• X-ray: triangular fragment at the base of metacarpal

Monteggia's fracture

• Dislocation of the proximal radioulnar joint in association with an ulna fracture


• Fall on outstretched hand with forced pronation
• Needs prompt diagnosis to avoid disability

Galeazzi fracture

• Radial shaft fracture with associated dislocation of the distal radioulnar joint
• Occur after a fall on the hand with a rotational force superimposed on it.
• On examination, there is bruising, swelling and tenderness over the lower end of
the forearm.
• X Rays reveal the displaced fracture of the radius and a prominent ulnar head
due to dislocation of the inferior radio-ulnar joint.

Barton's fracture

• Distal radius fracture (Colles'/Smith's) with associated radiocarpal dislocation


• Fall onto extended and pronated wrist

Scaphoid fractures

• Scaphoid fractures are the commonest carpal fractures.


• Surface of scaphoid is covered by articular cartilage with small area available for
blood vessels (fracture risks blood supply)
• Forms floor of anatomical snuffbox
• Risk of fracture associated with fall onto outstretched hand (tubercle, waist, or
proximal 1/3)
• The main physical signs are swelling and tenderness in the anatomical snuff box,
and pain on wrist movements and on longitudinal compression of the thumb.
• Ulnar deviation AP needed for visualization of scaphoid
• Immobilization of scaphoid fractures difficult

Radial head fracture

• Fracture of the radial head is common in young adults.


• It is usually caused by a fall on the outstretched hand.
• On examination, there is marked local tenderness over the head of the radius,
impaired movements at the elbow, and a sharp pain at the lateral side of the
elbow at the extremes of rotation (pronation and supination).

***
Metacarpal fractures may require surgical fixation and, as such, should be discussed
with plastic surgery
This patient has sustained a 'Boxer's fracture'. Metacarpal fracture may require surgical
repair, and so should normally be discussed with plastic or hand surgery. A volar slab is an
appropriate technique to immobilise the hand in the meantime.

15. Stress fractures

Repetitive activity and loading of normal bone may result in small hairline fractures.
Whilst these may be painful they are seldom displaced. Surrounding soft tissue injury
is unusual. They may present late following the injury, in which case callus formation
may be identified on radiographs. Such cases may not require formal immobilisation,
injuries associated with severe pain and presenting at an earlier stage may benefit
from immobilisation tailored to the site of injury.

This describes the female athlete triad of excessive exercise associated with reduced bone
mineral density, menstrual irregularities/ amenorrhoea and eating disorder or inadequate
calorie intake for expenditure. This combination increases the risk of stress fractures,
particularly when starting a new sport or increasing training for example in preparation for a
big event. When seeing women with a history of stress fracture it is important to discuss
their eating habits and menstrual cycle as this information may not be offered.

16. Meralgia paraesthetica

Meralgia paraesthetica comes from the Greek words meros for thigh and algos for
pain and is often described as a syndrome of paraesthesia or anaesthesia in the
distribution of the lateral femoral cutaneous nerve (LFCN). It is an entrapment
mononeuropathy of the LFCN, but can also be iatrogenic after a surgical procedure,
or result from a neuroma. Although uncommon, meralgia paraesthetica is not rare
and is hence probably underdiagnosed.

Anatomy

• The LFCN is primarily a sensory nerve, carrying no motor fibres.


• It most commonly originates from the L2/3 segments.
• After passing behind the psoas muscle, it runs beneath the iliac fascia as it
crosses the surface of the iliac muscle and eventually exits through or under the
lateral aspect of the inguinal ligament.
• As the nerve curves medially and inferiorly around the anterior superior iliac
spine (ASIS), it may be subject to repetitive trauma or pressure.
• Compression of this nerve anywhere along its course can lead to the
development of meralgia paraesthetica.

Epidemiology

• The majority of cases occur in people aged between 30 and 40.


• In some, both legs may be affected.
• It is more common in men than women.
• Occurs more commonly in those with diabetes than in the general population.

Risk factors 3

• Obesity
• Pregnancy
• Tense ascites
• Trauma
• Iatrogenic, such as pelvic osteotomy, spinal surgeries, laparoscopic hernia repair
and bariatric surgery. In some cases, may result from abduction splints used in
the management of Perthe's disease.
• Various sports have been implicated, including gymnastics, football,
bodybuilding and strenuous exercise.
• Some cases are idiopathic.

Patients typically present with the following symptoms in the upper lateral aspect of
the thigh:

• Burning, tingling, coldness, or shooting pain


• Numbness
• Deep muscle ache
• Symptoms are usually aggravated by standing, and relieved by sitting
• They can be mild and resolve spontaneously or may severely restrict the patient
for many years.

Signs:

• Symptoms may be reproduced by deep palpation just below the ASIS (pelvic
compression) and also by extension of the hip.
• There is altered sensation over the upper lateral aspect of the thigh.
• There is no motor weakness.

Investigations:

• The pelvic compression test is highly sensitive, and often, meralgia paraesthetica
can be diagnosed based on this test alone
• Injection of the nerve with local anaesthetic will abolish the pain. Using
ultrasound is effective both for diagnosis and guiding injection therapy in
meralgia paraesthetica
• Nerve conduction studies may be useful.

17. Hip fracture

The hip is a common site of fracture, especially in osteoporotic, elderly females. The
blood supply to the femoral head runs up the neck and thus avascular necrosis is a
risk in displaced fractures.

Features
• pain
• the classic signs are a shortened and externally rotated leg
• patients with non-displaced or incomplete neck of femur fractures may be able
to weight bear

Classification

Location

• intracapsular (subcapital): from the edge of the femoral head to the insertion of
the capsule of the hip joint
• extracapsular: these can either be trochanteric or subtrochanteric (the lesser
trochanter is the dividing line)

The Garden system is one classification system in common use.

• Type I: Stable fracture with impaction in valgus


• Type II: Complete fracture but undisplaced
• Type III: Displaced fracture, usually rotated and angulated, but still has boney
contact
• Type IV: Complete boney disruption

Blood supply disruption is most common following Types III and IV.

Intracapsular hip fracture

Undisplaced Fracture:

• internal fixation, or hemiarthroplasty if unfit.

Displaced Fracture:

• NICE recommend replacement arthroplasty (total hip replacement or


hemiarthroplasty) to all patients with a displaced intracapsular hip fracture
• total hip replacement is favoured to hemiarthroplasty if patients:
o were able to walk independently out of doors with no more than the use
of a stick and
o are not cognitively impaired and
o are medically fit for anaesthesia and the procedure.

Extracapsular hip fracture

Management
• stable intertrochanteric fractures: dynamic hip screw
• if reverse oblique, transverse or subtrochanteric fractures: intramedullary device

***
Hemiarthroplasty would be the management for an intracapsular neck of femur fracture
in unfit patients.

An intramedullary nail would be the surgical management for patients presenting with a
subtrochanteric femoral fracture.

Total hip replacement would be the management for fit patients presenting with an
intracapsular neck of femur fracture.

Dynamic hip screws are the preferred surgical management for intertrochanteric
(extracapsular) proximal femoral fracture

18. Ankle injury: Ottawa rules

The Ottawa Rules with for ankle x-rays have a sensitivity approaching 100%

An ankle x-ray is required only if there is any pain in the malleolar zone and any one
of the following findings:

• bony tenderness at the lateral malleolar zone (from the tip of the lateral
malleolus to include the lower 6 cm of posterior border of the fibular)
• bony tenderness at the medial malleolar zone (from the tip of the medial
malleolus to the lower 6 cm of the posterior border of the tibia)
• inability to walk four weight bearing steps immediately after the injury and in
the emergency department

There are also Ottawa rules available for both foot and knee injuries

The Ottawa ankle rules determine the need to perform an ankle x-ray for patients presenting
with foot or ankle pain. If an ankle x-ray is not indicated by the Ottawa ankle rules the
probability of a fracture is very low. The rules state an x-ray is only required if there is an
inability to weight bear both immediately after the injury and on assessment, or there is
tenderness along the distal 6cm of the posterior edge of the tibia or fibula or distal tip of
either malleoli.

In this case, the patient has tenderness of the anterior aspect of the fibula due to the very
common sprain of the anterior talofibular ligament which inserts in the anterior part of the
fibula.

***
The Ottawa ankle rules clearly state the following indications for imaging post trauma. First
line imaging would be x-ray.

1. Point tenderness over the distal 6 cm of the lateral malleolus


2. Point tenderness over the distal 6 cm of the medial malleolus
3. Inability to weight bear by at least four steps immediately post trauma and in the
emergency department

Therefore this patient does not require an x-ray

19. Olecranon bursitis

Olecranon bursitis describes inflammation of the olecranon bursa, the fluid-filled sac
overlying the olecranon process at the proximal end of the ulna. This bursa exists to
reduce friction between the posterior aspect of the elbow joint and the overlying soft
tissues. Inflammation may result from trauma, infection, or systemic conditions such
as rheumatoid arthritis or gout. Olecranon bursitis is also known as 'student's elbow'
because the repetitive mild trauma of leaning on a desk using the elbows is a
common cause. It is categorised as septic or non-septic depending on whether an
infection is present.

Epidemiology

• More common in men


• Typically presents between age 30 and 60

Causes

• Repetitive trauma (29%) - writers and students leaning on elbows, plumbers,


miners
• Direct trauma (17%)
• Infection (33%) - 50% of cases occur in immunosuppressed patients (alcohol
abuse, diabetes, taking steroids, renal failure, malignancy). 90% of cases due to
Staphylococcus aureus.
• Gout (7%)
• Rheumatoid arthritis (5%)
• Idiopathic (5%)

Patients with non-septic olecranon bursitis typically present with a subacute onset of:

• swelling over the olecranon process (100%)


For many patients, this will be the only symptom. Some patients with non-septic
olecranon bursitis also complain of:

• tenderness over the bursa (45%)


• erythema over the bursa (25%)

Patients with septic bursitis are more likely to have pain and fever:
-tenderness over the bursa (92-100%)
-fever (40%)

Signs:

• Swelling over the posterior aspect of the elbow, usually fluctuant and well-
circumscribed, appearing over hours to days4
• Tenderness on palpation of the swollen area
• Redness and warmth of the overlying skin
• Fever
• Skin abrasion overlying the bursa
• Effusions in other joints if associated with rheumatoid arthritis
• Tophi if associated with gout

Movement at the elbow joint should be painless until the swollen bursa is
compressed in full flexion.

Investigations:
Not always needed if a clinical diagnosis can be made and there is no concern about
septic arthritis, e.g. a well patient without pain, fever or erythema of the bursa.

• Aspiration of bursal fluid for microscopy (Gram stain and crystals) and culture is
essential if septic bursitis is considered. Purulent fluid suggests infection
whereas straw-coloured bursal fluid favours a non-infective cause.

20. Iliotibial band syndrome

Iliotibial band syndrome is a common cause of lateral knee pain in runners, occurring
in around 1 in 10 people who run regularly.

tenderness 2-3cm above the lateral joint line

Management

• activity modification and iliotibial band stretches


• if not improving then physiotherapy referral
***
Iliotibial band syndrome is a common cause of knee pain, particularly in runners

This is a classic history and examination for iliotibial band syndrome, a common cause of
lateral knee pain in keen runners.

Osgood-Schlatter disease, also known as tibial apophysitis, would typically cause pain and
swelling over the tibial tubercle.

Osteochondritis dissecans would typically cause locking and swelling of the joint as well as
tenderness.

Patellar tendonitis would also cause pain after exercise, but this would normally be located at
the lower aspect of the patella.

Patellofemoral pain syndrome, previously known as chondromalacia patellae, would


classically cause anterior knee pain worsened by going up or down stairs.

21. Achilles tendon disorders

Achilles tendon disorders are the most common cause of posterior heel pain.
Possible presentations include tendinopathy (tendinitis), partial tear and complete
rupture of the Achilles tendon.

Risk factors

• quinolone use (e.g. ciprofloxacin) is associated with tendon disorders


• hypercholesterolaemia (predisposes to tendon xanthomata)

Achilles tendinopathy (tendinitis)

• gradual onset of posterior heel pain that is worse following activity


• morning pain and stiffness are common

The management is typically supportive

• simple analgesia
• reduction in precipitating activities
• calf muscle eccentric exercises: this may be self-directed or under the guidance
of physiotherapy

Achilles tendon rupture


Achilles tendon rupture should be suspected if the person describes the following
whilst playing a sport or running; an audible 'pop' in the ankle, sudden onset
significant pain in the calf or ankle or the inability to walk or continue the sport.

An examination should be conducted using Simmond's triad, to help exclude Achilles


tendon rupture. This can be performed by asking the patient to lie prone with their
feet over the edge of the bed. The examiner should look for an abnormal angle of
declination; Achilles tendon rupture may lead to greater dorsiflexion of the injured
foot compared to the uninjured limb. They should also feel for a gap in the tendon
and gently squeeze the calf muscles if there is an acute rupture of the Achilles
tendon the injured foot will stay in the neutral position when the calf is squeezed.

Ultrasound is the initial imaging modality of choice for suspected Achilles tendon
rupture

An acute referral should be made to an orthopaedic specialist following a suspected


rupture.

***
Achilles tendonitis. The management of which is: rest, NSAIDs, and physio if symptoms
persist beyond 7 days.

Although ibuprofen would help it should be avoided in patients with asthma.

Reiterating the need to rest would be ignoring your patients request to step up his
treatment and need for further management.

A referral to orthopaedics would be inappropriate - if not settling a rheumatology referral


may be considered.

Although steroids may benefit the patient the evidence is variable and injection directly into
the tendon increases the risk of tendon rupture.

22. Talipes equinovarus/ Club foot

Talipes equinovarus, or club foot, describes an inverted (inward turning) and plantar
flexed foot. It is usually diagnosed on the newborn exam.

Talipes equinovarus is twice as common in males than females and has an incidence
of 1 per 1,000 births. Around 50% of cases are bilateral.

Most commonly idiopathic.


Associations include:
• spina bifida
• cerebral palsy
• Edward's syndrome (trisomy 18)
• oligohydramnios
• arthrogryposis

The diagnosis is clinical (the deformity is not passively correctable) and imaging is
not normally needed.

Management*

• in recent years there has been a move away from surgical intervention to more
conservative methods such as the Ponseti method
• the Ponseti method consists of manipulation and progressive casting which
starts soon after birth. The deformity is usually corrected after 6-10 weeks. An
Achilles tenotomy is required in around 85% of cases but this can usually be
done under local anaesthetic
• night-time braces should be applied until the child is aged 4 years. The relapse
rate is 15%

23. Osteomyelitis

Osteomyelitis describes an infection of the bone. It may be subclassified into:

• haematogenous osteomyelitis
o results from bacteraemia
o is usually monomicrobial
o most common form in children
o vertebral osteomyelitis is the most common form of haematogenous
osteomyelitis in adults
o risk factors include: sickle cell anaemia, intravenous drug user,
immunosuppression due to either medication or HIV, infective
endocarditis
• non-haematogenous osteomyelitis:
o results from the contiguous spread of infection from adjacent soft tissues
to the bone or from direct injury/trauma to bone
o is often polymicrobial
o most common form in adults
o risk factors include: diabetic foot ulcers/pressure sores, diabetes mellitus,
peripheral arterial disease

Microbiology
• Staph. aureus is the most common cause except in patients with sickle-cell
anaemia where Salmonella species predominate

Investigations

• MRI is the imaging modality of choice, with a sensitivity of 90-100%

Management

• flucloxacillin for 6 weeks


• clindamycin if penicillin-allergic

***
Blood and bone infections caused by non-typhi salmonella (NTS) are typically associated
with malaria and homozygous sickle cell disease, especially in children. The reason for this
perceived susceptibility is not fully understood - but it may be in part due to the haemolysis
and subsequent iron availability to the bacteria, which is 'siderophilic' in nature.

E.coli and P. aeruginosa are not typically linked to sickle cell disease and Staphylococcus
aureus is a gram positive coccus.

The haemoglobin level is normal for a child homozygous for sickle cell disease. Therefore
'aplastic anaemia' should not be considered and parvovirus can be ruled out. Parvovirus
does not cause osteomyeltitis.

24. Scaphoid bone

The scaphoid has a concave articular surface for the head of the capitate and at the
edge of this is a crescentic surface for the corresponding area on the lunate.

Proximally, it has a wide convex articular surface with the radius. It has a distally sited
tubercle that can be palpated. The remaining articular surface is to the lateral side of
the tubercle. It faces laterally and is associated with the trapezium and trapezoid
bones.

The narrow strip between the radial and trapezial surfaces and the tubercle gives rise
to the radial collateral carpal ligament. The tubercle receives part of the flexor
retinaculum. This area is the only part of the scaphoid that is available for the entry
of blood vessels. It is commonly fractured and avascular necrosis may result.

Scaphoid bone
***The mechanism and examination are highly suspicious for a scaphoid fracture.
Conservative management with splinting, analgesia and physiotherapy are all inappropriate
due to the risk of avascular necrosis due to the blood supply to the scaphoid bone.

An outpatient x-ray is a possibility but even at this stage, some scaphoid fractures do not
show on x-ray and require CT scan or MRI. This patient therefore needs to be seen in an
orthopaedic clinic - you may be able to refer directly or this may need to be via your local
emergency department.

25. Osteoarthritis: joint replacement

Joint replacement (arthroplasty) remains the most effective treatment for


osteoarthritis patients who experience significant pain.

Selection criteria

• around 25% of patients are now younger than 60-years-old


• whilst obesity is often thought to be a barrier to joint replacement there is only
a slight increase in short-term complications. There is no difference in long-term
joint replacement survival

Surgical techniques

• for hips the most common type of operation is a cemented hip replacement. A
metal femoral component is cemented into the femoral shaft. This is
accompanied by a cemented acetabular polyethylene cup
• uncemented hip replacements are becoming increasingly popular, particularly in
younger more active patients. They are more expensive than conventional
cemented hip replacements
• hip resurfacing is also sometimes used where a metal cap is attached over the
femoral head. This is often used in younger patients and has the advantage that
the femoral neck is preserved which may be useful if conventional arthroplasty is
needed later in life

Post-operative recovery

• patients receive both physiotherapy and a course of home-exercises


• walking sticks or crutches are usually used for up to 6 weeks after hip or knee
replacement surgery

Patients who have had a hip replacement operation should receive basic advice
to minimise the risk of dislocation:

• avoiding flexing the hip > 90 degrees


• avoid low chairs
• do not cross your legs
• sleep on your back for the first 6 weeks

Complications

• wound and joint infection


• thromboembolism: NICE recommend patients receive low-molecular weight
heparin for 4 weeks following a hip replacement
• dislocation

26. Trigger finger

Trigger finger is a common condition associated with abnormal flexion of the digits.
It is thought to be caused by a disparity between the size of the tendon and pulleys
through which they pass. In simple terms the tendon becomes 'stuck' and cannot
pass smoothly through the pulley.

Associations* (idiopathic in the majority)

• more common in women than men


• rheumatoid arthritis
• diabetes mellitus

Features

• more common in the thumb, middle, or ring finger


• initially stiffness and snapping ('trigger') when extending a flexed digit
• a nodule may be felt at the base of the affected finger

Management

• steroid injection is successful in the majority of patients. A finger splint may be


applied afterwards
• surgery should be reserved for patients who have not responded to steroid
injections

*there is scanty evidence to support a link with repetitive use

27. Ankle sprain

The bony components of the ankle joint include the distal tibia and fibula and the superior
aspect of the talus. Their configuration is such that they form a mortise, with the body of the
talus acting as the tenon. This arrangement is secured by a number of ligamentous
structures:

• The syndesmosis binds the distal tibia and fibula together (another example of a
syndesmosis is the distal radio-ulnar joint). It is composed of the anterior inferior
tibiofibular ligament (AITFL), posterior inferior tibiofibular ligament (PITFL),
interosseous ligament (IOL) and the interosseous membrane.
• The distal fibular is secured to the to the talus by the anterior and posterior
talofibular ligaments (ATFL and PTFL) and to the calcaneus by the calcaneofibular
ligament. These ligaments are sometimes referred to collectively as the lateral
collateral ligaments.
• The distal tibia is secured to the talus by the deltoid ligament, in view of its triangular
shape.

A sprain is a stretching, partial or complete tear of a ligament. In the ankle, this can be
divided into high ankle sprains involving the syndesmosis and low ankle sprains involving the
lateral collateral ligaments.

Low ankle sprains

Presentation:

• most common (>90%) with injury to the ATFL the most common offender
• inversion injury most common mechanism
• pain, swelling, tenderness over affected ligaments and sometimes bruising
• patients usually able to weight bear unless severe
• they can be classified as follows
Grade Ligament disruption Bruising and swelling Pain on weight bearing
Grade I (mild) Stretch or micro tear minimal normal
Grade II (moderate) Partial tear moderate minimal
Grade III (severe) Complete tear Severe Severe

Investigation:

• Radiographs should be done according to the Ottawa ankle rules as 15% of sprains
are associated with a fracture.
• MRI if persistent pain and useful for evaluating perineal tendons.

Treatment:

• Non-operative with rest, ice, compression and elevation (the so-called RICE protocol).
• Occasionally a removable orthosis, cast and/or crutches may be required for short-
term symptom relief.
• If symptoms fail to settle or there is significant joint instability then an MRI and
surgical intervention may be contemplated, but this is rare.

High ankle sprains

Presentation:

• Injuries to the syndesmosis are rare (about 0.5%) and severe.


• The mechanism of injury is usually external rotation of the foot causing the talus to
push the fibula laterally.
• Patients frequently find weight-bearing painful in comparison to low ankle sprains.
• Pain when the tibia and fibula are squeezed together at the level of the mid-calf
(Hopkin’s squeeze test).

Investigations:

• Radiographs may show widening of the tibiofibular joint (diastasis) or ankle mortise.
• MRI if high suspicion of syndesmotic injury, but normal plain films.

Treatment:

• If no diastasis then non-weight-bearing orthosis or cast until pain subsides.


• If diastasis or failed non-operative management then operative fixation is usually
warranted.

Isolated injuries to the deltoid ligament are rare as they are frequently associated with a
fracture and one should always be on the lookout for Maisonneuve fracture of the proximal
fibula. Provided the ankle mortise is anatomically reduced then treatment can be as per a
low ankle sprain, if not then reduction and fixation may be warranted.

28. Upper limb anatomy

The information below contains selected facts which commonly appear in


examinations:

Typical mechanism of
Nerve Motor Sensory
injury & notes
Elbow flexion
Isolated injury rare -
Musculocutaneous (supplies biceps Lateral part of
usually injured as part of
nerve (C5-C7) brachii) and the forearm
brachial plexus injury
supination
Humeral neck
Inferior region of fracture/dislocation
Axillary nerve Shoulder abduction
the deltoid
(C5,C6) (deltoid muscle)
muscle Results in flattened
deltoid
Small area
Humeral midshaft
Extension (forearm, between the
Radial nerve (C5- fracture
wrist, fingers, dorsal aspect of
C8)
thumb) the 1st and 2nd
Palsy results in wrist drop
metacarpals
LOAF* muscles

Features depend on
the site of the lesion:

• wrist:
paralysis of
thenar Palmar aspect of
Median nerve (C6, Wrist lesion → carpal
muscles, lateral 3½
C8, T1) tunnel syndrome
opponens fingers
pollicis
• elbow: loss of
pronation of
forearm and
weak wrist
flexion

Intrinsic hand Medial 1½ Medial epicondyle


Ulnar nerve (C8, T1)
muscles except fingers fracture
Typical mechanism of
Nerve Motor Sensory
injury & notes
LOAF*
Damage may result in a
Wrist flexion 'claw hand'
Often during sport e.g.
following a blow to the
ribs. Also possible
Long thoracic nerve complication of
Serratus anterior
(C5-C7) mastectomy

Damage results in a
winged scapula

Diagram of the brachial plexus

Erb-Duchenne palsy ('waiter's tip')

• due to damage of the upper trunk of the brachial plexus (C5,C6)


• may be secondary to shoulder dystocia during birth
• the arm hangs by the side and is internally rotated, elbow extended
Klumpke injury

• due to damage of the lower trunk of the brachial plexus (C8, T1)
• as above, may be secondary to shoulder dystocia during birth. Also may be
caused by a sudden upward jerk of the hand
• associated with Horner's syndrome

*LOAF muscles

• Lateral two lumbricals


• Opponens pollis
• Abductor pollis brevis
• Flexor pollis brevis

29. Myxoid cyst

Myxoid cysts (also known as mucous cysts) are benign ganglion cysts usually found
on the distal, dorsal aspect of the finger. There is usually osteoarthritis in the
surrounding joint. They are more common in middle-aged women.

30. Forearm flexor muscles

Nerve
Muscle Origin Insertion Action
supply
Flexes and
abducts the
Common flexor origin Front of bases of
Flexor carpi carpus, part flexes
and surrounding second and third Median
radialis the elbow and
fascia metacarpals
part pronates
forearm
Palmaris Apex of palmar
Common flexor origin Median Wrist flexor
longus aponeurosis
Small humeral head
arises from the
common flexor origin Flexes and
Flexor carpi Pisiform and base of Ulnar
and adjacent fascia. adducts the
ulnaris the fifth metacarpal nerve
Ulnar head comes carpus
from medial border of
olecranon and
Nerve
Muscle Origin Insertion Action
supply
posterior border of
ulna
Via tendons in the
Long linear origin fibrous flexor sheath. Flexor of
from common flexor At the level of the metacarpophalan
Flexor
tendon, adjacent metacarpophalangea geal joint and
digitorum Median
fascia and septa and l joint each tendon proximal
superficialis
medial border of the split into two, these interphalangeal
coronoid process bands pass distally joint
to their insertions
Upper two thirds of Medial
the medial and Via tendons that lie part =
anterior surface of the deep to those of ulnar, Flexes the distal
Flexor
ulna, medial side of flexor digitorum lateral interphalangeal
digitorum
the olecranon, medial superficialis to insert part joints and the
profundus
half of the into the distal =anterior wrist
interosseous phalanx interosseo
membrane us nerve
OPHTHALMOLOGY
1.Corneal ulcer

Corneal ulcers are more common in contact lens users


Features
eye pain

• photophobia
• watering of eye
• focal fluorescein staining of the cornea

Fluorescein is an orange dye that is applied to the ocular surface. It fluoresces


green/yellow when observed under a blue light if it is taken up by the corneal
epithelium, for example in corneal ulceration. Dry eye can also cause fluorescence,
though usually has a diffuse, speckled appearance - 'superficial punctate staining'.

**Prescribing anaesthetic eye drops for patients with corneal ulcer is not advisable as
it may cause more harm- delays healing of the ulcer.

Oral analgesics should be prescribed for pain relief. Abuse or long-term use of
topical anaesthetic eye drops have toxic effects on the cornea and decreases corneal
re-epithelialisation. This will prolong the healing time and the eye becomes more
susceptible to further infections. It is for this reason most practitioners avoid
prescribing topical anaesthetics for regular unsupervised use.

2.Infective conjunctivitis

Conjunctivitis is the most common eye problem presenting to primary care. It is


characterised by sore, red eyes associated with a sticky discharge

Type Features
Purulent discharge
Bacterial conjunctivitis
Eyes may be 'stuck together' in the morning)
Serous discharge
Viral conjunctivitis Recent URTI
Preauricular lymph nodes

Management of infective conjunctivitis

• normally a self-limiting condition that usually settles without treatment within 1-


2 weeks
• topical antibiotic therapy is commonly offered to patients, e.g. Chloramphenicol.
Chloramphenicol drops are given 2-3 hourly initially where as chloramphenicol
ointment is given qds initially
• topical fusidic acid is an alternative and should be used for pregnant
women. Treatment is twice daily
• contact lens should not be worn during an episode of conjunctivitis
• advice should be given not to share towels
• school exclusion is not necessary

Adenoviral conjunctivitis is highly contagious. In this case, this patient's boyfriend


had the same condition a week ago and it is not surprising that she has the same
problem now. This kind of viral conjunctivitis is also the most common type of viral
conjunctivitis

Bacterial or viral conjunctivitis may be associated with a purulent discharge and bilateral
symptoms where as preauricular lymph nodes are only a feature of viral or Chlamydial
conjunctivitis.

Herpes simplex is unlikely here because there was no mentioning of cold sores or
fluorescein uptake showing dendritic ulcers.

Bacterial conjunctivitis is unlikely here especially with the lack of purulent discharge.
The same goes for chlamydial conjunctivitis where there isn't any purulent discharge.
In the history, there was not any suggestion of any history of atopy and itchiness so
allergic conjunctivitis is unlikely here.

**Orbital lymphoma could present like a chronic unilateral conjunctivitis (Painless)


resistant to treatment

Orbital lymphoma causes a gradual onset of symptoms (non-resolving unilateral


conjunctivitis). The persistence of the condition despite being on treatment should
prompt the practitioner to rule out more sinister causes of red-eye. All the other
options listed are not correct as they are associated with a painful red eye

**Iritis would present with a red eye too but the patient would be in pain, have
photophobia, a small or abnormal shaped pupil, and blurred vision.
Acute glaucoma would present with severe eye pain, seeing halos, blurred vision,
nausea and vomiting
Keratitis can present with a painful, red eye, with tearing and discharge. Vision can be
blurry and decreased, with some photophobia. Can be more common with contact
lens wearers.
Blepharitis would not involve the conjunctiva, it involves the eyelids. The eyes would
be itchy, red and swollen, with some flaky skin around the eye.
3.Blepharitis

Blepharitis is inflammation of the eyelid margins. It may due to either meibomian


gland dysfunction (common, posterior blepharitis) or seborrhoeic
dermatitis/staphylococcal infection (less common, anterior blepharitis). Blepharitis is
also more common in patients with rosacea

The meibomian glands secrete oil on to the eye surface to prevent rapid evaporation
of the tear film. Any problem affecting the meibomian glands (as in blepharitis) can
hence cause drying of the eyes which in turns leads to irritation

Features

• symptoms are usually bilateral


• grittiness and discomfort, particularly around the eyelid margins
• eyes may be sticky in the morning
• eyelid margins may be red. Swollen eyelids may be seen in staphylococcal
blepharitis
• styes and chalazions are more common in patients with blepharitis
• secondary conjunctivitis may occur

Management

• softening of the lid margin using hot compresses twice a day


• 'lid hygiene' - mechanical removal of the debris from lid margins
o cotton wool buds dipped in a mixture of cooled boiled water and baby
shampoo is often used
o an alternative is sodium bicarbonate, a teaspoonful in a cup of cooled
water that has recently been boiled
• artificial tears may be given for symptom relief in people with dry eyes or an
abnormal tear film

4.Acute angle closure glaucoma

Glaucoma is a group of disorders characterised by optic neuropathy due, in the


majority of patients, to raised intraocular pressure (IOP). It is now recognised that a
minority of patients with raised IOP do not have glaucoma and vice versa.
In acute angle-closure glaucoma (AACG) there is a rise in IOP secondary to an
impairment of aqueous outflow. Factors predisposing to AACG include:

• hypermetropia (long-sightedness)
• pupillary dilatation
• lens growth associated with age
Features

• severe pain: may be ocular or headache


• decreased visual acuity
• symptoms worse with mydriasis (e.g. watching TV in a dark room)
• hard, red-eye
• haloes around lights
• semi-dilated non-reacting pupil
• corneal oedema results in dull or hazy cornea
• systemic upset may be seen, such as nausea and vomiting and even abdominal
pain

Management

The management of AACG is an emergency and should prompt urgent referral to an


ophthalmologist. Emergency medical treatment is required to lower the IOP with
more definitive surgical treatment given once the acute attack has settled.

There are no guidelines for the initial medical treatment emergency treatment. An
example regime would be:

• combination of eye drops, for example:


o a direct parasympathomimetic (e.g. pilocarpine, causes contraction of the
ciliary muscle → opening the trabecular meshwork → increased outflow
of the aqueous humour)
o a beta-blocker (e.g. timolol, decreases aqueous humour production)
o an alpha-2 agonist (e.g. apraclonidine, dual mechanism, decreasing
aqueous humour production and increasing uveoscleral outflow)
• intravenous acetazolamide
o reduces aqueous secretions

Definitive management

• laser peripheral iridotomy


o creates a tiny hole in the peripheral iris → aqueous humour flowing to the
angle

**Topical beta blockers such as timolol work by decreasing production of aqueous.


Topical alpha agonists such as brimonidine work by decreasing production of
aqueous and increasing drainage. Topical prostaglandin analogues such as
latanoprost work by increasing the outflow of aqueous. Topical carbonic anhydrase
inhibitors such as dorzolamide work by decreasing production of aqueous. They can
all be used to control the intraocular pressure in glaucoma patients. However, they
are not definitive treatments for AACG.
**A 68-year-old man with a history of type 2 diabetes mellitus presents with
worsening eye sight. Mydriatic drops are applied and fundoscopy reveals pre-
proliferative diabetic retinopathy. A referral to ophthalmology is made. Later in the
evening whilst driving home he develops pain in his left eye associated with
decreased visual acuity. What is the most likely diagnosis?

Acute angle closure glaucoma-Mydriatic drops are a known precipitant of acute


angle closure glaucoma. This scenario is more common in exams than clinical
practice.

5.Optic neuritis

Causes

• multiple sclerosis: the commonest associated disease


• diabetes
• syphilis

Features

• unilateral decrease in visual acuity over hours or days


• poor discrimination of colours, 'red desaturation'
• pain worse on eye movement
• relative afferent pupillary defect
• central scotoma

Management

• high-dose steroids
• recovery usually takes 4-6 weeks

Prognosis

• MRI: if > 3 white-matter lesions, 5-year risk of developing multiple sclerosis is c.


50%

**A 32-year-old female patient presents to the eye hospital emergency department
with a 1-week history of blurred vision and right eye pain worse on movement. The
patient also reports a missing area of the visual field in the right eye. On examination,
there is decreased visual acuity and impaired colour vision in the right eye and a
right relative afferent pupillary defect. Fundoscopy reveals mildly swollen right optic
disc. Intraocular pressure is 18 mmHg in the right eye. Her past medical history
includes type 1 diabetes mellitus and hypertension.
--This patient has typical symptoms of optic neuritis. This could be idiopathic or
secondary to multiple sclerosis.

--Visual loss, eye pain and red desaturation are all classical symptoms of optic
neuritis, which is often the presenting symptom of multiple sclerosis (MS). Lethargy
is a common non-specific symptom of early MS.

--Colour vision is affected in optic neuritis and typically there is periocular pain
associated with eye movement. Visual field defects can also occur, most commonly in
the form of a central scotoma. There is typically a relative afferent pupillary defect if
the other eye is uninvolved.

Even though she has some risk factors for stroke but stroke would not cause eye pain
or affect colour vision.

Anterior ischaemic optic neuropathy usually causes painless visual loss and usually in
someone who is above 50 years of age.

She does not have acute angle closure glaucoma as her intraocular pressure is within
normal range.

She does not have papilloedema because that only one optic disc is swollen.

**The diagnosis of optic neuritis is supported by the classic triad of visual loss,
periocular pain, and dyschromatopsia (change in colour perception). Classic features
on examination include a central scotoma and relative afferent pupillary defect.

Open-angle glaucoma presents with painless, gradual loss of vision usually affecting
the peripheries rather than a central scotoma.

Anterior ischaemic optic neuropathy is normally seen in people over the age of 50. It
causes sudden, painless loss of vision.

Optic nerve glioma is rarely seen after the age of 20 and can be seen with
neurofibromatosis. The tumour causes visual defects and headaches, but you would
not expect eye movement pain or colour desaturation.

Retinitis pigmentosa causes a more gradual loss of peripheral vision rather than a
central scotoma. As it is a genetic condition it would normally have a family history,
bilateral eye involvement, and pigmented lesions are visible along the peripheral
retina.

6.Subconjunctival haemorrhage
Subconjunctival haemorrhages result from the bleeding of blood vessels into the
subconjunctival space. The vessels that bleed are usually the ones responsible for
supplying the conjunctiva or episclera. The cause is most commonly traumatic
followed by spontaneous idiopathic cases, Valsalva manoeuvres and several systemic
diseases. Subconjunctival haemorrhages can look dramatic and cause worry to
patients, however they are rarely an indicator of anything sinister.

Epidemiology:

• Subconjunctival haemorrhages are more common in women than in men when


there has been no history of trauma
• The annual incidence of non-traumatic subconjunctival haemorrhages is 0.6%,
the risk increases with age as the risk factors are more common in elderly
patients. Newborns are also more susceptible
• The incidence of both traumatic and non-traumatic subconjunctival
haemorrhages is 2.6%

Risk factors:

• Trauma and contact lens usage (68%): these are the most common causes
generally, as well as being often the sole risk factor in younger patients
• Idiopathic
• Valsalva manoeuvre e.g. coughing, straining
• Hypertension
• Bleeding disorders
• Drugs such as aspirin, NSAIDs and anticoagulants
• Diabetes
• Arterial disease and hyperlipidaemia

Symptoms:

• Red-eye, usually unilateral


• Subconjunctival haemorrhages are mostly asymptomatic, however mild irritation
may be present

Signs:

• Flat, red patch on the conjunctiva. It will have well-defined edges and normal
conjunctiva surrounding it. Patches can vary in size depending on the size of the
bleed, and can involve the whole conjunctiva
• Traumatic haemorrhages are most common in the temporal region (40.5%), with
the inferior conjunctiva as the next most commonly affected area
• Vision should be normal, including acuity, visual fields and range of eye
movements
• On examination, the fundus should be normal

Investigations:

• The diagnosis of a subconjunctival haemorrhage is a clinical one


• If there is no obvious traumatic cause, check the patient's blood pressure. If
raised, refer the patient appropriately
• If the patient is taking warfarin, check the INR. If raised, refer for appropriate
adjustments to the dose to bring the INR back into the target range
• If you cannot see the whole border of the haemorrhage i.e. the full extent of the
haemorrhage is unclear, it may be associated with an intracranial bleed or an
orbital roof fracture. This is often corroborated with a history of trauma and a
black eye. Further appropriate investigations should then be done, including a
full cranial nerve exam looking for neurological signs as well as a CT head, after
discussion with a senior
• Recurrent or spontaneous, bilateral subconjunctival haemorrhages warrant
investigations for bleeding disorders or other pathology

Management:

• Reassure the patient that whilst a subconjunctival haemorrhage may look


alarming, it is a benign condition that will resolve on its own in 2 to 3 weeks. If
the patient presents to services immediately after the haemorrhage occurs, it
can spread over a couple of days until it begins to resolve. The colour of the
patch may change to yellow/ green, like a bruise, but preparing the patient for
this will prevent any alarm if this happens.
• If the cause is traumatic consider a referral to the ophthalmologist to ensure no
other damage has been caused to the eye
• Advise the patient to consult their GP if this happens again because, as
previously mentioned, recurrent subconjunctival haemorrhages warrant
investigations for bleeding disorders or other potential pathology
• Artificial tears can be used for a couple of weeks if there is any mild irritation

Prognosis:

• This is a mild, self-limiting illness and should resolve on its own in 2-3 weeks

Subconjunctival haemorrhage could be an early and clinically important tool in


detecting warfarin over anticoagulation

**Warfarin over anticoagulation could cause a non-traumatic subconjunctival


haemorrhage. Hence, the INR level should be checked in this patient prior to carrying
out any other intervention. An urgent ophthalmology review is not warranted in this
patient as his vision is not affected. If the INR level is normal, the patient could be
given the reassurance that this condition is self-limiting and should resolve in a
couple of weeks. Topical lubricants could be prescribed if there is any irritation
secondary to the subconjunctival haemorrhage. An urgent skull x-ray is not relevant
to the question as there is no history of trauma.

7.Age related macular degeneration

Age-related macular degeneration is the most common cause of blindness in the UK.
Degeneration of the central retina (macula) is the key feature with changes usually
bilateral. ARMD is characterised by degeneration of retinal photoreceptors that
results in the formation of drusen which can be seen on fundoscopy and retinal
photography. It is more common with advancing age and is more common in
females.

Risk factors

• advancing age itself is the greatest risk factor for ARMD


o the risk of ARMD increases 3 fold for patients aged older than 75 years,
versus those aged 65-74.
• smoking
o current smokers are twice as likely as non-smokers to have ARMD
related visual loss, and ex-smokers have a slightly increased risk of
developing the condition, (OR 1.13).
• family history is also a strong risk factor for developing ARMD
o first degree relatives of a sufferer of ARMD are thought to be four times
more likely to inherit the condition.
• other risk factors for developing the condition include those associated with
increased risk of ischaemic cardiovascular disease, such as hypertension,
dyslipidaemia and diabetes mellitus.

Classification

Traditionally two forms of macular degeneration are seen:

• dry macular degeneration


o 90% of cases
o also known as atrophic
o characterised by drusen - yellow round spots in Bruch's membrane
• wet macular degeneration
o 10% of cases
o also know as exudative or neovascular macular degeneration
o characterised by choroidal neovascularisation
o leakage of serous fluid and blood can subsequently result in a rapid loss
of vision
o carries worst prognosis

Recently there has been a move to a more updated classification:

• early age-related macular degeneration (non-exudative, age-related


maculopathy): drusen and alterations to the retinal pigment epithelium (RPE)
• late age-related macular degeneration (neovascularisation, exudative)

Clinical features

Patients typically present with a subacute onset of visual loss with:

• a reduction in visual acuity, particularly for near field objects


• difficulties in dark adaptation with an overall deterioration in vision at night
• fluctuations in visual disturbance which may vary significantly from day to day
• they may also suffer from photopsia, (a perception of flickering or flashing
lights), and glare around objects

Signs:

• distortion of line perception may be noted on Amsler grid testing


• fundoscopy reveals the presence of drusen, yellow areas of pigment deposition
in the macular area, which may become confluent in late disease to form a
macular scar.
• in wet ARMD well demarcated red patches may be seen which represent intra-
retinal or sub-retinal fluid leakage or haemorrhage.

Investigations:

• slit-lamp microscopy is the initial investigation of choice, to identify any


pigmentary, exudative or haemorrhagic changes affecting the retina which may
identify the presence of ARMD. This is usually accompanied by colour fundus
photography to provide a baseline against which changes can be identified over
time.
• fluorescein angiography is utilised if neovascular ARMD is suspected, as this can
guide intervention with anti-VEGF therapy. This may be complemented with
indocyanine green angiography to visualise any changes in the choroidal
circulation.
• ocular coherence tomography is used to visualise the retina in three dimensions,
because it can reveal areas of disease which aren't visible using microscopy
alone.
Management

Treatment

• the AREDS trial examined the treatment of dry ARMD in 3640 subjects. It
showed that a combination of zinc with anti-oxidant vitamins A,C and E reduced
progression of the disease by around one third. Patients with more extensive
drusen seemed to benefit most from the intervention. Treatment is therefore
recommended in patients with at least moderate category dry ARMD.
• vascular endothelial growth factor (VEGF)
o VEGR is a potent mitogen and drives increased vascular permeability in
patients with wet ARMD
o a number of trials have shown that use of anti-VEGF agents can limit
progression of wet ARMD and stabilise or reverse visual loss
o evidence suggests that they should be instituted within the first two
months of diagnosis of wet ARMD if possible
o examples of anti-VEGF agents include ranibizumab, bevacizumab and
pegaptanib,. The agents are usually administered by 4 weekly injection.
• laser photocoagulation does slow progression of ARMD where there is new
vessel formation, although there is a risk of acute visual loss after treatment,
which may be increased in patients with sub-foveal ARMD. For this reason anti-
VEGF therapies are usually preferred.

**When you suspect AMD, it is crucial you refer to ophthalmology urgently


within 1 week to get a formal diagnosis because if the diagnosis is wet AMD it
can be treated with anti-VEGF injections and will improve the patient's long-
term visual outcome. In this case, even though drusen is present and is more
suggestive of dry AMD, the referral should still be done urgently within 1 week.

Administer anti-VEGF injections is a treatment for wet AMD but not dry AMD.
Referring to Ophthalmology within 1 month will delay the formal diagnosis and
commencement of treatment in this patient. Multi-vitamins may be useful in dry
AMD but is not the next step in management. Atorvastatin is used to lower blood
cholesterol level and will not help in treating drusen.

8.Scleritis

Features

• red eye
• classically painful (in comparison to episcleritis), but sometimes only mild
pain/discomfort is present
• watering and photophobia are common
• gradual decrease in vision

Scleritis presents as a painful red eye with reduced visual acuity and blurred vision. It
can be unilateral or bilateral and peak incidence is age 40-60. It is associated with
systemic illness in 50% of cases such as SLE, rheumatoid arthritis, herpes zoster and
other infections. Treatment is with steroids and urgent ophthalmology referral.

Episcleritis can be differentiated from scleritis by the absence of pain. It is also more
commonly unilateral although a third of cases are bilateral. Treatment is with non-
steroidal anti-inflammatories.

Anterior uveitis presents with a painful red eye with photophobia, blurred vision and
reduced visual acuity. Its presentation differs from scleritis in that the affected pupil is
often small and there may be pus in the anterior chamber (a hypopyon) on
examination. Patients often have worsening pain on convergence as the pupil
constricts. Anterior uveitis is associated with seronegative arthropathies such as
ankylosing spondylitis in 30% of cases. If suspected patients should be urgently
referred to ophthalmology.

Blepharitis is caused by inflammation of the lid margins often caused by ingrowing


eyelashes. Patients report dry red uncomfortable eyes, however, no visual problems
making this the incorrect answer. Treatment is conservative with regular eye care
using hot cloth.

Conjunctivitis is another cause of red eye but would present with eye discharge and
sticky eyes particularly on waking. There is also no effect on visual acuity in
conjunctivitis.

9.Herpes zoster ophthalmicus

Herpes zoster ophthalmicus (HZO) describes the reactivation of the varicella-zoster


virus in the area supplied by the ophthalmic division of the trigeminal nerve. It
accounts for around 10% of case of shingles.

• vesicular rash around the eye, which may or may not involve the actual eye itself
• Hutchinson's sign: rash on the tip or side of the nose. Indicates nasociliary
involvement and is a strong risk factor for ocular involvement
• This gentleman is exhibiting Hutchinson's sign which suggests a high likelihood of
ocular (corneal) involvement. This occurs because the nasociliary branch of the
trigeminal nerve innervates both the cornea and the nasal tip.

Management
• oral antiviral treatment for 7-10 days
o ideally started within 72 hours
o intravenous antivirals may be given for very severe infection or if the
patient is immunocompromised
o topical antiviral treatment is not given in HZO
• topical corticosteroids may be used to treat any secondary inflammation of the
eye
• ocular involvement requires urgent ophthalmology review

Complications

• ocular: conjunctivitis, keratitis, episcleritis, anterior uveitis


• ptosis
• post-herpetic neuralgia

10.Mydriasis

Causes of mydriasis (large pupil)

• third nerve palsy


• Holmes-Adie pupil
• traumatic iridoplegia
• phaeochromocytoma
• congenital

Drug causes of mydriasis

• topical mydriatics: tropicamide, atropine


• sympathomimetic drugs: amphetamines, cocaine
• anticholinergic drugs: tricyclic antidepressants

Anisocoria may result in apparent mydriasis, due to the difference with the other
pupil.

**Argyll-Robertson pupil (not a mydriatic pupil) is one of the classic pupillary


syndromes. It is sometimes seen in neurosyphilis. Typically the pupil accommodates
but doesn't react. A mnemonic used for the Argyll-Robertson Pupil (ARP) is
Accommodation Reflex Present (ARP) but Pupillary Reflex Absent (PRA)
Features

• small, irregular pupils


• no response to light but there is a response to accommodate

Causes
• diabetes mellitus
• syphilis

Diabetes is now the most common cause of Argyll-Robertson pupil in the UK


because neurosyphilis is now rarely seen.

A patient's eyes have both pupils reduce in size on a near object but do not constrict
when exposed to bright light. The condition is Argyll-Robertson pupil. Although it is
highly specific to neurosyphilis, this question is asking what is the most common
cause of it in the United Kingdom. Neurosyphilis is now rarely seen in the UK because
of effective treatment with antibiotics. Hence, the most common cause of an Argyll-
Robertson pupil is diabetes mellitus. Chlamydia or gonorrhoea are sexually
transmitted diseases but they do not cause an Argyll-Robertson pupil. Optic neuritis
does not cause an Argyll-Robertson pupil but it does cause a Marcus Gunn pupil, i.e.
Relative afferent pupillary defect.

10.Holmes-Adie pupil

Holmes-Adie pupil is a benign condition most commonly seen in women. It is one of


the differentials of a dilated pupil.

Overview

• unilateral in 80% of cases


• dilated pupil
• once the pupil has constricted it remains small for an abnormally long time
• slowly reactive to accommodation but very poorly (if at all) to light

Holmes-Adie syndrome

• association of Holmes-Adie pupil with absent ankle/knee reflexes

11.Eyelid problems

Eyelid problems commonly encountered include:

• blepharitis: inflammation of the eyelid margins typically leading to a red eye


• stye: infection of the glands of the eyelids
• chalazion (Meibomian cyst)
• entropion: in-turning of the eyelids
• ectropion: out-turning of the eyelids

Stye
Different types of stye are recognised:

• external (hordeolum externum): infection (usually staphylococcal) of the glands


of Zeis (sebum producing) or glands of Moll (sweat glands).
• internal (hordeolum internum): infection of the Meibomian glands. May leave a
residual chalazion (Meibomian cyst)
• management includes hot compresses and analgesia. CKS only recommend
topical antibiotics if there is an associated conjunctivitis

A chalazion (Meibomian cyst) is a retention cyst of the Meibomian gland. It


presents as a firm painless lump in the eyelid. The majority of cases resolve
spontaneously but some require surgical drainage

A chalazion has a more rounded appearance and would not have the yellow 'head'
like hordeolum externum

**Stye: infection of the glands of the eyelids,

Hordeolum externum is an infection of the glands of the external eyelid. This, which
is a type of stye.
A chalazion is also called meibomian cyst and is typically on the internal eyelid.
blepharitis is the inflammation of the eyelid margins. It does not usually cause a
lump.
Ectropion is the out-turning of the eyelids.
acne vulgaris does not usually affect the eyelids.

12.Allergic conjunctivitis

Allergic conjunctivitis may occur alone but is often seen in the context of hay fever

• Bilateral symptoms conjunctival erythema, conjunctival swelling (chemosis)


• Itch is prominent
• the eyelids may also be swollen
• May be a history of atopy
• May be seasonal (due to pollen) or perennial (due to dust mite, washing powder
or other allergens)

Management of allergic conjunctivitis

• first-line: topical or systemic antihistamines


• second-line: topical mast-cell stabilisers, e.g. Sodium cromoglicate and
nedocromil
for symptoms typical of allergic conjunctivitis:Topical antihistamines are the first-line
treatment in this condition. Oral antihistamines may be used if there are other
symptoms such as rhinosinusitis. Topical mast cell stabilisers (e.g. sodium
cromoglycate) may be used but NICE Clinical Knowledge Summaries advise:

13.Anterior uveitis

Anterior uveitis is one of the important differentials of a red eye. It is also referred to
as iritis. Anterior uveitis describes inflammation of the anterior portion of the uvea -
iris and ciliary body. It is associated with HLA-B27 and may be seen in association
with other HLA-B27 linked conditions (chronic back pain may be HLA-B27 related,
which is associated with anterior uveitis).

Features

• acute onset
• ocular discomfort & pain (may increase with use)
• pupil may be small +/- irregular due to sphincter muscle contraction
• photophobia (often intense)
• blurred vision
• red eye
• lacrimation
• ciliary flush: a ring of red spreading outwards
• hypopyon; describes pus and inflammatory cells in the anterior chamber,
often resulting in a visible fluid level
• visual acuity initially normal → impaired

Associated conditions

• ankylosing spondylitis
• reactive arthritis
• ulcerative colitis, Crohn's disease
• Behcet's disease
• sarcoidosis: bilateral disease may be seen. Bilateral anterior uveitis in young
adults could be an early manifestation of sarcoidosis

Management

• urgent review by ophthalmology


• cycloplegics (dilates the pupil which helps to relieve pain and photophobia) e.g.
Atropine, cyclopentolate
• steroid eye drops
** Anterior uveitis is an important differential of a red eye. It is characterised by acute
onset ocular discomfort & pain (may increase with use), lacrimation, intense
photophobia, blurred vision and pupil may be irregular and small. There is also ciliary
flush.

This patient is presenting with an acutely painful red eye associated with
photophobia, lacrimation and reduced visual acuity. This, along with his past medical
history of ankylosing spondylitis (associated with HLA B27) and examination findings
of a small, irregularly shaped pupil suggests a diagnosis of anterior uveitis, a
condition characterised by inflammation of the anterior portion of the uvea (middle
layer of the eye). Management of anterior uveitis is with steroid and cycloplegic
(mydriatic) eye drops.

Acetazolamide and cycloplegic (mydriatic) eye drops is incorrect. Acetazolamide is a


carbonic anhydrase inhibitor and is used to reduce aqueous secretions in acute
angle-closure glaucoma.

Acetazolamide and pilocarpine (causes pupillary constriction) eye drops is incorrect


as this is the management of acute angle-closure glaucoma.

Chloramphenicol eye drops is incorrect as this is the management of infective


conjunctivitis.

Oral steroids and pilocarpine eye drops is incorrect as topical, rather than oral
steroids are used in the management of anterior uveitis and pilocarpine induces
pupil constriction and will therefore have a negative effect in anterior uveitis

Conjunctivitis does not usually cause intense photophobia and does not affect the
pupils.

Episcleritis does not affect visual acuity.

Posterior uveitis typically causes floaters and symptoms are more subtle.

Acute angle-closure glaucoma is a differential of red-eye and ocular pain but no


mention of ocular pressure makes it the less likely option.

14.Sudden loss of vision

Sudden loss is a frightening symptom for patients. It may represent an ongoing issue
or only be temporary. The term transient monocular visual loss (TVML) describes a
sudden, transient loss of vision that lasts less than 24 hours.
The most common causes of a sudden painless loss of vision are as follows:

• ischaemic/vascular (e.g. thrombosis, embolism, temporal arteritis etc). This


includes recognised syndromes e.g. occlusion of central retinal vein and
occlusion of central retinal artery
• vitreous haemorrhage
• retinal detachment
• retinal migraine

Ischaemic/vascular

• often referred to as 'amaurosis fugax'


• wide differential including large artery disease (atherothrombosis, embolus,
dissection), small artery occlusive disease (anterior ischemic optic neuropathy,
vasculitis e.g. temporal arteritis), venous disease and hypoperfusion
• may represent a form of transient ischaemic attack (TIA). It should therefore be
treated in a similar fashion, with aspirin 300mg being given
• altitudinal field defects are often seen: 'curtain coming down'
• ischaemic optic neuropathy is due to occlusion of the short posterior ciliary
arteries, causing damage to the optic nerve

Central retinal vein occlusion

• incidence increases with age, more common than arterial occlusion


• causes: glaucoma, polycythaemia, hypertension
• severe retinal haemorrhages are usually seen on fundoscopy

Central retinal artery occlusion

• due to thromboembolism (from atherosclerosis) or arteritis (e.g. temporal


arteritis)
• features include afferent pupillary defect, 'cherry red' spot on a pale retina

Vitreous haemorrhage

• causes: diabetes, bleeding disorders, anticoagulants


• features may include sudden visual loss, dark spots

Retinal detachment

• features of vitreous detachment, which may precede retinal detachment, include


flashes of light or floaters (see below)

Differentiating posterior vitreous detachment, retinal detachment and vitreous haemorrhage


Posterior vitreous
Retinal detachment Vitreous haemorrhage
detachment
Dense shadow that starts Large bleeds cause sudden
Flashes of light
peripherally progresses visual loss
(photopsia) - in the
towards the central vision Moderate bleeds may be
peripheral field of vision
A veil or curtain over the field described as numerous
Floaters, often on the
of vision dark spots
temporal side of the
Straight lines appear curved Small bleeds may cause
central vision
Central visual loss floaters
** Flashes and floaters are symptoms of vitreous detachment. The patient is at risk of retinal
detachment and should be referred urgently to an ophthalmologist

15.Chorioretinitis

Causes

• syphilis
• cytomegalovirus
• toxoplasmosis
• sarcoidosis
• tuberculosis

Chorioretinitis suspected = must do HIV test

Toxoplasmosis retinitis classically presents with white focal retinitis with overlying
vitreous inflammation.

Cytomegalovirus retinitis classically has the appearance of a 'pizza pie', with retinal
spots and flame haemorrhages.

Retinitis pigmentosa is an inherited condition which has bone-spicules on the retina.

Hypertensive retinopathy and diabetic retinopathy do not present with single lesions
or vitreous inflammation.

One of the most common causes of chorioretinitis is cytomegalovirus, which in itself


affects up to 40% of patients with AIDS. It is essential an HIV test is performed in
such patients.

Protein electrophoresis is irrelevant in chorioretinitis.

16.Red eye
There are many possible causes of a red eye. It is important to be able to recognise
the causes which require urgent referral to an ophthalmologist. Below is a brief
summary of the key distinguishing features

Acute angle closure glaucoma

• severe pain (may be ocular or headache)


• decreased visual acuity, patient sees haloes
• semi-dilated pupil
• hazy cornea

Anterior uveitis

• acute onset
• pain
• blurred vision and photophobia
• small, fixed oval pupil, ciliary flush

Scleritis

• severe pain (may be worse on movement) and tenderness


• may be underlying autoimmune disease e.g. rheumatoid arthritis

Conjunctivitis: purulent discharge if bacterial, clear discharge if viral


Subconjunctival haemorrhage: history of trauma or coughing bouts
Endophthalmitis: typically red eye, pain and visual loss following intraocular surgery

**Red eye - glaucoma or uveitis?

•glaucoma: severe pain, haloes, 'semi-dilated' pupil

•uveitis: small, fixed oval pupil, ciliary flush

17.Episcleritis

Features

• red eye
• classically not painful (in comparison to scleritis), but mild pain may be present
• watering and mild photophobia may be present
• in episcleritis, the injected vessels are mobile when gentle pressure is applied on
the sclera. In scleritis, vessels are deeper, hence do not move
• phenylephrine drops may be used to differentiate between episcleritis and
scleritis. Phenylephrine blanches the conjunctival and episcleral vessels but not
the scleral vessels. If the eye redness improves after phenylephrine a diagnosis
of episcleritis can be made

Approximately 50% of cases are bilateral.


Management

• conservative
• artificial tears may sometimes be used

**Phenylephrine drops can be used to distinguish between episcleritis and scleritis

Phenylephrine 10% eye drops can be used to distinguish between episcleritis and
scleritis. It blanches the redness in episcleritis but doesn't in scleritis. This is a very
useful skill to obtain especially working in an Emergency Medicine rotation.

**Episcleritis and scleritis present in similar ways - unilateral red eye (usually not, or
only mildly, painful) with tearing and no photophobia. Vision is not commonly
affected. The main method of differentiating the two presentations is the use of
phenylephrine or neosynephrine eye drops. These drops will cause blanching of the
blood vessels in episcleritis, but not in scleritis.

Closed-angle glaucoma will present typically with a painful, unilateral red eye. The
pupil may be mid-dilated and the cornea will be hazy. The patient will often complain
of nausea and vomiting and may remark on 'haloing' around lights.

A hordeolum (stye) is an infection of one of the glands in the eyelid. This causes a
red, tender bump on the edge of the eyelid. It is most commonly a bacterial infection
by Staphylococcus aureus.

Retrobulbar haemorrhage can occur due to AV malformation, trauma or raised


venous pressure. It presents with a painful red eye which may protrude. Symptoms
are characteristically worse at night and associated with reduced vision, nausea &
vomiting, and diplopia

18.Cataracts

A cataract is a common eye condition where the lens of the eye gradually opacifies
i.e. becomes cloudy. This cloudiness makes it more difficult for light to reach the back
of the eye (retina), thus causing reduced/blurred vision. Cataracts are the leading
cause of curable blindness worldwide.
Epidemiology

• Cataracts are more common in women than in men


• The incidence of cataracts increases with age. One study found that 30% of
individuals aged 65 and over had a visually-impairing cataract in either one or
both eyes

Causes

• Normal ageing process: most common cause

Other possible causes

• Smoking
• Increased alcohol consumption
• Trauma
• Diabetes mellitus
• Long-term corticosteroids
• Radiation exposure
• Myotonic dystrophy
• Metabolic disorders: hypocalcaemia

Patients typically present with a gradual onset of:

• Reduced vision
• Faded colour vision: making it more difficult to distinguish different colours
• Glare: lights appear brighter than usual
• Halos around lights

Signs:

• A Defect in the red reflex: the red reflex is essentially the reddish-orange
reflection seen through an ophthalmoscope when a light is shone on the retina.
Cataracts will prevent light from getting to the retina, hence you see a defect in
the red reflex.

Investigations:

• Ophthalmoscopy: done after pupil dilation. Findings: normal fundus and optic
nerve
• Slit-lamp examination. Findings: visible cataract

Classification

• Nuclear: change lens refractive index, common in old age


• Polar: localized, commonly inherited, lie in the visual axis
• Subcapsular: due to steroid use, just deep to the lens capsule, in the visual axis
• Dot opacities: common in normal lenses, also seen in diabetes and myotonic
dystrophy

Management

• Non-surgical: In the early stages, age-related cataracts can be managed


conservatively by prescribing stronger glasses/contact lens, or by encouraging
the use of brighter lighting. These options help optimise vision but do not
actually slow down the progression of cataracts, therefore surgery will eventually
be needed.
• Surgery: Surgery is the only effective treatment for cataracts. This involves
removing the cloudy lens and replacing this with an artificial one. NICE suggests
that referral for surgery should be dependent upon whether a visual impairment
is present, impact on quality of life, and patient choice. Also whether both eyes
are affected and the possible risks and benefits of surgery should be taken into
account. Prior to cataract surgery, patients should be provided with information
on the refractive implications of various types of intraocular lenses. After
cataract surgery, patients should be advised on the use of eye drops and
eyewear, what to do if vision changes and the management of other ocular
problems. Cataract surgery has a high success rate with 85-90% of patients
achieving 6/12 corrected vision (on a Snellen chart) postoperatively.

**Once a cataract surgery is done, it is impossible to get a recurrence of cataract


because the lens has already been replaced with an artificial lens. Cystoid macular
oedema can occur days to weeks after cataract surgery due to inflammation and can
cause blurring of vision. But in this case, our patient had cataract surgery years ago
so this is unlikely. Optic disc swelling can certainly cause blurring of vision, but there
was no history of eye pain indicating optic neuritis or signs of raised intracranial
pressure. So optic disc swelling is unlikely to be the cause. Corneal ulcer is simply
wrong because there is no pain or any history e.g. contact lens wearer, herpes
simplex/zoster infection suggesting an infection of the ulcer

Complications following surgery

• Posterior capsule opacification: thickening of the lens capsule: Blurring of


vision again years after cataract surgery can occur due to posterior capsule
opacification.
• Retinal detachment
• Posterior capsule rupture
• Endophthalmitis: inflammation of aqueous and/or vitreous humour

**Hypocalcaemia is a cause of cataracts. The patient likely has hypocalcaemia, as


demonstrated by the positive Chvostek and Trousseau signs. The history of tetany
and convulsions are also likely related to the hypocalcaemia. Hypocalcaemia is a
cause of cataracts.

She is still too young to have cataracts related to age.

Hypercalcaemia is not a common cause of cataracts.

Steroids are a cause of cataracts but she is not taking a steroid.

She does not have diabetes mellitus given the normal HbA1c.

19.Hypertensive retinopathy

The table below shows the Keith-Wagener classification of hypertensive retinopathy

Stage Features
Arteriolar narrowing and tortuosity
I
Increased light reflex - silver wiring
II Arteriovenous nipping
Cotton-wool exudates
III
Flame and blot haemorrhages
IV Papilloedema

20.Herpes simplex keratitis

Herpes simplex keratitis most commonly presents with a dendritic corneal ulcer.
Features

• red, painful eye


• photophobia
• epiphora
• visual acuity may be decreased
• fluorescein staining may show an epithelial ulcer
• fluorescein staining demonstrates a small, feathery area of abnormal uptake

Management

• immediate referral to an ophthalmologist


• topical aciclovir

**A 30-year-old woman presented to your GP clinic with cold sore as well as left eye
pain. She complained of a 3-day history of a red, painful eye with watering and
photophobia. On fluorescein examination, you can see a dendritic ulcer on the
cornea which was stained with fluorescein-- This is herpes simplex keratitis and
should be treated with topical aciclovir. If there is an option saying immidiate
referral- tick that first.

Antibiotics have no value in treating a viral disease.

Steroids will aggravate this condition and shouldn't be used. If the patient is on
topical steroids for other ocular disease, the dosage should be reduced.

Topical anaesthetic drops may temporarily relieve the pain but should never be used
as a treatment.

21.Horner's syndrome

Features

• miosis (small pupil)


• ptosis
• enophthalmos* (sunken eye)
• anhidrosis (loss of sweating one side)

Distinguishing between causes

• heterochromia (difference in iris colour) is seen in congenital Horner's


• anhidrosis: see below

Pre-ganglionic
Central lesions Post-ganglionic lesions
lesions
Anhidrosis of the face, arm and
Anhidrosis of the face No anhidrosis
trunk
Stroke Carotid artery dissection
Pancoast's tumour
Syringomyelia Carotid aneurysm
Thyroidectomy
Multiple sclerosis Cavernous sinus
Trauma
Tumour thrombosis
Cervical rib
Encephalitis Cluster headache

*in reality the appearance is due to a narrow palpebral aperture rather than true
enophthalmos

**

Select Pupillary Defects


Pupillary
Comments
Defect
• Pathogenesis inolves impairment in sympathetic pathways to the
eye and face which results in
o ptosis
 secondary to impaired innervation into the upper lid
Horner smooth muscles
syndrome o miosis
 secondary to impaired innervation into the pupillary
dilator muscles
o anhidrosis

• Describes an afferent pupillary defect resulting in


o the affected eye is not sensitive to the light
 there is minimal or absent pupillary constriction of
the affected eye to light
• A lesion may affect
o eye
Marcus
o retina
Gunn pupil
o optic nerve
• Afferent pupillary defects can be detected with
o swinging flashlight test which will show
 the affected pupil dilate when exposed to the
penlight

• A type of light-near dissociation where


Argyll o the eye does not constrict in response to light as much as
Robertson it does with accomodation
pupil • Associated with neurosyphilis

• A type of light-near dissociation where


o the eye does not constrict in response to light as much as
it does with accomodation
Adie's
• Secondary to
myotonic
o degeneration of the
pupil
 ciliary ganglion
 postganglionic parasympathetic neurons

Disorder Notes
Tonically dilated pupil, slowly reactive to light with more definite
accommodation response. Caused by damage to parasympathetic
Adie pupil
innervation of the eye due to viral or bacterial infection. Commonly
seen in females, accompanied by absent knee or ankle jerks.
Disorder Notes
Relative afferent pupillary defect, seen during the swinging light
examination of pupil response. The pupils constrict less and therefore
Marcus-Gunn
appear to dilate when a light is swung from unaffected to affected
pupil
eye. Most commonly caused by damage to the optic nerve or severe
retinal disease.
Miosis (pupillary constriction), ptosis (droopy eyelid), apparent
enophthalmos (inset eyeball), with or without anhidrosis (decreased
Horner's
sweating) occurring on one side. Caused by damage to the
syndrome
sympathetic trunk on the same side as the symptoms, due to trauma,
compression, infection, ischaemia or many others.
Unilaterally dilated pupil which is unresponsive to light. A result of
Hutchinson's compression of the occulomotor nerve of the same side, by an
intracranial mass (e.g. tumour, haematoma)
Argyll-
Bilaterally small pupils that accommodate but don't react to bright
Robertson
light. Causes include neurosyphilis and diabetes mellitus
pupil
22.Keratitis

Keratitis describes inflammation of the cornea. Microbial keratitis is not like


conjunctivitis - it is potentially sight threatening and should therefore be urgently
evaluated and treated.
Aetiology

• bacterial
o typically Staphylococcus aureus
o Pseudomonas aeruginosa is seen in contact lens wearers.

• fungal
• amoebic
o acanthamoebic keratitis
o accounts for around 5% of cases
o increased incidence if eye exposure to soil or contaminated water
• parasitic: onchocercal keratitis ('river blindness')

Remember, other factors may causes keratitis:

• viral: herpes simplex keratitis


• environmental
o photokeratitis: e.g. welder's arc eye
o exposure keratitis
o contact lens acute red eye (CLARE)

Clinical features

• red eye: pain and erythema


• photophobia
• foreign body, gritty sensation
• hypopyon may be seen

Evaluation and management

Referral

• contact lens wearers


o assessing contact lens wearers who present with a painful red eye is
difficult
o an accurate diagnosis can only usually be made with a slit-lamp, meaning
same-day referral to an eye specialist is usually required to rule out
microbial keratitis

Management

• stop using contact lens until the symptoms have fully resolved
• topical antibiotics
o typically quinolones are used first-line
• cycloplegic for pain relief
o e.g. cyclopentolate

Complications may include:

• corneal scarring
• perforation
• endophthalmitis
• visual loss

**Whilst a hypopyon can of course be seen in anterior uveitis the combination of a


normal pupillary reaction and contact lens use make a diagnosis of keratitis more
likely.

23.Central retinal artery occlusion

Central retinal artery occlusion


• causes sudden unilateral visual loss
• due to thromboembolism (from atherosclerosis) or arteritis (e.g. temporal
arteritis)
• features include afferent pupillary defect, 'cherry red' spot on a pale retina

24.Central retinal vein occlusion

Central retinal vein occlusion (CRVO) is a differential for sudden painless loss of
vision.
Risk factors

• increasing age
• glaucoma
• polycythaemia

Features

• sudden, painless reduction or loss of visual acuity, usually unilaterally


• severe retinal haemorrhages are usually seen on fundoscopy
• appearance is compared to a cheese and tomato pizza

25.Retinitis pigmentosa

Retinitis pigmentosa primarily affects the peripheral retina resulting in tunnel vision.
Retinitis pigmentosa is a classically inherited retinal disorder

• night blindness is often the initial sign


• tunnel vision due to loss of the peripheral retina (occasionally referred to as
funnel vision)
• fundoscopy: black bone spicule-shaped pigmentation in the peripheral retina,
mottling of the retinal pigment epithelium

Associated diseases

• Refsum disease: cerebellar ataxia, peripheral neuropathy, deafness, ichthyosis


• Usher syndrome
• abetalipoproteinemia
• Lawrence-Moon-Biedl syndrome
• Kearns-Sayre syndrome
• Alport's syndrome

** Tunnel vision
Tunnel vision is the concentric diminution of the visual fields. Causes are-

• papilloedema
• glaucoma
• retinitis pigmentosa
• choroidoretinitis
• optic atrophy secondary to tabes dorsalis
• hysteria

26.Relative afferent pupillary defect

Also known as the Marcus-Gunn pupil, a relative afferent pupillary defect is found by
the 'swinging light test'. It is caused by a lesion anterior to the optic chiasm i.e. optic
nerve or retina. This is the 'swinging light test' and reveals a relative afferent pupillary
defect. As there is a defect in the afferent nerve on the left side the pupils constrict
less than normal, giving the impression of dilation

Finding

• the affected and normal eye appears to dilate when light is shone on the
affected

Causes

• retina: detachment
• optic nerve: optic neuritis e.g. multiple sclerosis

Pathway of pupillary light reflex

• afferent: retina → optic nerve → lateral geniculate body → midbrain


• efferent: Edinger-Westphal nucleus (midbrain) → oculomotor nerve

**A 34-year-old woman presents complaining of headaches. Examination of her


pupils using a light shone alternately in each eye reveals that when the light is shone
in the right eye both pupils constrict but when the light source immediately moves to
the left eye both eyes appear to dilate.What is the most likely diagnosis?

--Given her age, multiple sclerosis causing optic neuritis is the likely underlying
diagnosis. Optic neuritis typically causes a dull ache in the region of the eye which is
aggravated by movement

27.Squint
Squint (strabismus) is characterised by misalignment of the visual axes. Squints may
be divided into concomitant (common) and paralytic (rare)

Concomitant Paralytic
Due to imbalance in extraocular muscles
Due to paralysis of extraocular muscles
Convergent is more common than divergent

Detection of a squint may be made by the corneal light reflection test - holding a
light source 30cm from the child's face to see if the light reflects symmetrically on
the pupils
The cover test is used to identify the nature of the squint

• ask the child to focus on an object


• cover one eye
• observe movement of uncovered eye
• cover other eye and repeat test

Management

• referral to secondary care


o eye patches may help prevent amblyopia

Refer children with a squint to ophthalmology. This child needs to be reviewed


by an ophthalmologist and orthoptist

Intermittent squint in newborns less than 3 months is normal due to


underdeveloped eye muscles and does not need to be investigated. If the squint
persists or there is concern regarding the newborns ability to see, then referral to
secondary care is appropriate. Eye patches may be trialled in older children to
prevent lazy eye.

28.Orbital cellulitis

Orbital cellulitis is the result of an infection affecting the fat and muscles posterior to
the orbital septum, within the orbit but not involving the globe. It is usually caused
by a spreading upper respiratory tract infection from the sinuses and carries a high
mortality rate. Orbital cellulitis is a medical emergency requiring hospital admission
and urgent senior review. Periorbital (preseptal) cellulitis is a less serious superficial
infection anterior to the orbital septum, resulting from a superficial tissue injury
(chalazion, insect bite etc...). Periorbital cellulitis can progress to orbital cellulitis.

Epidemiology
• Mean age of hospitalisation 7-12 years.

Risk factors

• Childhood
• Previous sinus infection
• Lack of Haemophilus influenzae type b (Hib) vaccination
• Recent eyelid infection/ insect bite on eyelid (Peri-orbital cellulitis)
• Ear or facial infection

Presentation

• Redness and swelling around the eye


• Severe ocular pain
• Visual disturbance
• Proptosis
• Ophthalmoplegia/pain with eye movements
• Eyelid oedema and ptosis
• Drowsiness +/- Nausea/vomiting in meningeal involvement (Rare)

Differentiating orbital from preseptal cellulitis

• reduced visual acuity, proptosis, ophthalmoplegia/pain with eye movements are


NOT consistent with preseptal cellulitis

Investigations

• Full blood count – WBC elevated, raised inflammatory markers.


• Clinical examination involving complete ophthalmological assessment –
Decreased vision, afferent pupillary defect, proptosis, dysmotility, oedema,
erythema.
• CT with contrast – Inflammation of the orbital tissues deep to the septum,
sinusitis.
• Blood culture and microbiological swab to determine the organism. Most
common bacterial causes – Streptococcus, Staphylococcus aureus, Haemophilus
influenzae B.

Management

• admission to hospital for IV antibiotics

Infection of the ethmoidal sinus is the commonest cause of orbital cellulitis in


children due to the relatively thin medial wall of the orbit
The right diagnosis in this scenario is orbital cellulitis due to the presence of
proptosis, decreased visual acuity and ocular motility, which are not characteristic of
preseptal cellulitis. Given the history of previous prolonged cold, the most likely
cause of orbital cellulitis in this patient is ethmoidal sinusitis. The thin walls of
ethmoidal sinuses facilitate the spread of infection to the orbital cavity. Otitis externa
is unrelated to this question as there is no associated earache or discharge. There is
no mention of atopy or seasonal predilection to suggest allergic rhinitis.

29. Chorioretinitis

Causes

• syphilis
• cytomegalovirus
• toxoplasmosis
• sarcoidosis
• tuberculosis

**A 50-year-old man with a past medical history of cytomegalovirus presented to the
emergency department with a 2-week history of blurring of vision, sensitivity to light,
floaters and redness of the eye all in the right eye. On fundus examination, you can
see a mixture of white and red retinal lesions.

The above fundus examination showed a classic pizza pie appearance in keeping
with a diagnosis of chorioretinitis. A patient with cytomegalovirus is susceptible to
the development of chorioretinitis, which is an inflammation of the choroid and the
retina. It is a form of posterior uveitis. Anterior uveitis will not present with any fundal
signs.

One may suspect that retinal detachment is a cause when there are floaters.
However, retinal detachment does not present as a pizza pie appearance on
fundoscopy.

Conjunctivitis is incorrect because it wouldn't show any fundus signs.

30. Papilloedema

Papilloedema describes optic disc swelling that is caused by increased intracranial


pressure. It is almost always bilateral.
The following features may be observed during fundoscopy:

• venous engorgement: usually the first sign


• loss of venous pulsation: although many normal patients do not have normal
pulsation
• blurring of the optic disc margin
• elevation of optic disc
• loss of the optic cup
• Paton's lines: concentric/radial retinal lines cascading from the optic disc

Causes of papilloedema

• space-occupying lesion: neoplastic, vascular


• malignant hypertension
• idiopathic intracranial hypertension
• hydrocephalus
• hypercapnia: Hyperventilation to induce hypocapnia may be used in the
emergency setting to reduce intracranial pressure

Rare causes include

• hypoparathyroidism and hypocalcaemia


• vitamin A toxicity

31. Lens dislocation

Causes

• Marfan's syndrome: upwards


• homocystinuria: downwards
• Ehlers-Danlos syndrome
• trauma
• uveal tumours
• autosomal recessive ectopia lentis

**Patients with Marfan syndrome are at risk of lens dislocation (which is the most
frequent ocular manifestation of the disease). The dislocation is typically
superotemporal and may be monocular or binocular.

Retinal detachment also causes painless, sudden loss of vision but is less likely than
lens dislocation and would usually be preceded by flashes, floaters, or visual field
defect.

32. Diabetic retinopathy


Diabetic retinopathy is the most common cause of blindness in adults aged 35-65
years-old. Hyperglycaemia is thought to cause increased retinal blood flow and
abnormal metabolism in the retinal vessel walls. This precipitates damage to
endothelial cells and pericytes
Endothelial dysfunction leads to increased vascular permeability which causes the
characteristic exudates seen on fundoscopy. Pericyte dysfunction predisposes to the
formation of microaneurysms. Neovasculization is thought to be caused by the
production of growth factors in response to retinal ischaemia

In exams, you are most likely to be asked about the characteristic features of the
various stages/types of diabetic retinopathy. Recently a new classification system has
been proposed, dividing patients into those with non-proliferative diabetic
retinopathy (NPDR) and those with proliferative retinopathy (PDR):

Traditional classification New classification


Mild NPDR
Background retinopathy
• 1 or more microaneurysm
• microaneurysms (dots)
• blot haemorrhages (<=3) Moderate NPDR
• hard exudates
• microaneurysms
Pre-proliferative retinopathy • blot haemorrhages
• hard exudates
• cotton wool spots ('soft • cotton wool spots, venous
exudates' - represent areas of beading/looping and intraretinal
retinal infarction) microvascular abnormalities (IRMA) less
• > 3 blot haemorrhages severe than in severe NPDR
• venous beading/looping
• deep/dark cluster Severe NPDR
haemorrhages
• more common in Type I DM, • blot haemorrhages and microaneurysms
treat with laser in 4 quadrants
photocoagulation • venous beading in at least 2 quadrants
• IRMA in at least 1 quadrant

Proliferative retinopathy

• retinal neovascularisation - may lead to vitrous haemorrhage


• fibrous tissue forming anterior to retinal disc
• more common in Type I DM, 50% blind in 5 years

Maculopathy
• based on location rather than severity, anything is potentially serious
• hard exudates and other 'background' changes on macula
• check visual acuity
• more common in Type II DM

**A 30-year-old woman, known to have type 1 diabetes mellitus, presented to your
GP clinic with sudden onset loss of vision in her right eye. On direct ophthalmoscopy,
you cannot see any features of the retina and the disc. The only thing you can see is
redness in the vitreous.

--Proliferative retinopathy is more common in T1DM

--Such presentation suggests vitreous haemorrhage, secondary to proliferative


retinopathy which is more common in type 1 diabetes than type 2. In proliferative
retinopathy, there is retinal neovascularization. These vessels are fragile and are
prone to bleeding.

Background, pre-proliferative diabetic retinopathy and diabetic maculopathy have no


features of retinal neovascularization and hence will not cause vitreous haemorrhage.

Hyphaema means blood in the anterior chamber. Although it can be obscure your
view in direct ophthalmoscopy, it is not a feature of diabetic retinopathy.

33.Rheumatoid arthritis: ocular manifestations

Ocular manifestations of rheumatoid arthritis are common, with 25% of patients


having eye problems

Ocular manifestations

• keratoconjunctivitis sicca (most common)


• episcleritis (erythema)
• scleritis (erythema and pain)
• corneal ulceration
• keratitis

Iatrogenic

• steroid-induced cataracts
• chloroquine retinopathy

**A 24-year-old man self-presents to the eye casualty department. He admits to being
persuaded to attend by his girlfriend as he is not normally keen on seeing doctors. The
patient has a past medical history of ulcerative colitis.
The doctor performs a preliminary examination of the eyes and decides that either the
episclera or the sclera are inflamed. He subsequently performs a slit lamp examination.
Which key feature of the history or examination classically differentiates between the two
diagnoses?

--The correct answer is the presence of pain. Classically, episcleritis is a milder


condition without any pain (although occasionally may be uncomfortable). Scleritis is
almost always painful and is a more serious condition. Whilst episcleritis is usually
self-limiting, scleritis requires intervention.

Association with autoimmune disease would not particularly help to differentiate


between episcleritis and scleritis. Both are commonly associated with a number of
autoimmune and rheumatological diseases, including rheumatoid arthritis, ulcerative
colitis and granulomatosis with polyangiitis.

Fluorescein staining would not help to differentiate between the two. This is used to
detect corneal abnormalities; as such, it does not help with regards to episcleritis and
scleritis.

Laterality is not a particularly useful discriminator between the two conditions either
- episcleritis is bilateral in about 40% of cases and, similarly, scleritis is bilateral in
approximately 50% of cases.

The redness of the eyes would not help to differentiate between scleritis and
episcleritis - in both conditions, the eyes will be visibly red.

34. Dry eyes

People with dry eye syndrome typically present with feelings of dryness, grittiness, or
soreness in both eyes, which worsen through the day, and watering of the eyes,
particularly when exposed to wind. Symptoms that are worse on wakening, eyelids
sticking together on waking, and redness of the eyelids suggest dry eye syndrome
caused by Meibomian gland dysfunction.

There may be no abnormalities on examination. Less commonly people present with


a complication of dry eye syndrome, for example, conjunctivitis or ulceration of the
cornea, suggested by severe pain, photophobia, marked redness, and loss of visual
acuity.

Eyelid hygiene is the most appropriate management step here. Eyelid hygiene helps
to control blepharitis. Most people with dry eye syndrome have blepharitis.
Bilateral gritty, sore eyes can be caused by dry eye syndrome

Dry eye syndrome is a common complication of Bell's palsy. In Bell's palsy, there is an
impaired action of blinking or closing the eye. Without these functions, it is
impossible to keep the cornea moist which subsequently leads to dry eyes. The first
three options are incorrect as both keratic precipitates and foreign body can be
visualised without fluorescein dye. Furthermore, keratic precipitates are found in
anterior uveitis and is not the condition described in the question above. Corneal
abrasion will present with decreased visual acuity. Punctate fluorescein staining of the
cornea is common in patients with dry eyes.

Chronic infection of tarsal glands will impair the formation of the precorneal tear film
which plays a role in delaying evaporation of tears. This will further lead to dry eyes.

Meibomian glands (tarsal glands) produce an oily secretion which forms the
outermost component of the precorneal tear film. The oily substance impedes tear
evaporation whilst improving tear stability and spreading. In the case of Meibomian
blepharitis, the secretion of meibomian oil is reduced causing too much water from
the tear film to evaporate, resulting in dry eyes. All the other listed options are
irrelevant to the question as they do not involve dysfunctioning meibomian glands.

** Punctal plug is helpful in providing temporary relief for dry eyes among patients
who need frequent dosing of eye drops or when prescription eye drops fail to
provide symptomatic relief

Punctal plugs are recommended in severe dry eyes where there is frequent use of
eye drops, ie every 2hours or hourly, with the absence of inflammatory process. If a
patient is using more than 6 drops a day in total from all ocular medications,
preservative-free drops should be considered. This is because preservatives found in
eye drops can further damage the corneal and conjunctival epithelium. With
moderate to severe dry eyes, the potential for preservative toxicity is higher due to
more frequent dosing and reduced tear secretion.

35. Primary open-angle glaucoma: management

Glaucomas are optic neuropathies associated with raised intraocular pressure (IOP).
They can be classified based on whether the peripheral iris is covering the trabecular
meshwork, which is important in the drainage of aqueous humour from the anterior
chamber of the eye. In open-angle glaucoma, the iris is clear of the meshwork. The
trabecular network functionally offers an increased resistance to aqueous outflow,
causing increased IOP.
Epidemiology

• affects 0.5% of people over the age of 40


• the prevalence increases with age up to 10% over the age of 80 years
• affects males and females equally

Causes

• increasing age
• genetics: first degree relatives of an open-angle glaucoma patient have a 16%
chance of developing the disease

Symptoms:

• characterised by a slow rise in intraocular pressure: symptomless for a long


period
• typically present following an ocular pressure measurement during a routine
examination by an optometrist

Signs:

• increased intraocular pressure


• visual field defect
• pathological cupping of the optic disc1

Case finding:

• optic nerve head damage visible under the slit lamp


• visual field defect
• IOP > 24 mmHg as measured by Goldmann-type applanation tonometry
• if suspected full investigations are performed

Diagnosis:

• Case finding and provisional diagnosis is done by an optometrist


• Referral to the ophthalmologist is done via the GP
• Final diagnosis is done by investigations as below

Investigations:

• automated perimetry to assess visual field


• slit lamp examination with pupil dilatation to assess optic neve and fundus for a
baseline
• applanation tonometry to measure IOP
• central corneal thickness measurement
• gonioscopy to assess peripheral anterior chamber configuration and depth
• Assess risk of future visual impairment, using risk factors such as IOP, central
corneal thickness (CCT), family history, life expectancy

The majority of patients with primary open-angle glaucoma are managed with eye
drops. These aim to lower intra-ocular pressure which in turn has been shown to
prevent progressive loss of visual field. A prostaglandin analogue should be used
first-line in patients with a history of asthma- Latanoprost

NICE guidelines:

• first line: prostaglandin analogue (PGA) eyedrop


• second line: beta-blocker, carbonic anhydrase inhibitor, or sympathomimetic
eyedrop
• if more advanced: surgery or laser treatment can be tried2

Reassessment

• important to exclude progression and visual field loss


• needs to be done more frequently if: IOP uncontrolled, the patient is high risk,
or there is progression

Medication Mode of action Notes


Once daily administration
Prostaglandin analogues (e.g. Increases uveoscleral
Adverse effects include brown
latanoprost) outflow
pigmentation of the iris,
increased eyelash length
Should be avoided in
Beta-blockers (e.g. timolol, Reduces aqueous
asthmatics and patients with
betaxolol) production
heart block
Avoid if taking MAOI or
Sympathomimetics (e.g. Reduces aqueous tricyclic antidepressants
brimonidine, an alpha2- production and
adrenoceptor agonist) increases outflow Adverse effects include
hyperaemia
Carbonic anhydrase inhibitors Reduces aqueous Systemic absorption may cause
(e.g. Dorzolamide) production sulphonamide-like reactions
Adverse effects included a
Miotics (e.g. pilocarpine, a Increases uveoscleral
constricted pupil, headache
muscarinic receptor agonist) outflow
and blurred vision
Surgery in the form of a trabeculectomy may be considered in refractory cases.

36. Nasolacrimal duct obstruction

Nasolacrimal duct obstruction is the most common cause of a persistent watery eye
in an infant. It is caused by an imperforate membrane, usually at the lower end of the
lacrimal duct. Around 1 in 10 infants have symptoms at around one month of age

Management

• teach parents to massage the lacrimal duct


• symptoms resolve in 95% by the age of one year. Unresolved cases should be
referred to an ophthalmologist for consideration of probing, which is done
under a light general anaesthetic

Congenital tear duct obstruction can lead to recurrent watery/sticky eyes in


neonates. Patients will often have recurrent appointments for suspected
conjunctivitis but will show only minimal improvement with chloramphenicol drops
and have negative swabs. The majority of cases resolve by 1 year of age and so
parents should be reassured. For persistent cases after 1-year routine ophthalmology
referral is appropriate.

37. Corneal abrasion

Corneal abrasions refer to any defect of the corneal epithelium and most commonly
come about from a recent history of local trauma (e.g. finger nail, branches).
Common clinical findings include eye pain, photophobia, decreased visual acuity in
the affected eye, foreign body sensation and conjunctival injection.
Fluorescein examination typically reveals a yellow stained abrasion (representative of
the de-epithelialized surface) which is usually visible to the naked eye.
Treatment with a topical antibiotic is recommended for these patients in order to
prevent bacterial superinfection.

It is essential that you use fluorescein to determine if the corneal epithelium is intact
and suspect any corneal damage to see extent of damage and ?penetrating injury.
Topical antibiotic such as chloramphenicol ointment is recommended for these
patients in order to prevent bacterial superinfection.

Topical lubricant can be used but it isn't the first-line treatment.

Topical anaesthesia can temporarily relieve the pain but it risks further corneal
abrasion since the cornea loses sensation. It is not given as a treatment.
Topical steroid is not used to treat corneal abrasion.

You will not just watch and wait for a corneal abrasion.
PALLIATIVE CARE
1.Palliative care prescribing: agitation and confusion

Underlying causes of confusion need to be looked for and treated as appropriate, for
example hypercalcaemia, infection, urinary retention and medication. If specific
treatments fail then the following may be tried:

• first choice: haloperidol


• other options: chlorpromazine, levomepromazine

In the terminal phase of the illness then agitation or restlessness is best treated with
subcutaneous midazolam.

2. Palliative care prescribing: pain

NICE guidelines

In 2012 NICE published guidelines on the use of opioids in palliative care. Selected
points are listed below. Please see the link for more details.

Starting treatment

• when starting treatment, offer patients with advanced and progressive disease
regular oral modified-release (MR) or oral immediate-release morphine
(depending on patient preference), with oral immediate-release morphine for
breakthrough pain
• if no comorbidities use 20-30mg of MR a day with 5mg morphine for
breakthrough pain. For example, 15mg modified-release morphine tablets twice
a day with 5mg of oral morphine solution as required
• oral modified-release morphine should be used in preference to transdermal
patches
• laxatives should be prescribed for all patients initiating strong opioids
• patients should be advised that nausea is often transient. If it persists then an
antiemetic should be offered
• drowsiness is usually transient - if it does not settle then adjustment of the dose
should be considered

SIGN guidelines

SIGN issued guidance on the control of pain in adults with cancer in 2008. Selected
points
• the breakthrough dose of morphine is one-sixth the daily dose of
morphine
• all patients who receive opioids should be prescribed a laxative
• opioids should be used with caution in patients with chronic kidney disease
o oxycodone is preferred to morphine in palliative patients with mild-
moderate renal impairment
o if renal impairment is more severe, alfentanil, buprenorphine and fentanyl
are preferred
• metastatic bone pain may respond to strong opioids, bisphosphonates or
radiotherapy. The assertion that NSAIDs are particularly effective for metastatic
bone pain is not supported by studies. Strong opioids have the lowest number
needed to treat for relieving the pain and can provide quick relief, in contrast to
radiotherapy and bisphosphonates*. All patients, however, should be considered
for referral to a clinical oncologist for consideration of further treatments such
as radiotherapy

Other points

When increasing the dose of opioids the next dose should be increased by 30-50%.
In addition to strong opioids, bisphosphonates and radiotherapy, denosumab may
be used to treat metastatic bone pain.

Metastatic bone pain may respond to analgesia, bisphosphonates or


radiotherapy

Opioid side-effects

Usually transient Usually persistent


Nausea
Constipation
Drowsiness

Conversion between opioids

From To Conversion factor


Oral codeine Oral morphine Divide by 10
Oral tramadol Oral morphine Divide by 10**

Oxycodone generally causes less sedation, vomiting and pruritis than morphine but
more constipation.
From To Conversion factor
Oral morphine Oral oxycodone Divide by 1.5-2***

The current BNF gives the following conversion factors for transdermal perparations
• a transdermal fentanyl 12 microgram patch equates to approximately 30 mg
oral morphine daily
• a transdermal buprenorphine 10 microgram patch equates to approximately 24
mg oral morphine daily.

From To Conversion factor


Oral morphine Subcutaneous morphine Divide by 2
Oral morphine Subcutaneous diamorphine Divide by 3
Oral oxycodone Subcutaneous diamorphine Divide by 1.5

*BMJ 2015;350:h315 Cancer induced bone pain

**this has previously been stated as 5 but the current version of the BNF states a
conversion of 10

***historically a conversion factor of 2 has been used (i.e. oral oxycodone is twice as
strong as oral morphine). The current BNF however uses a conversion rate of 1.5

>> Alfentanil, buprenorphine and fentanyl are the preferred opioids in patients with
chronic kidney disease

>> A 65-year-old man with metastatic prostate cancer is admitted electively to the oncology
ward for radiotherapy to bone metastases at T7, T9 and L1.

His usual medications include: paracetamol 1g four times a day, tamsulosin 400 micrograms
once daily, atorvastatin 80mg once nocte, morphine sulphate modified release (Zomorph)
30mg twice daily, and ramipril 5mg once daily.

During your afternoon review, he tells you that he is still suffering with pain in his back,
despite taking his regular medications this morning. He thinks he usually takes another
analgesic as needed, but is unsure of the name or the dose.
--Breakthrough dose = 1/6th of daily morphine dose: Oromorph 10mg PO

>> A 72-year-old man has been an inpatient on the elderly care ward for the last 2 weeks. He
has a new diagnosis of metastatic lung cancer. On the morning ward round, he complains
that his pain is not being adequately controlled. He currently takes oral morphine sulphate
20mg four times a day along with codeine 30mg four times a day and regular ibuprofen.
What is the correct breakthrough dose of oral morphine to give this man?

Oral codeine to morphine (divide by 10). Therefore, oral codeine 10mg = oral morphine
1mg.
30mg x 4 = 120mg codeine. This equals 12mg morphine.
20mg x 4 = 80mg morphine.
Total morphine = 80mg + 12mg = 92mg.
The breakthrough dose of morphine is 1/6th of the total dose of morphine in 24 hours. This
main takes 92mg of morphine in 24 hours. 1/6th of this is 15mg.

>> You are asked to review an 85-year-old man who was admitted 5 days ago with
community acquired pneumonia. He has a past medical history of type 2 diabetes mellitus,
angina, chronic obstructive pulmonary disease (COPD) and spinal stenosis.

Unfortunately, despite optimal ward-based treatment including IV co-amoxiclav, the patient


has continued to deteriorate. He current scores 11 on the Glasgow coma scale. His pupils are
3mm bilaterally and reactive to light. He has been unable to take his morning medications
which include morphine sulphate modified release (Zomorph) 30mg twice daily, oramorph
10mg as required, and metoclopramide 10mg three times a day. In the past 24 hours, he has
used 4 doses of PRN oramorph.

He is reviewed on the consultant ward round and the decision is made that he should be for
end of life care. He is currently comfortable, with no evidence of hallucinations, pruritis or
myoclonus. The nurse asks you to convert his medications to a syringe driver.

What will you prescribe?

Ans: Divide by two for oral to subcutaneous morphine conversion


This question is asking you to convert oral morphine to subcutaneous morphine for use in a
syringe driver - also known as a continuous subcutaneous infusion (CSCI).

The first step to calculate doses for use in a CSCI is to calculate the total 24-hour usage of
the drug. We are told this patient is taking both zomorph (modified release morphine), and
oramorph (immediate release) - we need to include both of these medications in our
calculation.

The patient is taking 30mg zomorph twice daily = 60mg/24 hours.


He has also taken 4 doses of 10mg oramorph = 40mg/24 hours.

This gives us a total of 60mg + 40mg = 100mg/24 hours of oral morphine. In order to
convert this to subcutaneous morphine, we must divide by two. Therefore the amount of
morphine needed in the CSCI is 100mg/2 = 50mg/24 hours.

The patient is comfortable, with no evidence of opioid toxicity, and so there is no indication
to change to oxycodone at the moment.

3. Palliative care prescribing: hiccups

Management of hiccups

• chlorpromazine is licensed for the treatment of intractable hiccups


• haloperidol, gabapentin are also used
• dexamethasone is also used, particularly if there are hepatic lesions
IMMUNOLOGY
1.Allergy tests

Most commonly used test as easy to perform and


inexpensive. Drops of diluted allergen are placed on the
skin after which the skin is pierced using a needle. A
large number of allergens can be tested in one session.
Normally includes a histamine (positive) and sterile
Skin prick test
water (negative) control. A wheal will typically develop if
a patient has an allergy. Can be interpreted after 15
minutes

Useful for food allergies and also pollen


Determines the amount of IgE that reacts specifically with
suspected or known allergens, for example IgE to egg
protein. Results are given in grades from 0 (negative) to 6
(strongly positive)
Radioallergosorbent
Useful for food allergies, inhaled allergens (e.g. Pollen)
test (RAST)
and wasp/bee venom

Blood tests may be used when skin prick tests are not
suitable, for example if there is extensive eczema or if the
patient is taking antihistamines
Useful for contact dermatitis. Around 30-40 allergens are
placed on the back. Irritants may also be tested for. The
Skin patch testing
patches are removed 48 hours later with the results being
read by a dermatologist after a further 48 hours

2. Anaphylaxis

Anaphylaxis may be defined as a severe, life-threatening, generalised or systemic


hypersensitivity reaction.
Common identified causes of anaphylaxis:
• food (e.g. nuts) - the most common cause in children
• drugs
• venom (e.g. wasp sting)

Features
Sudden onset and rapid progression of:

• generalised pruritus
• widespread erythematous or urticarial rash
• swelling of throat and tongue may result in a hoarse voice and stridor
• respiratory wheeze, dyspnoea
• hypotension (feeling faint, dizziness, collapse), tachycardia

Management

Anaphylaxis is one of the few times when you would not have time to look up the
dose of a medication. The Resuscitation Council guidelines on anaphylaxis have
recently been updated. Adrenaline is by far the most important drug in anaphylaxis
and should be given as soon as possible. The recommended doses for adrenaline,
hydrocortisone and chlorphenamine are as follows:

Adrenaline Hydrocortisone Chlorphenamine


150 micrograms (0.15ml 1 250
< 6 months 25 mg
in 1,000) micrograms/kg
150 micrograms (0.15ml 1
6 months - 6 years 50 mg 2.5 mg
in 1,000)
300 micrograms (0.3ml 1 in
6-12 years 100 mg 5 mg
1,000)
Adult and child > 500 micrograms (0.5ml 1 in
200 mg 10 mg
12 years 1,000)

Adrenaline can be repeated every 5 minutes if necessary. The best site for IM
injection is the anterolateral aspect of the middle third of the thigh.

Management following stabilisation:

• patients who have had emergency treatment for anaphylaxis should be


observed for 6–12 hours from the onset of symptoms, as it is known that
biphasic reactions can occur in up to 20% of patients
• sometimes it can be difficult to establish whether a patient had a true episode of
anaphylaxis. Serum tryptase levels are sometimes taken in such patients as they
remain elevated for up to 12 hours following an acute episode of anaphylaxis.

3. Oral allergy syndrome

Oral allergy syndrome, also known as pollen-food allergy, is an IgE-mediated


hypersensitivity reaction to specific raw, plant-based foods including fruits,
vegetables, nuts and certain spices. It typically presents with mild tingling or pruritus
of the lips, tongue and mouth. This hypersensitivity reaction is initiated by cross-
reaction with a non-food allergen, most commonly birch pollen whereby the protein
in the food is similar but not identical in structure to the original allergen. This
explains why OAS is strongly linked with pollen allergies and presents with seasonal
variation. It also explains why cooking culprit food, and thereby denaturing the
proteins, prevents symptoms occurring.

OAS vs food allergies:

• A food allergy is caused by direct sensitivity to a protein present in food. OAS is


caused by cross-sensitisation to a structurally similar allergen present in pollen.
• Non-plant foods do not cause OAS because there are no cross-reactive
allergens in pollen which would be structurally similar to meat. Food allergies
may be caused by plant or non-plant foods and allergens occur in response to
substances that are more stable and able to survive stomach enzymes and acid
processing and cooking.
• Food allergies are more likely to cause systemic symptoms (vomiting and
diarrhoea) and are more likely to lead to anaphylaxis because the allergens are
not readily broken down. The symptoms of OAS are normally limited to the
oropharynx because the proteins that cause the symptoms are denatured by
stomach enzymes.

Epidemiology:

• About 2% of the UK population has OAS but this is probably under-diagnosed.


• About 1/2 of patients with a general pollen allergy and 3/4 of patients with an
allergy to birch pollen (the most common allergen in the UK) report symptoms
of OAS.

Associations:

• Birch pollen allergy (most common)


• Rye grass pollen allergy
• Rubber latex allergy

Presentation:

• OAS should be suspected in patients with a history of seasonal allergy


symptoms (rhinitis, conjunctivitis and asthma) who present with symptoms
minutes after eating a specific raw food.
• Itching and tingling of the lips, tongue and mouth are the most common
symptoms
• Mild swelling and redness of the lips, tongue and throat may occur.
• In severe cases, patients develop nausea and vomiting.
• Symptoms fully resolve within one hour of contact.
• Anaphylaxis is a very rare complication.

Signs:

• Most patients will have no visible signs during or between episodes of OAS.
• Some patients may have visible swelling and redness of the lips, tongue and oral
mucosa.
• Patients with anaphylaxis present with wheeze, hives, hypotension and collapse.

Investigations:

• OAS is a clinical diagnosis but further tests can be used to rule out diagnoses
such as food allergies and to confirm the diagnosis when the history is unclear.
• Standard IgE RAST and skin prick testing may be performed to identify common
allergens such as pollens which co-exist with OAS. Note that common pollen
allergies have specific associated foods which cause OAS. For example, birch
allergy is associated with sensitivity to, amongst others, carrot, parsley and
spinach.
• Food-specific IgE testing: this is unreliable and rarely performed clinically
because the food does not survive processing.
• Skin prick testing with culprit foods is more commonly used. A positive test
would cause a small, itchy lump to develop on the skin.

Management:

• Avoidance of culprit foods is the only required modification for the vast majority
of patients.
• If culprit food is eaten and symptoms develop, an oral antihistamine can be
taken.
• If symptoms of anaphylaxis develop, an ambulance should be called.
Intramuscular adrenaline may be required.
• Patients should be informed that once cooked, culprit foods should not cause
symptoms.

**A 23-year-old male medical student presents with swollen lips for the last 20 minutes. He
is concerned he is having an anaphylactoid reaction. Upon further questioning, he ate some
strawberries directly before the swelling began.
He has got this same reaction a dozen times before with strawberries and it has always
settled after taking an antihistamine. He has not taken an antihistamine this time as he had
run out. There is no vomiting or diarrhoea.
He has a past medical history of well-controlled asthma on salbutamol inhalers and hay fever
in the spring months well-controlled with antihistamines. There is no family history of atopy.
On examination his lips are erythematous and mildly swollen. There is no swelling or redness
elsewhere and he can talk to you in full sentences and is not short of breath. All his
observations are within normal limits.

He is given an antihistamine and kept in for observation.


What is the most likely diagnosis?

Answer:
Oral allergy syndrome is strongly linked with pollen allergies and presents with seasonal variation

Oral allergy syndrome (OAS) is the correct option as the case above describes the typical
history of pollen-food allergy, with the raw strawberries being coated with the allergen. This
is most commonly associated with birch pollen allergy as described above with the
“springtime hay fever”. OAS is the most likely choice in this case due to the red and swollen
lips triggered by raw strawberries and the previous history of symptoms resolving with
antihistamines.

Anaphylaxis is incorrect as there is no wheezing, hives or hypotension described in this case.

Hay fever is incorrect as this is not a reaction to pollen in the air, but more localised to the
mouth from eating of the strawberries.

Strawberry allergy is incorrect as the symptoms are localised as in OAS, whereas with specific
food allergy you would expect systemic symptoms such as vomiting and diarrhoea.
Moreover, anaphylaxis is a more common complication of food allergy compared to OAS.

C1 esterase inhibitor deficiency is incorrect as there is no family history reported in this case
and the swelling in this condition has an obvious trigger which would not be present in the
deficiency syndrome. The presentation is also different, with angioedema and abdominal
pain most commonly.
GERIATRIC MEDICINE
1.Acute confusional state

Acute confusional state is also known as delirium or acute organic brain syndrome. It
affects up to 30% of elderly patients admitted to hospital.

Predisposing factors include:

• age > 65 years


• background of dementia
• significant injury e.g. hip fracture
• frailty or multimorbidity
• polypharmacy

The precipitating events are often multifactorial and may include:

• infection: particularly urinary tract infections


• metabolic: e.g. hypercalcaemia, hypoglycaemia, hyperglycaemia, dehydration
• change of environment: New surroundings can cause delirium in cognitively
impaired patients- eg: move from house to old home
• any significant cardiovascular, respiratory, neurological or endocrine condition
• severe pain
• alcohol withdrawal
• constipation

Features - a wide variety of presentations

• memory disturbances (loss of short term > long term)


• may be very agitated or withdrawn
• disorientation
• mood change
• visual hallucinations
• disturbed sleep cycle
• poor attention

Management

• treatment of the underlying cause


• modification of the environment
• the 2006 Royal College of Physicians publication 'The prevention, diagnosis and
management of delirium in older people: concise guidelines' recommended
haloperidol 0.5 mg as the first-line sedative
• the 2010 NICE delirium guidelines advocate the use of haloperidol or olanzapine
• management can be challenging in patients with Parkinson's disease, as
antipsychotics can often worsen Parkinsonian symptoms
o careful reduction of the Parkinson medication may be helpful
o if symptoms require urgent treatment then the atypical antipsychotics
quetiapine and clozapine are preferred

**All antipsychotics may worsen the symptoms of Parkinson's disease and should be
avoided if possible. A small dose of oral lorazepam may be an alternative in such a
situation.

2. Lewy body dementia

Lewy body dementia is an increasingly recognised cause of dementia, accounting for


up to 20% of cases. The characteristic pathological feature is alpha-synuclein
cytoplasmic inclusions (Lewy bodies) in the substantia nigra, paralimbic and
neocortical areas.

The relationship between Parkinson's disease and Lewy body dementia is


complicated, particularly as dementia is often seen in Parkinson's disease. Also, up to
40% of patients with Alzheimer's have Lewy bodies.

Features

• progressive cognitive impairment


o in contrast to Alzheimer's, early impairments in attention and executive
function rather than just memory loss
o cognition may be fluctuating, in contrast to other forms of dementia
o usually develops before parkinsonism
• parkinsonism
• visual hallucinations (other features such as delusions and non-visual
hallucinations may also be seen)

Diagnosis

• usually clinical
• single-photon emission computed tomography (SPECT) is increasingly used. It is
currently commercially known as a DaTscan. Dopaminergic iodine-123-
radiolabelled 2-carbomethoxy-3-(4-iodophenyl)-N-(3-fluoropropyl) nortropane
(123-I FP-CIT) is used as the radioisotope. The sensitivity of SPECT in diagnosing
Lewy body dementia is around 90% with a specificity of 100%

eManagement
• both acetylcholinesterase inhibitors (e.g. donepezil, rivastigmine) and
memantine can be used as they are in Alzheimer's. NICE have made detailed
recommendations about what drugs to use at what stages. Please see the link
for more details
• neuroleptics should be avoided in Lewy body dementia as patients are
extremely sensitive and may develop irreversible parkinsonism. Questions may
give a history of a patient who has deteriorated following the introduction of an
antipsychotic agent

>>Lewy body dementia typically presents with fluctuating cognition in contrast to


other forms of dementia

** A 70-year-old man comes to the GP surgery with his wife because she is growing
increasingly concerned about his health. Five years ago he began to suffer from periods of
confusion and sleepiness that seemed to come and go at random. More recently he has also
developed a unilateral tremor in his right hand.
Upon questioning, his wife tells you that she has slept in a separate bed for the last 30 years
because her husband suffers from bad nightmares.
What is the most likely diagnosis?

Answer: Dementia with Lewy bodies is the correct answer. It is easy to get
confused by the different types of dementia, especially non-Alzheimer's disease
dementia. The key here is in the chronology of this man's symptoms and the
classification of dementias. Lewy body dementia is a category of dementia that
includes 2 dementias:

• 1. Dementia with Lewy bodies (DLB).


• 2. Parkinson's disease dementia.

Both DLB and Parkinson's disease dementia have overlapping symptoms, however, in
Parkinson's disease dementia the tremor, bradykinesia, and rigidity will develop
before dementia. In DLB, the opposite is true and this is the key to this question.
Other clues that point towards DLB include the 'periods of confusion and sleepiness
that seemed to come and go at random' referring to fluctuating cognition and 'his
wife tells you that she has slept in a separate bed for the last 30 years because her
husband suffers from bad nightmares', which is alluding to the REM-sleep disorder
that is a distinctive feature of DLB. Patients and their partners may report violently
acting out their dreams as many as 40 years before the onset of the dementia
symptoms.

Alzheimer's disease is incorrect. Alzheimer's disease dementia classically presents as


slowly worsening memory loss and forgetfulness that progresses to affect speech,
orientation, and concentration. It is not associated with tremor, fluctuating cognition,
or REM-sleep disturbances.
Frontal lobe dementia is incorrect. Frontal lobe dementia classically presents
primarily with behavioural changes and reckless behaviour. It is not associated with
tremor, fluctuating cognition, or REM-sleep disturbances.

Vascular dementia is incorrect. Vascular dementia classically presents as a step-wise


decline in cognitive function associated with a series of cerebrovascular events.
Vascular dementia does not fluctuate. It is not associated with tremor, fluctuating
cognition, or REM-sleep disturbances.

** An 84-year-old woman comes to see her GP because of concerns she has about her
husband. She says that over the last year or so her husband has become more confused and
forgetful and can often be seen talking to 'Alice', a dog the couple owned who died several
years ago. She has noticed that more recently he is also falling over more and moving
around the house slower.
Which of the following is the most likely diagnosis?

ans:In order to distinguish between the answers it is important to break down the symptoms
this patients has. First, given his age and the time frame in question (one year) his cognitive
decline would make you suspicious of a dementia process. Then the more specific symptoms
of a visual hallucinations (involving an animal) is highly suggestive of lewy-body dementia.
Finally, the reducing movement and increased rate of falls could point towards a
parkinsonism which further points towards the correct answer of Lewy-body dementia

3. Dementia

Dementia is thought to affect over 700,000 people in the UK and accounts for a large
amount of health and social care spending. The most common cause of dementia in
the UK is Alzheimer's disease followed by vascular and Lewy body dementia. These
conditions may coexist.

Features

• diagnosis can be difficult and is often delayed


• assessment tools recommended by NICE for the non-specialist setting include:
10-point cognitive screener (10-CS), 6-Item cognitive impairment test (6CIT)
• assessment tools not recommended by NICE for the non-specialist setting
include the abbreviated mental test score (AMTS), General practitioner
assessment of cognition (GPCOG) and the mini-mental state examination
(MMSE) have been widely used. A MMSE score of 24 or less out of 30 suggests
dementia

Management
• in primary care, a blood screen is usually sent to exclude reversible causes (e.g.
Hypothyroidism). NICE recommend the following tests: FBC, U&E, LFTs, calcium,
glucose, TFTs, vitamin B12 and folate levels. Patients are now commonly referred
on to old-age psychiatrists (sometimes working in 'memory clinics').
• in secondary care, neuroimaging is performed* to exclude other reversible
conditions (e.g. Subdural haematoma, normal pressure hydrocephalus) and help
provide information on aetiology to guide prognosis and management

*in the 2011 NICE guidelines structural imaging was said to be essential in the
investigation of dementia

There are multiple causes of dementia as outlined below, the majority of which are
progressive and irreversible. There are medications that can be used to slow progression, but
as clinicians we also have a responsibility to ensure that our patients aren't taking
medications which may make things worse.

The STOPP-START Criteria (Gallagher et al., 2008) outlines medications that we should
consider withdrawing in the elderly. One example of this is the use of tricyclic
antidepressants in patients with dementia, due to the risk of worsening cognitive
impairment.

4. Vascular dementia

Vascular dementia (VD) is the second most common form of dementia after
Alzheimer disease. It is not a single disease but a group of syndromes of cognitive
impairment caused by different mechanisms causing ischaemia or haemorrhage
secondary to cerebrovascular disease. Vascular dementia has been increasingly
recognised as the most severe form of the spectrum of deficits encompassed by the
term vascular cognitive impairment (VCI). Early detection and an accurate diagnosis
are important in the prevention of vascular dementia.

Epidemiology
• VD is thought to account for around 17% of dementia in the UK
• Prevalence of dementia following a first stroke varies depending on location and
size of the infarct, definition of dementia, interval after stroke and age among
other variables. Overall, stroke doubles the risk of developing dementia.
• Incidence increases with age
The main subtypes of VD:

• Stroke-related VD – multi-infarct or single-infarct dementia


• Subcortical VD – caused by small vessel disease
• Mixed dementia – the presence of both VD and Alzheimer’s disease

Risk factors
• History of stroke or transient ischaemic attack (TIA)
• Atrial fibrillation
• Hypertension
• Diabetes mellitus
• Hyperlipidaemia
• Smoking
• Obesity
• Coronary heart disease
• A family history of stroke or cardiovascular

Rarely, VD can be inherited as in the case of CADASIL (cerebral autosomal dominant


arteriopathy with subcortical infarcts and leukoencephalopathy.

Patients with VD typically presents with


• Several months or several years of a history of a sudden or stepwise
deterioration of cognitive function.

Symptoms and the speed of progression vary but may include:


• Focal neurological abnormalities e.g. visual disturbance, sensory or motor
symptoms
• The difficulty with attention and concentration
• Seizures
• Memory disturbance
• Gait disturbance
• Speech disturbance
• Emotional disturbance
Diagnosis is made based on:

• A comprehensive history and physical examination


• Formal screen for cognitive impairment
• Medical review to exclude medication cause of cognitive decline
• MRI scan – may show infarcts and extensive white matter changes

National Institute for health and care excellence (NICE) recommends that diagnosis
be made using the NINDS-AIREN criteria for probable vascular dementia

Presence of cognitive decline that interferes with activities of daily living, not due to
secondary effects of the cerebrovascular event

• established using clinical examination and neuropsychological testing

Cerebrovascular disease
• defined by neurological signs and/or brain imaging

A relationship between the above two disorders inferred by:

• the onset of dementia within three months following a recognised stroke


• an abrupt deterioration in cognitive functions
• fluctuating, stepwise progression of cognitive deficits

General management

• Treatment is mainly symptomatic with the aim to address individual problems


and provide support to the patient and carers
• Important to detect and address cardiovascular risk factors – for slowing down
the progression

Non-pharmacological management

• Tailored to the individual


• Include: cognitive stimulation programmes, multisensory stimulation, music and
art therapy, animal-assisted therapy
• Managing challenging behaviours e.g. address pain, avoid overcrowding, clear
communication

Pharmacological management

• There is no specific pharmacological treatment approved for cognitive


symptoms
• Only consider AChE inhibitors or memantine for people with vascular dementia
if they have suspected comorbid Alzheimer’s disease, Parkinson’s disease
dementia or dementia with Lewy bodies.
• There is no evidence that aspirin is effective in treating patients with a diagnosis
of vascular dementia.
• No randomized trials found evaluating statins for vascular dementia

>> Tight control of vascular risk factors, rather than antidementia medication,
is recommended by NICE in vascular dementia

** Stepwise deterioration in cognitive function? - think vascular dementiay

The stepwise deterioration in cognitive function along with risk factors for cerebrovascular
disease (hypertension and smoking) point towards a diagnosis of vascular dementia. In this
case, cognitive impairment is caused by ischaemia or haemorrhage secondary to
cerebrovascular disease.

Alzheimer's dementia is the most common form of dementia. It usually develops gradually
and progresses slowly.

Frontotemporal dementia affects the frontal and temporal lobes of the brain, leading to
changes in personality, behaviour, language and attention. Like Alzheimer's, it tends to
develop gradually, as those parts of the brain begin to atrophy.

Lewy body dementia is typically associated with auditory, visual or even olfactory
hallucinations. You may also notice Parkinsonian symptoms such as tremor and bradykinesia.

Parkinson's dementia develops in patients with Parkinson's disease (PD) when their disease
begins to impair thought processes, mental function, and memory. You would expect to find
signs of PD on physical examination.

5. Alzheimer's disease: management

Alzheimer's disease is a progressive degenerative disease of the brain accounting for


the majority of dementia seen in the UK

Non-pharmacological management

• NICE recommend offering 'a range of activities to promote wellbeing that are
tailored to the person's preference'
• NICE recommend offering group cognitive stimulation therapy for patients with
mild and moderate dementia
• other options to consider include group reminiscence therapy and cognitive
rehabilitation

https://passmedicine.com/database/dbnotes.php
Pharmacological management

• NICE updated it's dementia guidelines in 2018


• the three acetylcholinesterase inhibitors (donepezil, galantamine and
rivastigmine) as options for managing mild to moderate Alzheimer's disease
• memantine (an NMDA receptor antagonist) is in simple terms the 'second-line'
treatment for Alzheimer's, NICE recommend it is used in the following situation
reserved for patients with
o moderate Alzheimer's who are intolerant of, or have a contraindication to,
acetylcholinesterase inhibitors
o as an add-on drug to acetylcholinesterase inhibitors for patients with
moderate or severe Alzheimer's
o monotherapy in severe Alzheimer's
Managing non-cognitive symptoms

• NICE does not recommend antidepressants for mild to moderate depression in


patients with dementia
• antipsychotics should only be used for patients at risk of harming themselves or
others, or when the agitation, hallucinations or delusions are causing them
severe distress

Donepezil

• is relatively contraindicated in patients with bradycardia


• adverse effects include insomnia

**Donepezil is generally avoided (relative contraindication) in patients with bradycardia and


is used with caution in other cardiac abnormalities
Donepezil may cause bradycardia and atrioventricular node block.
ENT
1.Acute otitis media
Acute otitis media is extremely common in young children, with around half of children
having three or more episodes by the age of 3 years.

Pathophysiology
• whilst viral upper respiratory tract infections (URTIs) typically precede otitis media,
most infections are secondary to bacteria, particularly Streptococcus
pneumonaie, Haemophilus influenzae and Moraxella catarrhalis
• viral URTIs are thought to disturb the normal nasopharyngeal microbiome, allowing
bacteria to infect the middle ear via the Eustachian tube
Clinical features and diagnosis
Features: otalgia+ some children may tug or rub their ear
• fever occurs in around 50% of cases
• hearing loss
• recent viral URTI symptoms are common (e.g. coryza)
• ear discharge may occur if the tympanic membrane perforates

Possible otoscopy findings:


• bulging tympanic membrane → loss of light reflex
• opacification or erythema of the tympanic membrane
• perforation with purulent otorrhoea
• decreased mobility if using a pneumatic otoscope

Whilst guidelines vary, the majority use the following criteria to diagnose otitis media:
• acute onset of symptoms
o otalgia or ear tugging
• presence of a middle ear effusion
o bulging of the tympanic membrane, or
o otorrhoea
o decreased mobility on pneumatic otoscopy
• inflammation of the tympanic membrane
o i.e. erythema
Management
Acute otitis media is generally a self-limiting condition that does not require an antibiotic
prescription. There are however some exceptions listed below. Analgesia should be given to
relieve otalgia. Parents should be advised to seek medical help if the symptoms worsen or do
not improve after 3 days.

Antibiotics should be prescribed immediately if:


• Symptoms lasting more than 4 days or not improving
• Systemically unwell but not requiring admission
• Immunocompromise or high risk of complications secondary to significant heart,
lung, kidney, liver, or neuromuscular disease
• Younger than 2 years with bilateral otitis media
• Otitis media with perforation and/or discharge in the canal

If an antibiotic is given, a 5-7 day course of amoxicillin is first-line. In patients with penicillin
allergy, erythromycin or clarithromycin should be given.

Sequelae and complications


Common sequelae include:
• perforation of the tympanic membrane → otorrhoea
o unresolved with acute otitis media with perforation may develop into chronic
suppurative otitis media (CSOM)
o CSOM is defined as perforation of the tympanic membrane with otorrhoea for
> 6 weeks
• hearing loss
• labyrinthitis

Complications:
• mastoiditis
• meningitis
• brain abscess
• facial nerve paralysis
This is a classic case of otitis media. Following the revised 2015 NICE guidelines on otitis
media, antibiotics should not be prescribed routinely. In this case, the child is systemically
well and has had symptoms for 2 days. The parent should be advised to re-attend if symptoms
persist beyond 4 days or there is a significant deterioration in the child's symptoms.
2. Deafness
The most common causes of hearing loss are ear wax, otitis media and otitis externa. The
table below details some of the characteristic features of other causes:

Condition Key features

Presbycusis Presbycusis describes age-related sensorineural hearing loss.


Patients may describe difficulty following conversations

Audiometry shows bilateral high-frequency hearing loss

Otosclerosis Autosomal dominant, replacement of normal bone by vascular


spongy bone. Onset is usually at 20-40 years - features include:
• conductive deafness
• tinnitus
• tympanic membrane - 10% of patients may have a
'flamingo tinge', caused by hyperaemia
• positive family history

Glue ear Also known as otitis media with effusion


• peaks at 2 years of age
• hearing loss is usually the presenting feature (glue ear is
the commonest cause of conductive hearing loss and
elective surgery in childhood)
• secondary problems such as speech and language delay,
behavioural or balance problems may also be seen

Meniere's disease More common in middle-aged adults


• recurrent episodes of vertigo, tinnitus and hearing loss
(sensorineural). Vertigo is usually the prominent symptom
• a sensation of aural fullness or pressure is now recognised
as being common
• other features include nystagmus and a positive Romberg
test
• episodes last minutes to hours

Drug ototoxicity Examples include aminoglycosides (e.g. Gentamicin), furosemide,


aspirin and a number of cytotoxic agents

Noise damage Workers in heavy industry are particularly at risk


Hearing loss is bilateral and typically is worse at frequencies of
3000-6000 Hz

Acoustic neuroma (more Features can be predicted by the affected cranial nerves
correctly called vestibular
• cranial nerve VIII: hearing loss, vertigo, tinnitus
schwannomas)
• cranial nerve V: absent corneal reflex

• cranial nerve VII: facial palsy


Bilateral acoustic neuromas are seen in neurofibromatosis type 2

Ototoxicity results from exposure to drugs or chemicals that damage the inner ear or the
vestibulocochlear nerve and hence causes disturbances in hearing and/or balance.
Ototoxic medicines include gentamicin, quinine, furosemide, aspirin and some chemotherapy
agents. Quinine may be given for nocturnal leg cramps or as an antimalarial. Other side
effects of quinine include tinnitus, sweating, low platelets and increased sweating.

3. Glue ear
Glue ear describes otitis media with an effusion (other terms include serous otitis media). It is
common with the majority of children having at least one episode during childhood

Risk factors
• male sex
• siblings with glue ear
• higher incidence in Winter and Spring
• bottle feeding
• day care attendance
• parental smoking
Features
• peaks at 2 years of age
• hearing loss is usually the presenting feature (glue ear is the commonest cause of
conductive hearing loss and elective surgery in childhood)
• secondary problems such as speech and language delay, behavioural or balance
problems may also be seen

Treatment options include:


• grommet insertion - to allow air to pass through into the middle ear and hence do the
job normally done by the Eustachian tube. The majority stop functioning after about
10 months
• adenoidectomy
This child has otitis media with effusion (glue ear). Following the revised NICE guidelines in
2011, children should be observed for 6-12 weeks as symptoms are normally self-limiting
and referral should be reserved if symptoms persist beyond this period.

However, referral should be earlier if:


Symptoms are significantly affecting hearing, development or education
Immediate referral in children with Downs syndrome or cleft palate

4. Rinne's and Weber's test


Performing both Rinne's and Weber's test allows differentiation of conductive and
sensorineural deafness.

Rinne's test
• tuning fork is placed over the mastoid process until the sound is no longer heard,
followed by repositioning just over external acoustic meatus
• 'positive test': air conduction (AC) is normally better than bone conduction (BC)
• 'negative test': if BC > AC then conductive deafness

Weber's test
• tuning fork is placed in the middle of the forehead equidistant from the patient's ears
• the patient is then asked which side is loudest
• in unilateral sensorineural deafness, sound is localised to the unaffected side
• in unilateral conductive deafness, sound is localised to the affected side

Interpretation of Rinne and Weber tests


Rinne result Weber result

Normal Air conduction > bone conduction Midline


bilaterally

Conductive hearing Bone conduction > air conduction in Lateralises to affected


loss affected ear ear
Air conduction > bone conduction in
unaffected ear
Sensorineural hearing Air conduction > bone conduction Lateralises to
loss bilaterally unaffected ear

In this patient, there is no evidence of conductive hearing loss as Rinne test is normal in both
ears with air conduction louder than bone. In sensorineural hearing loss, the sound is
localised to the normal ear i.e the sound is loudest in the unaffected ear.

As such the other answers are all incorrect.

Causes of sensorineural hearing loss include -


Meniere's disease
Acoustic neuroma
Infections e,g measles, meningitis
Loud noise
Presbyacusis
Drugs e.g furosemide, aminoglycosides

5. Nasal septal haematoma


Nasal septal haematoma is an important complication of nasal trauma which should always
be looked for. It describes the development of a haematoma between the septal cartilage and
the overlying perichondrium.

Features
• may be precipitated by relatively minor trauma
• the sensation of nasal obstruction is the most common symptom
• pain and rhinorrhoea are also seen
• on examination, classically a bilateral, red swelling arising from the nasal septum
• this may be differentiated from a deviated septum by gently probing the swelling.
Nasal septal haematomas are typically boggy whereas septums will be firm

Management
• surgical drainage
• intravenous antibiotics

If untreated irreversible septal necrosis may develop within 3-4 days. This is thought to be
due to pressure-related ischaemia of the cartilage resulting in necrosis. This may result in a
'saddle-nose' deformity
6. Chronic rhinosinusitis
Chronic rhinosinusitis affects up to 1 in 10 people. It is generally defined as an inflammatory
disorder of the paranasal sinuses and linings of the nasal passages that lasts 12 weeks or
longer.

Predisposing factors include:


• atopy: hay fever, asthma
• nasal obstruction e.g. Septal deviation or nasal polyps
• recent local infection e.g. Rhinitis or dental extraction
• swimming/diving
• smoking

Features
• facial pain: typically frontal pressure pain which is worse on bending forward
• nasal discharge: usually clear if allergic or vasomotor. Thicker, purulent discharge
suggests secondary infection
• nasal obstruction: e.g. 'mouth breathing'
• post-nasal drip: may produce chronic cough

Management of recurrent or chronic sinusitis


• avoid allergen
• intranasal corticosteroids
• nasal irrigation with saline solution

Red flags symptoms


• unilateral symptoms
• persistent symptoms despite compliance with 3 months of treatment
• epistaxis
Nasal irrigation with saline solution is an appropriate first-line treatment for uncomplicated
chronic rhinosinusitis. Other recommended treatments include; smoking cessation, avoidance
of allergic triggers, and a 3-month course of a steroid nasal spray eg, fluticasone or
mometasone.

The first option is incorrect as the patient has been suffering for six months and the symptoms
are unlikely to resolve without some lifestyle intervention or treatment.
Optimising asthma control is an important part of managing chronic rhinosinusitis, though
there is no mention of his asthma being poorly controlled in the question here.

Long-term antibiotics should not be initiated without specialist advice and there is little
evidence of their efficacy in managing this condition.

Referral to ENT may need to be considered at some point but this condition is usually
managed in primary care unless any of the following, as listed below (from NICE guidelines)
are present.
Unilateral symptoms (consider urgent referral as this increases suspicion of neoplasia).
Persistent symptoms despite compliance with 3 months of treatment.
Nasal polyps complicating assessment or treatment, particularly if present in children.
Recurrent episodes of otitis media and pneumonia in a child.
Unusual opportunistic infections.
Symptoms that significantly interfere with functioning and quality of life.
Allergic or immunologic risk factors that need investigating.

7. Viral labyrinthitis
Labyrinthitis is an inflammatory disorder of the membranous labyrinth, affecting both the
vestibular and cochlear end organs. Labyrinthitis can be viral, bacterial or associated with
systemic diseases. Viral labyrinthitis is the most common form of labyrinthitis.

Labyrinthitis should be distinguished from vestibular neuritis as there are important


differences: vestibular neuritis is used to define cases in which only the vestibular nerve is
involved, hence there is no hearing impairment; Labyrinthitis is used when both the
vestibular nerve and the labyrinth are involved, usually resulting in both vertigo and hearing
impairment.

Epidemiology
• The average age of presentation is 40-70 years

Patients typically present with an acute onset of:


• vertigo: not triggered by movement but exacerbated by movement
• nausea and vomiting
• hearing loss: may be unilateral or bilateral, with varying severity
• tinnitus
• preceding or concurrent symptoms of upper respiratory tract infection
Signs of labyrinthitis:
• spontaneous unidirectional horizontal nystagmus towards the unaffected side
• sensorineural hearing loss: shown by Rinne's test and Weber test
• abnormal head impulse test: signifies an impaired vestibulo-ocular reflex
• gait disturbance: the patient may fall towards the affected side
The diagnosis is largely based on history and examination.

Management
• episodes are usually self-limiting
• prochlorperazine or antihistamines may help reduce the sensation of dizziness

8. Benign paroxysmal positional vertigo


Benign paroxysmal positional vertigo (BPPV) is one of the most common causes of vertigo
encountered. It is characterised by the sudden onset of dizziness and vertigo triggered by
changes in head position. The average age of onset is 55 years and it is less common in
younger patients.

Features
• vertigo triggered by change in head position (e.g. rolling over in bed or gazing
upwards)
• may be associated with nausea
• each episode typically lasts 10-20 seconds
• positive Dix-Hallpike manoeuvre

BPPV has a good prognosis and usually resolves spontaneously after a few weeks to months.
Symptomatic relief may be gained by:
• Epley manoeuvre (successful in around 80% of cases)
• teaching the patient exercises they can do themselves at home, termed vestibular
rehabilitation, for example Brandt-Daroff exercises

Medication is often prescribed (e.g. Betahistine) but it tends to be of limited value.

Around half of people with BPPV will have a recurrence of symptoms 3–5 years after their
diagnosis
09. Hoarseness
Causes of hoarseness include:
• voice overuse
• smoking
• viral illness
• hypothyroidism
• gastro-oesophageal reflux
• laryngeal cancer
• lung cancer

When investigating patients with hoarseness a chest x-ray should be considered to exclude
apical lung lesions.
Suspected laryngeal cancer: referral guidelines-

A suspected cancer pathway referral to an ENT specialist should be considered for people
aged 45 and over with:
• persistent unexplained hoarseness or
• An unexplained lump in the neck.

10. Otosclerosis
Otosclerosis describes the replacement of normal bone by vascular spongy bone. It causes a
progressive conductive deafness due to fixation of the stapes at the oval window. Otosclerosis
is autosomal dominant and typically affects young adults

Onset is usually at 20-40 years - features include:


• conductive deafness
• tinnitus
• normal tympanic membrane*
• positive family history
Management
• hearing aid
• stapedectomy

*10% of patients may have a 'flamingo tinge', caused by hyperaemia

11. Tonsillitis and tonsillectomy


Complications of tonsillitis include:
• otitis media
• quinsy - peritonsillar abscess
• rheumatic fever and glomerulonephritis very rarely

The indications for tonsillectomy are controversial. NICE recommend that surgery should be
considered only if the person meets all of the following criteria
• sore throats are due to tonsillitis (i.e. not recurrent upper respiratory tract infections)
• the person has five or more episodes of sore throat per year
• symptoms have been occurring for at least a year
• the episodes of sore throat are disabling and prevent normal functioning

Other established indications for a tonsillectomy include


• recurrent febrile convulsions secondary to episodes of tonsillitis
• obstructive sleep apnoea, stridor or dysphagia secondary to enlarged tonsils
• peritonsillar abscess (quinsy) if unresponsive to standard treatment

Complications of tonsillectomy
• primary (< 24 hours): haemorrhage in 2-3% (most commonly due to inadequate
haemostasis), pain
• secondary (24 hours to 10 days): haemorrhage (most commonly due to infection),
pain

12. Otitis externa


Otitis externa is a common reason for primary care attendance in the UK.

Causes of otitis externa include:


• infection: bacterial (Staphylococcus aureus, Pseudomonas aeruginosa) or fungal
• seborrhoeic dermatitis
• contact dermatitis (allergic and irritant)

Features
• ear pain, itch, discharge
• otoscopy: red, swollen, or eczematous canal
The recommended initial management of otitis externa is:
• topical antibiotic or a combined topical antibiotic with a steroid
• if the tympanic membrane is perforated aminoglycosides are traditionally not used*
• if there is canal debris then consider removal
• if the canal is extensively swollen then an ear wick is sometimes inserted

Second-line options includee


• consider contact dermatitis secondary to neomycin
• oral antibiotics (flucloxacillin) if the infection is spreading
• taking a swab inside the ear canal
• empirical use of an antifungal agent

If a patient fails to respond to topical antibiotics then the patient should be referred to ENT.

Malignant otitis externa is more common in elderly diabetics. In this condition, there is
extension of infection into the bony ear canal and the soft tissues deep to the bony canal.
Intravenous antibiotics may be required.

*many ENT doctors disagree with this and feel that concerns about ototoxicity are unfounded
Topical antibiotic drops are the first-line treatment for otitis externa and can be combined
with a steroid. Alternatively, aluminium acetate drops can be as effective as antibiotics drops.

Olive oil drops can be used in the treatment of excessive hard ear wax but would not be of
use in otitis externa.

Oral antibiotics are not the first line as they do not improve outcome and are generally only
used where topical/local treatment has failed. Alternatively, in children or severe cases e.g
very swollen ear canal, or significant debris in canal, oral antibiotics could be considered if
there is likely to be difficulty administering the drops.

Otitis media, not otitis externa can self-resolve. 80% of patients with otitis media report
improved symptoms in less than 4 days. Antibiotics can shorten the duration of symptoms but
are associated with side effects and drug resistance. As such treatment is usually delayed
unless symptoms persist, patients are systemically unwell or symptoms are bilateral.

In extreme cases ear syringing is used as a second-line treatment to remove ear canal debris.
13. Meniere's disease
Meniere's disease is a disorder of the inner ear of unknown cause. It is characterised by
excessive pressure and progressive dilation of the endolymphatic system. It is more common
in middle-aged adults but may be seen at any age. Meniere's disease has a similar prevalence
in both men and women.

Features
• recurrent episodes of vertigo, tinnitus and hearing loss (sensorineural). Vertigo is
usually the prominent symptom
• a sensation of aural fullness or pressure is now recognised as being common
• other features include nystagmus and a positive Romberg test
• episodes last minutes to hours
• typically symptoms are unilateral but bilateral symptoms may develop after a number
of years

Natural history
• symptoms resolve in the majority of patients after 5-10 years
• the majority of patients will be left with a degree of hearing loss
• psychological distress is common

Management
• ENT assessment is required to confirm the diagnosis
• patients should inform the DVLA. The current advice is to cease driving until
satisfactory control of symptoms is achieved
• acute attacks: buccal or intramuscular prochlorperazine. Admission is sometimes
required
• prevention: betahistine and vestibular rehabilitation exercises may be of benefit

14. Post-nasal drip

Post-nasal drip (PND) occurs as a result of excessive mucus production by the nasal
mucosa. This excess mucus accumulates in the throat or in the back of the nose
resulting in a chronic cough and bad breath.

**Nasal tumours present with nosebleeds, persistent blocked nose, blood stained mucus
draining from the nose and a decreased sense of smell.

A smoker's cough is a chronic cough that occurs as a result of damage and destruction of
the protective cilia of the respiratory tract.
A nasal polyp results in nasal obstruction, sneezing, rhinorrhoea and a poor sense of taste
and smell. Sinister features include unilateral symptoms or bleeding.

Nasal foreign bodies are usually seen in children and are commonly peas, beads, buttons,
seeds and sweets.

15. Facial pain

The table below gives characteristic exam question features for conditions causing
facial pain

Condition Characteristic exam feature


Facial 'fullness' and tenderness
Sinusitis
Nasal discharge, pyrexia or post-nasal drip leading to cough
Unilateral facial pain characterised by brief electric shock-like pains,
Trigeminal
abrupt in onset and termination
neuralgia
May be triggered by light touch, emotion
Pain typical occurs once or twice a day, each episode lasting 15
mins - 2 hours
Cluster
Clusters typically last 4-12 weeks
headache
Intense pain around one eye
Accompanied by redness, lacrimation, lid swelling, nasal stuffiness
Temporal Tender around temples
arteritis Raised ESR

16. Benign paroxysmal positional vertigo

Benign paroxysmal positional vertigo (BPPV) is one of the most common causes of
vertigo encountered. It is characterised by the sudden onset of dizziness and vertigo
triggered by changes in head position. The average age of onset is 55 years and it is
less common in younger patients.

Features

• vertigo triggered by change in head position (e.g. rolling over in bed or gazing
upwards)
• may be associated with nausea
• each episode typically lasts 10-20 seconds
• positive Dix-Hallpike manoeuvre

BPPV has a good prognosis and usually resolves spontaneously after a few weeks to
months. Symptomatic relief may be gained by:
• Epley manoeuvre (successful in around 80% of cases)
• teaching the patient exercises they can do themselves at home, termed
vestibular rehabilitation, for example Brandt-Daroff exercises

Medication is often prescribed (e.g. Betahistine) but it tends to be of limited value.


Around half of people with BPPV will have a recurrence of symptoms 3–5 years after
their diagnosis.

17. Perforated tympanic membrane

The most common cause of a perforated tympanic membrane is infection. Other


causes include barotrauma or direct trauma.
A perforated tympanic membrane may lead to hearing loss depending on the size
and also increase the risk of otitis media.

Management

• no treatment is needed in the majority of cases as the tympanic membrane will


usually heal after 6-8 weeks. It is advisable to avoid getting water in the ear
during this time
• it is common practice to prescribe antibiotics to perforations which occur
following an episode of acute otitis media. NICE support this approach in the
2008 Respiratory tract infection guidelines
• myringoplasty may be performed if the tympanic membrane does not heal by
itself

A perforated eardrum will usually heal by itself within 6-8 weeks. Patients with a perforation
should be advised that the eardrum is a skin-like structure and therefore it heals in the same
way as a cut on the skin. They should avoid getting water into the ear as this can impair
healing and increase the chance of infection.

It would be inappropriate to refer to ENT either urgently or routinely before 6 weeks as this
would be an inappropriate use of NHS resources. Leaving it beyond 12 months to refer
would also be inappropriate as it could cause long-term complications.

18. Allergic rhinitis

Allergic rhinitis is an inflammatory disorder of the nose where the nose become
sensitized to allergens such as house dust mites and grass, tree and weed pollens. It
may be classified as follows, although the clinical usefulness of such classifications
remains doubtful:
• seasonal: symptoms occur around the same time every year. Seasonal rhinitis
which occurs secondary to pollens is known as hay fever
• perennial: symptoms occur throughout the year
• occupational: symptoms follow exposure to particular allergens within the work
place

Features

• sneezing
• bilateral nasal obstruction
• clear nasal discharge
• post-nasal drip
• nasal pruritus

Management of allergic rhinitis

• allergen avoidance
• if the person has mild-to-moderate intermittent, or mild persistent symptoms:
o oral or intranasal antihistamines
• if the person has moderate-to-severe persistent symptoms, or initial drug
treatment is ineffective
o intranasal corticosteroids
• a short course of oral corticosteroids are occasionally needed to cover important
life events
• there may be a role for short courses of topical nasal decongestants (e.g.
oxymetazoline). They should not be used for prolonged periods as increasing
doses are required to achieve the same effect (tachyphylaxis) and rebound
hypertrophy of the nasal mucosa (rhinitis medicamentosa) may occur upon
withdrawal

19. Head and neck cancer

Head and neck cancer is an umbrella term. It typically includes:

• Oral cavity cancers


• Cancers of the pharynx (including the oropharynx, hypopharynx and
nasopharynx)
• Cancers of the larynx

Features

• neck lump
• hoarseness
• persistent sore throat
• persistent mouth ulcer

NICE suspected cancer pathway referral criteria (for an appointment within 2 weeks)

Laryngeal cancer

• Consider a suspected cancer pathway referral (for an appointment within 2


weeks) for laryngeal cancer in people aged 45 and over with:
o persistent unexplained hoarseness or
o an unexplained lump in the neck

Oral cancer

• Consider a suspected cancer pathway referral (for an appointment within 2


weeks) for oral cancer in people with either:
o unexplained ulceration in the oral cavity lasting for more than 3 weeks or
o a persistent and unexplained lump in the neck.
• Consider an urgent referral (for an appointment within 2 weeks) for assessment
for possible oral cancer by a dentist in people who have either:
o a lump on the lip or in the oral cavity or
o a red or red and white patch in the oral cavity consistent with
erythroplakia or erythroleukoplakia.

Thyroid cancer

• Consider a suspected cancer pathway referral (for an appointment within 2


weeks) for thyroid cancer in people with an unexplained thyroid lump.

** A history of unilateral earache with no obvious cause, persisting for more than 4 weeks
warrants referral to ENT under the 2-week wait to investigated for possible underlying
malignancy. This is especially important in a smoker, who is at greater risk for head and neck
malignancy.

Topical antibiotic/steroid drops are inappropriate in the absence of pathology on otoscopy.


Amitriptyline may aid symptoms but ignores the possible underlying diagnosis, as does a
nasal steroid spray. Routine referral is inappropriate given the possible malignancy.

20. Sore throat

Sore throat encompasses pharyngitis, tonsillitis, laryngitis


Clinical Knowledge Summaries recommend:

• throat swabs and rapid antigen tests should not be carried out routinely in
patients with a sore throat
Management

• paracetamol or ibuprofen for pain relief


• antibiotics are not routinely indicated
• there is some evidence that a single dose of oral corticosteroid may reduce the
severity and duration of pain, although this has not yet been incorporated into
UK guidelines

NICE indications for antibiotics

• features of marked systemic upset secondary to the acute sore throat


• unilateral peritonsillitis
• a history of rheumatic fever
• an increased risk from acute infection (such as a child with diabetes mellitus or
immunodeficiency)
• patients with acute sore throat/acute pharyngitis/acute tonsillitis when 3 or
more Centor criteria are present

Scoring systems

The Centor criteria are: score 1 point for each (maximum score of 4)

• presence of tonsillar exudate


• tender anterior cervical lymphadenopathy or lymphadenitis
• history of fever
• absence of cough

Centor score Likelihood of isolating Streptococci


0 or 1 or 2 3 to 17%
3 or 4 32 to 56%

The FeverPAIN criteria are: score 1 point for each (maximum score of 5)

• Fever over 38°C.


• Purulence (pharyngeal/tonsillar exudate).
• Attend rapidly (3 days or less)
• Severely Inflamed tonsils
• No cough or coryza

FeverPAIN score Likelihood of isolating Streptococci


0 or 1 13 to 18%
2 or 3 34% to 40%
4 or 5 62% to 65%
If antibiotics are indicated then either phenoxymethylpenicillin or erythromycin (if the
patient is penicillin allergic) should be given. Either a 7 or 10 day course should be
given.

21. Audiograms

Audiograms are usually the first-line investigation that is performed when a patient
complains of hearing difficulties. They are relatively easy to interpret as long as some
simple rules are followed:

• anything above the 20dB line is essentially normal (marked in green on the
audiogram below)
• in sensorineural hearing loss both air and bone conduction are impaired
• in conductive hearing loss only air conduction is impaired
• in mixed hearing loss both air and bone conduction are impaired, with air
conduction often being 'worse' than bone

This audiogram is essentially normal. Note all the values are above the 20dB line, highlighted in green
on this image

22. Vertigo

Vertigo may be defined as the false sensation that the body or environment is
moving.
The table below lists the main characteristics of the most important causes of vertigo
Disorder Notes
Recent viral infection
Sudden onset
Viral labyrinthitis
Nausea and vomiting
Hearing may be affected
Recent viral infection
Vestibular neuronitis Recurrent vertigo attacks lasting hours or days
No hearing loss
Gradual onset
Benign paroxysmal
Triggered by change in head position
positional vertigo
Each episode lasts 10-20 seconds
Associated with hearing loss, tinnitus and sensation of
Meniere's disease
fullness or pressure in one or both ears
Elderly patient
Vertebrobasilar ischaemia
Dizziness on extension of neck
Hearing loss, vertigo, tinnitus
Acoustic neuroma Absent corneal reflex is important sign
Associated with neurofibromatosis type 2

Other causes of vertigo include

• posterior circulation stroke


• trauma
• multiple sclerosis
• ototoxicity e.g. gentamicin

** This woman has a left sensorineural hearing loss and prolonged vertigo following a viral
illness. The most likely cause of this is labyrinthitis, a viral inner ear infection. Vertigo in
labyrinthitis comes on acutely and tends to be more prolonged and persistent in the first few
days before it begins to ease. In some cases, rest cannot take away the sensation of vertigo.
Given the history of a recent viral infection, viral labyrinthitis is the most likely answer.

Vestibular neuritis is inflammation of the vestibular nerve. It presents very similarly to


labyrinthitis. However, hearing loss is not a feature of vestibular neuritis, making labyrinthitis
the correct option.

Meniere’s disease typically presents as a triad of tinnitus, vertigo and sensorineural hearing
loss. Vertigo tends to last for minutes-hours at a time, and the disease follows a relapsing
and remitting course. A sensation of fullness in the ear is also a common feature.

Benign paroxysmal positional vertigo (BPPV) is classically characterised by sudden attacks of


vertigo of brief duration (< 30 seconds) brought on by changes in position. Common
symptoms include sudden onset of vertigo whilst turning in bed or sitting upright. It is a
common condition and becomes more common in middle-age. However, sensorineural
hearing loss and prolonged vertigo are not features of BPPV, making it a less likely diagnosis.

A cholesteatoma is a growth of keratinised epithelium in the middle ear. It commonly


presents with conductive hearing loss and discharge from the affected ear. In later stages of
the disease, a disturbance in balance and vertigo can present. However, this suggests that
the cholesteatoma has invaded the inner ear.

23. Sudden-onset sensorineural hearing loss

When a patient presents with sudden onset hearing loss it is important to examine
them carefully to differentiate between conductive and sensorineural hearing loss →
sudden-onset sensorineural hearing loss (SSNHL) requires urgent referral to ENT to
rule out rare serious causes such as acoustic neuroma. .
The majority of SSNHL cases are idiopathic.
MRI scan is usually performed to exclude a vestibular schwannoma.
High-dose oral corticosteroids are used by ENT for all cases of SSNHL.

24. Ramsay Hunt syndrome

Ramsay Hunt syndrome (herpes zoster oticus) is caused by the reactivation of the
varicella zoster virus in the geniculate ganglion of the seventh cranial nerve. Vesicles
do not necessarily appear in the ear canal and can be present on the anterior 2/3rds
of the tongue or the soft palate. Unless the patient is systemically very unwell, oral
aciclovir and prednisolone can be given.

Features

• auricular pain is often the first feature


• facial nerve palsy
• vesicular rash around the ear
• other features include vertigo and tinnitus

25. Cholesteatoma

Cholesteatoma is a non-cancerous growth of squamous epithelium that is 'trapped'


within the skull base causing local destruction. It is most common in patients aged
10-20 years. Being born with a cleft palate increases the risk of cholesteatoma
around 100 fold. A combination of cranial nerve abnormalities, ear discharge and
hearing loss, Otoscopy shows 'attic crust'- The most likely diagnosis is a
cholesteatoma.

Main features

• foul-smelling, non-resolving discharge


• hearing loss

Other features are determined by local invasion:

• vertigo
• facial nerve palsy
• cerebellopontine angle syndrome

Otoscopy

• 'attic crust' - seen in the uppermost part of the ear drum

Management

• patients are referred to ENT for consideration of surgical removal

26. Nasal polyps

Around in 1% of adults in the UK have nasal polyps. They are around 2-4 times more
common in men and are not commonly seen in children or the elderly.

Associations

• asthma (particularly late-onset asthma)


• aspirin sensitivity
• infective sinusitis
• cystic fibrosis
• Kartagener's syndrome
• Churg-Strauss syndrome

The association of asthma, aspirin sensitivity and nasal polyposis is known as


Samter's triad.

Features

• nasal obstruction
• rhinorrhoea, sneezing
• poor sense of taste and smell

Unusual features which always require further investigation include unilateral


symptoms or bleeding. Nasal polyps are lesions that arise from the nasal mucosa and
can occur at any site in the nasal cavity or paranasal sinuses. A unilateral nasal polyp
is a red-flag feature, which needs to be referred urgently to ENT as it could signify
nasal cancer. However, unilateral nasal polyps can also be caused by nose picking,
foreign bodies, misapplication of nasal spray or cystic fibrosis.
Measures such as antibiotics, oral steroids nasal drops or cautery are unlikely to be
effective in relieving symptoms. Furthermore, they are likely to delay the diagnosis of
a potentially sinister lesion.

Management

• all patients with suspected nasal polyps should be referred to ENT for a full
examination
• topical corticosteroids shrink polyp size in around 80% of patients

27. Tinnitus

Tinnitus is the perception of sounds in the ears or head that do not come from an
outside source. Around 1 in 10 people will experience an episode of tinnitus at some
point in their life. Although sometimes considered a 'minor' symptom of 'ringing in
the ears' it can be distressing to patients and may occasionally be a sign of a serious
underlying condition.

Causes of tinnitus include:

An underlying cause will not be


Idiopathic
found in the majority of patients.
Associated with hearing loss, vertigo,
Meniere's disease tinnitus and sensation of fullness or
pressure in one or both ears
Onset is usually at 20-40 years
Conductive deafness
Tinnitus
Normal tympanic membrane
Otosclerosis although 10% of patients may have a
'flamingo tinge', caused by
hyperaemia

Positive family history


Hearing loss, vertigo,
tinnitus
Sudden onset Absent corneal reflex is
Around 80% of
sensorineural hearing important sign
Acoustic neuroma
loss (SSNHL) Associated with
neurofibromatosis type
2
An underlying cause will not be
Idiopathic
found in the majority of patients.
Causes include excessive loud noise
Hearing loss
and presbycusis
Aspirin/NSAIDs
Aminoglycosides
Drugs
Loop diuretics
Quinine
Impacted ear wax Usually obvious on otoscopy

Assessment

• audiological assessment
o detect underlying hearing loss
• imaging
o not all patients will require imaging. Generally, non-pulsatile tinnitus does
not require imaging unless it is unilateral or there are other neurological
or ontological signs. MRI of the internal auditory meatuses (IAM) is first-
line
o pulsatile tinnitus generally requires imaging as there may be an
underlying vascular cause. Magnetic resonance angiography (MRA) is
often used to investigate pulsatile tinnitus

Management

• investigate and treat any underlying cause


• amplification devices
o more beneficial if associated hearing loss
• psychological therapy may help a limited group of patients
o examples include cognitive behavioural therapy
• tinnitus support groups

28. Glue ear

Glue ear describes otitis media with an effusion (other terms include serous otitis
media). It is common with the majority of children having at least one episode during
childhood

Unilateral glue ear in an adult needs evaluation for a posterior nasal space tumour.
Tumours in the post-nasal space can obstruct the openings of the Eustachian tubes, leading
to persistent middle ear effusion. Adult patients should be referred to ENT under the 2 week
wait for assessment for malignancy.

29. Malignant otitis externa

Basics

• Uncommon type of otitis externa that is found in immunocompromised


individuals (90% cases found in diabetics)
• most commonly caused by Pseudomonas aeruginosa
• Infection commences in the soft tissues of the external auditory meatus, then
progresses to involve the soft tissues and into the bony ear canal
• Progresses to temporal bone osteomyelitis

Key features in history

• Diabetes (90%) or immunosuppression (illness or treatment-related)


• Severe, unrelenting, deep-seated otalgia
• Temporal headaches
• Purulent otorrhea
• Possibly dysphagia, hoarseness, and/or facial nerve dysfunction

Diagnosis

• A CT scan is typically done

Treatment

• non-resolving otitis externa with worsening pain should be referred urgently to


ENT
• Intravenous antibiotics that cover pseudomonal infections

** Otitis externa in diabetics: treat with ciprofloxacin to cover Pseudomonas


This patient is presenting with a history suggestive of malignant otitis externa (also known as
necrotizing otitis externa), a condition in which invasive bacterial infection spreads from the
soft tissues of the external auditory meatus to involve the bony ear canal and may progress
to temporal bone osteomyelitis. The condition is most common in immunocompromised
individuals and diabetics and is most commonly caused by Pseudomonas aeruginosa.
Management, therefore, requires coverage of Pseudomonas aeruginosa and is usually with
intravenous ciprofloxacin. Please note, diabetic patients presenting with non-malignant otitis
externa should be treated with ciprofloxacin as these patients are at high risk of progressing
to malignant otitis externa.
30. Salivary glands

Sialadenitis - inflammation of the salivary gland likely secondary to obstruction by a


stone impacted in the duct. The duct from the submandibular gland drain into the
floor of the mouth and purulent discharge from this duct causes a foul taste in the
mouth.
There are 3 main salivary glands:

• the parotid glands are anterior and inferior to each ear


• the submandibular glands lies below the angle of the jaw
• the sublingual glands lie beneath the tongue

Disorders of these glands occur due to infection, inflammation, obstruction or


malignancy.
Submandibular gland: Causes of swelling of this gland:

• Obstruction: stone
• Tumour: benign or malignant

**parotid (serous) - most tumours,

**submandibular (mixed) - most stones,

**sublingual (mucous)

Pathology

• tumours: '80% parotid, 80% of these = pleomorphic adenomas, 80% superficial


lobe
• malignant rare: short hx, painful, hot skin, hard, fixation, CN VII involvement

Pleomorphic adenomas (benign, 'mixed parotid tumour', 80%)

• middle age
• slow growing, painless lump
• superficial parotidectomy; risk = CN VII damage

Warthin's tumour (benign, 'adenolymphomas', 10%)

• males, middle age


• softer, more mobile and fluctuant (although difficult to differentiate)

Stones

• recurrent unilateral pain & swelling on eating


• may become infected → Ludwig's angina
• 80% are submandibular
• plain x-rays; sialography
• surgical removal

Other causes of enlargement

• acute viral infection e.g. mumps


• acute bacterial infection e.g. 2nd to dehydration diabetes
• sicca syndrome and Sjogren's (e.g. RA)

31. Vestibular neuronitis

Vestibular neuronitis is a cause of vertigo that often develops following a viral


infection.

Features

• recurrent vertigo attacks lasting hours or days


• nausea and vomiting may be present
• horizontal nystagmus is usually present
• no hearing loss or tinnitus

Differential diagnosis

• viral labyrinthitis
• posterior circulation stroke: the HiNTs exam can be used to distinguish
vestibular neuronitis from posterior circulation stroke

Management

• vestibular rehabilitation exercises are the preferred treatment for patients who
experience chronic symptoms
• buccal or intramuscular prochlorperazine is often used to provide rapid relief for
severe cases
• a short oral course of prochlorperazine, or an antihistamine (cinnarizine,
cyclizine, or promethazine) may be used to alleviate less severe cases

** vestibular neuronitis in which patients develop recurrent vertigo attacks lasting hours to
days. Patients can also experience nausea or vomiting but no hearing loss, tinnitus, or
neurological features, helping to distinguish it from other causes of vertigo. (During attacks,
she feels nauseous. She does not report any hearing disturbance or tinnitus. Her symptoms
are not worsened by head movement. Attacks last around 4-5 hours and in-between she
feels completely well. She recalls having a viral illness the week before her symptoms
started.)

Meniere's disease presents with a triad of vertigo, hearing loss and tinnitus and as such was
not the correct answer.

Benign paroxysmal positional vertigo occurs following head movement or change in position
with vertigo lasting seconds to minutes. There is no suggestion of symptoms on head
movement here and symptoms last longer than seconds to minutes making this less likely to
be the correct answer.

Acoustic neuromas are slow growing neurofibromas and can present as an acute unilateral
sensorineural hearing loss with tinnitus and facial nerve palsy. There is no hearing loss, in this
case, making vestibular neuronitis more likely.

Vertebrobasilar insufficiency is incorrect as this occurs in elderly patients on extension and


rotation of the head. Patients may also report neck pain.

32. Sudden-onset sensorineural hearing loss

When a patient presents with sudden onset hearing loss it is important to examine
them carefully to differentiate between conductive and sensorineural hearing loss →
sudden-onset sensorineural hearing loss (SSNHL) requires urgent referral to ENT.

The majority of SSNHL cases are idiopathic.

An MRI scan is usually performed to exclude a vestibular schwannoma.

High-dose oral corticosteroids are used by ENT for all cases of SSNHL

**Acute sensorineural hearing loss is an emergency and requires urgent referral to


ENT for audiology assessment and brain MRI

When Weber's test lateralized to the left, it indicates a conductive hearing loss in the
left ear or sensorineural loss in the right ear. Rinne's test found air conduction was
louder than bone conduction in both ears. Therefore this patient has a sensorineural
hearing loss in the right ear. Acute sensorineural hearing loss is an emergency and
requires urgent referral to ENT for audiology assessment and brain MRI. This is
because serious pathology such as a vestibular schwannoma needs to be ruled out
immediately.
There are no obvious features of infection so antibiotics would not be indicated.
Infection of the external ear, acute otitis externa, is often treated with a topical acetic
acid spray containing neomycin. Acute otitis externa symptoms include itching,
discharge and pain.

Otitis media may be treated with oral antibiotics, however, the majority of cases are
caused by a virus following an upper respiratory tract infection. Otitis media
symptoms include conductive hearing loss and pain.

Examination by otoscopy revealed wax in the right ear, however, this would cause
conductive hearing loss. Referral for ear syringing is therefore not required. Where
problems are associated with wax accumulation, topical treatments such as olive oil
can be tried first to soften the wax.

Routine referral to ENT would not be quick enough. Patients may have to wait a
number of months before being seen in a routine clinic. Acute sensorineural hearing
loss requires an urgent referral for audiology assessment and brain MRI.

33. Mouth lesions

All mouth ulcers persisting for greater than 3 weeks should be sent to oral surgery as a 2
week wait referral.
2 week wait referrals to oral surgery should be done in all of the following cases:

• Unexplained oral ulceration or mass persisting for greater than 3 weeks


• Unexplained red, or red and white patches that are painful, swollen or bleeding
• Unexplained one-sided pain in the head and neck area for greater than 4 weeks,
which is associated with ear ache, but does not result in any abnormal findings
on otoscopy
• Unexplained recent neck lump, or a previously undiagnosed lump that has
changed over a period of 3 to 6 weeks
• Unexplained persistent sore or painful throat
• Signs and symptoms in the oral cavity persisting for more than 6 weeks, that
cannot be definitively diagnosed as a benign lesion

The level of suspicion should be higher in patients who are over 40, smokers, heavy
drinkers and those who chew tobacco or betel nut (areca nut).

34. Parotid gland swelling


Swelling of this gland is either unilateral or bilateral. Disorders of the gland cause
swelling and pain which may be pronounced on eating or talking. There may be
associated fever and a foul taste in the mouth.

Bilateral causes

• viruses: mumps
• sarcoidosis
• Sjogren's syndrome
• lymphoma
• alcoholic liver disease

Unilateral causes

• tumour: pleomorphic adenomas


• stones
• infection

**In view of a dry cough and bilateral parotid swellings, the patient is likely to have
sarcoidosis. Sarcoidosis is a cause of bilateral parotid swellings.
Parotid tumours are usually unilateral.
Mumps causes painful parotid enlargement and would not cause 6 months of a dry cough.
Lyme disease does not usually cause parotid swellings.
Sjogren's syndrome is a cause of bilateral parotid swellings but does not usually cause
pulmonary symptoms.

35. Gingival hyperplasia

Drug causes of gingival hyperplasia

• phenytoin
• ciclosporin
• calcium channel blockers (especially nifedipine)

Other causes of gingival hyperplasia include

• acute myeloid leukaemia (myelomonocytic and monocytic types)

36. Rinne's and Weber's test

Performing both Rinne's and Weber's test allows differentiation of conductive and
sensorineural deafness.

Rinne's test
• tuning fork is placed over the mastoid process until the sound is no longer
heard, followed by repositioning just over external acoustic meatus
• 'positive test': air conduction (AC) is normally better than bone conduction (BC)
• 'negative test': if BC > AC then conductive deafness

Weber's test

• tuning fork is placed in the middle of the forehead equidistant from the patient's
ears
• the patient is then asked which side is loudest
• in unilateral sensorineural deafness, sound is localised to the unaffected side
• in unilateral conductive deafness, sound is localised to the affected side

Rinne result Weber result


Air conduction > bone conduction
Normal Midline
bilaterally
Bone conduction > air conduction in
Conductive hearing affected ear Lateralises to affected
loss Air conduction > bone conduction in ear
unaffected ear
Sensorineural hearing Air conduction > bone conduction Lateralises to
loss bilaterally unaffected ear

The Rinne and Weber tests are used to distinguish conductive from sensorineural hearing
loss.
This gentleman has wax blocking the right ear canal, so you would expect to find a
conductive hearing loss on the right side.
When performing the Weber test, the patient should localise the sound to the side of a
conductive hearing loss, as bone conduction is increased. The sound will localise away from a
sensorineural hearing loss.
The Rinne test is negative if there is a conductive hearing loss, as bone conduction is better
than air conduction. It is positive if air conduction is better than bone conduction, which can
be the case for mild-moderate sensorineural hearing loss or if there is normal hearing.

In this case, the wax causes a conductive hearing loss on the right side, so when performing
the Weber test sound should localise to the right, and Rinne should be negative on the right
side and positive on the left.

37. Black hairy tongue

Black hairy tongue is relatively common condition which results from defective
desquamation of the filiform papillae. Despite the name the tongue may be brown, green,
pink or another colour.

Predisposing factors

• poor oral hygiene


• antibiotics
• head and neck radiation
• HIV
• intravenous drug use

The tongue should be swabbed to exclude Candida

Management

• tongue scraping
• topical antifungals if Candida

38. Viral labyrinthitis

Labyrinthitis is an inflammatory disorder of the membranous labyrinth, affecting both the


vestibular and cochlear end organs. Labyrinthitis can be viral, bacterial or associated with
systemic diseases. Viral labyrinthitis is the most common form of labyrinthitis.

Labyrinthitis should be distinguished from vestibular neuritis as there are important


differences: vestibular neuritis is used to define cases in which only the vestibular nerve is
involved, hence there is no hearing impairment; Labyrinthitis is used when both the
vestibular nerve and the labyrinth are involved, usually resulting in both vertigo and hearing
impairment.

Epidemiology

• The average age of presentation is 40-70 years

Patients typically present with an acute onset of:

• vertigo: not triggered by movement but exacerbated by movement


• nausea and vomiting
• hearing loss: may be unilateral or bilateral, with varying severity
• tinnitus
• preceding or concurrent symptoms of upper respiratory tract infection

Signs of labyrinthitis:

• spontaneous unidirectional horizontal nystagmus towards the unaffected side


• sensorineural hearing loss: shown by Rinne's test and Weber test
• abnormal head impulse test: signifies an impaired vestibulo-ocular reflex
• gait disturbance: the patient may fall towards the affected side
The diagnosis is largely based on history and examination.

Management

• episodes are usually self-limiting


• prochlorperazine or antihistamines may help reduce the sensation of dizziness

**This condition is viral labyrinthitis, which typically presents as sudden onset horizontal
nystagmus, hearing disturbances, nausea, vomiting and vertigo. Patients will typically present
with symptoms such as a previous ear infection, tinnitus, or previous coryzal symptoms.
Meniere's disease can present in a similar manner, but usually patients will report a feeling of
pressure felt deep inside the ear. Meniere's disease is likely to result in recurring episodes of
symptoms, whilst this is less likely with labyrinthitis.
Migrainous vertigo is unlikely given that this lady reports a very different headache to her
usual migrainous headaches. Additionally other symptoms such as the hearing loss and
horizontal nystagmus also make this less likely.
Otitis media is unlikely given that she has no ear pain.
Benign paroxysmal positional vertigo can cause nausea and vertigo but usually occurs only
for a few seconds and with movement.

39. Chronic rhinosinusitis

Chronic rhinosinusitis affects up to 1 in 10 people. It is generally defined as an inflammatory


disorder of the paranasal sinuses and linings of the nasal passages that lasts 12 weeks or
longer.

Predisposing factors include:

• atopy: hay fever, asthma


• nasal obstruction e.g. Septal deviation or nasal polyps
• recent local infection e.g. Rhinitis or dental extraction
• swimming/diving
• smoking

Features

• facial pain: typically frontal pressure pain which is worse on bending forward
• nasal discharge: usually clear if allergic or vasomotor. Thicker, purulent discharge
suggests secondary infection
• nasal obstruction: e.g. 'mouth breathing'
• post-nasal drip: may produce chronic cough

Management of recurrent or chronic sinusitis

• avoid allergen
• intranasal corticosteroids
• nasal irrigation with saline solution

Red flags symptoms

• unilateral symptoms
• persistent symptoms despite compliance with 3 months of treatment
• epistaxis

**Urgent referral to ENT is recommended if there are unilateral symptoms, unilateral polyps
or blood-stained discharge as this increases suspicious of cancer. This is the case for this
patient.
If there are no red flag symptoms, intranasal corticosteroid spray or drops can be used to
manage chronic sinusitis or nasal polyps. Options include intranasal mometasone furoate,
fluticasone furoate, or fluticasone propionate. Drops may be preferred if there is severe
obstruction.
Intranasal antihistamine spray is licensed for allergic rhinitis.
Oral antibiotics may be considered in acute sinusitis if a person has had symptoms for around
10 days or more with no improvement and there is suspicion of bacterial infection.

40. Epistaxis

Epistaxis is split into anterior and posterior bleeds, whereby the former often has a visible
source of bleeding and usually occurs due to an insult to the network of capillaries that form
Kiesselbach’s Plexus. Posterior haemorrhages, on the other hand, tend to be more profuse and
originate from deeper structures. They occur more frequently in older patients and confer a
higher risk of aspiration and airway compromise.

While most cases of epistaxis tend to be benign and self-limiting, they may be an indicator of
serious pathology. The most common cause is trauma to the nose- this can range from the
insertion of foreign bodies, nose picking and nose blowing. Bleeding can also indicate
platelet function disorders such as thrombocytopenia, splenomegaly, leukaemia,
Waldenstrom’s macroglobulinaemia and ITP. In adolescent males, juvenile angiofibroma is a
benign tumour that may bleed as it is highly vascularised. If the nasal septum looks abraded
or atrophied, inquire about drug use. This is because inhaled cocaine is a powerful
vasoconstrictor and repeated use may result in obliteration of the septum. In the elderly,
hereditary haemorrhagic telangiectasia may cause prolonged nasal bleeding. Granulomatosis
with polyangiitis and pyogenic granuloma may also present with nosebleeds.

Management

If the patient is haemodynamically stable, bleeding can be controlled with first aid measures.
This involves:

• Asking the patient to sit with their torso forward and their mouth open- avoid lying
down unless they feel faint. This decreases blood flow to the nasopharynx and allows
the patient to spit out any blood in their mouth. It also reduces the risk of aspirating
blood.
• Pinch the cartilaginous (soft) area of the nose firmly and consistently for at least 20
minutes and ask the patient to breathe through their mouth.

If first aid measures are successful, consider using a topical antiseptic such as Naseptin
(chlorhexidine and neomycin) to reduce crusting and the risk of vestibulitis. Cautions to this
include patients that have peanut, soy or neomycin allergies, and Mupirocin is a viable
alternative.

Admission and follow up care may be considered in patients under if a comorbidity (e.g.
coronary artery disease, or severe hypertension) is present, an underlying cause is suspected
or if they are aged under 2 years (as underlying causes such as haemophilia or leukaemia are
more likely in this age group).

If bleeding does not stop after 10-15 minutes of continuous pressure on the nose, consider
cautery or packing. Cautery should be used initially if the source of the bleed is visible and
cautery is tolerated- it is not so well-tolerated in younger children! Packing may be used if
cautery is not viable or the bleeding point cannot be visualised. If the nose is packed in
primary care, the patient should be admitted to hospital for review.

Cautery:

• Ask the patient to blow their nose in order to remove any clots. Be wary that bleeding
may resume.
• Use a topical local anaesthetic spray (e.g. Co-phenylcaine) and wait 3-4 minutes for it to
take effect
• Identify the bleeding point and apply the silver nitrate stick for 3-10 seconds until it
becomes grey-white. Avoid touching areas which do not require treatment, and only
cauterise one side of the septum as there is a risk of perforation.
• Dab the area clean with a cotton bud and apply Naseptin or Muciprocin

Packing:

• Anaesthetise with topical local anaesthetic spray (e.g. Co-phenylcaine) and wait for 3-4
minutes
• Pack the patient’s nose while they are sitting with their head forward, following the
manufacturer’s instructions
• Pressure on the cartilage around the nostril can cause cosmetic changes and this should
be reviewed after inserting the pack.
• Examine the patient’s mouth and throat for any continuing bleeding, and consider
packing the other nostril as this increases pressure on the septum and offending vessel.
• Patients should be admitted to hospital for observation and review, and to ENT if
available

Patients that are haemodynamically unstable or compromised should be admitted to the


emergency department- control bleeding with first aid measures in the interim. Patients with a
bleed from an unknown or posterior source (i.e. the bleeding site cannot be located on
speculum, bleeding from both nostrils or profuse) should be admitted to hospital.

Epistaxis that has failed all emergency management may require sphenopalatine ligation in
theatre
Self-care advice involves reducing the risk of re-bleeding. Patients should be informed that
blowing or picking the nose, heavy lifting, exercise, lying flat, drinking alcohol or hot drinks
should be avoided. The same applies for patients who have just been cauterised, as any strain
on the nostril may induce a re-bleed.
**Emergency management of epistaxis includes the following:

• adequate first aid for 20 minutes (squeeze both nasal ala firmly and sit forward. Ice in
the mouth can help)
• topical adrenaline/local anaesthetic
• topical tranexamic acid
• nasal packing (e.g. with Rapid Rhino. Initially insert into the affected nostril. If
unsuccessful, a pack in the other nostril may help. Posterior bleeds can be packed with a
posterior pack, or with a Foley catheter).
• surgical intervention (sphenopalatine artery ligation).

41. Geographic tongue

Geographic tongue is a benign, chronic condition of unknown cause. It is present in around 1-


3% of the population and is more common in females.

Features

• erythematous areas with a white-grey border (the irregular, smooth red areas are said to
look like the outline of a map)
• some patients report burning after eating certain food

Management

• reassurance about benign nature

42. Macroglossia

Causes

• hypothyroidism
• acromegaly
• amyloidosis
• Duchenne muscular dystrophy
• mucopolysaccharidosis (e.g. Hurler syndrome)

Patients with Down's syndrome are now thought to have apparent macroglossia due to a
combination of mid-face hypoplasia and hypotonia

43. Neck lumps


The table below gives characteristic exam question features for conditions causing neck
lumps:

Condition Notes
Reactive By far the most common cause of neck swellings. There may be a
lymphadenopathy history of local infection or a generalised viral illness
Rubbery, painless lymphadenopathy
Lymphoma The phenomenon of pain whilst drinking alcohol is very uncommon
There may be associated night sweats and splenomegaly
May be hypo-, eu- or hyperthyroid symptomatically
Thyroid swelling
Moves upwards on swallowing

More common in patients < 20 years old


Usually midline, between the isthmus of the thyroid and the hyoid bone
Thyroglossal cyst
Moves upwards with protrusion of the tongue
May be painful if infected

More common in older men


Represents a posteromedial herniation between thyropharyngeus and
cricopharyngeus muscles
Pharyngeal pouch Usually not seen but if large then a midline lump in the neck that gurgles
on palpation
Typical symptoms are dysphagia, regurgitation, aspiration and chronic
cough

A congenital lymphatic lesion (lymphangioma) typically found in the


Cystic hygroma neck, classically on the left side
Most are evident at birth, around 90% present before 2 years of age

An oval, mobile cystic mass that develops between the


sternocleidomastoid muscle and the pharynx
Branchial cyst Develop due to failure of obliteration of the second branchial cleft in
embryonic development
Usually present in early adulthood

More common in adult females


Cervical rib
Around 10% develop thoracic outlet syndrome
Carotid aneurysm Pulsatile lateral neck mass which doesn't move on swallowing
DERMATOLOGY
1.Chondrodermatitis nodularis helicis

Chondrodermatitis nodularis helicis (CNH) is a common and benign condition characterised by the
development of a painful nodule on the ear. It is thought to be caused by factors such as persistent
pressure on the ear (e.g. secondary to sleep, headsets), trauma or cold. CNH is more common in men
and with increasing age.

Management
• reducing pressure on the ear: foam 'ear protectors' may be used during sleep
• other treatment options include cryotherapy, steroid injection, collagen injection
• surgical treatment may be used but there is a high recurrence rate

2. Strawberry naevus

Strawberry naevi (capillary haemangioma) are usually not present at birth but may develop rapidly in
the first month of life. They appear as erythematous, raised and multilobed tumours.

Typically they increase in size until around 6-9 months before regressing over the next few years
(around 95% resolve before 10 years of age).

Common sites include the face, scalp and back. Rarely they may be present in the upper respiratory
tract leading to potential airway obstruction

Capillary haemangiomas are present in around 10% of white infants. Female infants, premature
infants and those of mothers who have undergone chorionic villous sampling are more likely to be
affected

Potential complications
• mechanical e.g. Obstructing visual fields or airway
• bleeding
• ulceration
• thrombocytopaenia

If treatment is required (e.g. Visual field obstruction) then propranolol is increasingly replacing
systemic steroids as the treatment of choice. Topical beta-blockers such as timolol are also sometimes
used.

Cavernous haemangioma is a deep capillary haemangioma.

Capillary haemangioma: These appear as a small red patch which develops in the first month of life,
increasing in size until around 9 months and becoming more vascular. They are not present at birth
and regress spontaneously. Parents should be reassured that no treatment is needed and there is no
sinister cause. The other answers by contrast do not develop and worsen over time but are present at
birth.
Port wine stain and naevus flammeus are the same condition

A strawberry naevus is a type of congenital haemangioma occurring in around one in 20 babies. They
tend to grown rapidly over the first few months of life then spontaneously regress over the course of a
few years. They generally do not require any treatment unless they are large or impairing vision,
hearing, breathing or feeding. Further investigation may be considered if they are large, atypical, or
sited over the lower spine (where they may indicate spina bifida).

3. Alopecia

Alopecia may be divided into scarring (destruction of hair follicle) and non-scarring (preservation of
hair follicle)

Scarring alopecia
• trauma, burns
• radiotherapy
• lichen planus
• discoid lupus
• tinea capitis*

Non-scarring alopecia
• male-pattern baldness
• drugs: cytotoxic drugs, carbimazole, heparin, oral contraceptive pill, colchicine
• nutritional: iron and zinc deficiency
• autoimmune: alopecia areata
• telogen effluvium
o hair loss following stressful period e.g. surgery
• trichotillomania

*scarring may develop in untreated tinea capitis if a kerion develops

This patient has had a physical stress in the last few months (childbirth) which is consistent with her
developing thin hair due to telogen effluvium.

The patterns of hair loss do not fit with alopecia areata, which causes well-circumscribed areas of total
hair loss, or alopecia totalis, which causes complete loss of all hair of the head and face.

Trichotillomania, which describes a disorder where people pull their own hair out, would give
asymmetrical, uneven hair loss, and might be preceded by other psychiatric complaints.

Androgenetic alopecia, which in this context would be female-pattern baldness, would be suggested
by a family history of the same, and is more likely in an older patient.

4. Venous ulceration

Venous ulceration is typically seen above the medial malleolus

Investigations
• ankle-brachial pressure index (ABPI) is important in non-healing ulcers to assess for poor
arterial flow which could impair healing
• a 'normal' ABPI may be regarded as between 0.9 - 1.2. Values below 0.9 indicate arterial
disease. Interestingly, values above 1.3 may also indicate arterial disease, in the form of false-
negative results secondary to arterial calcification (e.g. In diabetics)

Management
• compression bandaging, usually four layer (only treatment shown to be of real benefit)
• oral pentoxifylline, a peripheral vasodilator, improves healing rate
• small evidence base supporting use of flavinoids
• little evidence to suggest benefit from hydrocolloid dressings, topical growth factors, ultrasound
therapy and intermittent pneumatic compression

5. Hyperhidrosis

Hyperhidrosis describes the excessive production of sweat.

Management options include


• topical aluminium chloride preparations are first-line. Main side effect is skin irritation
• iontophoresis: particularly useful for patients with palmar, plantar and axillary hyperhidrosis
• botulinum toxin: currently licensed for axillary symptoms
• surgery: e.g. Endoscopic transthoracic sympathectomy. Patients should be made aware of the
risk of compensatory sweating
This patient has hyperhidrosis. The first line management of this condition is aluminium chloride
which can be given in the form of roll-ons applied at nighttime. Underlying anxiety should also be
treated.

Botulinum toxin injections and topical glycopyrrolate (an antimuscarinic agent) can be used in
secondary care for the management of hyperhidrosis.

Steroid medication such as Clobetasol propionate 0.05% and calcium channel blockers such as
nifedipine are not used in the treatment of hyperhidrosis

6. Skin disorders associated with diabetes

Note whilst pyoderma gangrenosum can occur in diabetes mellitus it is rare and is often not included
in a differential of potential causes

Necrobiosis lipoidica
• shiny, painless areas of yellow/red/brown skin typically on the shin
• often associated with surrounding telangiectasia

Infection
• candidiasis
• staphylococcal

Neuropathic ulcers

Vitiligo

Lipoatrophy

Granuloma annulare*
• papular lesions that are often slightly hyperpigmented and depressed centrally

*it is not clear from recent studies if there is actually a significant association between diabetes
mellitus and granuloma annulare, but it is often listed in major textbooks.
Sweet's syndrome is also known as acute febrile neutrophilic dermatosis has a strong association with
acute myeloid leukaemia. It is not associated with diabetes mellitus
7. Hereditary haemorrhagic telangiectasia

Also known as Osler-Weber-Rendu syndrome, hereditary haemorrhagic telangiectasia (HHT) is an


autosomal dominant condition characterised by (as the name suggests) multiple telangiectasia over the
skin and mucous membranes. Twenty percent of cases occur spontaneously without prior family
history.

There are 4 main diagnostic criteria. If the patient has 2 then they are said to have a possible diagnosis
of HHT. If they meet 3 or more of the criteria they are said to have a definite diagnosis of HHT:

• epistaxis : spontaneous, recurrent nosebleeds


• telangiectases: multiple at characteristic sites (lips, oral cavity, fingers, nose)
• visceral lesions: for example gastrointestinal telangiectasia (with or without bleeding),
pulmonary arteriovenous malformations (AVM), hepatic AVM, cerebral AVM, spinal AVM
• family history: a first-degree relative with HHT

8. Keratoacanthoma

Keratoacanthoma is a benign epithelial tumour. They are more common with advancing age and rare
in young people.

Features - said to look like a volcano or crater

• initially a smooth dome-shaped papule


• rapidly grows to become a crater centrally-filled with keratin

Spontaneous regression of keratoacanthoma within 3 months is common, often resulting in a scar.


Such lesions should however be urgently excised as it is difficult clinically to exclude squamous cell
carcinoma. Removal also may prevent scarring.

9. Dermatitis herpetiformis

Dermatitis herpetiformis is an autoimmune blistering skin disorder associated with coeliac disease. It
is caused by deposition of IgA in the dermis.

Features
• itchy, vesicular skin lesions on the extensor surfaces (e.g. elbows, knees, buttocks)

Diagnosis
• skin biopsy: direct immunofluorescence shows deposition of IgA in a granular pattern in the
upper dermis

Management
• gluten-free diet
• dapsone
Dermatitis herpetiformis, a rash strongly associated with coeliac disease. This is a chronic blistering
skin condition that despite its name is neither caused by or associated with infection by the herpes
virus. It is worth knowing that Dermatitis herpetiformis responds well to dapsone (another common
exam question). It also responds well to a gluten free diet.
10. Zinc deficiency

Features
• perioral dermatitis: red, crusted lesions
• acrodermatitis
• alopecia
• short stature
• hypogonadism
• hepatosplenomegaly
• geophagia (ingesting clay/soil)
• cognitive impairment
• angular cheiliosis (also caused by vit B12 def)
11. Alopecia

Alopecia may be divided into scarring (destruction of hair follicle) and non-scarring (preservation of
hair follicle)

Scarring alopecia
• trauma, burns
• radiotherapy
• lichen planus
• discoid lupus
• tinea capitis*

Non-scarring alopecia
• male-pattern baldness
• drugs: cytotoxic drugs, carbimazole, heparin, oral contraceptive pill, colchicine
• nutritional: iron and zinc deficiency
• autoimmune: alopecia areata
• telogen effluvium
o hair loss following stressful period e.g. surgery
• trichotillomania

*scarring may develop in untreated tinea capitis if a kerion develops

12. Pyogenic granuloma

Pyogenic granuloma is a relatively common benign skin lesion. The name is confusing as they are
neither true granulomas nor pyogenic in nature. There are multiple alternative names but perhaps
'eruptive haemangioma' is the most useful.

The cause of pyogenic granuloma is not known but a number of factors are linked:
• trauma
• pregnancy
• more common in women and young adults

Features
• most common sites are head/neck, upper trunk and hands. Lesions in the oral mucosa are
common in pregnancy
• initially small red/brown spot
• rapidly progress within days to weeks forming raised, red/brown lesions which are often
spherical in shape
• the lesions may bleed profusely or ulcerate

Management
• lesions associated with pregnancy often resolve spontaneously post-partum
• other lesions usually persist. Removal methods include curettage and cauterisation, cryotherapy,
excision
13. Shin lesions

The differential diagnosis of shin lesions includes the following conditions:


• erythema nodosum
• pretibial myxoedema
• pyoderma gangrenosum
• necrobiosis lipoidica diabeticorum

Below are the characteristic features:

Erythema nodosum
• symmetrical, erythematous, tender, nodules which heal without scarring
• most common causes are streptococcal infections, sarcoidosis, inflammatory bowel disease and
drugs (penicillins, sulphonamides, oral contraceptive pill)

Pretibial myxoedema
• symmetrical, erythematous lesions seen in Graves' disease
• shiny, orange peel skin

Pyoderma gangrenosum
• initially small red papule
• later deep, red, necrotic ulcers with a violaceous border
• idiopathic in 50%, may also be seen in inflammatory bowel disease, connective tissue disorders
and myeloproliferative disorders

Necrobiosis lipoidica diabeticorum


• shiny, painless areas of yellow/red skin typically on the shin of diabetics
• often associated with telangiectasia

14. Molluscum contagiosum

Molluscum contagiosum is a common skin infection caused by molluscum contagiosum virus (MCV),
a member of the Poxviridae family. Transmission occurs directly by close personal contact, or
indirectly via fomites (contaminated surfaces) such as shared towels and flannels. The majority of
cases occur in children (often in children with atopic eczema), with the maximum incidence in
preschool children aged 1-4 years.

Typically, molluscum contagiosum presents with characteristic pinkish or pearly white papules with a
central umbilication, which are up to 5 mm in diameter. Lesions appear in clusters in areas anywhere
on the body (except the palms of the hands and the soles of the feet). In children, lesions are
commonly seen on the trunk and in flexures, but anogenital lesions may also occur. In adults, sexual
contact may lead to lesions developing on the genitalia, pubis, thighs, and lower abdomen. Rarely,
lesions can occur on the oral mucosa and on the eyelids.

Self-care advice:
• Reassure people that molluscum contagiosum is a self-limiting condition.
• Spontaneous resolution usually occurs within 18 months
• Explain that lesions are contagious, and it is sensible to avoid sharing towels, clothing, and baths
with uninfected people (e.g. siblings)
• Encourage people not to scratch the lesions. If it is problematic, consider treatment to alleviate
the itch
• Exclusion from school, gym, or swimming is not necessary

Treatment is not usually recommended. If lesions are troublesome or considered unsightly, use simple
trauma or cryotherapy, depending on the parents' wishes and the child's age:
• Squeezing (with fingernails) or piercing (orange stick) lesions may be tried, following a bath.
Treatment should be limited to a few lesions at one time
• Cryotherapy may be used in older children or adults, if the healthcare professional is
experienced in the procedure
• Eczema or inflammation can develop around lesions prior to resolution. Treatment may be
required if:
o Itching is problematic; prescribe an emollient and a mild topical corticosteroid (e.g.
hydrocortisone 1%)
o The skin looks infected (e.g. oedema, crusting); prescribe a topical antibiotic (e.g.
fusidic acid 2%)

Referral may be necessary in some circumstances:


• For people who are HIV-positive with extensive lesions urgent referral to a HIV specialist
• For people with eyelid-margin or ocular lesions and associated red eye urgent referral to an
ophthalmologist
• Adults with anogenital lesions should be referred to genito-urinary medicine, for screening for
other sexually transmitted infections

15. Port wine stains

Port wine stains are vascular birthmarks that tend to be unilateral. They are deep red or purple in
colour. Unlike other vascular birthmarks such as salmon patches and strawberry haemangiomas, they
do not spontaneously resolve, and in fact often darken and become raised over time. Treatment is with
cosmetic camouflage or laser therapy (multiple sessions are required).

The correct answer here is port wine stain. These typically occur on the face as a purplish/red macule
with irregular contours. They can be associated with intracranial vascular abnormalities like Sturge-
Weber-Syndrome. They do not spontaneously resolve and can become darker or lumpy in later life.
Treatment is with cosmetics or laser therapy.

Strawberry naevus or capillary haemangioma appear as a small red patch which develops in the first
month of life, increasing in size until around 9 months and becoming more vascular. They are not
present at birth and regress spontaneously.

Salmon patches or stork marks are pink telangiectatic macules present at birth and commonly found
on the forehead or back of neck. They do not require treatment and often self-resolve. They are not
associated with underlying conditions.
Mongolian blue spots are areas of bluish discolouration over the lower back and buttock which often
disappear by 1 year of age. They should be documented and highlighted to parents as they can be
mistake for bruising.

Melanocytic naevi are raised brown/black nodules which can be hairy and up to 20cm in diameter.
There is a risk of developing melanomas from these and so they should be closely monitored.

16, Seborrhoeic dermatitis in adults

Seborrhoeic dermatitis in adults is a chronic dermatitis thought to be caused by an inflammatory


reaction related to a proliferation of a normal skin inhabitant, a fungus called Malassezia furfur
(formerly known as Pityrosporum ovale). It is common, affecting around 2% of the general
population.

Features
• eczematous lesions on the sebum-rich areas: scalp (may cause dandruff), periorbital, auricular
and nasolabial folds
• otitis externa and blepharitis may develop

Associated conditions include


• HIV
• Parkinson's disease

Scalp disease management


• over the counter preparations containing zinc pyrithione ('Head & Shoulders') and tar
('Neutrogena T/Gel') are first-line
• the preferred second-line agent is ketoconazole
• selenium sulphide and topical corticosteroid may also be useful

Face and body management


• topical antifungals: e.g. ketoconazole
• topical steroids: best used for short periods
• difficult to treat - recurrences are common
17. Scabies

Scabies is caused by the mite Sarcoptes scabiei and is spread by prolonged skin contact. It typically
affects children and young adults.

The scabies mite burrows into the skin, laying its eggs in the stratum corneum. The intense pruritus
associated with scabies is due to a delayed-type IV hypersensitivity reaction to mites/eggs which
occurs about 30 days after the initial infection.

Features

• widespread pruritus
• linear burrows on the side of fingers, interdigital webs and flexor aspects of the wrist
• in infants, the face and scalp may also be affected
• secondary features are seen due to scratching: excoriation, infection
Management
• permethrin 5% is first-line
• malathion 0.5% is second-line
• give appropriate guidance on use (see below)
• pruritus persists for up to 4-6 weeks post eradication
Patient guidance on treatment (from Clinical Knowledge Summaries)
• avoid close physical contact with others until treatment is complete
• all household and close physical contacts should be treated at the same time, even if
asymptomatic
• launder, iron or tumble dry clothing, bedding, towels, etc., on the first day of treatment to kill off
mites.

The BNF advises to apply the insecticide to all areas, including the face and scalp, contrary to the
manufacturer's recommendation. Patients should be given the following instructions:
• apply the insecticide cream or liquid to cool, dry skin
• pay close attention to areas between fingers and toes, under nails, armpit area, creases of the
skin such as at the wrist and elbow
• allow to dry and leave on the skin for 8-12 hours for permethrin, or for 24 hours for malathion,
before washing off
• reapply if insecticide is removed during the treatment period, e.g. If wash hands, change nappy,
etc
• repeat treatment 7 days later

Crusted (Norwegian) scabies


Crusted scabies is seen in patients with suppressed immunity, especially HIV.
The crusted skin will be teeming with hundreds of thousands of organisms.

Ivermectin is the treatment of choice and isolation is essential

18. Acne vulgaris: management

Acne vulgaris is a common skin disorder which usually occurs in adolescence. It typically affects the
face, neck and upper trunk and is characterised by the obstruction of the pilosebaceous follicles with
keratin plugs which results in comedones, inflammation and pustules.

Acne may be classified into mild, moderate or severe:


• mild: open and closed comedones with or without sparse inflammatory lesions
• moderate acne: widespread non-inflammatory lesions and numerous papules and pustules
• severe acne: extensive inflammatory lesions, which may include nodules, pitting, and scarring

A simple step-up management scheme often used in the treatment of acne is as follows:
• single topical therapy (topical retinoids, benzoyl peroxide)
• topical combination therapy (topical antibiotic, benzoyl peroxide, topical retinoid)
• oral antibiotics:
o tetracyclines: lymecycline, oxytetracycline, doxycycline
o tetracyclines should be avoided in pregnant or breastfeeding women and in children
younger than 12 years of age
o erythromycin may be used in pregnancy
o minocycline is now considered less appropriate due to the possibility of irreversible
pigmentation
o a single oral antibiotic for acne vulgaris should be used for a maximum of three months
o a topical retinoid (if not contraindicated) or benzoyl peroxide should always be co-
prescribed with oral antibiotics to reduce the risk of antibiotic resistance developing.
Topical and oral antibiotics should not be used in combination
o Gram-negative folliculitis may occur as a complication of long-term antibiotic use -
high-dose oral trimethoprim is effective if this occurs
• combined oral contraceptives (COCP) are an alternative to oral antibiotics in women
o as with antibiotics, they should be used in combination with topical agents
o Dianette (co-cyrindiol) is sometimes used as it has anti-androgen properties. However, it
has an increased risk of venous thromboembolism compared to other COCPs, therefore
it should generally be used second-line, only be given for 3 months and women should
be appropriately counselled about the risks
• oral isotretinoin: only under specialist supervision
o pregnancy is a contraindication to topical and oral retinoid treatment
# Patients with scarring should be referred for oral retinoin

There is no role for dietary modification in patients with acne.


This scenario assesses your understanding of the treatment of severe acne: defined by NICE as
predominantly papular/pustular with nodules and scarring. Ultimately this patient requires specialist
dermatology assessment and treatment possibly with oral isotretinoin. In the mean time, you should
prescribe both an oral antibiotic and topical retinoid or benzoyl peroxide.

The guidelines below are quoted from http://cks.nice.org.uk/acne-vulgaris#!scenario:2

In severe acne, there are nodules and cysts (nodulocystic acne), as well as a preponderance of
inflammatory papules and pustules. There is a high risk of scarring (or scarring may already be
evident), and there is likely to be considerable psychosocial morbidity.
Refer all people with severe acne for specialist assessment and treatment (for example with oral
isotretinoin).

Consider prescribing an oral antibiotic in combination with a topical drug whilst waiting for an
appointment. Benzoyl peroxide or a topical retinoid are recommended as adjunctive treatment for
most people. Azelaic acid is an alternative, but avoid the use of topical antibiotics with oral
antibiotics.

Oral tetracycline, oxytetracycline, doxycycline, or lymecycline are first-line antibiotic options.


Erythromycin is an alternative if tetracyclines are poorly tolerated or contraindicated (such as in
pregnancy). Minocycline is not recommended.

Do not prescribe an antibiotic alone.

Do not combine a topical and an oral antibiotic.

Consider prescribing a combined oral contraceptive (COC) in women who require contraception.

For most women, a 'standard' COC is suitable.

Consider co-cyprindiol (Dianette®) only when topical treatment or systemic antibiotics has failed.
However, co-cyprindiol is not licensed for the sole purpose of contraception and should be
discontinued three to four menstrual cycles after the woman's acne has resolved.

19. Hirsutism and hypertrichosis

Hirsutism is often used to describe androgen-dependent hair growth in women, with hypertrichosis
being used for androgen-independent hair growth

Polycystic ovarian syndrome is the most common causes of hirsutism. Other causes include:
• Cushing's syndrome
• congenital adrenal hyperplasia
• androgen therapy
• obesity: thought to be due to insulin resistance
• adrenal tumour
• androgen secreting ovarian tumour
• drugs: phenytoin, corticosteroids

Assessment of hirsutism
• Ferriman-Gallwey scoring system: 9 body areas are assigned a score of 0 - 4, a score > 15 is
considered to indicate moderate or severe hirsutism

Management of hirsutism
• advise weight loss if overweight
• cosmetic techniques such as waxing/bleaching - not available on the NHS
• consider using combined oral contraceptive pills such as co-cyprindiol (Dianette) or
ethinylestradiol and drospirenone (Yasmin). Co-cyprindiol should not be used long-term due to
the increased risk of venous thromboembolism
• facial hirsutism: topical eflornithine - contraindicated in pregnancy and breast-feeding

Causes of hypertrichosis
• drugs: minoxidil, ciclosporin, diazoxide
• congenital hypertrichosis lanuginosa, congenital hypertrichosis terminalis
• porphyria cutanea tarda
• anorexia nervosa
20. Livedo reticularis

Livedo reticularis describes an purplish, non-blanching, reticulated rash caused by obstruction of the
capillaries resulting in swollen venules.

Causes

• idiopathic (most common)


• polyarteritis nodosa
• systemic lupus erythematosus
• cryoglobulinaemia
• antiphospholipid syndrome
• Ehlers-Danlos Syndrome
• homocystinuria

Erythema ab igne can give a lace-like appearance, but usually it is redder in colour. It is caused by
heat exposure and is more common in areas that are directly exposed to hot objects, such as the
abdomen or back after using hot water bottles for prolonged periods. Therefore, the distribution of this
rash and the history of worsening symptoms in the cold are not suggestive of erythema ab igne.

Raynaud's phenomenon is precipitated by cold weather but causes symptoms in the more distal
extremities. The appearance is of very pale, blanched skin, then becoming erythematous once blood
supply is restored.

The description of the lesions is not consistent with erythema nodosum. This gives painful,
erythematous nodules that often occur on both shins.

Lupus panniculitis causes tender, firm nodules on the face, shoulders, arms, breasts and buttocks.
21. Impetigo

Impetigo is a superficial bacterial skin infection usually caused by either Staphylcoccus aureus or
Streptococcus pyogenes. It can be a primary infection or a complication of an existing skin condition
such as eczema (in this case), scabies or insect bites. Impetigo is common in children, particularly
during warm weather.

The infection can develop anywhere on the body but lesions tend to occur on the face, flexures and
limbs not covered by clothing.

Spread is by direct contact with discharges from the scabs of an infected person. The bacteria invade
the skin through minor abrasions and then spread to other sites by scratching. Infection is spread
mainly by the hands, but indirect spread via toys, clothing, equipment and the environment may
occur. The incubation period is between 4 to 10 days.

Features
• 'golden', crusted skin lesions typically found around the mouth
• very contagious

Management: Limited, localised disease


• NICE Clinical Knowledge Summaries now recommend hydrogen peroxide 1% cream for
'people who are not systemically unwell or at a high risk of complications'
o the change was announced in 2020 by NICE and Public Health England and seems
aimed at cutting antibiotic resistance
o the evidence base shows it is just as effective at treating non-bullous impetigo as a
topical antibiotic
• topical antibiotic creams:
o topical fusidic acid
o topical mupirocin should be used if fusidic acid resistance is suspected
o MRSA is not susceptible to either fusidic acid or retapamulin. Topical mupirocin
should, therefore, be used in this situation

Extensive disease
• oral flucloxacillin
• oral erythromycin if penicillin-allergic

School exclusion
• children should be excluded from school until the lesions are crusted and healed or 48 hours
after commencing antibiotic treatment

22. Eczema herpeticum

Eczema herpeticum describes a severe primary infection of the skin by herpes simplex virus 1 or 2.

It is more commonly seen in children with atopic eczema and often presents as a rapidly progressing
painful rash.

On examination, monomorphic punched-out erosions (circular, depressed, ulcerated lesions) usually


1–3 mm in diameter are typically seen.

As it is potentially life-threatening children should be admitted for IV aciclovir.


This child has eczema herpeticum which is a medical emergency. This child needs urgent admission
and treatment with antivirals.

Eczema herpeticum usually begins with a cluster of itchy and painful blisters. It can affect any site but
is most often seen on the face and neck. Blisters can occur in normal skin or on skin that is affected by
atopic dermatitis or another skin disease. The patient is unwell, has swollen lymph nodes and a fever.

23. Vitiligo

Vitiligo is an autoimmune condition which results in the loss of melanocytes and consequent
depigmentation of the skin. It is thought to affect around 1% of the population and symptoms
typically develop by the age of 20-30 years.

Features
• well-demarcated patches of depigmented skin
• the peripheries tend to be most affected
• trauma may precipitate new lesions (Koebner phenomenon)

Associated conditions
• type 1 diabetes mellitus
• Addison's disease
• autoimmune thyroid disorders
• pernicious anaemia
• alopecia areata

Management
• sunblock for affected areas of skin
• camouflage make-up
• topical corticosteroids may reverse the changes if applied early
• there may also be a role for topical tacrolimus and phototherapy, although caution needs to be
exercised with light-skinned patients
24. Granuloma annulare

Basics

• papular lesions that are often slightly hyperpigmented and depressed centrally
• typically occur on the dorsal surfaces of the hands and feet, and on the extensor aspects of the
arms and legs

A number of associations have been proposed to conditions such as diabetes mellitus but there is only
weak evidence for this

25. Malignant melanoma: prognostic factors

The invasion depth of a tumour (Breslow depth) is the single most important factor in determining
prognosis of patients with malignant melanoma

Breslow Thickness Approximate 5 year survival


< 0.75 mm 95-100%
0.76 - 1.50 mm 80-96%
Breslow Thickness Approximate 5 year survival
1.51 - 4 mm 60-75%
> 4 mm 50%

26. Pruritus

The table below lists the main characteristics of the most important causes of pruritus

Condition Notes
History of alcohol excess
Stigmata of chronic liver disease: spider naevi, bruising, palmar erythema,
Liver disease
gynaecomastia etc
Evidence of decompensation: ascites, jaundice, encephalopathy
Pallor
Iron deficiency
Other signs: koilonychia, atrophic glossitis, post-cricoid webs, angular
anaemia
stomatitis
Pruritus particularly after warm bath
'Ruddy complexion'
Polycythaemia
Gout
Peptic ulcer disease
Lethargy & pallor
Chronic kidney
Oedema & weight gain
disease
Hypertension
Night sweats
Lymphadenopathy
Lymphoma
Splenomegaly, hepatomegaly
Fatigue

Other causes:
• hyper- and hypothyroidism
• diabetes
• pregnancy
• 'senile' pruritus
• urticaria
• skin disorders: eczema, scabies, psoriasis, pityriasis rosea
27. Contact dermatitis

There are two main types of contact dermatitis

• irritant contact dermatitis: common - non-allergic reaction due to weak acids or alkalis (e.g.
detergents). Often seen on the hands. Erythema is typical, crusting and vesicles are rare
• allergic contact dermatitis: type IV hypersensitivity reaction. Uncommon - often seen on the
head following hair dyes. Presents as an acute weeping eczema which predominately affects
the margins of the hairline rather than the hairy scalp itself. Topical treatment with a potent
steroid is indicated

Cement is a frequent cause of contact dermatitis. The alkaline nature of cement may cause an irritant
contact dermatitis whilst the dichromates in cement also can cause an allergic contact dermatitis
Contact dermatitis may be irritant or allergic in nature. Patch testing is the investigation of choice for
suspected allergic contact dermatitis. Various allergens are applied to the patient's back, and the skin
assessed at 48 hours and 7 days for any reaction. Standard batteries of allergens are used, in addition
to samples of any substances the patient suspects.

28. Lichen sclerosus

Lichen sclerosus was previously termed lichen sclerosus et atrophicus. It is an inflammatory condition
which usually affects the genitalia and is more common in elderly females. Lichen sclerosus leads to
atrophy of the epidermis with white plaques forming

Features
• itch is prominent

The diagnosis is usually made on clinical grounds but a biopsy may be performed if atypical features
are present*

Management
• topical steroids and emollients

Follow-up:
• increased risk of vulval cancer

*the RCOG advise the following

Skin biopsy is not necessary when a diagnosis can be made on clinical examination. Biopsy is
required if the woman fails to respond to treatment or there is clinical suspicion of VIN or cancer.

and the British Association of Dermatologists state the following:

A confirmatory biopsy, although ideal, is not always practical, particularly in children. It is not
always essential when the clinical features are typical. However, histological examination is
advisable if there are atypical features or diagnostic uncertainty and is mandatory if there is any
suspicion of neoplastic
change. Patients under routine follow-up will need a biopsy if:
• (i) there is a suspicion of neoplastic change, i.e. a persistent area of hyperkeratosis, erosion or
erythema, or new warty or papular lesions;
• (ii) the disease fails to respond to adequate treatment;
• (iii) there is extragenital LS, with features suggesting an overlap with morphoea;
• (iv) there are pigmented areas, in order to exclude an abnormal melanocytic proliferation; and
• (v) second-line therapy is to be used.

Lichen
• planus: purple, pruritic, papular, polygonal rash on flexor surfaces. Wickham's striae over
surface. Oral involvement common
• sclerosus: itchy white spots typically seen on the vulva of elderly women

29.Acanthosis nigricans
Describes symmetrical, brown, velvety plaques that are often found on the neck,
axilla and groin.

Causes
• type 2 diabetes mellitus
• gastrointestinal cancer
• obesity
• polycystic ovarian syndrome
• acromegaly
• Cushing's disease
• hypothyroidism
• familial
• Prader-Willi syndrome
• drugs
o combined oral contraceptive pill
o nicotinic acid

Pathophysiology
• insulin resistance → hyperinsulinemia → stimulation of keratinocytes and dermal
fibroblast proliferation via interaction with insulin-like growth factor receptor-1
(IGFR1)

30. Hirsutism and hypertrichosis

Hirsutism is often used to describe androgen-dependent hair growth in women, with


hypertrichosis being used for androgen-independent hair growth

Polycystic ovarian syndrome is the most common causes of hirsutism. Other causes
include:

• Cushing's syndrome
• congenital adrenal hyperplasia
• androgen therapy
• obesity: thought to be due to insulin resistance
• adrenal tumour
• androgen secreting ovarian tumour
• drugs: phenytoin, corticosteroids

Assessment of hirsutism

• Ferriman-Gallwey scoring system: 9 body areas are assigned a score of 0 - 4, a


score > 15 is considered to indicate moderate or severe hirsutism
Management of hirsutism

• advise weight loss if overweight


• cosmetic techniques such as waxing/bleaching - not available on the NHS
• consider using combined oral contraceptive pills such as co-cyprindiol (Dianette)
or ethinylestradiol and drospirenone (Yasmin). Co-cyprindiol should not be used
long-term due to the increased risk of venous thromboembolism
• facial hirsutism: topical eflornithine - contraindicated in pregnancy and breast-
feeding

Causes of hypertrichosis

• drugs: minoxidil, ciclosporin, diazoxide


• congenital hypertrichosis lanuginosa, congenital hypertrichosis terminalis
• porphyria cutanea tarda
• anorexia nervosa

31. Psoriasis: management

NICE released guidelines in 2012 on the management of psoriasis and psoriatic


arthropathy. Please see the link for more details.

Chronic plaque psoriasis management

NICE recommend a step-wise approach for chronic plaque psoriasis

• regular emollients may help to reduce scale loss and reduce pruritus
• first-line: NICE recommend:
o a potent corticosteroid applied once daily plus vitamin D analogue
applied once daily
o should be applied separately, one in the morning and the other in the
evening)
o for up to 4 weeks as initial treatment
• second-line: if no improvement after 8 weeks then offer:
o a vitamin D analogue twice daily
• third-line: if no improvement after 8-12 weeks then offer either:
o a potent corticosteroid applied twice daily for up to 4 weeks, or
o a coal tar preparation applied once or twice daily
• short-acting dithranol can also be used

Secondary care management:


Phototherapy:

narrowband ultraviolet B light is now the treatment of choice. If possible this should
be given 3 times a week

• photochemotherapy is also used - psoralen + ultraviolet A light (PUVA)


• adverse effects: skin ageing, squamous cell cancer (not melanoma)

Systemic therapy:

• oral methotrexate is used first-line. It is particularly useful if there is associated


joint disease
• ciclosporin
• systemic retinoids
• biological agents: infliximab, etanercept and adalimumab
• ustekinumab (IL-12 and IL-23 blocker) is showing promise in early trials

Scalp psoriasis management

Outline

• NICE recommend the use of potent topical corticosteroids used once daily for 4
weeks
• if no improvement after 4 weeks then either use a different formulation of the
potent corticosteroid (for example, a shampoo or mousse) and/or a topical
agents to remove adherent scale (for example, agents containing salicylic acid,
emollients and oils) before application of the potent corticosteroid

Face, flexural and genital psoriasis management

Outline

• NICE recommend offering a mild or moderate potency corticosteroid applied


once or twice daily for a maximum of 2 weeks

Notes on medication

Using topical steroids in psoriasis

• as we know topical corticosteroid therapy may lead to skin atrophy, striae and
rebound symptoms
o the scalp, face and flexures are particularly prone to steroid atrophy so
topical steroids should not be used for more than 1-2 weeks/month
• systemic side-effects may be seen when potent corticosteroids are used on large
areas e.g. > 10% of the body surface area
• NICE recommend that we aim for a 4-week break before starting another course
of topical corticosteroids
• they also recommend using potent corticosteroids for no longer than 8 weeks at
a time and very potent corticosteroids for no longer than 4 weeks at a time

Vitamin D analogues

• examples of vitamin D analogues include calcipotriol (Dovonex), calcitriol and


tacalcitol
• they work by ↓ cell division and differentiation → ↓ epidermal proliferation
• adverse effects are uncommon
• unlike corticosteroids they may be used long-term
• unlike coal tar and dithranol they do not smell or stain
• they tend to reduce the scale and thickness of plaques but not the erythema
• they should be avoided in pregnancy
• the maximum weekly amount for adults is 100g

Dithranol

• inhibits DNA synthesis


• wash off after 30 mins
• adverse effects include burning, staining

Coal tar

• mechanism of action not fully understood - probably inhibit DNA synthesis

32. Leukoplakia

Leukoplakia is a premalignant condition which presents as white, hard spots on the


mucous membranes of the mouth. It is more common in smokers.

Leukoplakia is said to be a diagnosis of exclusion. Candidiasis and lichen planus


should be considered, especially if the lesions can be 'rubbed off'

Biopsies are usually performed to exclude alternative diagnoses such as squamous


cell carcinoma and regular follow-up is required to exclude malignant transformation
to squamous cell carcinoma, which occurs in around 1% of patients.

33. Shingles
Shingles (herpes zoster infection) is an acute, unilateral, painful blistering rash caused
by reactivation of the varicella-zoster virus (VZV). Following primary infection with
VZV (chickenpox), the virus lies dormant in the dorsal root or cranial nerve ganglia.

Risk factors

• increasing age
• HIV: strong risk factor, 15 times more common
• other immunosuppressive conditions (e.g. steroids, chemotherapy)

The most commonly affected dermatomes are T1-L2.

Features

• prodromal period
o burning pain over the affected dermatome for 2-3 days
o pain may be severe and interfere with sleep
o around 20% of patients will experience fever, headache, lethargy
• rash
o initially erythematous, macular rash over the affected dermatome
o quickly becomes vesicular
o characteristically is well demarcated by the dermatome and does not
cross the midline. However, some 'bleeding' into adjacent areas may be
seen

The diagnosis is usually clinical.

Management

• remind patients they are potentially infectious


o may need to avoid pregnant women and the immunosuppressed
o should be advised that they are infectious until the vesicles have crusted
over, usually 5-7 days following onset
o covering lesions reduces the risk
• analgesia
o paracetamol and NSAIDs are first-line
o if not responding then use of neuropathic agents (e.g. amitriptyline) can
be considered
o oral corticosteroids may be considered in the first 2 weeks in
immunocompetent adults with localized shingles if the pain is severe and
not responding to the above treatments
• antivirals
o NICE Clinical Knowledge Summaries makes recommendations on when to
use antivirals
o in practice, they recommend antivirals within 72 hours for the majority of
patients, unless the patient is < 50 years and has a 'mild' truncal rash
associated with mild pain and no underlying risk factors
o one of the benefits of prescribing antivirals is a reduced incidence of
post-herpetic neuralgia, particularly in older people
o aciclovir, famciclovir, or valaciclovir are recommended

Complications

• post-herpetic neuralgia
o the most common complications
o more common in older patients
o affects between 5%-30% of patients depending on age
o most commonly resolves with 6 months but may last longer
• herpes zoster ophthalmicus (shingles affecting affecting the ocular division of
the trigeminal nerve) is associated with a variety of ocular complications
• herpes zoster oticus (Ramsay Hunt syndrome): may result in ear lesions and
facial paralysis

34. Lichen planus

Lichen planus is a skin disorder of unknown aetiology, most probably being immune-
mediated.
Features

• itchy, papular rash most common on the palms, soles, genitalia and flexor
surfaces of arms
• rash often polygonal in shape, with a 'white-lines' pattern on the surface
(Wickham's striae)
• Koebner phenomenon may be seen (new skin lesions appearing at the site of
trauma)
• oral involvement in around 50% of patients: typically a white-lace pattern on the
buccal mucosa
• nails: thinning of nail plate, longitudinal ridging

Lichenoid drug eruptions - causes:

• gold
• quinine
• thiazides

Management

• potent topical steroids are the mainstay of treatment


• benzydamine mouthwash or spray is recommended for oral lichen planus
• extensive lichen planus may require oral steroids or immunosuppression

35. Actinic keratoses

Actinic, or solar, keratoses (AK) is a common premalignant skin lesion that develops
as a consequence of chronic sun exposure

Features

• small, crusty or scaly, lesions


• may be pink, red, brown or the same colour as the skin
• typically on sun-exposed areas e.g. temples of head
• multiple lesions may be present

Management options include

• prevention of further risk: e.g. sun avoidance, sun cream


• fluorouracil cream: typically a 2 to 3 week course. The skin will become red and
inflamed - sometimes topical hydrocortisone is given following fluorouracil to
help settle the inflammation
• topical diclofenac: may be used for mild AKs. Moderate efficacy but much fewer
side-effects
• topical imiquimod: trials have shown good efficacy
• cryotherapy
• curettage and cautery

36. Erythema nodosum

Overview

• inflammation of subcutaneous fat


• typically causes tender, erythematous, nodular lesions
• usually occurs over shins, may also occur elsewhere (e.g. forearms, thighs)
• usually resolves within 6 weeks
• lesions heal without scarring
Causes

• infection
o streptococci
o tuberculosis
o brucellosis
• systemic disease
o sarcoidosis
o inflammatory bowel disease
o Behcet's
• malignancy/lymphoma
• drugs
o penicillins
o sulphonamides
o combined oral contraceptive pill
• pregnancy

37. Antihistamines

Antihistamines (H1 inhibitors) are of value in the treatment of allergic rhinitis and
urticaria.

Examples of sedating antihistamines

• chlorpheniramine

As well as being sedating these antihistamines have some antimuscarinic properties


(e.g. urinary retention, dry mouth).

Examples of non-sedating antihistamines

• loratidine
• cetirizine

Of the non-sedating antihistamines there is some evidence that cetirizine may cause
more drowsiness than other drugs in the class.

38. Erythema ab igne


Erythema ab igne is a skin disorder caused by over exposure to infrared radiation.
Characteristic features include reticulated, erythematous patches with
hyperpigmentation and telangiectasia. A typical history would be an elderly women
who always sits next to an open fire.

If the cause is not treated then patients may go on to develop squamous cell skin
cancer.

39. Guttate psoriasis

Guttate psoriasis is more common in children and adolescents. It may be precipitated


by a streptococcal infection 2-4 weeks prior to the lesions appearing.

Features

• tear drop papules on the trunk and limbs

Management

• most cases resolve spontaneously within 2-3 months


• there is no firm evidence to support the use of antibiotics to eradicate
streptococcal infection
• topical agents as per psoriasis
• UVB phototherapy
• tonsillectomy may be necessary with recurrent episodes

Differentiating guttate psoriasis and pityriasis rosea

Guttate psoriasis Pityriasis rosea


Classically preceded by a
Many patients report recent respiratory tract
Prodrome streptococcal sore throat
infections but this is not common in questions
2-4 weeks
Herald patch followed 1-2 weeks later by
multiple erythematous, slightly raised oval
lesions with a fine scale confined to the outer
aspects of the lesions.
'Tear drop', scaly papules
Appearance
on the trunk and limbs
May follow a characteristic distribution with
the longitudinal diameters of the oval lesions
running parallel to the line of Langer. This
may produce a 'fir-tree' appearance
Guttate psoriasis Pityriasis rosea
Most cases resolve
spontaneously within 2-
Treatment /
3 months
natural Self-limiting, resolves after around 6 weeks
Topical agents as per
history
psoriasis
UVB phototherapy

40. Acne rosacea

Acne rosacea is a chronic skin disease of unknown aetiology.

Features

• typically affects nose, cheeks and forehead


• flushing is often first symptom
• telangiectasia are common
• later develops into persistent erythema with papules and pustules
• rhinophyma
• ocular involvement: blepharitis
• sunlight may exacerbate symptoms/

Management

• topical metronidazole may be used for mild symptoms (i.e. Limited number of
papules and pustules, no plaques)
• topical brimonidine gel may be considered for patients with predominant
flushing but limited telangiectasia
• more severe disease is treated with systemic antibiotics e.g. Oxytetracycline
• recommend daily application of a high-factor sunscreen
• camouflage creams may help conceal redness
• laser therapy may be appropriate for patients with prominent telangiectasia
• patients with a rhinophyma should be referred to dermatology

41. Psoriasis and psoriatic arthropathy: nail changes

Psoriatic nail changes affect both fingers and toes and do not reflect the severity of
psoriasis but there is an association with psoriatic arthropathy - around 80-90% of
patients with psoriatic arthropathy have nail changes.

Nail changes seen in psoriasis

• pitting
• onycholysis
• subungual hyperkeratosis
• loss of nail

42. Keloid scars

Keloid scars are tumour-like lesions that arise from the connective tissue of a scar
and extend beyond the dimensions of the original wound

Predisposing factors

• ethnicity: more common in people with dark skin


• occur more commonly in young adults, rare in the elderly
• common sites (in order of decreasing frequency): sternum, shoulder, neck, face,
extensor surface of limbs, trunk

Keloid scars are less likely if incisions are made along relaxed skin tension lines*

Treatment

• early keloids may be treated with intra-lesional steroids e.g. triamcinolone


• excision is sometimes required

*Langer lines were historically used to determine the optimal incision line. They were
based on procedures done on cadavers but have been shown to produce worse
cosmetic results than when following skin tension lines

43. Erythema multiforme

Erythema multiforme is a hypersensitivity reaction which is most commonly triggered


by infections. It may be divided into minor and major forms.

Previously it was thought that Stevens-Johnson syndrome (SJS) was a severe form of
erythema multiforme. They are now however considered as separate entities.

Features

• target lesions
• initially seen on the back of the hands / feet before spreading to the torso
• upper limbs are more commonly affected than the lower limbs
• pruritus is occasionally seen and is usually mild

Causes

• viruses: herpes simplex virus (the most common cause), Orf*


• idiopathic
• bacteria: Mycoplasma, Streptococcus
• drugs: penicillin, sulphonamides, carbamazepine, allopurinol, NSAIDs, oral
contraceptive pill, nevirapine
• connective tissue disease e.g. Systemic lupus erythematosus
• sarcoidosis
• malignancy

The more severe form, erythema multiforme major is associated with mucosal involvement.
*Orf is a skin disease of sheep and goats caused by a parapox virus

44. Pemphigus vulgaris

Pemphigus vulgaris is an autoimmune disease caused by antibodies directed against


desmoglein 3, a cadherin-type epithelial cell adhesion molecule. It is more common
in the Ashkenazi Jewish population.

Features

• mucosal ulceration is common and often the presenting symptom. Oral


involvement is seen in 50-70% of patients
• skin blistering - flaccid, easily ruptured vesicles and bullae. Lesions are typically
painful but not itchy. These may develop months after the initial mucosal
symptoms. Nikolsky's describes the spread of bullae following application of
horizontal, tangential pressure to the skin
• acantholysis on biopsy

45. Skin type

Skin type is an important risk factor for skin cancer. Skin types may be classified
according to Fitzpatrick classification:

• I: Never tans, always burns (often red hair, freckles, and blue eyes)
• II: Usually tans, always burns
• III: Always tans, sometimes burns (usually dark hair and brown eyes)
• IV: Always tans, rarely burns (olive skin)
• V: Sunburn and tanning after extreme UV exposure (brown skin, e.g. Indian)
• VI: Black skin (e.g. Afro-Caribbean), never tans, never burns
46.Nickel dermatitis

• Nickel is a common cause allergic contact dermatitis and is an example of a type IV


hypersensitivity reaction. It is often caused by jewellery such as watches
It is diagnosed by a skin patch test.
47. Skin disorders associated with pregnancy

Atopic eruption of pregnancy

• is the commonest skin disorder found in pregnancy


• it typically presents as an eczematous, itchy red rash.
• no specific treatment is needed

Polymorphic eruption of pregnancy

• pruritic condition associated with last trimester


• lesions often first appear in abdominal striae
• management depends on severity: emollients, mild potency topical steroids and
oral steroids may be used

Pemphigoid gestationis

• pruritic blistering lesions


• often develop in peri-umbilical region, later spreading to the trunk, back,
buttocks and arms
• usually presents 2nd or 3rd trimester and is rarely seen in the first pregnancy
• oral corticosteroids are usually required

48. Spider naevi

Spider naevi (also called spider angiomas) describe a central red papule with
surrounding capillaries. The lesions blanch upon pressure. Spider naevi are almost
always found on the upper part of the body.

Spider naevi can be differentiated from telangiectasia by pressing on them and


watching them fill. Spider naevi fill from the centre, telangiectasia from the edge .

Around 10-15% of people will have one or more spider naevi and they are more
common in childhood. Other associations

• liver disease
• pregnancy
• combined oral contraceptive pill

49. Pityriasis versicolor

Pityriasis versicolor, also called tinea versicolor, is a superficial cutaneous fungal


infection caused by Malassezia furfur (formerly termed Pityrosporum ovale)

Features

• most commonly affects trunk


• patches may be hypopigmented, pink or brown (hence versicolor). May be more
noticeable following a suntan
• scale is common
• mild pruritus

Predisposing factors

• occurs in healthy individuals


• immunosuppression
• malnutrition
• Cushing's

Management

• topical antifungal. NICE Clinical Knowledge Summaries advise ketoconazole


shampoo as this is more cost effective for large areas
• if failure to respond to topical treatment then consider alternative diagnoses
(e.g. send scrapings to confirm the diagnosis) + oral itraconazole

50. Retinoids

Isotretinoin is an oral retinoid used in the treatment of severe acne. Two-thirds of patients
have a long-term remission or cure following a course of oral isotretinoin.

Adverse effects

• teratogenicity
o females should ideally be using two forms of contraception (e.g. Combined
oral contraceptive pill and condoms)
• dry skin, eyes and lips/mouth
o the most common side-effect of isotretinoin
• low mood
o whilst this is a controversial topic, depression and other psychiatric problems
are listed in the BNF
• raised triglycerides
• hair thinning
• nose bleeds (caused by dryness of the nasal mucosa)
• intracranial hypertension: isotretinoin treatment should not be combined with
tetracyclines for this reason
• photosensitivity

51. Seborrhoeic keratoses

Seborrhoeic keratoses are benign epidermal skin lesions seen in older people.

Features

• large variation in colour from flesh to light-brown to black


• have a 'stuck-on' appearance
• keratotic plugs may be seen on the surface

Management

• reassurance about the benign nature of the lesion is an option


• options for removal include curettage, cryosurgery and shave biopsy

52. Burns pathology

Extensive burns

• Haemolysis due to damage of erythrocytes by heat and microangiopathy


• Loss of capillary membrane integrity causing plasma leakage into interstitial space
• Extravasation of fluids from the burn site causing hypovolaemic shock (up to 48h after
injury)- decreased blood volume and increased haematocrit
• Protein loss
• Secondary infection e.g. Staphylococcus aureus
• ARDS
• Risk of Curlings ulcer (acute peptic stress ulcers)
• Danger of full thickness circumferential burns in an extremity as these may develop
compartment syndrome

Healing
• Superficial burns: keratinocytes migrate to form a new layer over the burn site
• Full thickness burns: dermal scarring. Usually need keratinocytes from skin grafts to
provide optimal coverage.

53.Skin disorders affecting the soles of the feet

• The table below gives characteristic exam question features for conditions affecting the
soles of the feet
Diagnosis Notes
Secondary to the human papilloma virus
Firm, hyperkeratotic lesions
Verrucas
Pinpoint petechiae centrally within the lesions
May coalesce with surrounding warts to form mosaic warts
More commonly called Athlete's foot
Tinea pedis
Affected skin is moist, flaky and itchy
A corn is small areas of very thick skin secondary to a reactive
hyperkeratosis
Corn and calluses
A callus is larger, broader and has a less well defined edge than a
corn
May be acquired or congenital
Describes a thickening of the skin of the palms and soles
Keratoderma
Acquired causes include reactive arthritis (keratoderma
blennorrhagica)
Affects people who sweat excessively
Patients may complain of damp and excessively smelly feet
Pitted keratolysis Usually caused by Corynebacterium
Heel and forefoot may become white with clusters of punched-out
pits
Crops of sterile pustules affecting the palms and soles
Palmoplantar pustulosis The skin is thickened, red. Scaly and may crack
More common in smokers
Affects children. More common in atopic patients with a history
Juvenile plantar of eczema
dermatosis Soles become shiny and hard. Cracks may develop causing pain
Worse during the summer

54. Sebaceous cysts

Sebaceous cysts is a general term which encompasses both epidermoid and pilar cysts. It is a
bit of a misnomer and probably best avoided where possible.

Epidermoid cysts are due to a proliferation of epidermal cells within the dermis. Pilar cysts
(also known as trichilemmal cysts or wen) derive from the outer root sheath of the hair
follicle.
Location: anywhere but most common scalp, ears, back, face, and upper arm (not palms of
the hands and soles of the feet).

They will typically contain a punctum.

Excision of the cyst wall needs to be complete to prevent recurrence.


55. Pityriasis rosea

Pityriasis rosea describes an acute, self-limiting rash which tends to affect young adults. The
aetiology is not fully understood but is thought that herpes hominis virus 7 (HHV-7) may
play a role.

Features

• in the majority of patients there is no prodrome, but a minority may give a history of a
recent viral infection
• herald patch (usually on trunk)
• followed by erythematous, oval, scaly patches which follow a characteristic distribution
with the longitudinal diameters of the oval lesions running parallel to the line of Langer.
This may produce a 'fir-tree' appearance

Management

• self-limitingm - usually disappears after 6-12 weeks

55. Fungal nail infections

Onychomycosis is fungal infection of the nails. This may be caused by

• dermatophytes - mainly Trichophyton rubrum, accounts for 90% of cases


• yeasts - such as Candida
• non-dermatophyte moulds

Risk factors include for fungal nail infections include diabetes mellitus andincreasing age.

Features

• 'unsightly' nails are a common reason for presentation


• thickened, rough, opaque nails are the most common finding

Differential diagnosis

• psoriasis
• repeated trauma
• lichen planus
• yellow nail syndrome

Investigation

• nail clippings
• scrapings of the affected nail
• the false-negative rate for cultures are around 30%, so repeat samples may need to be
sent if the clinical suspicion is high

Management

• do not need to be treated if it is asymptomatic and the patient is not bothered by the
appearance
• diagnosis should be confirmed by microbiology before starting treatment
• dermatophyte infection:
o oral terbinafine is currently recommended first-line with oral itraconazole as an
alternative
o 6 weeks - 3 months therapy is needed for fingernail infections whilst toenails
should be treated for 3 - 6 months
o treatment is successful in around 50-80% of people
• Candida infection:
o mild disease should be treated with topical antifungals (e.g. Amorolfine) whilst
more severe infections should be treated with oral itraconazole for a period of 12
weeks
• if topical topical treatment is given treatment should be continued for 6 months for
fingernails and 9-12 months for toenails.

56. Erythroderma

Erythroderma is a term used when more than 95% of the skin is involved in a rash of any
kind.

Causes of erythroderma

• eczema
• psoriasis
• drugs e.g. gold
• lymphomas, leukaemias
• idiopathic
Erythrodermic psoriasis

• may result from progression of chronic disease to an exfoliative phase with plaques
covering most of the body. Associated with mild systemic upset
• more serious form is an acute deterioration. This may be triggered by a variety of factors
such as withdrawal of systemic steroids. Patients need to be admitted to hospital for
management

57. Purpura

Purpura describes bleeding into the skin from small blood vessels that produces a non-
blanching rash. Smaller petechiae (1-2 mm in diameter) may also be seen. It is typically
caused by low platelets but may also be seen with bleeding disorders, such as von Willebrand
disease.

It is important to recognise purpura as it can indicate the presence of serious underlying


disease. Children with a new purpuric rash should be admitted immediately for investigations
as it may be a sign of meningococcal septicaemia or acute lymphoblastic leukaemia.
Parenteral antibiotics should be given prior to transfer if meningococcal septicaemia is
suspected.

Causes

Children Adults
• Meningococcal septicaemia • Immune thrombocytopenic purpura
• Acute lymphoblastic leukaemia • Bone marrow failure (secondary to leukaemias,
myelodysplasia or bone metastases)
• Congenital bleeding disorders • Senile purpura
• Immune thrombocytopenic purpura • Drugs (quinine, antiepileptics, antithrombotics)
• Henoch-Schonlein purpura • Nutritional deficiencies (vitamins B12, C and
• Non-accidental injury folate)

Raised superior vena cava pressure (e.g. secondary to a bad cough) may cause petechiae in
the upper body but would not cause purpura.
58. Psoriasis

Psoriasis is a common (prevalence around 2%) and chronic skin disorder. It generally
presents with red, scaly patches on the skin although it is now recognised that patients with
psoriasis are at increased risk of arthritis and cardiovascular disease.

Pathophysiology

• multifactorial and not yet fully understood


• genetic: associated HLA-B13, -B17, and -Cw6. Strong concordance (70%) in identical
twins
• immunological: abnormal T cell activity stimulates keratinocyte proliferation. There is
increasing evidence this may be mediated by a novel group of T helper cells producing
IL-17, designated Th17. These cells seem to be a third T-effector cell subset in addition
to Th1 and Th2
• environmental: it is recognised that psoriasis may be worsened (e.g. Skin trauma,
stress), triggered (e.g. Streptococcal infection) or improved (e.g. Sunlight) by
environmental factors

Recognised subtypes of psoriasis

• plaque psoriasis: the most common sub-type resulting in the typical well-demarcated
red, scaly patches affecting the extensor surfaces, sacrum and scalp
• flexural psoriasis: in contrast to plaque psoriasis the skin is smooth
• guttate psoriasis: transient psoriatic rash frequently triggered by a streptococcal
infection. Multiple red, teardrop lesions appear on the body
• pustular psoriasis: commonly occurs on the palms and soles

Other features

• nail signs: pitting, onycholysis


• arthritis

Complications

• psoriatic arthropathy (around 10%)


• increased incidence of metabolic syndrome
• increased incidence of cardiovascular disease
• increased incidence of venous thromboembolism
• psychological distress

59. Squamous cell carcinoma of the skin

Squamous cell carcinoma is a common variant of skin cancer. Metastases are rare but may
occur in 2-5% of patients.

Risk factors include:

• excessive exposure to sunlight / psoralen UVA therapy


• actinic keratoses and Bowen's disease
• immunosuppression e.g. following renal transplant, HIV
• smoking
• long-standing leg ulcers (Marjolin's ulcer)
• genetic conditions e.g. xeroderma pigmentosum, oculocutaneous albinism
Treatment
Surgical excision with 4mm margins if lesion <20mm in diameter. If tumour >20mm then
margins should be 6mm. Mohs micrographic surgery may be used in high-risk patients and in
cosmetically important sites.

Prognosis
Good Prognosis Poor prognosis
Well differentiated tumours Poorly differentiated tumours
<20mm diameter >20mm in diameter
<2mm deep >4mm deep
No associated diseases Immunosupression for whatever reason

60.Milia

• Milia are small, benign, keratin-filled cysts that typically appear around the face. They
may appear at any age but are more common in newborns.
61. Toxic epidermal necrolysis

Toxic epidermal necrolysis (TEN) is a potentially life-threatening skin disorder that is most
commonly seen secondary to a drug reaction. In this condition, the skin develops a scalded
appearance over an extensive area. Some authors consider TEN to be the severe end of a
spectrum of skin disorders which includes erythema multiforme and Stevens-Johnson
syndrome,

Features

• systemically unwell e.g. pyrexia, tachycardic


• positive Nikolsky's sign: the epidermis separates with mild lateral pressure

Drugs known to induce TEN

• phenytoin
• sulphonamides
• allopurinol
• penicillins
• carbamazepine
• NSAIDs

Management

• stop precipitating factor


• supportive care
o often in an intensive care unit
o volume loss and electrolyte derangement are potential complications
• intravenous immunoglobulin has been shown to be effective and is now commonly used
first-line
• other treatment options include: immunosuppressive agents (ciclosporin and
cyclophosphamide), plasmapheresis

62. Basal cell carcinoma

Basal cell carcinoma (BCC) is one of the three main types of skin cancer. Lesions are also
known as rodent ulcers and are characterised by slow-growth and local invasion. Metastases
are extremely rare. BCC is the most common type of cancer in the Western world.

Features

• many types of BCC are described. The most common type is nodular BCC, which is
described here
• sun-exposed sites, especially the head and neck account for the majority of lesions
• initially a pearly, flesh-coloured papule with telangiectasia
• may later ulcerate leaving a central 'crater'

Referral

• generally, if a BCC is suspected, a routine referral should be made

Management options:

• surgical removal
• curettage
• cryotherapy
• topical cream: imiquimod, fluorouracil
• radiotherapy

** Basal cell carcinomas can generally be referred on a routine basis for surgical excision.
Exceptions to this are lesions where delay may cause damage, for example the eyelids and
nasal ala. These should be referred urgently under the 2-week wait.

63. Stevens-Johnson syndrome

Stevens-Johnson syndrome is a severe systemic reaction affecting the skin and mucosa that is
almost always caused by a drug reaction.

Previously it was thought that Stevens-Johnson syndrome (SJS) was a severe form of
erythema multiforme. They are now however considered as separate entities.

Features
• rash is typically maculopapular with target lesions being characteristic. May develop
into vesicles or bullae
• mucosal involvement
• systemic symptoms: fever, arthralgia

Causes

• penicillin
• sulphonamides
• lamotrigine, carbamazepine, phenytoin
• allopurinol
• NSAIDs
• oral contraceptive pill

Management

• hospital admission is required for supportive treatment

** This woman is suffering from Stevens-Johnson syndrome. It is commonly caused by anti-


epileptics such as carbamazepine and is characterised by a rash affecting <10% of body
surface area and mucosal involvement.

Toxic epidermal necrolysis also involves mucous membranes but the rash is more extensive
affecting at least 30% of body surface area. It is also triggered by drugs.

The cause of erythema multiforme is not known. The rash usually has an acral distribution
and lesions are often raised (erythema multiforme minor). The major form has the same rash
as the minor form but also has mucous membrane involvement.

Erythroderma is erythematous dermatitis affecting over 90% of the skin. Exacerbation of


psoriasis is a common cause.

64. Alopecia areata

Alopecia areata is a presumed autoimmune condition causing localised, well demarcated


patches of hair loss. At the edge of the hair loss, there may be small, broken 'exclamation
mark' hairs

Hair will regrow in 50% of patients by 1 year, and in 80-90% eventually. Careful explanation
is therefore sufficient in many patients. Other treatment options include:

• topical or intralesional corticosteroids


• topical minoxidil
• phototherapy
• dithranol
• contact immunotherapy
• wigs
**Watchful waiting for spontaneous remission is another option. Neither the British
Association of Dermatologists or Clinical Knowledge Summaries recommend screening for
autoimmune disease

65. Mycosis fungoides

Mycosis fungoides is a rare form of T-cell lymphoma that affects the skin.

Features

• itchy, red patches which are <="" psoriasis="" or="" eczema="" with="" confused="">
• lesions tend to be of different colours in contrast to eczema/psoriasis where there is
greater homogenicity

**Given the slightly unusual history, previous diagnostic uncertainty and the laboratory
findings, cutaneous T-cell lymphoma is the most likely diagnosis. Pautrier microabscesses
are typical of mycosis fungoides on biopsy.
Harlequin ichthyosis is a genetic condition, so symptoms would have been present since
birth.
Syphilis can give a variety of different skin findings, but the biopsy results are not consistent
with this diagnosis.
Discoid eczema, also known as nummular dermatitis, has a different natural history to this
patient, and again the biopsy results do not point towards it.
Pityriasis lichenoides is seen in younger patients, and the history of the lesions is much
different to that described above.

66. Acne rosacea

Acne rosacea treatment:

• mild/moderate: topical metronidazole


• severe/resistant: oral tetracycline

Acne rosacea is a chronic skin disease of unknown aetiology.

Topical metronidazole gel is the first line treatment for acne rosacea.
Benzoyl peroxide gel, tetracycline tablets and erythromycin tablets are all used in the
treatment of acne vulgaris.
Hydrocortisone 1% ointment is used in the treatment of mild inflammatory skin disorders.

67. Skin disorders associated with malignancy

Paraneoplastic syndromes associated with internal malignancies:


Skin disorder Associated malignancies
Acanthosis nigricans Gastric cancer
Acquired ichthyosis Lymphoma
Acquired hypertrichosis lanuginosa Gastrointestinal and lung cancer
Dermatomyositis Ovarian and lung cancer
Erythema gyratum repens Lung cancer
Erythroderma Lymphoma
Migratory thrombophlebitis Pancreatic cancer
Necrolytic migratory erythema Glucagonoma
Pyoderma gangrenosum (bullous and
Myeloproliferative disorders
non-bullous forms)
Haematological malignancy e.g. Myelodysplasia -
Sweet's syndrome
tender, purple plaques
Tylosis Oesophageal cancer

68. Bullous pemphigoid

Bullous pemphigoid is an autoimmune condition causing sub-epidermal blistering of the skin.


This is secondary to the development of antibodies against hemidesmosomal proteins BP180
and BP230.

Bullous pemphigoid is more common in elderly patients. Features include

• itchy, tense blisters typically around flexures


• the blisters usually heal without scarring
• there is usually no mucosal involvement (i.e. the mouth is spared)*

Skin biopsy

• immunofluorescence shows IgG and C3 at the dermoepidermal junction

Management

• referral to a dermatologist for biopsy and confirmation of diagnosis


• oral corticosteroids are the mainstay of treatment
• topical corticosteroids, immunosuppressants and antibiotics are also used

**in reality around 10-50% of patients have a degree of mucosal involvement. It would,
however, be unusual for an exam question to mention mucosal involvement as it is seen as a
classic differentiating feature between pemphigoid and pemphigus.
**Blisters/bullae

• no mucosal involvement (in exams at least*): bullous pemphigoid


• mucosal involvement: pemphigus vulgaris
69. Shingles

Shingles (herpes zoster infection) is an acute, unilateral, painful blistering rash caused by
reactivation of the varicella-zoster virus (VZV). Following primary infection with VZV
(chickenpox), the virus lies dormant in the dorsal root or cranial nerve ganglia.

Risk factors

• increasing age
• HIV: strong risk factor, 15 times more common
• other immunosuppressive conditions (e.g. steroids, chemotherapy)

The most commonly affected dermatomes are T1-L2.

Features

• prodromal period
o burning pain over the affected dermatome for 2-3 days
o pain may be severe and interfere with sleep
o around 20% of patients will experience fever, headache, lethargy
• rash
o initially erythematous, macular rash over the affected dermatome
o quickly becomes vesicular
o characteristically is well demarcated by the dermatome and does not cross the
midline. However, some 'bleeding' into adjacent areas may be seen

The diagnosis is usually clinical.

Management

• remind patients they are potentially infectious


o may need to avoid pregnant women and the immunosuppressed
o should be advised that they are infectious until the vesicles have crusted over,
usually 5-7 days following onset
o covering lesions reduces the risk
• analgesia
o paracetamol and NSAIDs are first-line
o if not responding then use of neuropathic agents (e.g. amitriptyline) can be
considered
o oral corticosteroids may be considered in the first 2 weeks in immunocompetent
adults with localized shingles if the pain is severe and not responding to the
above treatments
• antivirals
o NICE Clinical Knowledge Summaries makes recommendations on when to use
antivirals
o in practice, they recommend antivirals within 72 hours for the majority of
patients, unless the patient is < 50 years and has a 'mild' truncal rash associated
with mild pain and no underlying risk factors
o one of the benefits of prescribing antivirals is a reduced incidence of post-
herpetic neuralgia, particularly in older people
o aciclovir, famciclovir, or valaciclovir are recommended

Complications

• post-herpetic neuralgia
o the most common complications
o more common in older patients
o affects between 5%-30% of patients depending on age
o most commonly resolves with 6 months but may last longer
• herpes zoster ophthalmicus (shingles affecting affecting the ocular division of the
trigeminal nerve) is associated with a variety of ocular complications
• herpes zoster oticus (Ramsay Hunt syndrome): may result in ear lesions and facial
paralysis

70.Pyoderma gangrenosum

Features

• typically on the lower limbs


• initially small red papule
• later deep, red, necrotic ulcers with a violaceous border
• may be accompanied systemic symptoms e.g. Fever, myalgia

Causes*

• idiopathic in 50%
• inflammatory bowel disease: ulcerative colitis, Crohn's
• rheumatoid arthritis, SLE
• myeloproliferative disorders
• lymphoma, myeloid leukaemias
• monoclonal gammopathy (IgA)
• primary biliary cirrhosis

Management

• the potential for rapid progression is high in most patients and most doctors advocate
oral steroids as first-line treatment
• other immunosuppressive therapy, for example ciclosporin and infliximab, have a role
in difficult cases
*note whilst pyoderma gangrenosum can occur in diabetes mellitus it is rare and is generally
not included in a differential of potential causes
71. Impetigo

Impetigo is a superficial bacterial skin infection usually caused by either Staphylcoccus


aureus or Streptococcus pyogenes. It can be a primary infection or a complication of an
existing skin condition such as eczema (in this case), scabies or insect bites. Impetigo is
common in children, particularly during warm weather.

The infection can develop anywhere on the body but lesions tend to occur on the face,
flexures and limbs not covered by clothing.

Spread is by direct contact with discharges from the scabs of an infected person. The bacteria
invade the skin through minor abrasions and then spread to other sites by scratching.
Infection is spread mainly by the hands, but indirect spread via toys, clothing, equipment and
the environment may occur. The incubation period is between 4 to 10 days.

Features

• 'golden', crusted skin lesions typically found around the mouth


• very contagious

Management

Limited, localised disease

• NICE Clinical Knowledge Summaries now recommend hydrogen peroxide 1% cream


for 'people who are not systemically unwell or at a high risk of complications'
o the change was announced in 2020 by NICE and Public Health England and
seems aimed at cutting antibiotic resistance
o the evidence base shows it is just as effective at treating non-bullous impetigo as
a topical antibiotic
• topical antibiotic creams:
o topical fusidic acid
o topical mupirocin should be used if fusidic acid resistance is suspected
o MRSA is not susceptible to either fusidic acid or retapamulin. Topical mupirocin
should, therefore, be used in this situation

Extensive disease

• oral flucloxacillin
• oral erythromycin if penicillin-allergic

School exclusion
• children should be excluded from school until the lesions are crusted and healed or 48
hours after commencing antibiotic treatment

72. Pompholyx

Pompholyx is a type of eczema which affects both the hands (cheiropompholyx) and the feet
(pedopompholyx). It is also known as dyshidrotic eczema.
Pompholyx eczema may be precipitated by humidity (e.g. sweating) and high temperatures.

Features

• small blisters on the palms and soles


• pruritic
o often intensely itchy
o sometimes burning sensation
• once blisters burst skin may become dry and crack

Management

• cool compresses
• emollients
• topical steroids

**Pompholyx eczema (also known as dyshidrotic eczema) is a type of vesicular hand


dermatitis. It is more common in young adult females and is commonly related to sweating,
with flares during hot weather and humid conditions. It manifests as recurrent crops of
vesicles and/or blisters on the palms which can cause intense itching and burning. The
vesicular stage is usually followed by a dry, desquamating phase in which the skin can peel,
crack or crust.

Treatment usually involves liberal use of emollients, avoidance of irritants and potent or very
potent topical corticosteroids.

Keratoderma blennorhagicum presents as vesico-pustular waxy yellow-brown lesions on the


palms and soles. It is associated with reactive arthritis.

Palmoplantar pustulosis is a chronic condition associated with fluid-filled blisters on the


palms and sole of the feet. Distinguishing this from pompholyx eczema clinically can be
difficult. However, about 25% of patients with palmoplantar pustulosis have psoriasis on
other body parts, and there is a very strong association with smoking. The correlation with
high temperatures and humidity is also not as strong compared with pompholyx eczema.

Scabies - caused by infestation by the parasite Sarcoptes scabiei - manifests as intense itching
of the hands and web spaces but is not usually a recurring condition. Clinical features include
linear burrows and erythematous papules, rather than vesicles.
Tinea manuum describes a dermatophyte infection of one or both hands. It usually appears as
a slowly extending area of dry, peeling and scaly erythematous skin and typically only affects
one hand. It is more common in those who do manual work who sweat profusely in their
hands.

73. Bullous disorders

Causes of skin bullae

• congenital: epidermolysis bullosa


• autoimmune: bullous pemphigoid, pemphigus
• insect bite
• trauma/friction
• drugs: barbiturates, furosemide

74. Granuloma annulare

Basics

• papular lesions that are often slightly hyperpigmented and depressed centrally
• typically occur on the dorsal surfaces of the hands and feet, and on the extensor
aspects of the arms and legs

A number of associations have been proposed to conditions such as diabetes


mellitus but there is only weak evidence for this

75. Skin disorders associated with diabetes

Note whilst pyoderma gangrenosum can occur in diabetes mellitus it is rare and is
often not included in a differential of potential causes

Necrobiosis lipoidica

• shiny, painless areas of yellow/red/brown skin typically on the shin


• often associated with surrounding telangiectasia

Infection

• candidiasis
• staphylococcal
Neuropathic ulcers

Vitiligo

Lipoatrophy

Granuloma annulare*

• papular lesions that are often slightly hyperpigmented and depressed centrally

*it is not clear from recent studies if there is actually a significant association between
diabetes mellitus and granuloma annulare, but it is often listed in major textbooks

You might also like